Pre-Medical Expert Answers

From Student Doctor Network Wiki
Revision as of 17:44, 11 December 2019 by SDN Team 1 (talk | contribs)
Jump to navigation Jump to search

Below are some frequently asked questions that have been answered by SDN Pre-Medical experts.

If you have a question that is not answered here, please visit the SDN Confidential Consult forum to ask your question of our expert panel.

Contents

How do I find a doctor who would be willing to let me shadow them?

I am a freshman pre-medical student attending school at the University of Central Florida. I recently began volunteering at the Nemours Children's Hospital in the surgical unit as I hope to become a pediatric surgeon or neurosurgeon. Based on the advice of my counselor, I was hoping I would have some form of interaction with the surgeons and perhaps be able to ask them to allow me to shadow them. However, I rarely see surgeons where I volunteer (post-op recovery) and there aren't any other areas that would allow me more access to them. I don't know any surgeons or doctors I can shadow in my area. I am really worried about gaining a competitive amount of shadowing hours before I apply to medical schools. Do you have any suggestions on how to find a doctor that would be willing to let me shadow them? Do they require certain classes to be taken before shadowing? How would be the best way to approach them? Any help would be greatly appreciated!

mark-ER

Shadowing docs is a good idea, though shadowing neurosurgeons while an undergrad, may be too much too early. I have heard of undergrads who do physiology/joint experiments shadow orthopods, or general surgeons or even OB/Gyn, but cardiothorasic and neurosurg is typically off limits to undergrads. You have to know how to scrub in, know these cases in & out to get anything out of the shadowing experience. These are typically very intense procedures, with a lot at state, where time under anesthesia is carefully accounted for and there is less time to teach or grasp complicated concepts even for residents, or medical students, much less undergrads.

What I would recommend instead, if you are truly interested in peds/neurosurg, is to shadow a pediatric neurologist or a pediatric geneticist with interest in neurological diseases. Peds docs are generally more amendable to be shadowed, are generally quite friendly and responsive to a request for teaching. Some might even let you come in and with some preparation to a brief SOAP note as/before they are seeing the patient -- perhaps with a resident (?), a useful experience and something to talk about during med school interviews.

My approach to this would be to look for a residency program in pediatric neurology in your area, and email the program director asking to point you to a doc who would be amendable to having an undergraduate with an interest in neuroscience (don't say anything about neurosurgery, at least an the outset) to shadow you.

Oh and a few tips for shadowing: be humble, be on time (which means early) and be very careful/gentle in attempts to help/chime in with answers. Medicine has some things in common with the military hierarchy in this aspect -- speak when spoken to, and generally it is best to stay out of the way. It is better not to be remembered, or thought of as bland/unoffensive, than overeager/annoying or a know-it-all. Please remember, you know next to nothing at this point, and if you happen to know some esoteric factoid, do not chime in obtrusively, to avoid to be perceived to overshadow senior trainees like med students/residents. Again, not necessarily directed at the original poster, but rather to other ambitious undergrads who might be reading this reply further on down the line.


Do I still have a chance?

I am a sophomore undergrad student at uconn who just finished her third semester. I'm not doing so well in my science classes. First semester I took a genetics class and gen chem I for which I got an A- and a B respectively. Second semester I took gen chem II and got a B-. This semester I'm ending with a C+ for orgo I. I'm doing pretty well in all my other classes. I was talking to a paid advisor about game plans and at the beginning of this semester he said this orgo class was my make or break. Now that I've told him that I got a C+, he is saying that I still have a chance if I really apply myself. I can't tell if he's being serious or if he is just trying to get more money out of me. If I were somehow able to get my crap together, would I still have a chance at med school or should I look into other career options? Thanks!

tantacles

It is absolutely possible that you could still get into medical school. However, I would say that your performance in physics (the last class in the pre-med series) will be very important. In addition, if you are able to do well in upper level science courses, this will also serve you well and show that you are able to handle medical school.

That being said, I think it is ALWAYS a good idea to have a backup plan. Think about what you would do if not medicine as even many qualified applicants are not able to gain admission to medical school.


Can you get away without showing all the transcripts in the medical school application?

So my story goes like this: I completed my associates degree in Biology through a program in High school called Running Start, where I attained college credits during high school. I got a 3.41 GPA and a 2.9 Science GPA. Once I graduated from High school with my diploma and Associates degree, I went to a 4 year institution, WSU - Pullman campus. My first semester was terrible, got an F in organic chemistry, a C in Genetics and A- in General Physics, (Phy 101). Knowing that Medical schools average out GPA in the medical school application, how would medical schools know if I just started fresh from another university next semester and retook the same classes and got a better GPA? And I would pretend nothing happened, and finish the rest of my degree over at the new University. And I would send my community college transcripts and the new University i went to start fresh from. Yes, I take Financial aid from every where I went. But would I have to write down that I took classes at WSU for one semester? What if I dont report and just started fresh at a newer university?

tantacles

You are required to show your transcript for every undergraduate institution you attended. Medical schools use a service to determine if you have submitted all transcripts, so if you were not to submit this transcript, it is likely that no medical school would interview you as they would see that you had omitted a transcript.


Am I done for? How can I improve?

Hi everyone! I just finished my first semester of college as a pre-medical student. I went through so many family problems this semester that it was difficult for me to focus entirely on classes. There were deaths in the family and my mom needed caring for due to health issues, so I would have to frequently go back home. My grades suffered and I ended up getting a 3.25 GPA (B- in gen chem 1, B in Calculus 1, B+ in writing, A in Spanish III). I know that none of this is an excuse for bad grades because Med school will be a hundred times harder, but I want to know if it’s time to rule out med school for the future. As I said, this was my first semester of college, so I don’t know if a lot of this had to do with having bad study habits as well.

Also, how can I improve my studying habits and be better at subjects that I know for a fact I could’ve performed so much better in?

tantacles

A 3.25 GPA is not a bad GPA and leaves a lot of room for improvement. If you are able to improve your GPA and do well on the MCAT, your performance this semester should not hold you back.


MD chances... DO?

Hi All,

I am posting just out of curiosity... I am a first generation student with plans on attending MD/DO school. Currently I am just finishing up my fall semester of my junior year with a sci gpa of 3.54 and an overall gpa of 3.46. With 2 pre reqs to take ( Physics I and Calc), as well as numerous sci classes to fulfill my major for my spring semester and senior year (Bio emphasis on A&P/Minor in chem) what are my chances pending a decent MCAT score (taking this summer) on getting into a mid- lower tier MD school or decent DO school? I should also point out that I have 100+ hours of community service, 100+ clinical research hrs in the ED, multiple leadership experiences, and 20+ hrs of shadowing currently.

Side not question- Does being a dual national effect my chances?

tantacles

If you get a good MCAT score, you have a good chance of admission. My suggestion is that you purchase the MSAR and take a look at GPA and MCAT averages for many schools and try to target those schools (excluding state schools where few out of state students are accepted). My biggest suggestion is that you do your absolute best for the MCAT and not take it until you are truly ready to do well; that may mean taking a gap year so that you have time to study.

Being a dual national typically does not affect the chance of getting into medical school.


Asking for a letter of recommendation from a professor who was a postdoc when they taught me? Is that a bad idea or fine?

Hi, I was wondering if it would be bad to ask for a letter of recommendation from a professor who was a post doc when they taught me (they also taught me in a class that is highly recommended you take before applying to med school)?

mark-ER

Short answer = yes, it's a perfectly appropriate source for a letter of recommendation. Couple of caveats: is the person now a professor in the sciences (esp. biomedical sciences)? Is he/she employed by a reputable school (i.e. accredited college, university of medical school, preferrably in the USA)? How long ago did you closely work together and is that person going to remember you and write you a STRONG letter? The questions are more to spark your thinking bout "is this appropriate person to write a letter" (most likely yes), but is this the best person to serve as a reference (i.e. maybe there is someone better).


Medical School Application Advice

Hello, I have what I would consider a bit of a unique situation and am in need of some advice on how to proceed with applying to medical school.

I am graduating undergrad with a BS in Biomedical Sciences with a roundabout 3.3-3.4 GPA and a 520 MCAT score.

During my second and third years as an undergraduate, four of my immediate family members were diagnosed with incurable cancers that required extensive care. I was a part of my family's care team, and part of my duties was to transport those family members to chemo appointments located out of state. My family also maked their living from multiple family businesses, so part of my time was also used to assist in the upkeep of these businesses until additional help could be found.

I have also worked a full-time job in healthcare for all 4 of my years as an undergraduate. During this time, I logged nearly 6,000 clinical hours. In addition to working full time, I currently lead a research team of RN's and PharmD's with a focus on diabetic education that will lead to a change in hospital protocol of patient diabetic education and in-house insulin dispensing guidelines. This research will be published by mid-2019. Over the past two years, I have spent my Spring Breaks in third world countries, volunteering my acquired skills to help treat the severely medically underserved communities.

Given this rundown of my college history, I would like advice on how to go about applying for medical school. Am I a good candidate for MD schools, or should I just apply to DO?

Any advice is greatly appreciated.

mark-ER

I think as you explain your situation in a thoughtful, well-organized personal statement and make a solid, but not overaggresive efforts to contact admissions offices so your application is seen, you have a solid shot at MD schools. Your biggest worry should be that you get 'screened out' due to low GPA, either thru a computer algorithm or by someone pretty low on the totem pole of Admin offices. You have a solid MCAT score, a lot of good experiences related to medicine, and significant life challenges that helped you to grow -- these are many attributes we all want among our future docs. That being said, you should apply to quite a few schools (15-25 allopathic, at least 5 more DO schools), to make sure you get in somewhere. Good luck.


If I get my Associates in high school via Dual enrollment. Can I still get into med school?

My high school offers dual enrollment courses that I could take at the community college and earn my AA. Once I do that I can “transfer” to a university and start off as a junior. If I complete some pre-req courses at the community college instead of university will med schools accept the credits? Or see me as a weak candidate.

I’m also concerned about the MCAT and all my clinical experience in undergrad. Because I’ll be started as a junior... I’ll have to take the MCAT that year and will only have basically 1 year to complete clinical experience.

Anything will help! Thank you.

Pathdocmd

Community college credit is handled differently from school to school. It should not hurt you. Yes, not having as much time as a person that did a more traditional college route gives you less time to get clinical experience. For most medical schools, clinical experience is quality over quantity. Also, there is no rush to get into medical school. You will be doing medicine for the rest of your life. You might want to take some time and do something else after graduation.


What can I do after an interview to tell a school I would really like to go there?

I interviewed at a medical school on October 1st. They are my number one pick, and I'd love to go there because I feel like I fit well with their program. Additionally, attending this school wouldn't require me to move, I'd have family and friends close by, and it wouldn't force me into a long distance relationship.

I haven't heard anything back from them yet, and I believe they don't accept most students until spring. I really just hate sitting around waiting if there's something I could do to help impact the decision that will be fairly life-changing for me.

What, if anything, can I do after an interview to tell a school I would really like to go there/ improve my odds of getting in?

Pathdocmd

Sit tight. Calling or e-mailing them frequently will not help. Look at their policy for updates and follow it.


retake 512?

I am really contemplating a retake... I scored a 512 (127/128/127/130) when I took it the first time. I have a lower GPA (around 3.5) but I went to Stanford, had a strong upward trajectory, and was a varsity athlete there. I am currently doing full time cancer research and hope to have a publication soon. I have good ECs and recs. I know my score is pretty decent, but I really want to go to Baylor (TX resident) or another top 20 school. Any help would be greatly appreciated!! :)

Pathdocmd

With your MCAT, GPA, undergrad school with the time commitment of a varsity athlete, and state residency I would say no if you want to stay in TX (all schools). If you want to get into a "Top 20" (whatever that means) then you might want to try. If it was me, then I would not. You have a lot of good choices in TX.


Undergraduate degree

I'm currently in school pursuing my bachelors in nutrition. Will medical schools consider a nutrition degree? I am interested in med-school but wondering if my undergraduate would be accepted or not.

mark-ER

Sure, any degree from an accredited, 4 year degree-granting university should be OK. As long as you take the required pre-requisites, do well (high GPA, especially science GPA) and do well on the MCAT. You can certainly do a lot with a degree with nutrition (GI, endocrinology comes to mind). Good luck.


MDPHD application - Am I stronger after a gap year?

Hello All! I am reaching out to SDN because I am struggling to decide whether I should take a gap year before applying MD/PhD. I am unsure whether I have a strong enough application to apply following the completion of my Junior year, and after having read forum after forum here, I have decided to reach out to the community! I thank anyone in advance for taking the time to skim through this info - I have spent months researching this topic and have no-one to turn to for concrete advice, and thought it could help to reach out to the experts and see what feedback I get!

A little background on me:

   I am a white male who attends a small private liberal arts college (NOT University, meaning we have no graduate school), which is decently well-known (top 70 school - nothing too crazy).  I am a pre-med student completing a double major in Chemistry and Spanish Language & Literature, and double minoring in Math and Biochemistry. By the end of my junior year, my stats should look as follows:

3.85 cGPA 3.80 BCPM 4.00 Non-BCPM

  • Haven’t taken MCAT yet: planning for the spring of 2019, as I am currently studying abroad in Spain.

Research Experience: By the time of graduation, I will have 2 co-authored publications from an NIH funded laboratory, and will have 2.5 years of research experience in that lab.

Clinical Experience: - 1000+ hours working as a Certified Nurses Assistant, particularly working in Dementia Care Unit (1.5 years as of now) - Hospice Volunteer - Dental Intern (150 hours) assisted in filling cavities and procedures.

EC’s: too many to really list, but I will try - Student government (100+ Hours) - Manage a volunteer program in an elementary school and coordinate and manage volunteer participation. (500+ hours) - Started a newspaper and started an article writing mentorship program in an underprivileged district - Chemistry Teaching Assistant: Assist in lecture - Tutor for Chemistry (College and High School) - Completed an internship in Spain whilst studying abroad (clinical setting) - Deans award for Outstanding performance in Chemistry - Deans Award for Spanish - Alpha Lambda Delta Member - Health Professions Society and American Chemical Society

Essentially, I am wondering whether I should aim to take the MCAT in the spring and submit my application in June of 2019 (finishing my Junior Year), or if I should shoot for option 2:

Option 2:

Apply MD/PhD at the end of my senior year of college, and take a gap year afterwards. I will have the opportunity to apply to a related lab following graduation, where all of the graduates of my lab go to do research until they begin graduate school, at a top 30 school with a well established professor. By the end of graduation, my stats are likely to look something as follows:

cGPA: 3.90 BCPM: 3.85 Non-BCPM: 4.00

Further, I will have more research experience from the lab, however it will not be continuous experience. I go to a college that does not have a graduate school, so I will have 2.5 years of research at my college upon graduation with 2 publications, and will then have 1 year of research in the top 30 university lab.

I know this is difficult without an MCAT score to really gauge my competitively, but I get the impression that my application would not be competitive if I were to apply following the completion of my Junior year. Any thoughts are genuinely appreciated, because I don’t really know who to turn to for advice. Thank you very much - I really appreciate any genuine thoughts on the subject!

Mr.Smile12

I can't say that either option is a bad one, but in my opinion, I would personally want to understand why I would want to sacrifice 8 or more years of my life to get the MD/PhD (or DO/PhD). Research is an option without having to do both, so I would want to get a really good idea of why both would be essential. The usual path for people pursuing this route is to develop academic physician leaders, but you lose a lot in terms of potential lost income. In addition, seeking a career path outside of academia with both degrees -- while desirable -- is not often a common topic that is discussed (though that can change). If you are applying MD/PhD, you need to understand that while you may be applying to a few schools, all of the program directors know each other well. And once you join the MD/PhD ranks, you'll know so many of your peers going to other MD/PhD programs, so you have to be on top with your interview skills and selective on which institutions/departments you want to do your research work. Sure getting the 97th percentile across the board on your MCAT is a daunting task, but you need to be sure this option is the one you want, knowing you can probably do the same with just an MD/DO at a medical school that requires a research thesis or a Ph.D. without all the debt.


Should I retake MCAT? Score, 514 (BB 130, CP 131, CARS 127, PS 126)

Many people have told me not to retake, but I feel as if my score is skewed in PS. However, I did average a 514 on my three AAMC exams, and a 515 on my Next Step exams. I have a 4.0, from a small school, and I have three years of both clinical and research experience. Planning to apply next cycle, should I retake?

mark-ER

Short answer = no. Not worth the time/hassle. Yes your score is a bit skewed, but not overwhelmingly so and re-taking an exam always means a risk of a lower score. In short, I think you have a solid stats and a well-rounded application, and with a well-thought out application that is submitted early in next cycle, you should be able to get in. Good luck.


Should I double major in Biomedical Sciences and Psychology for Pre-med?

I am a junior in college with a major in Biomedical Sciences and a minor in Psychology. Should I up my minor to a major in psychology? I can handle it credits wise and was wondering how much of an impact it would make on my application to medical schools.

mark-ER

Short answer = minimal (likely none) impact. If you want to double major, then do it, but don't do it solely for a greater chance of getting accepted. Both bio and psych undergrad majors are pretty 'vanilla', and frankly your grades (esp. in science) and MCAT score and your passion/evidence of commitement to medicine is what carries weight, not a specific major.


Sufficient application for this cycle or improve application for next cycle?

I was complete to every medical school September 19. I have a 505 MCAT 127,127,126,125 with a 3.8 cGPA and a 3.85 sGPA. I have had one interview at CUSOM and was not offered on the seat. I double majored in Biology and Spanish and double minored in Chemistry and General Science at Bowling Green State University in Ohio. I studied abroad in Costa Rica for 9 months while also completing some volunteer work teaching English to local elementary school children for 30 hours. I achieved 160 hours of clinical experience by volunteering in a local clinic and hospital in La Ceiba, Honduras. I do not have any research nor do I have any further volunteer/clinical experience although I did shadow a plastic surgeon for 10 hours.

I completely believed my statistics were good enough to receive many interviews from DO schools and was being optimistic about receiving applications from MD schools. My question is do you think my application was strong or weak. I believe the thing that hurt me the most was submitting in September. I am already planning on improving my application for next cycle as I just received a full-time job as an inpatient pharmacy technician at UC medical center level 1 trauma. I also will try to shadow a DO doctor for 40 hours and volunteer to teach english to local latinos.

My overall question, because I have incorporated a lot of information in this post, is am I overreacting already from not receiving an acceptance? Is my application strong enough to receive an acceptance this cycle? and most importantly, if I do not get accepted, are the changes I plan to make strong enough to get me an acceptance if I apply June 1???????? If not what would you recommend I do to strengthen my application for the following cycle.

Thanks

mark-ER

I think submitting a bit late did hurt you, and overall you have a reasonably well-rounded application and you seem to show very strong commitment to a career in medicine. However (probably no surprise), my advice would be to re-focus and put all of your energy to studying and re-taking MCAT. Even a few extra point (say getting to a 510) would make a huge difference in your application. As it is, you will get screened-out by a lot of adcoms, with either a staff lower in the decision making process or no humans (algorithms) screening your application out.

So in short, make sure you get a good year of experience, study hard & re-take the MCAT, put together a solid personal statements and letters and submit on time to a range of schools (both DO and MD) and you should have a reasonable shot. Good luck.


whether I should continue in the pre-med track in college

I am a freshman in college right now and I am currently failing gen chem 1 with a 60%. In high school I took AP chem and passed the class but didn't pass the AP exam, I also took IB chem and did not pass that exam either. Should I drop this class and take it again next semester? Should I stay in the class and try to study harder? Or am I just not cut out for chemistry?

MusicDOc124

If your aim is for med school, it would be better to drop a class, even if it means a W, than to continue on and risk an F, a D, or even a C or low B.

With that said, retaking it later should only happen when you realize what's going wrong. You're in college taking gen chem and failing. You previously did not pass the AP exam. You MUST absolutely figure out what is inhibiting passing first. Are you not putting in the time? Are you not focused while studying and only studying passively? Are you studying the incorrect info? Do you need a tutor but haven't gotten one? Do you utilize office hours? These are all questions you need to ask yourself and then some. Retaking it would be pointless if you don't correct the error because you'll be right back in the same boat, but with 2 Ws for the same class, which is not good.

Depending on how much time you have before you can still drop for a W (assuming its past the point to drop without it on your transcript), consider getting a tutor who can help with the material and possibly study habits for that class and see if your grade starts to turn around while in the class currently.


Research for during undergrad

I am going to be applying for medical schools this coming Spring. My application is pretty solid throughout with one exception: I have no research. Research has never been something that I was passionate about, so I never pursued and opportunities. I was wondering how critical research is for medical schools, and whether it will make or break my application.

Mr.Smile12

It will depend on the schools where you are sending applications. You may want to talk with admissions staff at the schools on how important research is for applicants and whether they have experience with similar candidates with no research when it comes to getting them into residencies they desire. Some schools are built for students to do research as part of their education, but research doesn't mean just bench research.


3.8 cGPA from state school, 517 MCAT, low clinical experience

I have a 3.8 GPA and 517 MCAT, but not much clinical experience. What schools could be realistic with this?

mark-ER

Are you applying this cycle or next cycle? What's the tier of your undergrad/major combination (some schools notoriously grade inflate, and some majors are harder than others)... that being said, for a middle-of-the road state school U, and a typical biology major those are reasonable stats that should get you into at least a middle-tier allopathic (MD) school -- so for most places, that means your state school, or mid-lower tier private school, perhaps with the right letters/personal statement/motivation even a top tier school (one caveat being california, which is notoriously more difficult).

If you're applying for this cycle (to start in 2019), you are behind the ball -- apply NOW, as in TODAY, since most medical schools have rolling admissions and apply broadly to 25-35 schools, because you are so late. If you are applying for the next cycle (2020), then you can improve your profile with clinical volunteering/research and could be more selective, applying to 3-5 top tier (top 20 USNWR) schools, 5-10 middle tier schools and 5 or so lower tier and in-state schools, for a total of about 15-18 schools. Good luck.


Choosing Universities and Medical Schools

Hi! I'm a high school junior starting to examine and research specific universities and colleges more. I'm not 100% sold on becoming a doctor, but it is one of my career ideas. That said, I would love to study abroad and then return to get my medical degree, or even get both of my degrees abroad as I would love to live in Germany or Switzerland. But, how would I go about this? For instance, I'd like to attend Oxford for biochemistry, but is that a sufficient "pre-med" qualification for American universities, or other European universities? Thanks for all your help!

Mr.Smile12

For the US system, first things first: get your undergraduate education but keep your eyes open about where you can apply in the US for medical school. A word of warning that university education outside the US and Canada is different than within, so many of the admissions committees may require you to get your foreign degree evaluated to see if you have fulfilled requirements and desired courses. Now if you're that good that you can get a top-merit scholarship at a US university that could include foreign study abroad at Oxford or Germany, that's another option that might work better for you if you intend to apply to US medical schools.


i have the option to take biostats and also to take ellementary statistics. what is the difference between them? which one is easier to learn and which should be better for mcat and medical school

Mr.Smile12

To answer this question, check out the syllabi of both courses and compare to the desired competencies tested for on the MCAT and for entry to medical school. I suspect that there is no difference except in the types of examples you will be exposed to in the classes.


Should I put on my resume that I am currently donating to UNICEF USA & UNICEF WORLD for 3 years?

I have been constantly donating to the Unicef USA and Unicef World Wide. Should I put that on my resume or application to apply for medical school? Or Should I only put the volunteer and community service hours on my application?

Thank you

mark-ER

No, personal monetary donations to charities are not relevant and simply reflect your financial means to do so. But your hands-on activities, volunteering your time and effort are meaningful.


Is 11-12 hrs of class time + 10 hours of volunteer, community service, research, ect. (Per week) enough for Post Bac?

Hi there!

I am wondering my my semester load is competitive enough?

11 credits (16 hours of class time due to labs) +10 hours total of outside class volunteer, community service and volunteer, ect.

Per week.

Any insight would be so appreciated. Thanks!!!

mark-ER

Good start. Whether you do more and what you do depends on other aspects of your applicant profile. Have you already attempted MCAT and did you do well? If not, perhaps focusing extra outside energy on the MCAT may be your best best. If you have a good MCAT score and/or are generally confident/capable of doing well on standardized exams, consider working for $, perhaps getting a paramedic license, something medically relevant. Also, it would be helpful to do something medically relevant in your volunteer activities, perhaps serving as a pharmacy tech/organizer in a free clinic, medical translator if you speak another language, or psych/suicide helpline if you enjoy/intend to go into counseling. Otherwise, do what you enjoy -- if you like research, do research; if you like community outreach, do that.


Will an LOR from a med school clinical professor improve admission chances?

Will having a letter of recommendation from a clinical professor at a medical school help me get an interview or improve my admissions chances?

Long story short I did some shadowing under a physician alongside M3s doing their family practice rotation. The physician has been teaching clinical rotations for a top university for years and let me know that he wrote me a strong letter of recommendation for grad school. I've done about 60 hours of shadowing with him and will hopefully at least double that by the time I apply to med school. While I obviously know the letter wont hurt me, I'm curious if (and how much) it will help me. I've had a hard time finding information in this area.

Other relevant information about me: My GPA sucks (3.3), I have ADHD and I've worked basically full time throughout undergrad while being in school full time. I'm in grad school now, working slightly less, and doing a masters of Bio in pre-med. I haven't taken my MCAT but I think it'll be about ~510. I took intentionally VERY difficult classes in my undergrad that were specifically designed to emulate med school courses, and did decently well in them. (Think pathophys, cadaver anatomy, and medical physiology. After pathophys I was able to answer about 40% of the USMLE1 questions without having taken the rest of the M1 and M2 curriculum.) I don't think getting through med school will be a problem, but I do think getting in will be due to my GPA. BUT, I have a REALLY good LOR from the clinical professor I shadowed with for two weeks. He knows I'm capable, based on seeing me in a clinical setting alongside M3s and being able to keep up with (and occasionally teach) them. I am currently in grad school and I will be applying for med school in summer 2019 or 2020. If its 2020, I'll likely have a second physician letter very similar to the first, but with a different university affiliation.


1) Will an LOR like that be enough to get me an interview at the university he's affiliated with? Other less-prestigious universities?

2) Would asking him to contact the admission committee recommending that they grant me an interview, based on his belief that I would be a good fit for the university, be appropriate? (He's volunteered that has a very favorable opinion of me and my ability to do well in this field.) He is a professor at that school...

3) Will the letter help in general with attesting to my competence to do well in med school, despite my sub par GPA?

4) Any general advice for my situation?

TLDR: Will having a great letter of recommendation from a clinical professor at a medical school get an interview, even though my grades suck?

mark-ER

Short answer -- nope, letter of recommendation from a physician will NOT help to compensate for poor undergraduate grades. 3.3 GPA will get your 'filtered out' unless your MCAT is amazing (518 or above), and possibly with the exception of your in-state school, some private allopathic schools on the lower end of the prestige spectrum and DO schools. You will have to do a lot of legwork, contacting admissions to make sure you are not automatically screened out. Regardless my advice is to take a couple more science undergrad courses at school where you are currently doing your grad school coursework, to improve your GPA, study hard for MCAT and rock it.

Last (& this is more for others, than just yourself): little humility please. Putting thoughts in writing/speaking out of turn like " don't think getting through med school will be a problem" or "in a clinical setting alongside M3s and being able to keep up with (and occasionally teach) them" is not great. I have been both thru grad school and med school, and medicine is a lot more hierarchical, almost army-like atmosphere with a more-or-less clearly defined pecking order. You have to realize early you don't know anything, ask questions, maintain a very humble attitude. Showing off, or worse yet denigrating someone's intelligence, even to someone with "just" a high school diploma like a lab tech or nursing aid, can get you a bad eval and tank your chances at a top residency. Tough words, but better come now in an anonymous forum, than later when it actually matters.


What options do I have for pre-med?

I am a US citizen currently studying Adult Nursing (BSc) in the UK. During my nursing studies, I've found a keen interest in medicine, and I realize that I may or may not be too late to get into the medical field. I plan to finish my degree and move back to the US and hopefully get into a medical school there. I have a few options and a few questions.

Options: 1) Either work in the UK for 1 year as a nurse (to note down experience for med school application) and move back to the US, or apply straight to any pre-med programs in the US as soon as I graduate.

and

2) Attend a formal post-bacc pre-med program, take the MCAT, and apply to a medical school

or

3) Attend a regular bachelors program and make a DIY post-bacc and possibly drop out of the program after fulfilling all medicine prereq's

Questions: 1) When I apply for med schools, would they accept my UK BSc degree along with the US prereq courses? Or do I need to complete another bachelors in the US for me to qualify for admission into medicine?

2) For option 1, would having that 1 year of work experience increase my chances of admission into a med school? Or would it be a waste of time? Should I just move back to the US as soon as I graduate to do my med prereq's?

3) For option 3, is it necessary to finish the bachelors, or would I be able to drop out of the program as soon as I finish all prereq courses?

4) What other options do I have?

mark-ER

Great questions, good way to sum things up... Perhaps a bigger question is: does your UK degree in nursing translate (or can translate) into a nursing certificate in the USA? If so, it's (almost) a no-brainer: come back to the US, get experience in the US system and if you are not 'picky' geographically you can also earn a very solid salary. The most flexible approach would be to do a DYI post-bacc at your own pace, while working part time. But if you think you need the structure and the support, both from admissions counseling and MCAT prep standpoint, you could go the formal post-bacc route -- more expensive, less flexible, but a lot more structured. You know yourself. Last option you should not completely take off the table: clearly you have connections and work permit for Europe. Have you considered a US-style (4 year) European medical school like Trinity college (Ireland) or Jagiellonian University (Poland)? Would be a faster way to get thru medical school, especially if you intend to go into a primary care field, not uber-competitive subspecialty.


Could I matriculate to a US medical school from Canadian undergrad?

I am living abroad and I plan to attend a premedical program at a Canadian university, due to its vastly cheaper cost. As a US citizen, would I struggle to return to the States for medical school?

Similarly, if I, as a US citizen, attends a Canadian medical school, would it be more difficult for me to match into a residency program in the US as compared to a US medical school graduate? In essence, would my citizenship aid me in coming back home or would it not make a difference?

Thank you for your help.

mark-ER

Good questions -- first the short answer. As a US citizen, you will be absolutely fine, with either option you describe.

Now a bit longer: You should be able to get into a good medical school with a Canadian undergrad, as long as you take the requisite premed courses and do well on the MCAT. In fact some higher ranked Canadian schools like UBC, or McGill hold enough prestige to be a better option than most US schools (IVY leage, Stanford and a few top-notch big state schools like Michigan and UNC notwithstanding). Second option of doing both undergrad and medical school, then transfer to residency in the states is a bit more difficult, but only a little bit. You will technically be a US-citizen FMG, but (maybe with the exception of Australia, UK and Ireland), you will be heads and shoulders above the rest of FMGs and with appropriate grades, step1 scores and extracurriculars, you will NOT be held back from any residency, even competitive subspecialties. The best way to check is look at the match lists for canadian schools. Either way, I commend you on your approach -- costs ARE very important and if you reside in Canada and want to live there at least for the duration of your training, why not take advantage of the lower cost of training?


Post Bacc Choices to fit my situation

My GPA coming out of undergrad is a flat 3.0 as a biochemistry major with a 508 mCAT. I had a large upward trend of dipping below 2.0 my freshman year to bringing it up from there. I have over 200+ hours of volunteering and about 50-60 hours being clinical. I got accepted to post baccs in SFSU, Elms College, Chatham University and CSUEB, but I am not sure which one would best suit my needs. I want this post-bacc to simply improve my GPA to show to medical schools as continuation in my upward trend and be able to have some clinical experiences so that I can get a good letter of recommendation/committee letter of recommendation.

Mr.Smile12

I hope you asked the program directors at each institution what you want. I'm not sure about clinical experiences each program offers, so hopefully you picked to apply to those programs specifically for that reason. You should ask what the process is to get very strong letters of recommendation or program letters for your intended application.


Kaiser med school?

Hey y'all, I'm applying to med school next cycle (2020) and I'm interested in Kaiser permanente. I know they were planning to have their first class matriculate in 2019. Anyone know anything about this? Their website is silent and AAMC doesn't have any data on them. Just curious! Thanks!

mark-ER

According to modernhealthcare.com, they do not seem to be LCME accredited (yet), but Kaiser is a very well-respected medical institution, should not be an issue. They have a board of directors, and I am sure the facilities are being prepared. It should be a good choice for the very crowded California medical school 'market', to start with at least, they should have a similar profile as say Loma Linda, or as a better comp Geisinger which is openining a new medical school in Scranton, PA. You can always 'tweet' at them to ask or just keep checking for updates, their twitter seems active.


I need help forming a school list; I am open to both MD and DO

Here are my current stats, and I need help forming a school list that I will be competitive in. I am currently a Tennessee resident.

GPA: 3.85 Mcat hasn't been taken yet, but hoping for at least 508

Extracurriculars: -200 hours ram clinic, 10 different locations -350 hours at VA working with physical therapy -3500 hours as paid medical transporter at hospital for 3.5 years (does this count as clinical experience?) -250 hours at Second Harvest Food Distribution -100 Hours as a Children's Ministry Leader at church -50 hours at a homeless shelter -100 hrs tutoring underprivileged youth -Underprivileged youth Mentor Program for two years -Tutoring For College underprivileged students 2 years (paid) -250 hours Shadowing Experience 8 specialties Involved in three clubs and working on research I have had memorable experiences and good talking points about all activities above.

mark-ER

Thanks for a relatively thorough run down. Couple of things that may influence the suggested list: are you URM (or conversely overrepresented)? What's the pedigree of your undergrad (ivy/top 10, top 50 USNWR college, major known liberal arts college, state schools, etc)? Also, I am not sure what you mean by "working on" research -- do you have any independent project(s) that you can easily talk about to an expert, and potentially publications (or en-route to publication)? And of course actual MCAT score. With a balanced 515 or above, you will get in somewhere and you should be more than OK with applying to a range of allopathic schools only, including in-state public, regional private and a few 'reaches'. Unless your MCAT score is absolutely stellar (520 and above) and you have research publications and some other major accomplishments (Rhodes scholar, amazing athlete, or overcame major life issues), I'd probably stay out of top-10 schools (Harvards, Hopkins, Stanfords) U Tennesee is hard to beat in terms of cost/value equation, but the triad of Tennessee, Vandy and Pittsburgh is realistically what you should be aiming for. Create a list of 15-20 schools, put those on the top. Good luck with your applications (presumably next cycle).


pre med Post Bac program or another bachelors degree?

Hi all,

I have been accepted into a pre-med program at Fordham University but now I am contemplating attending. I’m not sure if I should do the pre med program or get another bachelors degree.

My previous BFA is in fashion design (8 years ago) and I graduated with a 3.2. I am wondering if I would be better off doing another 4 year program (maybe majoring in biology) since my gpa was so low. A pre med Post Bac program will only raise my gpa so much.

Any advice on what would be the best path to increase my chances of getting into med school would be great. Thank you!!!

mark-ER

Good questions -- Post-bac adds on to your existing grades, so it would average out with your 3.2. 3.2 is low, but fixable. If you get straight As or close to it, you can bring up your average to 3.5 and with a good MCAT score (above 510) you ought to be able to get in. So a 2 year post-bacc is probably the way to go for you, not to mention the support and guidance you receive, which is not always a given in a traditional 4-year biology or even pre-med pathway at many colleges. So if you are truly dedicated to becoming a physician (though please keep an open mind to other allied health professions -- genetic counselor, PA, podiatrist, nutritionist), the abbreviated 2-year pathway thru a formal post-bacc may be the best route for a non-traditional student coming from a non-science background, such as yourself. Good luck.


Should I withdraw from my SMP? Horrible second exam

Can’t really say much. hurricane Florence really held me back. I had to travel across the country and it led me to do terrible on my second exam. It was a straight F. I was already borderline C+ to begin with after the first exam but this F really set me back. I ask you guys should I keep on co tinhig and hope for a B which at this point looks impossible with two exams to go or should I drop out and retry elsewhere ? I get that SMP are dress rehearsal for medical schools and they predict success but I sincerely get the material and am interested. I just got wrecked by this second exam and nd now it seems like my entire dream is in jeopardy. (U cum gpa- 3.03, sc gpa - 2.89 MCAT -507

I applied for 10 different DO schools. Would getting a C in one of the classes in SMP end me ? Should I withdraw before I do more damage ?

Mr.Smile12

I am sorry to hear things didn't go so well with you. I think withdrawing is going to do a lot of damage to any timeline that you may want to get you into medical school in the near future while staying with the SMP and getting support from the faculty about your situation would give you some chance. You have to shape up or else there really isn't much more hope to give you outside of getting into medical school at a Caribbean program.


Should I retake my class or withdraw? Pt. 2

Hey, I just wanted to let mark-ER that the class is not combined or averaged. Is it bad to think a C in the lab is okay? Cause if I end up with the A in the lecture and a C in the lab, that's still a 3.5 gpa overall. I would rather just go into physics 2 with a pretty solid gpa, then have to withdraw and just take the physics 1 lab in the spring. My school doesn't allow me to go into physics 2 lecture without the physics 1 lab (makes sense). So I'll talk to my lab professor in a few weeks about my grades in his lab. Anyways, Thank you for the advice.

mark-ER

Yes, your thinking is right. As long as you are learning the material and you are getting what you want from the class, just stick with it. Also, if you are seeing bad grades in the lab component, try to address it NOW not in a couple of weeks. The rest of my advice still applies -- read ahead, work extra hard... it might mean more time dedicated to this class than you might want, but that's how the cookie crumbles sometimes.


Should I retake my class or withdraw?

Hey guys, I am a little more than halfway through the semester and am questioning if I should withdraw from my physics I lab. I am doing very well in my lecture, I got a 95 on my last exam, but the lab has been really difficult. The professor is pretty bad. He is a few weeks ahead of lecture, so its kind of difficult to understand what the material is and how to apply it. His grade for the entire lab is 70% lab reports (I have an 85 average on.) and 30% quizzes (like a 63 average on.). I can probably get at least a C in the lab, but it seems to be kind of a bummer especially if I get an A in the lecture. Does it look weird to the adcoms if I get a C in lab but an A in lecture? should I suck it up and take C or withdraw from the lab? Thank you

mark-ER

Does lab & lecture components of the course get averaged together? Basically what do you see on your transcript? Double check that, and go to the prof who teaches the lab and/or TA office hours and gently/carefully try to understand what they want you to get out of the lab component of the course. No grade-grubbing just a calm, rational conversation. If that fails, you should adjust and focus more energy on the class perhaps by reading ahead, if the lab component is ahead of the lecture component. Others may be doing better on the quizzes for the lab component, because they may have had the course material covered perviously (i.e. AP Physics in high school). You will see this plenty both in college & in real life and it is not always fair. Bottom line is: you are still earning solid if not stellar grades. If you are getting something out of the class (i.e. learning the material) and are just frustrated with the lab component/arbitrary nature of the quizzes, stick thru it, adjust and control what you can. Yes, you can withdraw and it's better to have a W than a C. But whether a W is better than a B is questionable and then you have to repeat the course again.


3.53 GPA 3.48 Science GPA 508 MCAT I haev no interviews yet and have not heard back from 18 MD schools

18 MD Schools have not replied, does that mean I am still being considered or do some not tell you you are rejected? DO you think I have a chance for an MD program with my numbers? I applied mainly in NY, Pa & NJ. Is there anything I can do now to help my current application? thank you

mark-ER

Hard to say. When did you apply and did you get/quickly fill out secondary applications (?)... I would not worry yet if you applied in September, but perhaps be somewhat concerned if you did so promptly on the first possible day (or week).

Your stats and MCAT are (slightly) below average, I also hope you applied to the right tier of medical schools (mid-lower prestige, local to your residency status) and perhaps a few DO schools as well. Perhaps other red flags (academic, something in the letters of recommendation or not enough exposure to healthcare thru volunteering or research)? Either way, do not panic, but act to control what you can control.

Briefly, gently and VERY respectfully inquire about your status at a handful (max 5) of medical schools where you most hope to attend. Depending on the feedback (almost regardless, unless you happen to get 3+ interviews pending), I would expand your application list by another 10-12 schools. Time is of the essence, most medical schools have rolling admissions. Last but not least, if one of your recommenders has strong ties to the medical school where you are applying, it may now be time to pull on that lever. Good luck.

Other things you can do to help (since you asked): did you have a major accomplishment in the meanwhile -- higher MCAT score on a retake, a 1st author publication, a Rhodes/Marshall scholarship or something of that nature? Then definitely, as you reach out to adcoms, update them/your profile.


Trying to make decisions about whether it would be worth it to still apply to medical school in the cycle or wait

I am currently a junior majoring in Neuroscience and History with a minor in Chemistry and possible Religious Studies Minor.

My GPA through my university is a 3.56

Going by AMCAS GPA policies: My BCPM GPA is a 3.24 My All Other GPA is a 3.7 My Running Overall GPA is 3.43

I have yet to take the MCAT, but I am still planning to do that in early May 2019. I am currently trying to decide if I have a chance at getting into medical school in the next cycle or if I should wait and do a post-bacc year-long program and apply in 2020. I am attempting to get into a clinical research lab right now and waiting to hear back on a clinical volunteer program I've applied to where I sit and comfort people who are close to dying and do not want to die alone. I have been extremely active in my service fraternity and have an excess of volunteer hours through them. I am working on gaining shadowing hours. I currently attend an OOS school for undergrad and would like to stay up north for medical school.

I would like to know if it is worth it for me to even consider applying next cycle for medical school? Like do people get into medical school with GPA's like mine? Also any alternative suggestions for how to make myself a better applicant especially if you believe I should wait a year or two?

mark-ER

Ultimately it depends on your MCAT score (aim for at least 515) and what your ultimate career goal happens to be.

If your ultimate career goal is to get into a competitive subspecialty, you probably ought to spend another year or more in a post-bacc to improve your science and overall GPA to 3.5-3.7, respectively. Postbac would also have the benefit of dedicated support and prep time for the MCAT.

Your alternative would be to apply to A LOT (and I mean 25-35) lower tier private allopathic schools and all your in-state schools as well as all DO schools. If you continue your exposure to volunteering/healthcare exposure opportunities, continue to earn excellent grades (all As from now on, especially in science) and get an excellent MCAT score and apply early you ought to get in somewhere. So if your ultimate goal is a less competitive subspecialty, just be on top of things and apply early next year; don't forget opportunity cost -- you 'save' a year in training, less expenses (tuition/living expenses) and earn an extra year of physician salary.


cGPA 3.2, sGPA 3.0, MCAT 509 - Plan of action advise please

I graduated in May of 2018 with cGPA 3.2, sGPA 3.0 from a top 5 liberal arts college with a non-science major. I applied with GRE scores [95% in verbal, Quantitative 55%, Writing 95%] in June for post bacc. I was rejected from Loyola [both programs], and Tufts post bacc.

I took my MCAT and got 509 [127,127,127,128]. Please help me with suggestions on what to do next. I am currently shadowing doctors, volunteering in hospitals. I have applied for a data coordinator job with a professor in a medical university - this includes clinical research and some shadowing].

My current plan is:

1) Apply for post bacc to start Fall of 2019. If so what programs would you recommend? 2) Should I apply for DO programs next June as I am doing post bacc [provided I get in to one]? 3) Should I wait until I finish my post bacc and them apply for DO and a few MD programs?

Or should I apply for a few DO programs now [is it too late]?

Please advise. Thanks in advance.

Mr.Smile12

Hopefully your institution has a health professions advisor or office. It's tough to figure out what your options are just based on the information you provide. I would say that you should look into DO programs in general but it's not clear what you know about osteopathic medicine or why that route would best suit why you want to go into medicine in the first place.


Low GPA/High MCAT Applicant; MD or focus on DO

Hey all, thanks in advance for taking the time to look at my question!

I am a current Master's student with a low undergrad GPA (3.26 cGPA, 3.24sGPA) and a high MCAT (520; 130/129/131/130). I am applying next cycle and want to get an idea of where I should be applying and if I should spend the time applying MD or just put maximum effort into DO schools.

I went to a SUNY undergrad school and graduated with a B.S. in Biochem with the above GPAs and am going to another SUNY school to do my MS in biochem as well. My Master's is research focused with a thesis requirement and I have a 3.9 in the classes I have taken there (25 credits worth, all biochem/analytical chem/ cell bio/ stuff like that).

I have ~3,000 hours direct patient care experience as a volunteer critical care technician on an ambulance (A bit lower on the totem pole than a paramedic but can do most of the same treatments and skills). Also significant leadership and teaching experience here, was elected to office and in charge of all training.

I have also been doing research in cancer biology for a little over 3 years now and that has resulted in 2 second author literature reviews and one second author manuscript in a mid-level journal. Plus I'll have my thesis to point to by the time I apply if that is taken into account?

My shadowing experience is scarce, only ~35 hours with a plastic surgeon whom I met serendipitously and he wrote me a great LOR. 

For non-clinical volunteering I began a social-support group on campus for students battling drug addiction (not in recovery myself but sibling died of an overdose so it is something I'm passionate about). That's it for my non-clinical volunteering though.

I guess I just want to know if my undergrad GPA will cause me to get tossed at MD programs and if I'm better off focusing my time/energy to DO programs . I was thinking maybe I can just apply to SUNY MD schools and have the rest of my apps be to DO programs I'm interested in. Any insight from experts would be greatly appreciated. Thank you all in advance!!

mark-ER

If your ultimate career goal is to get into a competitive subspecialty, I think you ought to spend another year or more in a post-bacc or in your case (with excellent MCAT score already in hand), a self-designed post-bacc, taking several science classes at a local college (above a community college, but does not have to be USNWR ranked). You don't have to be in school full time, maybe 3 classes in the fall, 2 in the spring should do it, but to get there given how late it is, you will have to spend 2018 and 2019 doing this, and apply 2020. In the background you can continue to work (and cover your expenses) in your current laboratory, maybe even get another manuscript published (1st author would help a lot, 2nd/3rd authors don't account for as much). This may be the best approach if your ultimate career goal is a competitive subspecialty practice.

Your alternative would be to apply to A LOT (and I mean 25-35) lower tier private allopathic schools and all your in-state schools as well as all DO schools. Given your background and MCAT score, if you asked this question in early August/late July I would have said pretty confidently with enough applications you could get in somewhere (likely DO, but perhaps at least 1 lower tier MD ??) But at this point with rolling admissions, it may be too late.

So my suggestion is to perhaps split the difference between the 2 plans, particularly if your ultimate goal is to be an internist, peds, psych or family doc (something less competitive when residency applications roll around). Enroll in 1 undergraduate science class (ASAP, like today), then a 2-3 undergraduate classes in the spring. With those, assuming A's, you should boost your GPA to 3.4 or so, which will help and probably be OK to get into a lower tier allopathic school next cycle (2019), given your application includes enough schools (20-25). Remember it is the undergraduate GPA that counts (& at this point might lead to a 'screen out' at a lot of allopathic schools) Just be on top of things and apply early next year.


Career advice: aspiring scientist interested cancer, cognition, and nutrition

My background:

I am currently a junior in college majoring in Neuroscience and Philosophy. I am heavily involved in basic science research (neurodegenerative diseases) and have been published in peer-reviewed journals/presented many abstracts. Currently, I intend to go to medical school for an MD/PhD and am studying for the MCAT (medical college admission test) in preparation for next year's application cycle.

My Question(s):

I am writing for advice on potential career options for someone who is interested in working at the intersection of neuroscience/cognition, nutrition, and cancer. In particular, my interests lie in how certain dietary changes/habits can enhance cognition and/or sensitize cancer to better, more efficient treatment modalities. Is there anyone who has/is currently working in this area? Is it possible to be productive in something that is so niche-specific?

As a follow up question, I have been working as an assistant in my current lab for 2 years (freshman/sophomore year) and have developed a strong relationship with my PI. She has written me countless letters for various scholarships/fellowships and will continue to do so as I progress in my career. That being said, my research interests have definitely changed over the course of my undergraduate career, and I am more interested in nutrition then I am Parkinson's/Alzheimer's. Would it be unethical to search for other labs this "deep" into college? I am currently working on a large project in the lab and am independent, so I would be leaving a lot behind.

Any advice and insight would be greatly appreciated!!

mark-ER

Short answer: it is productive to be very focused and "niche", but not so early in your career. Keep an open mind, do not restrict yourself to one line of thinking. In fact, vast majority (myself included) of research-oriented physicians benefit from multiple lines of investigation, often changing throughout their careers. What you are interested in undergraduate != interests in grad school != interests as a pos-doc/resident and may not necessarily = research you pursue as faculty.

So that also answers your second questions = no, stick with your current lab. Be productive, get something out of it (poster, publication and most of all learn how to do science; and large part of science is persistence). You can search for a lab that fits your niche during your grad-school phase interviews and lab rotations that are a part of MD/PhD.

Overall, interesting major combo & sounds like MD/PhD (esp. neuroscience/psych) may be the right track for you. Just remember it is a LOOOOONG road. Most people do not get established as an independent (though still junior) investigator until their late 30s/early 40s, whereas your MD-only colleagues will be earning a 'real doctor' salary with real doctor medical responsibilities for 8-10 years by then. Make sure you are OK with that; check out physician-scientist forum here on SDN.


Math and the MCATS

Hi there!

I will be attending a Post Bac pre med program at Fordham university next fall.

This is a huge career change for me as I graduated with a BFA in fashion design 8 years ago. The only math I had to take for my degree was algebra 1 and 2 but since it has been so long I have forgotten everything. To help I am receiving private tutoring in both prior to my program start at Fordham to prepare me better for Calculus 1. BUT I came across a list of all the math I must know for the MCATS, which involves geometry and trigonometry. I never learned either. I was thinking of maybe getting my BA in biology instead that way I can take all these math classes as cutting corners scares me. I guess my question is if I should continue with my plan to attend the pre med program or apply to a four year program? Or is the math I will be using for all my sciences classes and the MCAT simple enough to learn without having to take geometry and trigonometry?

Any guidance would be so appreciated. Thanks!!!!

mark-ER

Ok, difficult question -- it depends on where you are headed and what you want both out of your preparation and your ultimate career. Yes, many (some argue most) medical subspecialties do NOT require calculus and or more advanced maths in general, but some (say critical care, nephrology) may. If your goal is to be a general practitioner, you can be OK or more than OK with "just" calculus at a post-bacc, perhaps supplemented with a statistics class. Overall, I think your approach is right -- hire a tutor and take a "remedial" (I hate that word) pre-calculus class on your own to fill out your holes, and do your best in calc in post-bacc. If you are just worried about the entrance exam, relax; there are VERY, VERY few questions on the MCAT that require snap thinking/top of your head trig, geometry or even calc.


Nurse to D.O. with WGU MSN

I am a registered nurse who has been practicing for eight years. I've worked in SICU settings and medical/surgical settings. I am currently working as a nurse educator and finishing my MSN in nursing education from Western Governors University. I am curious with WGU being a competency based school and my odd background if there were any medical school (osteopathic or allopathic) that would be interested in me. I plan on starting to study for the MCAT after I am done with my MSN degree but I didn't want to waste my time if it's unlikely to get into a school.

Mr.Smile12

The best thing to do is connect with admissions staff at some of the medical schools you are interested in. The fact you have been pursuing nursing (graduate degree) would interest some medical schools just because you'd have an insight into clinic that is hardly covered in medical education. I'm sure that you may find some of the schools more accepting of your journey.


3.9 GPA from Cornell and a 513 MCAT?

I ended up doing a lot worse on my MCAT (129,126,130,128) than all my practice scores but have a 3.9 total and science GPA in addition to a lot of clinical, community service, and research hours. Is my MCAT okay for this cycle?

Experience: 1,200 hours of research 1,200 clinical work/exposure orgo tutor and a few other clubs/jobs Honor society for top 10% of Cornell students

mark-ER

You are still OK, but (as you suspect) it will now be a bit more of an uphill climb. You should NOT be screened out, except among the most competitive medical schools. As such, you ought to avoid the top 10, though you could throw your hat into the ring at one or two places in the top 25 (for instance for Cornell Medical school). In your shoes I would focus on solid in-state institutions where you maintain home residency (that may or may not be New York), geographically close to your undergrad/home (maybe UBuffalo, Pitt), schools where you have some personal attachment to and mentors who can advocate for you. You also ought to put in an application into a few lower tier schools, and perhaps up your number of applications to 25 or so.

You still really should be able to get in somewhere, but time is now of the essence -- remember most medical schools have rolling admissions and interviews have started.


3.96 cGPA, 4.00 sGPA, 501 MCAT, nontraditional applicant, Asian: Chances for MD school?

Hello, So I applied to 15 MD schools for the 2018-2019 cycle... here are my stats:

cGPA 3.96, 4.00 sGPA, 501 (128/123/127/123) MCAT (I know, it's horrible)

I am Asian American; Registered Nurse with over two years of experience working in post-surgical care and intermediate critical care; have shadowed two MDs; Participated and currently participating in an Organic Chemistry research lab, Private tutor for the biochemical sciences; volunteered for 5 years as a Mandarin-English translator; Participated in Medical/Nursing Mission Trips to Taiwan & Honduras.

I know my MCAT score is a very low score, which was lower than what I scored on my practice MCATs. Is there any possibility that I would stand a chance of acceptance at any MD schools?

mark-ER

There is always a chance, but due to your low MCAT score you will be "screened out" at a lot of places. At this point, best chance is with DO schools, though you can try a few lower tier allopathic schools as well.

However, my overall recommendation would be to retake the MCAT. If you scored well in prior MCAT practice tests and there is a reason why you think you performed poorly (got sick, really nervous), I would re-schedule MCAT and just retake it as soon as possible. Even a few point improvement (say to 510) would do wonders for your chances. Otherwise your application is quite good.


How to approach a doctor when asking for clinical observation opportunities?

Hello!

I recently reached out to two doctors (in different specialties) asking if I could contact them and ask questions about the medical field and their experiences. I would eventually like to ask them if they would be open to being shadowed or if they may refer me to someone. However, I would like to go about this in the correct manner and do not know how to approach the situation or how to ask. I have both of their contact phone numbers and email addresses. They both told me calling is the best way to get in touch. Should I ask over the phone or is this too informal? I have never before approached or had contact with doctors in this manner before and this could potentially be my first clinical observation if all works out.

My primary questions are these: When I call, what sorts of questions should I ask the doctors? Should I ask about clinical observation/shadowing over the phone (and in the first conversation)? Or should I simply introduce myself ask a few questions and then ask them if I can call them again in the future (ie build the relationship a little more before asking)?

mark-ER

Very good questions, I wish more students in your shoes would ask. First point is to find someone who would be amendable. Probably the best way to do so is under the auspices of a formal internship program (my hospital has one of these for undergraduates over the summer). Alternatively, perhaps you know someone in your family or a friend who would be amendable, or who can recommend a colleague, perhaps even in a specialty of particular interest to you. It is always best to have an 'in' rather than call or email a doc out of the blue. If you do not have any other connections, I recommend singing up to volunteer with the hospital's volunteer department... it does not have to be an onerous commitment, maybe every other week x 6 months. You could be delivering flowers, be a mobile library, greeter, etc. and can get contacts with docs and other allied medical professionals (podiatrists, etc) that way.

In terms of how to ask and what to ask -- informal phone call or conversation can be OK, as long as you have actually previously met the person. If not, email is probably best at least for the younger docs. The few questions you may need to know or ask ahead of time -- do I need a short white coat or is business casual attire ok? Should I furnish you with a HIPAA training certificate? (you can do a quick 30min training course online, google it). Mention that you are low-mainenance, will keep in the shadow with not a peep out (as you should), but then ask if this is a specific clinic day (say endocrinology, child neuro, etc) and if so if they recommend a brief topical review you could read ahead of time so you can get as much as possible out of your day.

Those are my recommendations; they are on a bit more ambitious/outgoing side, but still within line of not being too overeager (it's a fine line to walk, as you will discover in your medical training).


Grad work in sciences or something non-science?

I am trying to decide on what post-bacc education I should explore. I am considering two things, on different sides of the spectrum and I'm not sure which would look better on a med-school application. On the one hand, I am considering doing an online masters program in biology. On the other hand, I'm considering getting an associates degree in American Sign Language (ASL) Interpreting and becoming a nationally certified interpreter, and potentially going on to do some work in medical interpreting. The former I know is good because it would show that I can handle the upper level science courses, but I've also heard that you should have something on your application/resume that most everyone else wouldn't have, which is where I think the ASL work could be beneficial. I have taken ASL classes in the past, and have been told by a Deaf instructor that I am very good and should considering getting into interpreting. Note: In the meantime, I will hopefully be working as an EMT no matter what (I just got my certification and have been applying) so that will give me some hands on experience in the health field.

mark-ER

You are right, both options can be appealing. Remember, masters (no matter if it's online or not) will not help your undergraduate GPA, but may improve your knowledge base and help you to have the time to study for the MCAT. On the flip side, unless you are interested in ASL in and of itself, that is not necessarily going to be helpful there. So it's more of a matter of what you find more interesting and how you plan to study for the MCAT. Good luck.


Pre-Med from PR where to apply?

I'm from the University of Puerto Rico. Most of the students who pursue their undergrad education in PR don't apply to US schools. I don't want to limit myself to the schools on the Island. I received my MCAT score and got a 512 (129/125/129/129), I have a 4.0 GPA, research experience and volunteer work. I'm Mostly lacking clinical exposure but am currently volunteering at a hospital. I've worked since my second year. I want to apply to US schools but don't know which are best in terms of considering out of state and Hispanic students.

I've applied to 2 schools in PR and 3 in the US: UCONN Health (did a summer program there), University of Central Florida and Penn State (long shot). Was thinking about applying to Quinnipiac or Miami Miller School. Want to apply to a really great school even if my chances are low but don't know which school is the best fit given my application and background. I'm still waiting for the AMCAS verification so I can't apply to any school with a deadline in October; yes, I'm late for applying I know.

mark-ER

Short answer apply where you feel comfortable living (if that's florida, then great) and where you have a solid chance to get in. That means where they take good number of out-of-state students and where they value diversity/Spanish-speaking skills. I would focus more on the former, than the latter. So no in-state public university (no University of Michigan, no UNC, no UIowa, etc), but perhaps mostly mid-lower tier private schools and maybe a sprinkling of higher tier/reach schools, though perhaps not the very tip, top-10. Among mid-lower tier think along the lines of Rosalind Franklin, Loyola, Wayne State, Jefferson, etc. Among reach schools maybe Vanderbilt or UChicago. Overall, I think you have the right attitude -- apply broadly, you never know where you are going to get in. Your stats and experiences are solid, though not super-star level, hence my recommendations. Read and research SDN forums and again as you point out do so quickly, as it is already nearly October and most medical schools have rolling admissions, so you are behind. Good luck.


Looking to hear postbacc stories

Hello - I am a current medical student at the University of Southern California. I mentor undergraduate students and have realized that it's really challenging to find information on post-bacc programs.

I would love you hear your perspective: +What's the value of post-bacc? Does it really help your odds of getting into med school? +What do students do after post-bacc if they don't get into medical school? +How should students think about picking a postbacc program that's right for them?


tantacles

Post-baccalaureate programs tend to be valuable for students who have not taken their pre-requisite courses for medical school as well as for students who want to boost their undergraduate GPAs after a poor performance.

Many students who do these programs go into other health professional fields if they do not get into medical school, like dentistry, PA school, or nursing.

The best post-baccalaureate program is one in which you have the appropriate support to get the highest grades possible. For many people, location plays a role in this as having family close by helps them to do better in their programs. Some post-baccalaureate programs also offer automatic interviews based on grades in the program and MCAT score.

Ultimately, all post-baccalaureate programs serve the same purpose - allowing students to take science courses for undergraduate credit to bolster their applications. The decision of which post-bacc program to choose is personal, but if you do well, the post-bacc program will be a help and not a hindrance to getting into medical school.


Help with school list

I have a 3.9 gpa and a 517 mcat. I am from Pennsylvania. Can you help me generate an appropriate school list?

tantacles

I would suggest that you purchase the MSAR:

https://apps.aamc.org/msar-ui/#/landing

This app will help you choose schools that have similar GPAs and MCAT scores to your own, and it will also provide information on how many out of state students are accepted to state schools. SDN also has myriad resources. I would suggest that you purchase the MSAR, choose 15-20 schools that seem in range with some schools that have averages below yours and some above yours and then repost either here or on the forums. Generating a school list will be quite difficult without knowing more of your preferences, but your MCAT score and GPA will not lock you out of any schools.


Secondary App Mistake

Hello! I was checking up on my application statuses on the different schools I applied to and I realized that I filled out the residency statement for UVM incorrectly. Is it okay for me to email their admissions with the corrected statement as an application update or should I just leave it as is? (The app asks for a specific format for this question that could only be found in the FAQ, which I obviously did not look at prior to submitting my app. It asks you to write the town you lived in at every age and the population of the town, whereas I simply wrote I lived in x state for my entire life.) Thanks for your help!

tantacles

I would honestly let this be. Don't draw attention to the fact that you filled out this portion of the application incorrectly. They likely know that you filled out this part incorrectly. I would imagine that if the rest of your application is up to snuff, the school will reach out to you and have you correct the information if they are interested. If your application is not up to snuff, they will likely not seek you out to update this information.


Complete secondary even though course requirements aren't met?

So, shame on me, I applied to a medical school without doing the proper research and I didn't realize that I didn't meet all of their course requirements.

Weirdly, however, I received a secondary from them (though it took much longer for it come than the other ones), and when filling out the course requirements tab on their secondary is when I found out I am missing some courses!

Is it worth it to still submit that secondary? Or am I literally just throwing money down the drain?

mark-ER

Yes, worth it IF (big if), you are only missing one or two pre-reqs that you can finish up in the next semester or two, before your matriculation into medical school. You took the MCAT, right? -- you must have had most of the pre-reqs, so you are missing one or two (?) Another approach would be to reach out to the admissions office for the medical school where you did receive a secondary.


Is it late to apply to DO schools now ( mid september) ?

I have a GPA of 3.5 and MCAT score of 506 and graduated from UCLA with a BS in Psychobiology . I have multiple volunteering and work experience however my application lacks research publication although I have worked as an RA in 2 different labs.

Mr.Smile12

While I don't think it is technically too late, you are playing against the odds. Many schools have already set up their interview dates through the end of November. Now you can still apply because schools are always trying to fill their classes up through the spring, but you should make sure everything is in order with your letters and transcripts to AACOMAS.


What are my chances?

I am a 5th year health sciences student. I have been through a lot since beginning undergrad. I have lost both of my parents, a grandparent, and went through a bad breakup and have suffered with anxiety and depression. My grades are not the best. I am going to have to retake at least 3 science courses I scored D's in. I am willing to apply to DO schools, my overall GPA is above a 3.0. It's hard to know exactly what it is because I transferred and my new school calculates my GPA based on the courses I've taken there, so right now according to their calculation I have a 3.5. I have took some courses at community college and have attended 5 different institutions due to everything I've been through. I have not taken the MCAT as yet. I am hoping to raise my science GPA to a ~3.3 by the time I graduate next fall. Will this be enough if I do well on the MCAT?

Mr.Smile12

I guess it all depends on what you mean by doing well on the MCAT and the schools you are trying to apply to. In general though you should probably focus on those courses you need to repeat and do your own AACOMAS calculations. You should also ask each school about their GPA considerations before you even apply.


Finishing undergrad overseas

My husband is in the military and we’re going to be moving to Germany. I have one year of my undergrad done- is it possible to finish a bachelors in biology in Germany and still have a good chance at getting into a US med school?

mark-ER

Yes (meaning you can proceed this way) and yes (Western European schools with teaching in English are viewed well by US med school ADComs). I answered a similar question, so search archives, but briefly: search out a school where teaching is done in English; make sure the school is appropriately accredited and you can transfer credits (confirm this BEFORE enrolling); ask if anyone else before you with a similar background (American Expat) was able to successfully navigate the medical school admission process with a degree from their university, then reach out to him/her. You should be able to get answers to these questions from the admissions office at most European universities. From what I read (I have not been back to Europe for 10+ years), German universities are becoming more desirable to international students, so much so that they are becoming crowded with higher student:faculty ratio and a lot of burn-out among faculty. If you are a type of person who benefits from more individualized attention (rather than self-learner), seek out solid but lower profile/prestige schools (for instance Frankfurt am Oder. Ultimately, how the whole thing plays out is largely up to you -- you will have to be very self-motivated and do well on the MCAT. Overall, the experience can be very good, but it is what you make of it and preparation is a big part of setting yourself up for success -- so kudos to you for asking the right questions.


Post Bacc Pre-Med program as an International Student

I have always wanted to become a doctor. Couldn't go to med school then owing to a couple of reasons. So pursued electronics engineering (2014 pass out) then did my MBA(2016 pass out) worked as a business analyst for a year and half. Now decided to do what I 've always wanted to. And now Post bacc Pre Med is my only way in. I would like to know if its a plausible thing as an International student? and what are my chances? If there is a fighting chance, how and where do I get things started? Has anybody taken the route of Post Bacc Pre-Med ->MCAT -> Med school as an International student?

Mr.Smile12

Without really addressing your citizenship status (US permanent resident or citizen or other eligible status for financial aid and residency standing), it could be a waste of your time and money until you have a timeline established for your status change. Otherwise, I know there should be a forum full of international students who have gotten US status and gotten into medical school (or there should be) whose advice you can seek.


Connections to Med School's communities

On many secondaries, medical schools often ask you to describe your connection to their school, their community, the state that it's in, etc (beyond "why do you want to go here?"). If you actually have NO connection (didn't go to school there, not born there, no relatives there, never visited, parents didn't go to school there, literally not a single connection), is there a strategic way to respond to these questions? Just to emphasize, the question isn't why do you want to come here, where you could do research on the school, the area, and the state to come up with your reason, the question is what connections do you have. If you don't have any, do you leave it blank? Or is there a way to go about this?

Thanks.

Mr.Smile12

Most applicants are upfront with their selection strategies when it comes to picking medical schools. After all there are 140-180+ different choices. Maybe the word "connection" is the semantic issue; usually I find that schools want to know why you would be attracted to their program, and so that would be the way I would suggest anyone think about their answer to such a question. You don't want to come across that you are just doing a random search.


Should I go to school to become an MA before going to medical school in order to get clinical experience?

Right now, I attend a community college, have a 3.7 GPA with 60+ credit hours. I am at a cross-roads in regards to where I go after I graduate in the upcoming spring semester, and I am researching universities to transfer to. One college that I've been seriously looking at, has a fantastic medical school but to increase my chances of getting in, I need to do undergraduate research. To get that opportunity, I'll need to get into the honors college. The good news is, I qualify because of my GPA so far, the bad news, only 22 students are accepted. This is path 1.

This is path 2: I will also need clinical experience to increase my chances of getting in. Medical assistants are needed badly in the U.S. right now. I could go to medical assistant school, become an MA, and get about a year's worth of clinical experience under my belt before I apply to medical school. But I can't do both college and MA school concurrently; I don't have enough time or money. I'm tempted to choose the first option, because my education was already delayed about a year and a half due to two surgeries. Clinical experience would be nice, but it's not a requirement to get into medical school. However, going to MA school might be the more logical option. What should I do? Go to MA school first, or continue my education as a pre-med student?

Thanks in advance.

mark-ER

The first option is better. Not enormously better, but better. 3.7 GPA is pretty solid, though you did not break out your science GPA (hopefully that's higher) and you do not have MCAT score on file yet. Many medical school AdComs will like to see you succeed academically at a higher-tier 4-year institution. Yes, research and clinical experiences are helpful, but you can get hit those checkboxes as extracurricular activities and during summers. Research experience can be done outside of an honors program (google UROP, there are plenty of undergrad research experiences likely within your desired undergraduate institution and outside). So yes, try to get into the honors program, but your option #2 may be to apply more broadly to 4-year colleges within your desired geographic area, rather than MA school. Of course if you have other extenuating circumstances (need to earn a salary or support a family), the calculus might change; but if your main goal at this time is to succeed in school, try for the best 4-year undergraduate institution you can get into, at a reasonable cost (that is often your state's flagship, i.e. "University of ").


could i get into medschool?

currently just became a junior, looking to see how i stack up and how/ where i can make improvements in my overall resume

-GPA: 3.62 (deans list 4 semesters) 1 year microbiology research 2 years bioengineering research (presented nationally) 6 months pharmacoepidemiology research (to be published) 1 year neuroscience research (to be presented this year)

club president, newsletter founder +150 hours shadowing experience +200 hours volunteering

just became a hospice volunteer

lost? could i be doing more? should i be?

Mr.Smile12

Hopefully you have been in touch with your health professions advisors to give you a good idea of where you stand for the moment. It is always too challenging to gauge where anyone stands without knowing how well you do on the MCAT and what schools you would be targeting.


Lots of clinical experience no volunteering

I have a 3.7 gpa I’m taking the mcat in the spring. About 1000 hours of direct patient care, 4 semesters of research in a psychology lab, 8 hours of shadowing and no volunteering. This summer I will be applying and I am wondering what kind of numbers I should strive for with my shadowing and volunteering? I will still be taking classes and mcat prepping. I am not sure how much schools value volunteering when you have a decent amount of medical experience so if you could shed some light on that I would appreciate it.

Pathdocmd

I would try to get something in, but your 1000 hours of patient care is more impressive to most committees. Try to find something you are interested in and it does not have to be medically related. Showing is good, but a lot isn't better. Besides, the patient care (active) work trumps shadowing (passive).


What should I do? I have lost all hope!

I am very worried about my upcoming school year going into second year, I did not do as well as I had hoped to do in my first year of University. My GPA was only a 3.1, making it seem next to impossible to raise it up. I am an extreme procrastinator with a touch of depression, that makes it difficult to succeed in my courses. If I study I know that I am capable of achieving high 80s and 90s, but the issue is commitment for me. I was wondering if anyone had a similar undergraduate GPA for their freshman year, and how did you bring it up? I want to raise my overall GPA to a 3.9 by the end of my 3rd year, and do not know how to achieve this with the current grades that I have!

mark-ER

Certainly not all hope is lost, some people have a harder time adjusting to college than others. You may also have very real, legitimate mental health issues (please strongly consider counseling, a lot of colleges have professional, free or nearly free, anonymous services). Couple lines of approach/advice: lower down your course load (you can still finish in 4 years, even if it may require an extra class in the summer), see if there is an academic help consult service available (my undergrad had a service like that; I used it my first semester, and by year 3 I became a tutor), and/or find a group of like-minded individuals both to help in studying/understanding concepts and motivation. Good luck, you can do it!


Help with School List

I have a 3.9 GPA (science and overall) and I recently scored a 516 mcat and I am from New York. Please help me make a school list of target schools

mark-ER

Very strong numbers (congrats) and quick question right to the point (I like it).

In your position, I would put schools into 3 tiers: likely acceptance, mid-tier and reach. The number of schools on the list may depend on how much you can spend on application/travel to interviews & how sure you want to be of the 'right fit'. I think anything more than 25 is overkill -- after a certain point you will get tired of the process and basically all US medical allopathic schools will get you where you want to be (albeit at a different costs (#1 consideration for many), geographic location and training environment). For likely acceptance tier, include all instate schools and maybe 2-3 lower tier private schools that take out of state residents in a geographic location that is desirable to you (SUNY Downstate, SUNY Stony Brook, SUNY Buffalo, Albany and for private schools maybe RUSH if you like big cities). Middle tier should be your largest group, and here again mid-higher prestige in-state and private schools in your general area (Einstein, Icahn, Thomas Jefferson, University of Rochester, Brown, Hofstra, Tufts, Rutgers, Drexel, University of Rochester, Pitt) and maybe a couple where you might want to live geographically (Georgetown, Uchicago, Vandy). Then last group is high tier/prestige schools or places where you really want to live geographically but have not ties to and/or you are out of state (very low chance). That's for you to decide, but keep that last group to fewer than 5 -- it's hard to get rejections and though your stats are very strong they are not absolutely stellar, so places like Harvard, Hopkins or Stanford are going to be quite a reach (on the other hand, you only live once).

The only schools I would really avoid, because it is a waste of your time are in-state schools that only take 2-3 out of state candidates per class and even then only with VERY strong family ties (alumni, lived there their whole lives), places like UHawaii or UC San Diego. If you want those locales, you can try them as a resident, much fewer geographic consideration in the selection process.


Should I do a post-bac or should I retake my science courses in my undergrad years?

Before I get into my current crisis, I am going to talk about my background.

My ultimate goal is to become a physician. In my freshman and sophomore year at my university, I was declared as a Chemistry major and it has been a difficult road. I took some prerequisites for medical school and my experience has caused me to stumble a lot. I passed one year of physics, one year of Calculus, and the first general chemistry class. However, I also took Gen Bio I & II, and Gen Chem II. I withdrew out of all of those courses (I was around a D in all of them, so I just took a W).

Now, I'm going into my junior year. I decided to major in Religious Studies & Spanish so I can still be on track to graduate on time. The Religious Studies major will help me as a prospective physician because each culture has their own beliefs, especially relating to medicine. In addition, Spanish is the second most-spoken language in the world. It's significant to have a connection with my patients rather than having a translator in the room.

I only have two options:

Option A: Should I still fit in one science course each semester? This method could work because I would only focus on one science course. I took two sciences in a semester twice and I had to drop out one other science course. I can complete my two majors on time and do this. Maybe I'll understand the science courses better since I have already taken them.

-OR-

Option B: I recently went to a Pre-Health conference at Stanford University and I was introduced to a post-bac program. It's available to those who have an interest in medicine and already have a university degree. It's a one year program where I can strictly focus on and retake my science courses, prepare for the MCAT, and shadow physicians at hospitals/clinics. It's about $30,000 for an entire year but it guarantees admission to medical school. Would this be a better option so I don't have to stress about retaking all of my prereqs?


I already took these courses and here are my grades: Physics I (B+) & Physics II (B+) Calculus I (C) & Calculus II (C) General Chemistry I (C)

I'm missing the following courses in order to complete the prerequisites: General Biology I (W) General Biology II (W) General Chemistry II (W) Organic Chem I (not taken) Organic Chem II (not taken)

Basically, My science GPA is 2.58 and my overall is 3.2

Besides school, I work a part-time job to support my family and I volunteer every week at Kaiser Permanente to gain experience in a healthcare setting.

Any thoughts? All comments will be appreciated! I just need a sense of direction. Thanks in advance!!!!

mark-ER

First, I commend you for taking the time to "stop" and re-evaluate your position, even if it required taking withdrawals. If you were to continue along this trend, you could have done irreparable damage to your grades and dashed your hopes of pursuing your career track in medicine. That being said, you still have quite a steep hill to climb. If lowering your course work and taking one class in science at a time, focusing on your other courses as well (it does not seem you have had straight As in your major either, given your overall GPA is also quite low), then that is exactly what you should do.

You will likely have to follow BOTH plan A and plan B, since you will have to do some re-mediation. I would suggest really putting your head down to the grindstone and trying to aim for AT LEAST a B+ in General chemistry and O-chem and one or two advanced biology courses, aiming to finish college in 4 years with your selected major. Then retake Calculus (at least calc 1), and take an additional biology course (perhaps a genetics) or a stats course in a post bacc -- either specifically designed post doc (like the one you mention for stanford), or self-designed one, while studying for MCAT and doing some research.

So I know that is a lot, but that is what is expected from successful US medical applicants. So also please seek help and academic counseling, talk to your classmates when choosing courses/professors and if you see yourself struggling drop the course BEFORE you have to take a W. Lastly, please do not despair or think of yourself as a failure if you cannot reach these lofty 'ideal' goals -- consider DO schools and consider shadowing other allied health professionals -- genetic counselors, PAs, CRNPs, etc. They all do important meaningful work, with a lot of autonomy but less hoops to jump through and a lot lower student loan debt than physicians.

Last, but not least upon re-reading your longish question -- NO pre-health post-bacc program can "guarantee" admission into medical school. You still have to have the grades, MCAT score and show yourself to be capable to handle the workload from both intellectual (and perhaps more importantly) self-motivation/delayed gratification standpoints.


Will it matter if the total units at my UC are different than the total units I receive at the CC?

I am planning to retake an organic chemistry class at a community college since my institution will only allow me to retake the class if I got below a C-. However, the total units I received at my college for the entire course will be different than the total units I'll receive from my community college, even though those classes are equivalent.

Will this cause any complications? The organic chemistry course at my quarter school was 15 units (5 units each quarter) total but at my community college it'll be 12 units (6 units each semester).

Also, If I don't finish the entire organic chemistry course on time before I apply, will this hurt my application since organic chemistry is a pre-requisite? I have taken it already but I just want to retake it since I didn't do so well.

Pathdocmd

You have to check with the individual medical school, but if it is a "full year" of either semester or quarter hours, it should be fine. The other thing to look into is if one school includes the lab in the lecture course or if the lab is a separate course. This may be the cause of the credit hour discrepancy. I would take the lab with the course


C's in Organic Chemistry but retake policy is only for anything below a C-. Can I still retake at a community college?

Hello, I have recently graduated from undergrad from a UC earning a B.S. and am planning to apply to med school next year in 2019. My GPA is on the lower side because I had accumulated 3 Cs in organic chemistry (with a C- in one of the courses in the series). These Cs are the only Cs I have gotten during my undergrad and I would love to retake the entire series to prove that I can handle the course load.

In my upper division biochemistry courses I had gotten all B's and above. I would like to retake the entire Ochem series so that I can redeem myself. Additionally, I understand that some medical schools require a C and above so my C- would be dragging my whole application down. However, at my institution you can only retake a class if you had gotten below a C- so I couldn't retake the course at my UC. With this policy though, can I still retake the course at a community college? Also, my UC was on a quarter system and the community colleges around me are all semester schools so would that cause any complications? How would I submit my grades on the AMCAS/AAMCOAS if I did retake the course would this class be part of a "post-bacc?"

If I cannot retake the course because of my UC's policy, then what are some options I have in terms of this C-. I would still be able to apply possibly to some schools with my C+'s but my C- would really be the limiting factor here since it might not even be accepted at all. What would I do in that case?

Any guidance or insight towards my problem would be of great help and appreciation. Thank you!

tantacles

You can absolutely retake these courses at a community college. AMCAS has no rules about which courses you retake, though all grades are factored into your GPA, which means your C- and the new grade you got would be averaged together. AAMCOAS, however, allows for grade replacement. The course would simply be factored into your GPA as an undergraduate course taken after you graduated.

If a school has a policy that they do not accept a C-and you choose not to retake the course, it is unlikely you will receive an interview from that school.


How does grade inflation/deflation affect medical school admissions?

I am currently a California HS senior enrolled full-time at a local community college, trying to decide where to transfer. I have been told GPA, MCAT, and extra curriculars are the most important things to admission committees, but do they consider the rigor of the undergrad curriculum? For example, would an applied mathematics major at UCLA with a 3.8 GPA be viewed in the same light as a gender studies major at a small private school with a 3.8 GPA?

Mr.Smile12

Most medical schools will consider where classes were taken or where the student was enrolled. I'm not sure to what extent they consider majors, though I suspect they will focus on common core courses that are generally required or preferred.


Pre med dance major

Hello, I am currently signed up to be a biology and dance major. However, due to my dance schedule it looks like the double major at my school will be impossible unless I take 18 credit hours a semester and take a fifth year. I don’t have money for a fifth year. I am on scholarship for dance so I would like to not drop the dance major. Will I still be able to get into medical school as a dance major? Also, due to my dance schedule, I will have to take some of my pre requisite courses for medical school over the summer at a community college. Will medical schools accept these credits?

mark-ER

Yes, you can get in with a dance major. As long as you do your prerequisites (talk with the pre-health advisers at your school), do well in your science classes and on the MCAT you will be OK. In fact, keeping up with a busy dance schedule would be a testament to your tenacity and following your passion and add diversity to the medical school class. As another example, per recent match list at harvard medical school, one person decided to forego residency for a year to pursue a stint with a professional dance company.


Adding more medical schools

If my AMCAS application has been verified since June and I have added additional schools, how long will it take me to receive those secondaries?

mark-ER

Pretty quick at this point. Expect within 1-2 weeks (remember, medical schools have rolling admissions).


Should I do a post-bacc?

Hello!

I was wondering if you had any information that would help me decide whether or not I should look into post-bacc programs (aimed toward academic record enhancers) or retaking 2-3 classes while working full time.

I graduated college with a 3.46 cGPA, 3.42 sGPA. I have 3.5 years of biomedical research experience (2 yrs full time since graduating undergrad). I am taking the MCATs in early 2019, so I can't comment on my performance on that exam.

I given this information, I wanted to know if I should just focus on post-bacc programs, or look into re-taking or taking additional classes to boost my low GPA. Given that a post-bacc is a significant financial commitment, I was hoping to get some advice or guidance.

Thank you in advance!

mark-ER

Your GPA is low, but something you can fix/improve/overcome with a couple of good, upper level classes and a very good MCAT score. So it really comes to knowing yourself -- if you think you can stick to a regular, strong study schedule every evening (for the MCAT), while working and taking classes part time, go for it. Also do you have access to your undergraduate schools med school advising/admissions staff? Choosing a do-it-yourself (DIY) route you would be saving yourself a good chunk of change in tuition, but loosing out on potential structure, help with admissions, exam and potentially link-in into medical schools. Good luck.


Obtained a unique experience after primary submission- should I send a letter to update schools?

This September I am taking a course to be a certified level 1 wine sommelier. While this is completely unrelated to medicine, I am unsure if schools might find it interesting enough to distinguish me from other applicants. What do you think?

mark-ER

No, not worth a formal letter update ,but it is something you could update as part of a larger update process for secondaries (plus publications, new sets of grades, etc). It also makes for an interesting snippet of conversation at interviews, particularly in & around wine country (California)? It certainly doesn't hurt, but also doesn't help that much (sorry to burst your bubble; there are a LOT of very interesting/dedicated people applying to medical schools.


Forgot an Institutional Action AMCAS Application

I just realized I forgot that I have an institutional action and did not mark "yes" on my AMCAS application. I was talking with a friend when the subject came up and it sparked my memory that I was written up the first week of college freshman year for alcohol under the age of 21 occurring in the dorms. It is on my conduct records under ResLife. I went to go change the answer on my application but it is "Ready for Review" so I can't change that section. The incident is not on my official transcript and I was never legally in trouble. I am not sure what I should do

Mr.Smile12

I don't know if you have begun to answer secondary applications yet, but you probably will have another opportunity to properly disclose this institutional action there. If you are interviewed, you should probably make sure you disclose it again to the admissions staff. At any rate, you can probably email the schools you have applied to now and let them know of the oversight.


Should I gain experience in a lab or work on an animal farm?

I am going into my senior year at IU and a professor offered me a job at a sheep and cow farm not to far away from campus, but she can also put me in touch with some professors who could put me in an animal behavior lab that studies the aggression of male hamsters when testing their territorial boundaries (run by the head of the Biology department at IU), or one that studies dung beetles in microbiomes.

I'm not sure which one would be more worth getting experience because lab experience with the head of the biology department would be great, but also working on a farm would be nice to relieve some stress and I believe it would make me stand out from other applicants.

tantacles

The best thing to do is to do the thing that you are most interested in. Generally, this will give you better topics to speak about at interviews and will make you stand out from other applicants.


Pre-Med Post bacc for International applicants

Has anybody taken the route of pre-medical post-baccalaureate, then MCAT, and finally make it to medical school in the States as an international applicant? If so help me with how and where do I start?

tantacles

I would strongly suggest that you ask about this in the pre-medical forums; there is a large number of applicants from all walks of life who may be able to share their stories and help you find a solution. You can find this forum at the below link:

https://forums.studentdoctor.net/forums/pre-medical-md.10/


Will DO schools or post Bachs accept this?

(Im a PA resident btw) So I took most of my prereqs and I’m headed into senior year, just have physics 2 and biochem left. I want to focus on my EC’s since I’m gonna be taking a Gap year. I had a lot going on during the spring semester junior year and got a C- in orgo. I really really don’t want to retake it because I absolutely hate my school’s chemistry department and it makes me super depressed whenever I’m in there. I want to apply mainly to both post Bach’s with linkages and DO schools after my senior year and go with whatever I can get into. My question is.... can I get into programs with that C- or do I have to retake it? If I can, please name which programs I have a chance at?

tantacles

It is possible that some programs will overlook a C-, but I would check individual programs' websites. Many programs state that a C or better is required for admission, so you may want to consider retaking organic chemistry regardless.


extra semester community college

I am currently in my third semester at my local CC. I could technically finish my AS degree next semester, but I am thinking about taking an extra semester before I transfer. This would be for personal reasons not academic. If I did take an extra semester I would complete all the necessary prerequitsities before transferring. My major at the university would either be neuroscience or physics. Considering I would be taking a ton of upper level science courses to finish my degree, would it hurt me to spend an extra semester at CC before transferring?

mark-ER

You are asking if it would hurt in terms of attaining medical school admission, right? If so, no extra 1 semester of CC is not going to hurt. You are just finishing your associates and transfering to a full 4-year university, so you are just completing your course of study. But please be advised, many medical school admissions committee do take CC grades/credits with a grain of salt, so please excel in your advanced coursework at the 4 year university and ace your MCAT. That being said, for many (? most) people, the financial/social benefits of 2 years of CC arguably outweigh doing full 4 years at a university, so good for you.


Technical MCAT Scoring Question & Advice on my MCAT Situation

1) Is it possible that by virtue of how AAMC standardizes MCAT scores that, say, if your cohort of test takers does worse on average than other groups that you get a higher score? Like, when I take my exam, who all is considered in scoring? Taking in August. Is it possible that summer test takers are more/less prepared than say January takers and so scores are different just by who I test with? AAMC Official Guide doesn't make it clear when the groups "reset" or if it depends on which version of the test you had, etc., other than alluding to scores resetting in 2018. In that case, if I have being standardized against a smaller sample, won't it be more likely that my score is de-/inflated compared to what it would be if it were standardized against years and years of this particular administration of the exam?

2) When I took the first time I didn't study & got a 501. I only practiced by taking AAMC FL 1 & 2, and got 501 & 502 resp. but w/a **** ton of variance in section scores (123-128). Now, I retook FL 1 & 2 and got more consistent, higher scores, 512, 515 resp. with only one 126 & the rest >127....

BUT... when I took FL 3 I only got a 511 (128/127/130/126). So I'm worried, were the scores inflated bc/I had taken FL1&2 before? (I didn't remember the answers or anything, only was familiar with some of the CARS passages but that's it). I test Friday and want a 510 and am nervous AF. Just gonna be memorizing and reviewing my weakest concepts until then I guess. Any advice? Esp. from FL3 vs. real scores?

mark-ER

Also, why are you taking the exam so many times? I can understand re-taking, maybe even twice, but what makes you think you can in a great way improve your score taking it 4-5 times? Ultimately it all evens out & you are putting yourself thru unnecessary stress (& expense). You have a decent score, go ahead and apply with what scores you have.

tantacles

Taking more practice tests is always a good idea to get a better idea of how you will do on the real exam. As far as how scores are standardized, it is possible that a poor cohort compared to you could mean a better score, but there are a large number of people on any given day taking the test, and thus it is unlikely.

It is hard to say how you will perform on this specific exam, but if you are unsure, I would suggest doing another practice test to see where you stand.


When to start worrying

Hello!

I've been watching many people who were complete after me receive interviews already. It's extremely discouraging because I feel like I have a strong application and come from a top 10 undergrad school. Furthermore, all of my friends who have gone through this process told me to submit early. I'm wondering if the time I submitted is correlated at all to when I'll receive interviews and if I should start worrying.

Thank you!

tantacles

It is hard to say exactly when schools will send out interviews. Some of this depends on which schools you have applied to, and some of it depends on how good a fit those specific schools feel you are for their program. That being said, many schools interview until April. If you do not have any interviews by October, I would start worrying about your application and thinking about a backup plan.


Does it matter what online certificate I do?

I graduated undergrad with a cGPA 2.5 and mostly C's and a couple of D's in my science courses. After uni I joined the military, EMT, attempting to go to medical school with no debt. But running into the same problem that my GPA is low, I'm attempting to fix it now after being in denial for bit. I am deploying right now so that limits me to online courses. While deployed I'm planning on redoing some of my prereq courses online and I'm trying to figure out what certificate to do while I do my prereqs. When I come back I'm going to try to enroll into a physical college and do badass on my prereqs. I'm obviously going to be trying to study for MCAT and do super well on that too. So if you could give me any advice on any online certs or advice to my plan that'd be great.

Thank you!

Mr.Smile12

The first thing to do is to contact medical school admissions officers at programs where you have a high interest attending and ask them for their advice about your situation. Obviously a certificate program isn't going to address or alter your GPA, so I'm not sure how much an online program is going to help at face value. (You likely already have a lot of desirable qualifications and experiences from your time as an EMT.)


Community College Coursework and Transferring

I am currently in my third semester at my local community college. Technically, I could finish my associates degree next semester and transfer, but I am thinking about taking an extra semester at CC. I have already taken multiple prerequisite courses. If I do decide to take an extra semester before transferring, I would complete all of my medical school prerequisites at CC. I am thinking about doing the neuroscience major at UofL where I would take some anatomy and physiology courses through their medical school. I would also be taking upper level biology and psychological sciences courses having to do with neuroscience. So my question is would it hurt my application to take an extra semester and finish the prerequisites before I transfer even though I would be taking dozens of upper level sciences courses at UofL? Also, UofL medical school is my number one choice, so I might get to have some of the professors that teach there. My current GPA is 3.77. Thanks for taking the time to answer me!

Mr.Smile12

I am hoping you have talked with admissions staff at U of L about your decision. Your upper-level science courses there are going to more likely dictate your chances about being considered for admission, but you also want to do what you can to network with students and faculty there during your time as an undergraduate (or even now), including opportunities for research or community service. Work carefully with your transfer student counselors to be sure that all your prerequisites count when you transfer (that is to say, help satisfy any entry requirements to take upper-level science courses).


How Much Damage is Done?

Hello Everyone,

I am currently in the summer of my 1st year of my Undergraduate career.

In a Bio Lab class I took my 1st quarter, my professor told us that "Nothing can erase a C in Ochem".

The way Ochem is split in my school is that there are 3 separate classes, each with their own labs. Each lecture is 3 units, while the lab is just 1. Unfortunately, I got a C in 1 lab, and A's in all the lectures and labs (the other 2).

My GPA currently is a 3.7 because I received a B+ in a general chemistry class and a B- in Bio (1 out of 3). (I've received either A's or A-'s in the rest of my classes).

I am also doing research for a lab (hoping to get published in the next year or so) and I have been shadowing a doctor for roughly 2 months now (will try to do this for as long as possible). I am doing this along with other extracurriculars. Best case scenario, if I get straight A's from here on out for the rest of my 3 years (would be ~3.85 maybe) and an excellent MCAT score, how much damage has this 1st year done for hopes of being accepted into a top tier medical school?

Mr.Smile12

I would want to know what you define as a "top tier" medical school and why you are insistent going to one. If you want to go there, how much research have you done (I do acknowledge your answer)? How much leadership and community service have you done? Have you considered a gap year, and what options are you thinking about? How much networking have you done with students attending those medical schools and alumni who graduated from there? And just as important, how much have you done working with your prehealth advisors on campus? I don't think one C is going to undermine your application, but having blinders on to just focus on the top tier may prevent you from seeing a lot of other unique opportunities many other medical schools have developed to help with the clinical education of its students.


Personal Statements

Hey, guys. I know this is rather late, but would anyone be willing to give me feedback on a draft of personal statement, please? Any help would be greatly appreciated. I'll PM you a link to the google document.

Thank you.

Mr.Smile12

The forums have specific individuals who volunteer to help with personal statements, so I hope you posted your request there.


Sending a CV or Resume post secondary submission

I am applying to both MD and DO schools for the 2018-2019 cycle. Most of my applications are considered complete. I noticed that many of my schools have the option to submit a CV or resume, and some have options to submit anything as an update. I am thinking of submitting my CV mainly because it contains the citations for many poster presentations I have given, but were not able to squeeze into my AMCAS or ACOMAS applications.

Is it a good idea to submit a CV to the schools that list it as an option? And what about the schools that don't specifically list CVs as an option, but allow you to submit an "update?"

Thanks in advance!

Mr.Smile12

If you feel that there is important information that is not included in your original application, I don't see it hurting you to take advantage of sending a CV that lists significant items such as publication/poster references/citations. I would think carefully about what you want to submit because you should be efficient and not send an update every month.


Is there hope?

I need all the answers I can get.

So I have a 2.4 gpa with only one class left for me to graduate my undergraduate studies as a biology major. I have yet to take the mcat. I have done research, volunteered in the hospital, and I've been a Ta for Organic chemistry. I really would like to be a doctor and have always dreamed of doing so, but I have clearly went off track. I would really like to fix my life and get back on some type of track. Do you have any advise for me? At this point I am not a competitive applicant for Medical schools in the U.S . nor even the accredited schools in the Caribbean . Please advice me on what to do at this moment. Also it would be great if you can estimate the time frame before I make it to medical school. I know post-bacc programs, and possibly SMPs are an option. If you have any programs in mind for me, also let me know. Is there hope?

tantacles

Right now, you are not a competitive applicant for medical school regardless of your MCAT score. If you want to be accepted to medical school, you have several options.

1. Wait several years and pursue a post-baccalaureate program after this period. This will allow schools to see that you've taken time and improved.

2. Do an SMP program. The SMP, if you can get accepted to it, may offer an interview to the school it is connected to. That will likely be your only interview given your GPA.

That being said, at this point, your GPA at this point may keep you out of even an SMP. I would also look into other options besides medical school.


non trad, low gpa, high mcat

More than 10 years ago I was a terrible undergrad, I've since had several careers, and recently became a paramedic, which feels like a calling, so I've done the pre-med courses. cGPA 3.2 sGPA 4.0 MCAT 524

I have no idea if or where I might be competitive with those stats. Any advice would be really appreciated.

tantacles

Based on your redemption and the fact that you've been able to do well in all of your science courses and perform excellently on the MCAT, you have a great chance of getting into medical school. It's hard to completely predict your chance of success, but provided you've done everything you can to get the appropriate experience (including being a a paramedic), you should absolutely receive interviews based on your statistics.


I worked in my Uncle's lab for a summer after freshman year, he is one of my reference letters and i am applying to the school he works at.

I worked in my Uncle's lab for a summer after freshman year, he is one of my reference letters and I am applying to the school he works at. I worked very hard doing research there. Will the adcomms look down on this? It's too late to retract the letter, and I have to disclose family members working at the school on my secondary.

mark-ER

You should be fine. Does your uncle have the same last name? Regardless, he should know better and mention you are related, plus he should make sure to incorporate verbatim "in quotes" positive comments from the senior people you worked with at the lab, grad students, technicians, postdocs. Ask your uncle if he already sent the letter (if he's like most academics, probably not) and briefly mention that to him, just to be extra safe.


Kicked out: failed 1 class due to grief, waiting for appeal any advice?

Kicked out of my program: failed a class due to grief. My mom passed away unexpectedly. I have a wife and child on the way, immersed in student loans. Waiting for the appeal. Any advice? My program didn’t let me take a year off. They won’t let me re-apply.

mark-ER

First of all, my deep condolences, very difficult situation. That is a lot on your plate. Please, first of all take care of yourself and your family. There are a lot of resources for free and confidential psychiatric help and counseling -- someone who will listen and help you to process. Take a deep breath and a pause. Your life and well being are VERY VERY important and I know it's hard to see being directly in this situation, but this one misstep will not drastically alter your trajectory in life.

Onto your actual question -- it looks like you are still a pre-med (undergrad), hopefully junior or below. It is not clear how this failure happened -- was it academic performance, academic dishonesty, tempter at a professor/TA? Regardless, but especially if it is the latter, please with support of your wife and a senior academic mentor (guidance counselor/another professor), write the professor teaching the class you failed a short succinct but very conciliatory note to smooth things over and so that the situation does not escalate. E-mail may be OK if it is a younger professors, but in my experience older academics are typically touched by a hand-written note. Then set up an appointment with the dean of your division or undergraduate, perhaps where your mentor/supporter can also attend. Some schools in your specific very difficult circumstances may be willing to offer a "W" (withdrawal) rather than a fail and allow you to retake the class. Especially if you set yourself up and get buy in from the professor teaching the class. Then ROCK the class on a retake, and you will have a wonderful turn-around story (worst case scenario) and a supporter/reference letter from the professor whose class you failed (best case scenario).


Chances of Acceptance Into md/do School/HELP

I am currently a senior. I wanted some help/opinions on what you guys think my odds are of getting into either MD or DO school. My current major is Molecular and Cell Biology, however I decided to stay an extra year to get a degree in Psych to help boost my cum gpa. Currently my cumgpa sits at a 2.44 and my sgpa is at about a 2.7. I have some volunteering experience doing MedLife in Peru and also helping out some food shelters in my area as well as ALOT of research experience working in a microbiology lab in addition to a regenerative engineering lab(program that I completed this summer). I have written a thesis in my microbiology lab as well as a review paper for the regenerative engineering program. In addition to this I have also done SMDEP (Summer Medical and Dental Program) in the Summer at Rutgers as well. What has caused me to do so poorly is I have had various extenuating family/domestic circumstances that I have been placed in and has effected my grades throughout my 4 years in college. I still havent taken the MCAT which i believe will hold a significant amount of weight in my application just wanted to get your thoughts on if I should retake some of the science courses I did poorly in or just try to finish the Psyc Degree. I am looking to take MCAT in Jan. and apply Spring 2019 so any type of feedback would help and be greatly appreciated. :) (Side Note: I am really trying to avoid having to do a Post-Bacc or Masters Program hence why I am trying to add the Psych degree for an extra year to boost my GPA)

mark-ER

To even consider applying to US medical schools (MD and/or DO), you will have to absolutely get straight As in all your remaining science courses, to bring your GPA to at least 3.4, science GPA to 3.2 and score at least 515 to make up for those very low grades. That is quite an uphill battle. Please seriously consider 'plan B'... you can still stay in healthcare, but perhaps a field that's a little bit more understanding/forgiving of academic difficulties: DPM, RN (and if you do well, then DNP or anesthesia), or maybe PA. You will still have a good degree of autonomy and professional respect, with a silver lining of a lot less educational debt and shorter training.


Getting into University of Iowa Carver College of Medicine

What are my chances of getting into Iowa if I have a 3.88 GPA, lots of research experience, around 100 hours of volunteering, around 30 hours of shadowing, and a low MCAT score of 504 (126/124/126/128). I am an Iowa resident and attended the U of I for undergrad. I know my MCAT score is very low for Iowa but curious if there is still a chance?

mark-ER

There is a chance, but a low chance (spitballing here, but 20%, maybe a tad bit higher if you are a URM). The best advice I have is please retake MCAT at your earliest possible slot. Even a few points of improvement like a 508 might greatly improve your chances. It may not be fair, but MCATs predict step 1 score, which in turn greatly influences residency match. So even with fantastic grades and good ECs/leadership you still need a solid MCAT score. If you are applying this cycle my best advice would be to expand your application to 30-40 schools (all in-state, many if not most low-tier allopathic programs, and many DO programs), and at the same time schedule to retake the MCAT. Do it today (!), time is of the essence, medical schools have rolling admissions.


University of Miami pre-med first semester course load.

I am taking General Bio with lab, General Chem with lab, Beginners Calculus 1 and Accelerated Elementary Spanish for a total of 16 credit hours. Is this too much?

mark-ER

Nope, that is not too heavy of a coursework, but of course that depends. On one hand, if you want to be a strong medical school candidate, you should be able to handle this coursework. Some high achievers do even more, do sports & work part time on the side. But you know yourself and your high school background -- did you already have calc in high school? Did you take AP bio and AP chem? If you think you would have trouble with this, you can substitute a sociology/history class for the bio class. I would NOT remove chem or calc -- you need calculus and it's easier to do it in continuity with HS maths. You need chemistry and it's best to do gen chem freshman year and O-chem sophomore year and perhaps P-chem (if you like chem) or biochem (if you like biology), so you have that background before MCAT in the winter/spring of junior year. It's good that you are thinking about it, though.


Med school app grammar

I'm at a loss for whether or not using "they" as a single, gender-neutral pronoun is appropriate in med school essays. Online grammar sites say that "they" in the singular form is only used in informal writing, and med school essays would probably count as formal writing. But at my undergraduate university, using the singular "they" was the default to be inclusive.

This issue has come up a few times in my secondaries. One school includes "him/her" in the scenario-based question itself, but it sounds really repetitive writing "him/her" over and over in response. In another case, the question asks "what would you do if a student..." and in my response I'm tempted to just use the singular "they" instead of "him/her." What should I do?

I know a simple grammar issue probably won't make or break my application, but an older professor reading over one of my essays circled every instance I used a singular "they" as incorrect. I think physicians are probably trained to avoid misgendering patients and so may understand my usage of "they," but I also don't want a similarly negative reaction from the admissions committee.

Thanks for any advice!

mark-ER

Use "he/she". By the letter, that is grammatically correct (University of Chicago manual). There is no time/space to explain the nuances of your pronoun choice in personal statements. This diction choice may arrive as fully acceptable shortly. For situation where you think you are using he/she too much, alternate and use forms such as "the student" or "the individual". Good luck.

Edit: if the personal statement question deals specifically with transgender issues, "they" is now considered acceptable. In this specific situation only, I would then suggest using he/she/they initially, and then using 'they'. Otherwise, safest bet is to use he/she.


Help with Texas University Choice

I want to know what school is good to go for pre med in Texas. I really like the UT Austin campus but I’ve heard that the research opportunities and shadowing opportunities there are bad. The classes are also really big making it hard to get close to the professor. I want to know what other good schools there are in Texas. I don’t know if it’ll help with finding more research and shadowing opportunities at UT Austin being trilingual in Italian, Spanish, and English. The other good school that I know of are Baylor and of course Rice. Also with the addition of the new Dell Seton hospital there should be more opportunities at UT (right?).

mark-ER

It is less important to find an undergraduate with an associated medical school, than a school that is a GOOD FIT for you and where you are going to excel. So kudos to you for considering the setup of the school, class size, who teaches the courses (TAs vs. tenured professors), and perhaps curricular strengths. Yes, UT-Austin may be "the best" (i.e. highest ranked per USNWR) state undergraduate institution, but it may not be the best choice for you. Still, consider it, Austin is a great place to spend 4 years. Otherwise, good schools in TX for undergraduate on a pre-health track are private Baylor & Rice (latter, particularly if you would consider engineering), University of Houston and among UT in-state schools are UT Galveston and UT El-Paso. Last, but certainly not least, consider the costs, but also take into the account that the stated tuition cost for an expensive, private school like Baylor & Rice may not be what you pay, so apply regardless, put in your FAFSA and even if the cost is still higher than you want, you can challenge the assessment.


509(123 CARS) 3.86/s4.0 does it kill my chances

509(123 CARS) 3.86/s4.0 does it kill my chances? I am already applying but I want to be realistic, still awaiting verification

I have ok EC, with one publication URM

my list, please as truthful as possible

Tuft

Penn State

Loyola

Central Michigan


U of Miami

New York medical college

Oakland University

Tulane

U of Vermont

Cincinnati(reach)

Thomas Jeffersons

Umass medical

Quinnipiac

Rosalind Franklin

University of Arizona College of Medicine Tuscon

Indiana

Virginia Commonwealth University School of Medicine

University of Minnesota Medical Scho

WVU

Creighton

mark-ER

Your GPA is great, EC and a publications are a 'cherry on top'. If you are indeed a URM, you should be good to go (despite probably for you) disappointing MCAT score. I think your list looks good, and perhaps a few more 'reach' places could be considered. Some 'reach' places to consider, based on your current geographic preferences may be Pitt, Case Western, Medical college of Wisconsin. Perhaps instead of some in-state schools like Indiana, Penn state and Vermont (look carefully at admission rates for in-state vs. out of state candidates). Good luck.


How bad is institutional action due to marijuana use?

I started undergraduate early, taking summer classes. I received a disciplinary probation for being found with marijuana and upon further search a couple alprazolam pills that were not mine, alcohol, and a fake id. This will not result in a criminal record, however, it has resulted in institutional action. I have full scholarships to a couple universities still standing for the fall. There is a chance my current school will expunge the record once the sanctions have been completed, yet I still may be compelled to report it when I apply to medical school. Should I withdraw from my current university and go to another, less prestigious, university to avoid this record being seen in my medical application?

mark-ER

It sounds like you are trying to avoid owning up to this. Do you consider this mistake, indiscretion and do you plan to continue to use? Regardless, I would stick with your current undergrad, try to learn from this experience and be open about it. It won't trail you forever -- once in medical school, it won't be on your record and won't follow you into residency and/or practice. Unless you happen to be a repeat offender... so treat it as lesson learned; do not repeat the mistake, own up to it and you should be fine. It won't prevent you from getting into medical school, but it will probably make things a bit harder. So overcompensate -- rock your GPA and MCAT.


Do I still have a chance to get into Med School?

I'm concerned about the introductory science classes I took as pre reqs for pre med last semester. I became ill in the middle of the semester due to my health condition. I managed to pass with C’s but I wanted to know when applying for med school, is there an option to explain why I got C’s in these classes? Currently I’m considered a Junior in my college, however I have not completed majority of the classes within my major. I just wanted to be aware before applying whenever I have completed my degree.

tantacles

There are options to explain C's on your application. That being said, many students are accepted to medical school with C's on their transcripts. It is important, however, that you do well in all of your other courses to improve your GPA so that you can be taken seriously as an applicant.


Adding Admissions Recruiters to your LinkedIn?? Professional or not?

Is it professional to send an invitation to connect to an admissions recruiter (LinkedIn, currently applying to this school)? We have exchanged several emails, and I think that there is a good communication link. This school is very important to me, and I definitely want to seem invested, especially since the recruiter has been emailing me, but I do not want to seem unprofessional or too eager (to the point when it is annoying).

What should I do?

mark-ER

Not while you are applying; maybe after you are done with the process.

(too many people are too loose with their facebook/linked in contacts, personal point of view but one that is shared by many professionals).


Past alcohol criminal history

Hi there,

When I was 15 I was charged with possession of alcohol. I went through a diversion program and my record was sealed. When I was 18 I again was arrested for underaged consumption of alcohol. Both of my records have been sealed but I'm wondering how/if I would say this on an application. I know that doesn't look good because it's like I didn't learn my lesson. Do I even need to talk about the one that happened when I was 15? Both records don't show up on BCI background checks but I know they run an FBI background check too. Any help is appreciated. Thanks.

tantacles

You must report these infractions when you apply. That being said, many medical schools are lenient with regard to charges related to alcohol, so the best way to approach this is to be honest and state that you have learned something from the citations. Several years will have passed since these events, so medical schools may be willing to overlook them if you are honest.


MCAT 508, GPA 3.83, SCRIBE 2500 HRS, COMM-LETTER, C-SERV. 300 HRS, RESEARCH, FLORIDA-RES, WHAT ARE MY CHANCES AND WHERE

tantacles

Given your MCAT and GPA, you have a good chance of getting interviews at mid-tier schools and possibly some mid-high tier schools. I would apply to all schools in Florida and use the MSAR to select schools that fit your MCAT and GPA combinations at around the median.


Multiple MCAT - (491<498<500<511), WAMC?

I did something stupid, and took MCAT in 2015-2016 several times before finishing all the prerequisites (no idea what I was thinking). Only the last MCAT 511 was taking after organic chem and biochem was finished.

My GPA is both 3.65 (over 250 credits, - 2 bachelor's and master's), I have military service (was a medical technician), other stats:

research: 500 hours (NIH) paid medical employment (not military): 2300 Learning Assistant (chemistry) : 480 hours Leadership (was manager of supply in medical unit, military): 1320 hours, clinical direct patient care experience: 250 hours.

What I my chances? I know that multiple MCAT score is bad, but how bad? do you think it will automatically ruin my chances of being accepted?

FYI: already submitted all secondaries (within the first two days of getting them, in the secondaries mentioned multiple MCAT as an experience that taught me to not rush, plan, etc.)

Drexel Katz Kimmel EVMS (in state) GW GT Howard MCW MUSC U vermont Tulane U. COnn U Mass UVA U Wisconsin VA tech Creighton Wake Forest WVU.

Should I add any more schools to increase my chances? (VA in state)

Thank you

mark-ER

Your chances are NOT ruined. Taking MCAT early is an uncommon mistake, but not completely unheard of. However taking it so many times is very unusual, and you should preemptively do something to address it. Pre-meds are often very ambitious driven individuals who like to test themselves. Often these individuals are used to taking exams early, think PSAT or SAT while still in elementary school or early high school. The problem with MCAT as opposed to SAT is that MCAT is much more content based than intelligence/critical thinking test. So no wonder not having formally had the material it's easy to do poorly. You did not realize it -- decent enough explanation. Put a similar respectful and succinct explanation of your situation as an addition to your application and I think some (though probably not all) schools will disregard your early attempts.

If you already submitted primaries and secondaries, please followup with the schools you have applied to and send them a formal letter to clarify this situation. Ask kindly for it to be included following your personal statement.

In terms of schools/school choice, it is a decent list, but perhaps short. With your stats you should consider adding 4-5 additional schools and if you are not getting a solid response (several interview offers by late October), then another 5-10 schools on top of that, perhaps even some osteopathic. You are a Virginia resident, so clearly you focused your applications to in-state VA, and generally mid-lower prestige tier private schools on the east coast (good choice). But I also see some schools in the midwest, particularly wisconsin -- do you have some personal ties to the area? If so, adding several more schools in the midwest might be helpful (perhaps both some 'reach' schools like Northwestern/Uchicago, Mayo but also some lower tier schools like Rosalind Franklin, Loyola, Rush, Michigan state). Oh and I think you are missing Virginia Commonwealth? Good luck.


Low Overall & Science GPA, can a high MCAT score allow my application to be considered?

I graduated from a CUNY institution (located in NYC) last year with a 3.05 overall GPA last year. My science GPA would be close to a 2.9 and my other GPA would be 3.6+. I am a female, first generation college student and I am Asian.

Since then I have been studying for the MCAT in the hopes that my application will be considered for an MD program?

I have participated in a number of programs to expose myself to the field of medicine. I have shadowed physicians, volunteered countless hours and organized community health fairs. I have also conducted research and I am a coauthor on a publication.

mark-ER

At this point, despite reasonable overall GPA -- presumably post-postBacc, you are bluntly a marginal candidate. A high (and I mean HIGH) MCAT score may ameliorate things a bit and help you to reach some lower tier allopathic and/or (many) osteopathic schools. MCAT is by far the biggest determinant, but do you have time/room for more classwork? Can you load it up with (relatively easy) science classes to improve your sGPA? Your overall GPA isn't too bad, but unless you had straight Bs in all prereqs, those Cs and below in science classes are going to hurt you.

As a note of encouragement: at this early stage of your career it's difficult NOT to have a tunnel vision, but there are a ton of rewarding careers within medicine that are as rewarding professionally -- DPM, masters of genetic counseling, optometry, PharmD... That being said, you *can* still reach for medicine, with some HARD study. Good luck.


HELP! Should I drop Pre-med?

Hello, I am about to be a 3rd-year student at FSU. My major is Women Studies and I am on the pre-med track. (As if that's not obvious) I struggled my two years in college and failed pre-calculus. I am fighting to get my GPA to a 3.0 and honestly I do not know if pre-med is for me. I have not done any research or volunteer yet. I'm involved on campus but not with organizations that are medically based. My peers are super involved, done research and volunteer work. And I have done nothing. I question if I even wanna be a Doctor cause it seems like so much work and my regular and science GPA is not that good. I have no motivation whatsoever. And my advisors do not help what so ever. I honestly feel like their as confused as I am. I've looked at other career fields and the only one of interest to me is Marketing. But I'm conflicted. This is my 3rd year... There is no more time for games. HELP?

tantacles

If you are truly motivated to become a physician, it is possible that you could redeem yourself. However, if you have no motivation and don't feel that you will be able to do well, I would consider another career option. Most medical schools will not even offer an interview with a 3.0 GPA, so you would need to get all A's if you wanted to have a chance at acceptance.


Where could I get accepted

Hello all,

So i am currently applying and would like to know where my best chance of getting accepted is. I am a Florida resident with a sGPA of 3.1 and overall GPA of 3.6 with a pretty low MCAT score. I will be retaking it this August but wont get scores back till September. I already applied to NOVA since it is in state and have already submitted my secondary.

Also does anyone have a list of the schools with master bridge programs that guarantee an interview? I know LECOM does but am not sure of any others.

Thanks!

tantacles

There is no current comprehensive list of programs that offer a guaranteed interview, but programs' websites usually give a good idea of whether or not they offer this as a feature of their program. If that does not work, you can also contact the admissions department with an e-mail address. If you do not feel comfortable using your real name, you can create an e-mail address with a false name.


Submit applications now or wait for MCAT retake?

So I took my first MCAT and got a 502, 127/123/126/126, I know its not good and I'm going to retake it on august 4th, I know its a little late but thats beside the fact. What I am wondering is if I should submit my applications (already verified) to the schools I want to apply to now with that bad MCAT score and just have on the app that I will be taking it again, or will the school just look at that MCAT and say no. Also, I'm applying both MD and DO, Florida resident, Hispanic, Male, I know the score is really bad for MD but I figure I still gotta try, at least my in state schools (i.e. FSU).

Mr.Smile12

It always depends on the schools where you are applying, and no single school behaves like another. If the MCAT is the only issue (assuming your GPA's are good), then I would encourage you to begin the application process now and let them know of your retake.


Secondary Applications

I was hoping to get some advice on when to submit secondary applications. I recently took the MCAT will not get my score back for another few weeks. I submitted the primary application the first week it opened and now I am receiving invitations for the secondary applications. Would you recommend waiting to submit these until I get my score back or submitting them anyway? Honestly I feel that my score could go either way. Any advice is appreciated.

Pathdocmd

Do not wait. Medical schools know if you are waiting on scores, so your secondary will be put on hold until the scores arrive. That how is it done at my school, so I can't speak for every school. If you do well, things will not be delayed. If you do poorly, then you are out the secondary fees.


Chances of getting accepted

Im a junior at UCF I'm majoring in pre-clinical health sciences and I am unsure about my future and am looking for guidance. I have 57 credits and a 3.33gpa after a rough 2 years. I have finally got myself together and I realize if I get a 4.0 for the next two years I will only have a 3.66 gpa which isn't a great gpa for med school acceptance. I volunteer at a free clinic, I start volunteering at a children's hospital in august, I start volunteering at a human milk lab in august, I am working towards getting my CNA license, and I plan on studying abroad (South America) shadowing doctors for a week during spring break and a month in the summer. What are my chances of getting into med school? Why mcat score should I get and what other options do I have if med school isn't in my range?

Pathdocmd

The upward trend is great and most schools will notice that. You really can't predict chances until you have your MCAT scores are available. Also, there are many more factors like state residency, the type of school you are applying to, etc.


Best Choice for Gap Year

1. Do an MBS program (SMP) to increase my GPA, get better letters of recommendation, and more research programs. I could also work on campus for convenience and study time.

2. Work at a pain management clinic where I’d be working alongside an anesthesiologist, shadowing him at BCM, and who I would be publishing papers with, and maybe take some classes on the side.

I can only do one option, and I'm torn between the two! Thanks!

mark-ER

I think either choice is OK... if your GPA is at least in the OK range (say 3.5-3.7), and you have a good MCAT score or you know yourself enough to think you can stick to an structured, independent study schedule, then go for option #2. My thoughts -- MBS costs $ (tuition, room/board), whereas for #2 you would be making a salary and potentially have time to get a few publications under your belt. If you do MBS, you will not have time to take classes, study for MCAT and do any meaningful research, but if research is "baked into" your job, then you are better off. But again, it's not a slam dunk decision, more to do with your pre-existing stats and how well you do in a structured vs. less structured study environment.


Already submitted primary but received low mcat score...what now

I was told by multiple people to submit my primary application as soon as possible even though my mcat date was not until June, meaning my score came out in July. So I listened and my primary has been completed and approved. I received my score and it is not competitive enough to get in and its unlikely I will get an interview even. I considered retaking it in late August but am afraid I will still not have enough time to get my score where I want it (I want to take a prep class before the next one). I have accepted that I will not be getting in this cycle with my current score and want to reapply next cycle. That being said, do I still complete the secondary application so the school knows I am interested and sees I turned it in when I apply next cycle? Or do I not send it in and hope it doesn't hurt my chances for next cycle with that school. I feel like I have been left in the dark and have received little or false information from my advisors so any feedback would be greatly appreciated.

Mr.Smile12

I guess if you have decided to delay your application until you get a better MCAT score, I don't see why you should submit secondary applications and fees. I would check with admissions staff and even some medical students you may know from the "inside" who have gone through the process and can help encourage you if it's appropriate. I don't think most medical schools pay that much attention if you withdrew from the process before you sent them a secondary application (and fee).


Advice for College- Switching Majors to Bio?

Hi,

I am currently going into my second year of university (undergrad). I am thinking about switching majors from music to bio for med school. I know med schools don't really care what your major is but my science background isn't too strong so I think it may help me prep for the MCAT.

Is this a smart move on my part? I really don't want to hurt my GPA but at the same time, I feel like I need these science courses to prep properly for my MCAT, rather than just doing the book courses.

mark-ER

You are correct in stating that the major does not matter; but you DO have to have a certain number of science courses, and specific type (math, chemistry, biology, physics) -- make sure you understand the prereqs you need before you apply and the background courses you would need before taking the MCAT. A good place to start:

https://students-residents.aamc.org/applying-medical-school/applying-medical-school-process/

For many people, there is just so much overlap between biology and med school prerequisites that they do a second major. If you are a rising 2nd year (i.e. just completed your first year of college), you will be fine, but if you are a rising 3rd year you will have a hard time to squeeze all these in 4 years, taking MCAT and applying w/o taking a year off.


Advice needed

I am currently a Biochemistry major with a 3.85 overall GPA (idk what my science GPA is but it is higher than that I'd say 3.88). I have junior standing as of right now. What I have so far is:

-50 hours of shadowing a doctor with 38 of those being an ER doctor and the other 12 with an urology surgeon. -53 hours of volunteering at a food bank and the local hospice combined -I was an Undergraduate TA last semester for a Cell biology and genetics class -Currently am in a research lab in which I started this Summer 2018. I should get published (3rd author?) by the beginning of next year from what I have been told. I put a lot of time in the lab. - Pre-med club member

Unrelated that might affect: I am fluent in Portuguese and English and I also am moderately good in Spanish. I am also a permanent resident card holder in the USA (I have lived here since I was 9, I am now 20). The school I go to is Washington State University.

I was wondering in what areas would be good for me to work on in order to increase my chances of being able to go to med school?

mark-ER

You are on a very solid footing. You have enough volunteer and medical shadowing, you dipped your toes in research, your grades are very good. Your main (if not only) concern is doing the best you possibly can on MCAT. The fantastic MCAT score vs. run-of-the-mill above average one can mean a difference between one or a couple of matriculation options versus several admits and perhaps even a merit-based scholarship.

A very, very minor point -- if really liked research and you intend to pursue grant-funded research in the future (perhaps as a physician-scientist) a US citizenship would be an asset. If you are already a permanent resident with a green card (that how I understand your status), you are all-set from a medical school admissions standpoint, but for some scholarships and many federally-funded grants it is beneficial if not required to have US citizenship.

Mr.Smile12

Congratulations on your hard work. So that said, what outreach efforts have you made with the school at Washington State? Have you met their students and know their AMSA officers or other club leaders? Do you know what they do when it comes to clinical and community service experience while in medical school? You can always shadow more physicians in different areas and volunteer more, but you want to keep your focus on the missions of the schools you want to get into and why. Think of your recommendations and doing what you can to have the strongest champions for you.


Advice needed

I am currently a Biochemistry major with a 3.85 overall GPA (idk what my science GPA is but it is higher than that I'd say 3.88). I have junior standing as of right now. What I have so far is:

-50 hours of shadowing a doctor with 38 of those being an ER doctor and the other 12 with an urology surgeon. -53 hours of volunteering at a food bank and the local hospice combined -I was an Undergraduate TA last semester for a Cell biology and genetics class -Currently am in a research lab in which I started this Summer 2018. I should get published (3rd author?) by the beginning of next year from what I have been told. I put a lot of time in the lab. - Pre-med club member

Unrelated that might affect: I am fluent in Portuguese and English and I also am moderately good in Spanish. I am also a permanent resident card holder in the USA (I have lived here since I was 9, I am now 20). The school I go to is Washington State University.

I was wondering in what areas would be good for me to work on in order to increase my chances of being able to go to med school?

mark-ER

You are on a very solid footing. You have enough volunteer and medical shadowing, you dipped your toes in research, your grades are very good. Your main (if not only) concern is doing the best you possibly can on MCAT. The fantastic MCAT score vs. run-of-the-mill above average one can mean a difference between one or a couple of matriculation options versus several admits and perhaps even a merit-based scholarship.

A very, very minor point -- if really liked research and you intend to pursue grant-funded research in the future (perhaps as a physician-scientist) a US citizenship would be an asset. If you are already a permanent resident with a green card (that how I understand your status), you are all-set from a medical school admissions standpoint, but for some scholarships and many federally-funded grants it is beneficial if not required to have US citizenship.

Mr.Smile12

Congratulations on your hard work. So that said, what outreach efforts have you made with the school at Washington State? Have you met their students and know their AMSA officers or other club leaders? Do you know what they do when it comes to clinical and community service experience while in medical school? You can always shadow more physicians in different areas and volunteer more, but you want to keep your focus on the missions of the schools you want to get into and why. Think of your recommendations and doing what you can to have the strongest champions for you.


Med schools asking about college finances?

In both the primary AMCAS application and a couple of my secondaries, med schools are asking how I paid for college. Why are they so interested in this and how do they use this information? Is it frowned upon to already have a lot of debt? Thanks!

Mr.Smile12

While I cannot speak for admissions committees' interest in this question, there is general recognition that higher education is costly, and medical education is going to add to that. Some of your peers may already have found a lot of grants or scholarships to offset their undergraduate debts, including those doing ROTC or perhaps a job taken at the same time as taking classes. Though ability to pay is not a factor in offering admission, it may help when it comes to "closing the deal" if you wind up being eligible for special financial aid opportunities.


Is it too late or how can I get on the right track when I am entering my fourth year of undergrad and the only aspect of med school i'm ready for is my GPA? (Little to no EC)

I am currently entering my fourth year of undergraduate studies in Biological Sciences and I have a GPA of 3.95. I was not sure what I wanted to do throughout university and just made sure I had good grades so that wouldn't hold me back when I graduated. Looking into careers I want to pursue I realized that all the summers and the last three years of my undergrad have been wasted as I currently have little to no ECs (I volunteered short-term with children with disabilities, church) and no experience doing research or abroad. I am considering extending my degree to 5 years so I have a little more time to gain experience during undergrad but I am worried it is too late. What can I do when there isn't much of this summer left and I feel like I've already wasted so much time?

mark-ER

First of, you are still very much on track. Most admission officers do not mind a candidate who 'discovers' medicine as a career option later on, but with a stellar academic record. So onwards to 4th year -- as you point out, you are very unlikely to be able to apply this cycle, but maybe not with the main reason you list (lack of ECs): with a biology major you might (but maybe not) have all the prerequisites, but you still have to prepare for and take the MCAT, getting letters, writing a PS and putting together an application is no joke, and remember medical school has rolling admissions. So you are behind the 8-ball here.

That being said, if you have enough courses to graduate (and please re-check into your medical school prereqs, adjusting your next 2 semester coursework accordingly) you should be able to graduate in 4 years. No real reason to shell out more $ for tuition/room & board. You can then take a year doing what YOU want to do, that you find interesting and useful. Several options: MPH (stats and community health), medical mission abroad, work as a tech in a research lab, post-bacc NIH program, or most likely a combination of several things. I would really, really recommend the last option -- perhaps you'd get the research bug and getting the experience of living in/around DC is fantastic. Whatever you decide to do for "5th" year, the key is to start now -- many masters or year-off program applications are due summer or fall of the preceeding year, some may require a GRE. Good luck.


Emory Secondary Prompt

I'm unsure about the "Briefly describe your health-related experiences. Be sure to include important experiences that are in your AMCAS application, as well as any recent experiences. (200 Words)" prompt.

My understanding is that it is only about clinical experiences, such as volunteering and shadowing, but my father says that I should include my basic research as well? Can someone shine some light on this dilemma?

Mr.Smile12

I would ask Emory about the prompt on whether research experiences are appropriate. Be prepared to talk in brief about your specific research if it is bench-related, field-related, or survey-related.


Deferring

I applied to medical school this year but I recently obtained a research position through Harvard Medical School and Brigham and Women's Hospital. It's possible that this position might be a two-year commitment. I am wondering how likely it would be that medical schools would accept this as a reason for deferment of attendance if I was accepted.

Mr.Smile12

Each school has a policy on deferrals, but many will not grant you that deferral, especially if you were aware of this opportunity ahead of time.


High ranked IMG vs Low Ranked US MD

Hey, I'm a non-US citizen looking at Neurology as a career choice. I currently have a 3.4 GPA (not taken the MCAT yet) and am in a position to get into a top-ranked medical school in my home country in Ireland. Given that I am not aiming for a very desirable residency program (Neurology) what do you all think about picking that school and being an IMG vs struggling to get into a low-ranked US school with my credentials?? I know that being a US MD is >>>>>> than a non-citizen IMG, but will the caliber of the medical schools cross things out when being evaluated by residencies? Or is even an unranked US medical program better than say an Oxford or UCL MBBS for purposes of matching into a less than desirable speciality?

I know there is always the chance that I will change my mind and then be stuck with a speciality where only 19% would do it again if given a chance....so yeah not sure mates

mark-ER

Let me reverse your question a little bit -- you have NOT yet taken MCAT and your GPA is pretty low. On top of that, you are NOT a US citizen. Your chances of getting into US medical school are not great, actually it's quite an uphill battle. I knew quite a few non-US citizens on a student visa (i.e. not a citizen and not a permanent legal citizen, i.e. w/o a green card), and even from top-notch undergrad, with top notch GPA and MCAT getting into medical school as a non-US citizen/permanent resident was still a MAJOR struggle. Only a handful of non-citizens each year get admitted w/o legal status/permanent residency, either DACA or special circumstances (MSTP/researchers, uber-talented individuals).

On the flip side -- I think the plan to matriculate in a well-known, western medical school, esp. if it's trinity college Dublin, is a great one. Tuition cost = low, chances to match into US low-competitive program like neurology is great, with the caveat (big if) that you do well on USMLE step 1 and do well in medical school. You are also not 'stuck' with neurology -- you are probably out of luck for the very tip-top competitive specialties (think derm or combined surg-plastics), but other than that it depends on how well you do on USMLE and med school (sorry to repeat myself, but that is the key if you pursue this path). In fact, you have a major leg-up on most IMGs, because: (1) native English speaker (2) you have lived or have connections in the US (??). Good luck.


Is it better to Void MCAT or get a low score then retake later?

I'm planning on applying to the 2019 cycle. My MCAT is in a week and I haven't been scoring a 500 or better on my practice exams. I was wondering, should I take the MCAT and Void it or should I see what I get and just retake it next year?

mark-ER

Short answer -- if you do not feel ready, do not take the exam. It sounds like you are not ready and not comfortable taking it yet. Have you had all of the prerequisite courses? Perhaps a dedicated study course is in order.


Undergrad questions about GPA.

I am currently attending a community college then transferring to a university, my freshman year first semester I got a D in psychology and contemporary mathematics the classes were not even hard I was just being lazy and unfocused which I regret. I got all A's & B's my second semester & took 3 summer classes which I received A's in to help boost my GPA. I'm about to start my sophomore year and was wondering should I retake the psychology & math course at some point maybe the upcoming summer? And would it replace the D's in my GPA if I did or just average the new grade I receive with the D? I am a biology major planning to attend med school so I want my GPA almost perfect. Thanks!

Mr.Smile12

You should check with your academic advisor about retaking those classes as they may affect your progress towards your degree. Your institution may have a grade replacement policy for calculating GPA's, but most of the application services do NOT replace grades when calculating GPA.


3.7 sGPA 3.8 cGPA MCAT 505 Competitive for MD/DO School(s)?

Hello!

I am going into my senior year and would like to gauge how competitive I am for the schools below. Any advice/insight is incredibly appreciated!

Degree: M.S. Biology with Distinction w/ minor in Community Health Sciences Caucasian/Not economically disadvantaged or first generation college

sGPA 3.7 cGPA 3.8 MCAT 505

  • No Clinical Employment*

Clinical Volunteering Medical Volunteer/Shadowing (2 weeks - Guatemala) 80 hours Medical Interpreter (Spanish) 200 hours (4 years) Hospital Volunteer (Surgical Services) 300 hours (2 years) Peer Health Educator/STD Clinic 200 hours (4 years) Trauma Intervention Volunteer 150 hours (3 years) Non-Clinical Volunteering Boys' and Girls' Club 500 hours (6 years) -Governor's Service Award, Presidential Service Award, Summer Camp Counselor for underprivileged, organized summer camp for teen leaders, volunteer services and teen internship (nonpaid)

Extracurricular Army National Guard | Motor Transportation Operator (8000 hours) - Basic Training Honor Graduate - Advanced Individual Training Distinguished Honor Graduate - E-5 Sergeant (non-commissioned officer) - Combat Life Saver - Master Driver - Air Assault Qualified - Basic Leaders Course - Soldier of Year Board Winner - Other army schools (HAZMAT, Ammo Handler, Resiliency Trainer, etc.)

Leadership Basic Training - Was selected by Cadre to be Platoon Guide where I lead 70 soldiers on a daily basis through first aid, marksmanship, physical training and land navigation. Advanced Individual Training - Was selected by Cadre to be Platoon Guide for 60 soldiers on a daily basis through physical training and specialized transportation training. Army Non-Commissioned Officer - Operated as a leader on several convoys. - Delivered several briefings to 50+ soldiers. Student Government Health Sciences Senator - Represented 6,400 college students - Chair of the Committee on Public Affairs - Sat on Academics, Budget and Finance, and Oversight Committees. Other: Anatomy Dissection Leader and Genetics Teaching Assistant

Research Ecology Based 500 hours Undergraduate Research Award Winner Senior Thesis

Misc. Toast Masters Skiing Work part time as a restaurant busser Fluent in Spanish I also received the Army Health Professions Scholarship Program Award (full-4 Year tuition) - should I indicate this on AMCAS?

Applying to many DO schools, some State MD schools, and a military medical school (Uniformed Services University of Health Sciences).

Again any help is greatly appreciated!

mark-ER

Your activities and leadership are outstanding, though your grades and MCAT are on the left edge of the bell-curve .. so I think your approach is right. Apply broadly (30-35 schools): many DO schools, state MD and lower-mid tier private schools that take people from out of state. A few 'flyers' or reach schools are OK, though I would not send in 5 applications to Stanford/Harvard/Hopkins/Penn (but many one or a few to mid-upper tier places like Columbia, Brown or Vanderbilt?) Other questions you have to answer for yourself -- do you have a desire to do military further? Do you specifically aim for DO vs. allopathic? If not (i.e. you are more motivated to get into the best possible medical school you want), then apply broadly and early and control what you can control -- good support letters, great personal statement, finalize things as early as possible -- remember medical schools have rolling admissions.


What are my chances for getting into Med School?

Hello! I have a few concerns regarding my chances of gettings in to medical school. I have taken Organic Chem. 1 three times, and I just passed it with a C. I haven't been the best student when it comes to Organic, because I had a high school mindset of not having to study for things, so I just gave up the first 2 times. However, I have matured, and I really am starting to see what it takes to become a doctor. I spoke to my advisor about the issue, and I told him that I am in a period of academic growth and development. I know I have messed up, but I know I can turn my direction back into the right one. I feel that my chances are horrible, but I am trying to retain some type of hope towards medical school.

Pathdocmd

One class will not make or break you; HOWEVER, you will have to shine in all your other science classes and do well on your MCAT.


Should I submit secondaries before I have all of my recommendation letters?

I set a goal to return my secondaries a week after I received them. I am on track to submit 3 of my secondaries now. However, I am still waiting on one letter of recommendation. The writer said the letter would be submitted by the end of next week. Should I wait for this letter to be submitted or should I submit the completed secondaries now?

mark-ER

Yes, submit secondaries now. Be polite, but persistent with the faculty writing the letter... it sounds like he/she is willing to do it, but might have a lot on his/her plate. Remember, medical school has rolling admissions, so sooner you finish your secondaries = better.


Foreign transcript

Hi, I’ve a Bachelors in Nursing degree from out of country and have done a masters and a certificate course in the US in Anatomical and translational sciences. Currently prep for the MCAT. Went through the application and found that foreign transcripts are waived. Please I’m in need of advice or betterstill are they schools that can deal with foreign transcripts or do I️ need to take a post bac course to augment my application.Also I’ve been a nurse and currently working in the US for past 5yrs I’ll appreciate your advice. Thanks

Pathdocmd

If you are applying to a US MD school (AMCAS), the your non-US/Canada grades will not count. Only your masters and certificate will count in the GPA calculation. I can't speak for DO or Texas schools.


civil infraction

I am applying for medical school 2018. I know I received civil infraction when I was 19 years old when I try to get into bar with my friend's ID. The application does not ask anything about civil infraction. Do I need to disclose anyway or wait till they ask? Not sure what I need to do. Anyone who can help me answer this question? thank you

mark-ER

Was it a misdemeanor, or worse yet a felony? If it was not classified as such, and given the rather innocuous nature of the infraction, probably just keep it to yourself. You can check your own criminal record (several good ways to do so online, landlord do it all the time)...


Should I retake the MCAT?

Although I consistently score a 509-510 on practices tests, I bombed the actual MCAT and scored at a 505. I have a 4.0 GPA, 7 peer-reviewed publications, over 200 shadowing hours, participated in activities like student government, and have lots of volunteer hours. I worry that if I opt to retake it now (which would mean me taking it in August), that my application wouldn't be considered until it is too late as I am applying to schools with rolling admission. Is it worth the risk of not improving or should I just apply this application cycle with the score I have now?

mark-ER

The short answer is "yes", if you think you can do significantly better (at least +2-3 points), then retake it. I think your grades and activities will compensate to some extent, but a better MCAT score will boost your chances of multiple acceptances and perhaps better financial aid offer (? scholarship). If you retake it in August, you can still update your application and it won't be too late. There is always a risk of a lower score, but weighing the upside & downside, it seems like value play in your situation would be to retest.


What are my chances? Could you guys recommend me a school list?

I'm a rising senior. I have 100 hours clinical volunteering, 2260 hours labwork (with leadership within lab and employment over the summers, and an honors capstone senior year), interim leadership for a club, and decent ECs. 160+ hours of varied shadowing.

3.94 cGPA, 3.90 sGPA

508 MCAT, committee letter not certain (my MCAT is one point below their minimum to write a letter so there is a possibility I won't get a committee letter, although I have a valid reason for a score this low. I was nauseous and threw up during my MCAT, my score was 3-4 points below what it was on every single practice test I had. I will be petitioning my committee to allow me to still get an interview).

Submitted my primary yesterday, and I have documentation of throwing up during my MCAT.

[More details on this: My father said it’s my last chance to take the MCAT and if I voided the exam this time, they wouldn’t let me take it again. I don’t want to tell med schools about this, though. He says I can retake it if I want now but this relied on me not voiding the exam I took. I will apply to more schools once I can make sure I can get a committee letter. I voided it once before due to food poisoning, so I knew my whole family would be devastated if I voided again because I was vomiting. Now they’re just sad I got a 508... I did my best while absurdly lightheaded, lol... If I can’t get a committee letter, I will ask a pre-med advisor what steps I should take (Retake MCAT? Apply to schools in the South? Etc). I really want to go to a school in the Northeast so as long as that’s possible with a 508, we’ll see. I really would love advice from you people online :). I’m happy about my GPA but have no idea how far I can go with this MCAT score.]

I've applied to

Robert Wood Johnson (Rutgers) Penn state NYMS

Do I have a shot at these schools, and which ones I should add?

mark-ER

My take: with your stats, extracurriulars, etc. you probably can get in somewhere with a 508 MCAT, though it is an uphill battle. My recommendation would be to submit early, submit to 30-35 medical schools (including a handful of DO schools), and still plan to retake MCAT. My best guess with your MCAT score as it stands: you will probably get in 1-2 schools, you will probably get on a couple additional waiting lists and if you retake the MCAT and get a better score, you should get off waiting lists and possibly get better financial aid offers (maybe even a scholarship, if you ROCK it, above 520).

Sorry to hear about the unfortunate event @ prior MCAT attempt; you can pursue remediation action, but it is just difficult issue to prove/ameliorate, easiest course of action is just to retake).  In terms of schools, there is close to enough in the NE, but you will probably want to expand into SE and midwest.  Apply to all in-state schools, all lower and mid tier private schools that take significant % out of state in your preferred geographic area and as previously stated several DO schools.  For now, worry about things you CAN control.  Study hard for retake, prepare for interviews, get great letters and personal statement.  Good luck.

Should I give up on my dream of becoming a doctor?

My main concern is whether I still have a chance at going to medical school? My transcript consist of A's and one B, but the catch is that I have a D in biology(retook it and got a C , it was a summer course which was much harder), C- in Physics. But I plan on retaking it next summer. I am currently a 1st term junior undergrad doing pre-med. I work as a trauma technician and as a volunteer EMT on my free time. I plan getting my GPA to 3.5-3.7 and take my mcat after graduation. I just want to know whether having those low grades in such an important courses will ruin my chances of going to medical school? I would appreciate any kind of advice, thank you.

mark-ER

Not necessarily, though as you point out it might take you longer -- an extra post-bacc year to push up your grades, studying extra, extra hard for MCAT, and even then you might have to be happy with either low tier MD or DO school, or maybe even going international. That being said, as long as you persist and work hard at it, there is still a chance. See how next year goes, if you can truly dedicate yourself and get all As in your science classes and get your GPA to 3.5 or above, then definitely still pursue it. If it winds up that however hard you try, you cannot get higher than a B, B- in science classes, then perhaps it's time to consider plan B (maybe still in medicine, perhaps PA school, optometry, podiatry, clinical genetics/genetic counseling there's lots of options).


DYI Post Bacc or SMP - Please advise

Hi Just graduated with a non-science major but I did the pre reqs for med school. My final GPA is cGPA 3.2, sGPA 3.0 [Went to a top 5 Liberal Arts School]. I do not have many shadowing hours or clinical volunteering hours. I did one summer of bench research, I worked with social justice programs for almost 2 years and worked in college library.

I am taking my MCAT this August.

Please advise which option is better:

1) Do science upper level or non-attempted science courses in local Community college or state school to build up the sGPA? If this is an option how many hours do I have to attempt [assuming I get staright As in all of them] to boost my sGPA from 3.0 to 3.5 in one year?

If I choose this option I can volunteer in hospitals, shadow doctors and build up my application. Assuming I can get a new cSGPA of 3.5, [my cGPA will also increase from 3.2 to 3.5+] and get MCAT over 510, will I be a reasonable candidate for application cycle starting June 2019?

2) I have applied to SMPs [very late application 2 weeks ago, so not sure if I will get in]. If I do get in to SMPs [Loyola, Tufts], will this be a better option that option 1? If I go this route and do well [get GPA over 3.6 in post bacc], then I can apply for the Jun 2020 application cycle [provided my MCAT scores are above 210].

3) Or is it better to go to Carribean?

Please advise, thanks in advance for your help.


Mark - Thanks for your advise. Appreciate it.

mark-ER

To address your questions directly -- this will be a hard road. Your GPA is pretty low (even if from top-notch school) and you are trying to cram in a year of science and to study for the MCAT all at the same time. I would NOT advise you to try to take the MCAT before you finish the vast majority of your coursework -- that is a recipe for a low score. I also would NOT advise you to do community college -- you already have a degree from a liberal art university, so CC is a step down. Some schools don't 'count' community college science classes, some discount the grade when figuring it into GPA. As you know SMPs are both fairly expensive and fairly selective. There is a third way -- is there a post-bacc (not necessarily SMP) at your local college(s)? It does not have to be a big-name college, just so it is a 4 year degree-granting institution. I suggest you do it thru a post-bacc, because of the camraderie and advice/support such a program will provide. SMPs are typically better, but as discussed more expensive and more selective... ultimately how/if you make it into medical school is up to YOU and (typically) cream will rise to the top.


Retaking Classes

Currently a senior with a 3.83 GPA and was curious on how medical schools view retaking classes. I retook GenChem 1 and my grade went from a B to an A. Also retook OChem 1 and went from a B+ to an A. Helped my GPA out but I heard conflicting info about if med school average those two attempts and make their own GPA or if they just take my schools GPA. DO medical schools look down on retaking classes?

mark-ER

Retaking to up your grades from a B to an A is OK, but I question why you decided to do it (perfectionist? want an absolute rock-solid handle on the foundational material? Make sure you have an answer to that question). Your GPA is well within norm (maybe even above average) for accepted medical school candidates. So do well on MCAT and take more advanced coursework, don't worry about retaking anything unless you get a C, C- or below.


LOW MCAT (501) and LOW SCIENCE GPA (3.3)- any hope for MD or DO school? Best chances?

Hi,

Here are my VERY low stats........

MCAT- 501 (took it one time this May) GPA- 3.4 Science GPA- 3.3

I am trying to apply for MD and DO schools this summer so these are the stats I will be submitting.

Any advice on MD or DO schools that accept these low stats? What are my best chance schools? Is there any hope for me? :(

mark-ER

Of course there is hope, but for now unless you have MAJOR extenuating circumstances (by major think - "I was an olympic athelete, and my training got in the way of studying for MCAT"), unfortunately you need to re-take the MCAT, if you hope to get any interview offers. That goes for both DO and MD schools. You need to aim for at least 507 and preferably much higher to compensate for your low GPA too. Talk to your pre-health adviser, get some more feedback from the forums here on SDN, but my strong recommendation is to take another year after getting your degree, do something to strengthen your application (research tech year, volunteer), while taking one or two science classes of interest to you (genetics, statistics) that would be useful for your future career, and studying HARD for the MCAT (think 4-5 hours a day for 3 months). Your road is not unusual (I am mentoring 3 undergrads in a similar position), and YOU can do it. But if you don't think you can handle that intense of a year, perhaps it's time to think of a 'plan B' for your career, maybe still in the medical field, but maybe not medical school.


UWO vs UTSC for pre med

Heyyyy! So as my name indicates, I have a really hard time making decisions & & I'm currently facing a severe dilemma lol. I am enrolled in the neuroscience specialist co-op program right now at UTSC and so far for first year, my marks have been around 76% (except for chem whoops). I am really afraid that my grades aren't gonna enable me to med (or rather, I KNOW that my grades aren't on par with med school standards). I was thinking of either switching my program and doing a double major in something like neuroscience & health studies (but then that might cause problems with getting all my prerequisites in-- which btw, as a sub question, what are the necessary prerequisites? & can one fulfill the prerequisites in a post-bacc program if not fulfilled during undergrad-- reason I ask is because what if someone had done an English major and then had a later epiphany in life that they wanted to do medicine but never actually took the necessary classes?) to avoid the really intense upper year science courses at UTSC that could cause my GPA to drop even more. That, or I was thinking of transferring over to Western University for their neuroscience program as well. However, before making any decision, I wanted some advice (preferably from someone who's taking courses at UTSC -- not UTSG (because I've found their system aslo runs a bit different, for example first year chem in UTSG was orgo in half a credit which isn't done here at UTSC until second year under a whole credit (two courses for the year) , AND WESTERN Uni. I currently live downtown with family and I love being in the city, so that's something I would miss if I were to go to Western. I also find the city helpful in that there's a lot of job opportunities as well as events/clubs/just life really to participate in whereas London (from what I hear) is mainly composed of the university & not much else? In terms of commute, it's been alright as some of my courses have been web-optioned & it's easier to catch up without physically attending some of the classes. I also am afraid of Western as the reputation I hear of it is that there's a lot of distractions. etc. Is this true? Regardless, my biggest priority are my grades & I was wondering if I am right in switching over to Western for attaining higher grades? Now, I will admit-- first year I have been dealing with personal issues, including bipolar disorder which has made it difficult for me at times to concentrate during exam season-- I find myself contemplating life more than studying & I've also skipped numerous classes during manic periods and assumed I could teach myself the whole curriculum a week before the exam. However, I AM working on fixing those issues with my therapist/psychiatrist. Do you think if I were to alter my studying habits and attend/watch every class, I could possible attain grades that are good enough for medical school? Or should I switch universities completely???? I can't determine the exact issue at hand. Thanks a bunch in advance! Xxx

Mr.Smile12

If you are dealing with mental health issues, that should be among your highest priorities. Get yourself the help you need at your home institution, and find out what would happen if you changed institutions and thus campus providers. It's important to note that if you change schools, you need to have a plan to transfer the counseling notes to the other institution.

Your prehealth advisors at both institutions should be able to give you a sense of the critical questions you need to ask before you make a decision to transfer out. Academic rigor does count a bit with medical school admissions as shown by grades and solid recommendations from faculty who know you well. You also have to consider what work you would have to do if you transfer out of your home institution, take classes at other institutions and try to transfer them into your transcript (because there is usually a process for this), and support from peers. Why don't you ask the advisors at the institutions you mention to find other students who can answer your questions?


Advice Needed on what I should do

Hello,

I am a 23 year old newly college graduate. During my undergrad year, I went to a Community College and graduated with a 3.7 gpa overall. When I transferred to my University, I left home and lived on campus. My first semester as a biology major my gpa was a 3.3 and it was my first time getting a C grade. As my semesters continued, I began to get ill almost every week. I was in a car accident when I was younger that flipped my life upside down, causing me to lose my spleen, as well as become very depressed and somewhat limited with mobility. Once I was on my own and got older my depression and health became worse and ultimately started to show in my school work. I started to do horrible and even had to drop classes due to being ill. I finished school with a 2.1 overall average, a 2.2 in biology, a 2.6 in my psychology minor and a 3.6 in my African studies minor. My plan is to go to medical school and become a doctor. But, I know with my grades my chances are not very high unless I bring them up.

I am taking a gap year, maybe even longer, but I am going back and forth with taking classes to go to nursing school and then attempting to go to medical school because I do feel very discouraged. I know my options are very slim but is there anything I can do to bring up my grades? I haven’t taken my MCATs yet but my goal is to take them in a year. I don’t know how long it would take for me to bring my grades up but I don’t want to give up on my dream of becoming a surgeon.

I know I’m not unintelligent but as time progressed my health and depression just won. But I have started taking care of myself and now I am ready to make a change.

Any suggestions help!

Mr.Smile12

Whatever you wind up doing, make sure you connect with the health professions advisor at the four-year institution/postbac program you decide to enroll in. You also should talk to a few admissions officers way in advance and make connections with practitioners as you shadow them. Nursing school is totally different from medical school and has its own pressures that you may not appreciate, especially if you want to be a surgeon. You should consider surgery assistants as a career (or other physician assistants, anesthesiology assistants, etc.) and see if that is something you want to do. Being a surgeon is really stressful, and anything you can do to avoid relapsing in your mental health problems is going to help you in the long run serve as many patients as you can.


Extracurr activities, volunteer experience: How many hrs is competitive for matriculation to medical school ? Advise on specific activities?

Volunteered in long term care hospital once (2-4hrs)/week for over a year Volunteered with a community organization helping food insecurity seasonal approx. 4 hrs/week for 16-20 weeks over 2 years Volunteered with a seasonal sporting competition as a safety team member 9hrs/day for approx a week per year over 3 years Volunteered tutoring high school students 2-3 hrs/ week. seasonal for 10 weeks over a year

How many hours is considered competitive by med school admission committees? And what do I do now?

I have enjoyed what I have done so far, but, I may want to explore more areas. Is that bad in the eyes of med school admission committees? Should I continue with the same activities? Some people pick one or 2 activities and continue for years; others like myself like a wide range of activities and enjoy new experiences and challenges.

Please advise.

mark-ER

Just to be clear/blunt: volunteer activities are pretty low on the list. Clear-cut stats (your major, GPA, MCAT score, the university you attended) are just much easier to evaluate and according to many academic articles at least somewhat predictive of your success in medical school & beyond. So the key is having done "something" to expose you to clinical medicine/clinical practice/sick patients. The minimum is ~50-60 hours on a regular basis -- 1-2hours a week for a year or so. You have way more than that, and more is better, but with rapidly declining "returns". Bottom line is: do what you want to do & do activities YOU enjoy. If I had to give more specific advice -- for someone with your extracurriculars the one thing that may be missing is exposure to research. Experience asking a scientific question, collecting & analyzing data, interpreting literature and maybe writing an article is of great value in medicine & science in general. Good Luck.


Chances of Acdceptance and Avg MCAT for potential target schools?

Am I a good applicant for this cycle with a 3.9cGPA 3.7+sGPA and a 508 MCAT (129/125/127/127). Having trouble determining the avg MCAT for my target schools URM: Puerto Rican and Peruvian

Shadowing

-Anesthesiologist: 55 hours
-Atlantis Project (shadowing doctors abroad in spain, utilizing my knowledge of Spanish): 72 hours split into 4 departments (Anesthesiology, ENT, Urology, Cardiology), received leadership scholarship for this program
-Orthopedic Surgeon: 50 hours
Research
-Patient Contact: Assisted people with their workout routines to test power vs. strength training in the Elderly, focusing on the effects to their ADLs. Over 84 hours in the one semester the lab ran
-Wet Lab: Working with Schwann cells and their ability to produce myelin to help with the repair of damage axons following a spinal cord injury (3 semesters: 400+ hours)
-Summer Undergraduate Research program @ Sloan Kettering (IP): Testing the ability of 77 different compounds to reduce cell viability in Diffuse Intrinsic Pontine Gliomas (poster presentation at the end) (So far: 130 hours, anticipated by the end: 380-400 hours)


Extracurricular
-Working within Biology Advising Office (3 semesters)
-Workshop leader for Intro biology courses (2 semesters)
-Tutor for student athletes (3 semesters)
-Tour guide for the university (House Leader for my specific group: entire organization split into 4 houses)
-Secretary for my fraternity (highest GPA on campus)
-Biochemistry tutor/proctor for professor (wrote LOR
Volunteering
- Note Taker for the office of disabilities (receive 60 hours of volunteering per class per semester: accumulated 300 hours)
-Blood Bank volunteering when I return from the atlantis project --> 30 hours
- Local Rescue Mission that I have been a part of since 2007. Volunteering for holiday dinners, was able to accumulate majority of the hours while in middle and high school, but had to slow down in college because I am not always home for the holidays (150: 100 before college, 50 since)
LOC
- All professors thought highly of me and spoke with them almost every class. No hesitation when I asked for LOR. A or A+ in their classes
GPA trend
-hard to say because I had a 4.0 my freshman year. Dipped down slighty when I took orgo 1 (B+) but got an A in orgo 2. Multiple semesters with a 4.0. Non science GPA is a 4.0. Mainly got non-As (few) in labs that ended up having significantly hard finals that had content we didn't learn in the class associated with it.
State of Residence: NJ

mark-ER

Your combination of grades, classes, extracurriculars make you a very solid candidate. You can likely get into medical school as is. But if your goal is to get into one of the top 10-20, elite schools, you should try to retake MCAT and score at least in the 512-515 range. Based on the experience and drive you seem to exhibit thus far, and some SERIOUS prep, that score should be achievable. Talk to your school's pre-health adviser and consider the pros and cons of applying now (with updated MCAT to come later) versus waiting to retake the MCAT. Besides having the choice of schools/potentially a higher-ranked medical school, testing up to a higher MCAT may mean better financial aid package (esp. scholarships). So think hard about it, but strongly consider retaking the exam.


Low Stats - Advice Needed!

Hi there,

I need advice about my GPA. I have terrible stats right now, with a 2.8 science GPA and 3.1 overall cummulative GPA. I am Biology major and intend to take the MCAT next summer, and have already started to take a course that my college offers for the biochemistry section of the MCAT. I have reflected and thought about my current situation for Medical School and I am getting extremely worried. With the appilcation process more competitive now than ever before, I dont even think there is a .1% chance I'll get in anywhere. Should I take a Gap year and take a couple of more classes to fix my GPA? Or should I just power through, try to make straight A's and hope for someone to look at my applicaion? I also have just finished my sophomore year of college, and I am so worried that even if I do decently well, say 3.6+, There still would be doubt if the ADCOMS look at my application - I have other extra curriculars ( research, leadership ,shadowing/volunteering), I felt like I needed to make up for what my GPA lacked, but without decent stats I'm afraid it would be all for nothing. I just have no idea what to do and this dream about becomig a doctor is slipping away. Your input and opinion on what would be the right course of action ( what you think) would be more than enough.

Thanks.

Mr.Smile12

You should have already gone to your prehealth advisor. If you are a sophomore, you are quite a bit early to discuss gap years especially with that GPA as it stands. You should do a lot of networking to be sure you know why you want to be a doctor in the first place. But no doubt your grades are very important and you need to change some of your habits and priorities if you're going to discipline yourself to be ready for the rigors of medical school and training.


Discouraged/depressed after mcat scores

I got my MCAT scores back yesterday, and i’m devastated to have gotten a 506. I took 4 practice exams, and scored above a 508 on all of them so I don’t know what happened. I have a 3.71 GPA, am a medical scribe, have done biomedica research at a hospital, and computational molecular biophysics research at my university. I have traveled the country presenting my research and am the president of a volunteer ACT tutoring organization in addition to significant other volunteering. Do you think I still have a chance to get into an in state school at least (i am from kansas). Beyond that, how do I forgive myself for this mistake and move on? I feel like such a failure, and like I dont deserve anything good after this.

mark-ER

With your MCAT score and GPA, and unless you have serious extenuating circumstances, you will have a very hard time with applications. You can try and I would submit multiple (25+ applications to MD schools, and many, if not all, DO schools). But my strong recommendation would be to unfortunately fight the war on 2 fronts -- redouble your efforts and retake MCAT as soon as possible, try to score 510 or above. If your scores come out in September-October, then release new score to schools you are applying to, and depending on how many interview offers you got, perhaps expand your application window. Good luck.


What to do after this?

I finished freshman year with an okay GPA (3.7) at a school knowing for its low GPAs, and I was happy with it (this included about 4-5 science classes, including Orgo 1 and both bio 1 and 2). I am pretty good at the sciences, but I wanted to study abroad junior year, so I decided to take physics 1 and 2 (calc based) at my home university this summer. However, I turned out to have really bad luck, and (while I'm not trying to make excuses), I mean I had a 100 on the last day of class and the final was just 10 MCQs and 50% of my grade, and I failed it so I ended up with a C (an A in the lab component but that doesn't affect the overall grade).

I don't know how to come up from that, because the logical answer would be to get an A in physics 2, but it's taught by the same professor, and while I know I could try harder, I don't think an A is possible at all! (or maybe that's the fear talking, idk... but seems risky to take a chance, since I thought I was going to get an A in physics 1  like 3 days ago because that's what I got on all other tests) 

Now I have two options - 1- I was wondering if I could take the 2nd physics class of the algebra based sequence. but idk how bad that looks? I mean I'll def get an A (bc a better teacher and an easier course overall), but does that look like a really bad idea? I mean med schools don't require calc-based physics courses anyways so why should I stay in one if it's killing my GPA. Ideas? 2- Go back to my college and take physics 2 there. This is also going to be a little tricky, as it will probably mean I can't do my minor in chemistry because I don't want to be taking 3 lab courses at the same time (I'm a bio major too, so I take like one bio course and one chem course each semester). This is also not going to give me an A (from what I know about the physics courses at my college) but there's no chance of a C or anything. But it ruins so many of my planned out classes and it's just very frustrating but seems like the best bet...?

any advice would be appreciated! I'm really upset because I think overall I'm a very strong candidate and would have become a really strong applicant with a high GPA and a high MCAT score (which is usually something my liberal arts college boasts about). I don't know what to do anymore.

Mr.Smile12

You should always check with your prehealth advisor when it comes to course selection. I'm not sure if your major required you to take calculus-based physics, but you should check if algebra-based physics would be satisfactory to your degree requirements. Otherwise, most medical schools won't care.


Drug screening requirement for incoming students at AUC

I am aware that drug screening takes place prior to, and during, clinical medicine/rotations (years 2-4). Does AUC require a drug screening during the first two years of medical sciences/upon matriculation? Just want to know the facts.

Mr.Smile12

AUC should be able to give you that information directly. I'm sure that you probably have to be tested or get a background check before starting clinic rotations as that is pretty standard protocol nowadays.


U.S. medical school pre-reqs in Europe?

I am a recent graduate with a bachelors degree from an accredited U.S. university. While there, I’ve completed some of my medical school pre-req but still need a couple (I.e. organic chemistry and physics). There is a strong possibility that I will be traveling to Europe (as a military spouse), but still want to pursue medicine. So, I was wondering would I be able to complete the pre-reqs abroad and have them be recognized by U.S. medical schools?

mark-ER

The short answer is 'yes', for many European universities, US schools will accept your classes and grades in lieu of course requirements. Couple of things to consider: make sure the classes are in English and the university is accredited by a body recognized in the USA. If you know where you will be stationed and what possible universities are in the area, you can even gently approach the AdCom for your in-state medical school to confirm (preferably in writing). The disadvantage is you might not be used to the way courses are structured and graded, plus there is considerable logistical issues with getting transcripts and figuring out grades/GPA esp. when combined with American coursework; advantages are cultural enrichment and (probably) very low and/or military-subsidized tuition.

Besides enrolling in classes at a local university, there is always an option of doing online courses, and many reputable universities (Drexler and New Hampshire come to mind) offer such programs, or if you are close to a major metropolitan area, a lot of US colleges/universities have study-abroad programs that offer credits for that university, which is again based in the USA... with US professors, grading, etc. The downside is likely expense and many of these away courses are in language/history/sociology not in sciences. Good luck.


What not to write in med school apps - geography?

Do med schools consider where their students want to practice medicine in the admissions process?

My school adviser told me to make sure I focus on wanting to practice in the US since I talked a lot about my international experience in my PS, so I'm aware of that. But I also read an off-hand comment on SDN about how med schools like their students to practice in the same general area after they graduate. I've been drafting some of my secondaries and focusing a lot on my home state, which is well known for its medical inequality, and how I'd want to eventually go back and work on these issues as a physician. The only problem is that most of the schools I'm applying to are across the nation from where I live, so after reading that comment I'm worried that admissions committees would be put-off by my forceful dedication to leaving the area I'm applying to. Should I change my essays to talk about dedication to patient-centered care (which I'm also passionate about) or something else?

Thanks for any advice!

Mr.Smile12

When picking schools to apply to, you have to read the mission statements to see if they are focused on providing service to the local community and the activities they organized are focused to that degree. If you are trying to apply across the nation, then you should be very smart in reading these mission statements, curricular activities, and state residency composition to help you select your preferred schools. Don't try to err to being more generic (patient-centered care... every medical school should be teaching this by now) if that is not what you want to do. Otherwise, most schools I don't think really have a strong preference about where you want to practice. Many of them know that students probably have a strong desire to return to their communities where they and their families were established ultimately.


Rejected an MD acceptance

In the 2017-2018 cycle, I applied to 24 schools, of which I interviewed at 2 and was accepted at 1. The acceptance came from USUHS (the military medical school). After a lot of deliberating, I decided to decline the offer (further explanation below). I am in the process of reapplying, however I began reading that schools to which I am applying this year may know that I was previously accepted and thereafter declined the offer to an MD program. Can anyone (ideally people on admissions committees/with other experience) confirm or deny this?

Additionally, because I am already in the process of applying, would it be reasonable to get out ahead of the story and make it abundantly clear in whatever way I can why I chose to decline the offer? This would have to be in the secondaries, seeing as I already submitted my primary.

Decision to reject: First and foremost, my decision came as a result of seeing two of my siblings (who are current officers in the military) become frustrated with the bureaucracy of the military (long story there, but basically one was a victim in a car accident and the process of reinstitution into his unit was extremely tedious and time-consuming). Another reason is that I became aware that my ability to pursue some specialties may be limited based on what the military match has to offer (USUHS graduates apply to a different match cycle rather than the citizen match). For example, I may be unable to pursue ENT as a specialty if the military does not need any more ENTs and thus does not create spots for postgraduate training. Additionally, the turbulent political climate dissuaded me.

About me: Graduated in 2017 with B.S. in biology from my state university. GPA 3.83. MCAT 516. Some college athletics. Some research experience. ~400 volunteer hours. Currently working as medical scribe and tutor. Child of MD. ORM.

Mr.Smile12

I'll admit that takes some courage to turn down an offer, and you will have to disclose that in your reapplication. I would talk with your prehealth advisor about how you should try to address this in your application. The problem is that you weren't that successful getting 1 of 24 schools you applied to accept you, so there's a lot more that you have to overcome.


Chances of Acceptance to American Medical School

Originally from NJ. Graduate of Wash U with Drama major/Bio minor, 3.4 GPA, 3.07 science GPA (with upward trend), and 499 MCAT. Spent two years scribing, doing clinical research, and taking upper level bio courses, in which maintained 4.0 as non degree student and now with 3.42 overall GPA and 3.25 or 3.28 science GPA. Applied to MD (20 schools) and DO (8 schools) and did not get in, with the exception of one interview from DO school. Re-took MCAT and scored 512 and now looking to reapply. Also have many hours of shadowing and volunteering. Want to know my chances of getting into an MD school and DO school? What would be a good list of MD schools to apply to?

Mr.Smile12

I can't quantify your chances, but I think you would get a bit more attention with the better MCAT result. It still depends on where you have an in-state residency advantage, but I would start there and apply regionally.

mark-ER

I second the other expert. Strictly numerically you are a borderline candidate, though your determination to improve your grades, do research/get clinical medicine exposure and improved MCAT speak well for you. If you think you can do better on the MCAT (at least 515), I also agree that retaking the test for a third time may be your best option. If you think this is the best you can do +/- 5 points, then apply this cycle and see how it goes. A list of 25 MD schools, focusing on all schools where you have a reasonable shot -- all in-state school, lower-tier private schools and schools where you have strong personal links to (grew up in the area, have family there) & take substantial % out of state, are the place to start. Several (8-10 DO) schools are a good idea too, as is a strong 'plan B' in case you don't get in anywhere. Sorry, I wish I could be more encouraging ... maximize what you can control (great support letters, great personal statement, practice/practice/practice your interview skills) and be on time with your application -- remember medical schools have rolling admissions. Good luck.


Choosing a Major fro Pre-Med with incoming Dual Enrollment credits HELP

Hello, I am going to a University this fall and am bringing in over 62 credits from dual enrollment. I really want to knock out the rest of my prerequisite courses for medical school by the spring of 2019 and take my MCAT that summer. However, the problem I am running into is choosing a major that would be "not too difficult" and allow me to give the MCAT my all. So my question is should I extend my University time to 3 years instead of 2? and if so should I go for a more challenging degree. Or should I stick with 2 years and possibly rush my MCAT and extracurriculars? I mean if I can get my MCAT and everything finished in 2 years (even if it is difficult) why shouldn't I.

mark-ER

Big question: why are you in such a rush? 2 years vs. 2.5-3 years is pretty trivial, and would allow you to get a bit more well-rounded college experience. If you are concerned about costs (appropriate) and/or are just a very focused, single minded individual (that's good too), then with your credits it sounds like you could put your head down, get a biology major and get thru college in 2 years, taking MCAT next summer. But that's a pretty intense goal, and a goal, which I strongly believe with the advantage of hindsight, you might eventually regret. If you are indeed that very goal-focused, single-minded individual even stretching things out a bit to 2.5 instead of 2 years, would give you added perspective and perhaps allow to take useful/enriching classes to your future career like molecular genetics, biochemistry, with an additional half year working as a tech in a lab on a project (perhaps present at a national conference or even publish).

Lastly, if your goal is getting into medical school, you should not worry about choosing "easy" major, but one that you have an interest in, that will enrich you as a person and give you insight into your patient's lives and difficult questions/situations you will run into in your own life. So venture a bit further afield; take some history, linguistics, philosophy, music... it might be your last chance to do so formally in an educational setting.


Should I retake the MCAT if I already sent in application?

I took the MCAT in May and just received the score back and I got a 502 (124 C/P, 126 CARs, 127 B, 125 P/S). This is disappointing because I was used to scoring 510 on practice exams, but I was extremely nervous that day and knew that I didn't do well on the C/P section so it threw me off for the rest of the exam. I submitted my primary application to only one school and it was processed already.

Should I retake the MCAT at the end of July and still be able to apply this cycle? If so, should I wait to add more schools to AMCAS until I get my scores back late August?

The rest of my application is as follows: I go to a top 10 public university with a 3.8 cumulative GPA, 3.68 science. I have solid letters of rec from 4 professors and a research professor, 2 full years as a research assistant while being first author on 2 conference abstracts, 150+ hours of clinical volunteer experience, 2.5 years of being a Chemistry TA, and 30+ hours of shadowing. I am willing to take a gap year if needed.

Mr.Smile12

If you know you can focus your energy on preparing for an MCAT retake, I would try to see about retaking the exam again, especially if you feel nerves got you. I don' know what other universities you may be looking at, especially ones close to you where you may have a state residency advantage, but I would consider expanding your net to include any such schools before your retake your exam. I don't think with your GPA that you would need to take a gap year, unless you're trying to really shoot for high-reputation high-metrics schools.


How can I best prepare my self for pre med courses as an incoming college freshman?

Hello, I am an incoming Pre Med Student as I will be a freshman this fall. Although I am majoring in Biochemistry and Molecular Biology, I do not have a good background in science. My Biology Honors teacher freshman year went on maternity leave one month into the school year and never returned, our substitute was a computer science teacher that volunteered. We hardly covered any material that year and were all just given completion grades. However, in Chemistry, I had the highest average in my class the entire year and I truly love the subject. I transferred schools midway through high school. My new school didn't allow me to take AP courses in Biology and Chemistry because of a weird policy regarding taking college-level courses. So heading into one of the most rigorous tacts of study in the sciences, I was wondering if I should try and self-study Biology this summer before attending college to make up for my lack of knowledge in the subject. Or do entry-level college bio courses build that foundation? Thank you!

mark-ER

Overall, I agree with the sentiment in your last sentence -- that's what entry level bio class is for. That being said, if you feel your HS preparation was very sub-par and you feel extra motivated, you can read a bit at night (1-2 hours couple of times a week) for the summer, you will get a bit of a flavor for college level classes. The book I most recommend, that's basic but well written with online modules is "Molecular Biology of the Cell" by Alberts. Now, if you are a super-duper-overachiever, you can even look for a free, online course (such as those offered by MIT), that uses this specific book to keep yourself on the schedule.

I would strongly recommend the middle-of-the-road approach of just setting yourself up on a schedule, and reading a chapter a week from the book I recommended above -- you should be able to find it in a library. Make sure you take some time in the summer to relax & recharge, too. I remember I worked 2 jobs in the summer between HS and college (60+ hours a week) and was quite tired by the end of the summer! Don't repeat my mistake, of being tired not enjoying freshman orientation as much as I should and having a hard time starting up.


Should I take a gap year before applying to medical school?

I am currently in the summer between my second and third years of undergraduate studies at a public university, majoring in Biology with a minor in Philosophy. Currently, I have a 3.95 cumulative GPA and a 3.96 cumulative Science/Math GPA that does include a year of upper-level coursework. Regarding my extra-curricular activities and experiences, I have completed approximately 200 hours of volunteering in under-served areas near my university, abroad, and inner-city neighborhoods around the state. This volunteer experience includes clinical and non-clinical work. I've additionally accrued over 600 hours of experience as a medical scribe in the Emergency Department, though I have no other clinical exposure. I also was elected as President of a Biological honor society chapter, participated in club sports, and will be a TA for a General Chemistry course in the upcoming academic year. I have not completed summer research yet, but I plan to do so next year; however, my Organic Chemistry course's laboratory component included a semester of collaborative research as part of an ongoing project by the professor.

Based on the information presented here, and assuming I continue to gain volunteer hours and research experience, is it reasonable to continue my plan of applying traditionally? I would take the MCAT in late May/ Early June 2019.

I apologize for the length and specificity, as my academic adviser recently retired and I have not been assigned a new one. Thank you for any input!

mark-ER

With minor caveats**, you will be fine, applying in a standard cycle w/o taking an additional year off. Interesting major-minor combo, lots of activities, top-notch GPA... if your MCAT score is in line with your other accomplishments, you will be in very good shape to matriculate into medical school, likely with multiple acceptances.

Caveats: you will need to have more research experience if you want to apply MD/PhD, or potentially more work/enterpreneurial experience if you want to go MD/MBA route.


Should I retake the MCAT?

I am in the process of applying to med school right now. I have a 3.90 overall GPA and a 3.97 BCPM GPA. Plenty of volunteering, shadowing, some leadership, research, etc.

I just got my MCAT score back today and I got a 512. This is extremely disappointing as I had been scoring 515-520 on the AAMC full length tests. It was basically just the CARS section that I did bad in (125) that pulled me down so far. Anyways, should I retake? This is probably the only thing that is holding me back. I have no desire to go to a top 20 school, and I really just want to go to my state school, which has a median MCAT score of 511.

mark-ER

You will be fine in terms of admissions. With a broad enough application pool (20-30 schools), focused on schools that are more likely to accept you (in-state, ties to mentors and family/geography), you ought to get at least one matriculation offer (likely multiple).

Yes, it may be disappointing to receive MCAT score lower than you expected; if you really want to (in part to prove yourself, in part to torture yourself), you can retake it again. If so, I would encourage you to do so later in the application process once you already have a few interview invites in hand, just in case something goes completely off.


Microbiology and Immunology together

I am considering on taking Microbiology (5 credit) , Immunology Lec (3 credit), Immunology lab (1 credit), and a 3 credit history class this fall 2018. I am wondering if it is a goof idea to take Immunology with Microbiology? Any advice? I would like to get an A in Microbiology, but I don't want to have too heavy of a load.

Your help will be gladly appreciated.

mark-ER

Do-able. Immunology and microbiology (somewhat) complement each other. They are both interesting courses and with history on the non-science side you have a bit of balance. This should not be considered a 'heavy' class load for somebody who is trying to get into a medical school (** I am assuming you are a rising junior, not a freshman with AP credits only; if you are a freshman that is a bit foolhardy).


How can I apply for Post-Baccalaureate Course for MD?

I am looking for some suggestions. I am living in Chicago. I want to apply for post-baccalaureate course (Pre-Med). I am a foreign pharmacist and I obtained my whole education outside of the United States. I have science background. Can someone guide me?

Mr.Smile12

There is an online directory of postbaccalaureate programs organized by the AAMC which you should use. Contact the program officers for those programs that are best suited for your situation.


College major

Hello,

I'm an incoming Freshman at UC Davis. My major is currently Biological sciences. I'm considering changing it to Human Development or Psychology. My senior year of high school, I really enjoyed studying AP psychology. My main concern is that when I'm applying to med school, will it be better to have a bio science or a psychology major? I've been told that majors do not really matter but I want to take the path that will prepare me for med school.

mark-ER

College major is less important than doing well in school (high GPA), enjoying what you do (taking classes you like) and getting a well-rounded education that will prepare you well for medicine, but life in general, too -- plenty of people change their mind about doing medical school in the process, so you want to keep in mind plan B (and C and D). That being said, if you do decide to change away from biology/physical sciences, you still want to take challenging science courses to fulfill medical school requirement, and then some. Make sure you connect well with faculty in your major, so you can get strong letters of evaluation and opportunities to interact with faculty/graduate students academically (get involved in a research project or two). Good luck.


MD or DO? And Post-bac or not?

I’m 27 years old and a mom, so I’m definitely a non-traditional applicant. I’ve been really struggling with the decision of what path to commit to, and I would love to get a bit of input.

The basics: I got my degree in biochemistry/cell biology with a 3.54 gpa. I did retake one class though, so if both grades get counted then it’s really a 3.48 (bummer). I got a masters in biochem with another meh gpa of 3.57. I have about 4 years of lab research experience and a couple publications where I’m 4th/5th author.

I can get good letters from professors I TAed for and from my most recent job (from a physician specifically). Aside from those relationships though, I don’t really have any undergrad professors to ask for a LOR.

I’ve always thought about being a doctor. I got distracted by the PhD research route and it took me a long time to realize that basic science research isn’t my strength. It also took a lot of time and attention away from getting more A’s honestly. Now I’m sure I want to be a doctor, but I’m torn on what path to take.

I haven’t taken the mcat yet but I’m confident in my ability to do great on it.

If my interest is in holistic, integrative medicine and I most likely want to do internal medicine, is it best to go for DO or MD? I’m not really interested in OMM (although I wouldn’t mind learning it either). I AM concerned about DO’s not being regarded as highly as MD’s in some settings, and about not having the best chances at working in a good hospital at the end of all this. I’m afraid to think “eh, it’ll be fine” like I did with the PhD route, and then realize I’m in the wrong place (again).

Is my gpa good enough to try for some DO schools, like Western University in Pomona? Should I do a post bac for a decent chance at DO and/or MD schools? If I do a post bac and I would like to finish it in a year (since I’m not working and I’m not getting any younger), what are good options in California? So far UCSD extension seems like a great choice but I might be missing some other solid options.

Really any guidance on any aspect of this (md, do, post-bac, mom applicants) would be awesome.

Renee_MD

Hi there,

Thanks for your question. I think the best way to for you to answer this question is to spend some time with an osteopathic physician. I recommend setting up a time to shadow an osteopathic physician and see for yourself if this is the type of medicine you would like to practice. Also, if you choose to apply to DO schools, most of them require you to have some experience with osteopathy so this will help in that aspect as well. In my own opinion, it sounds like osteopathy may be the best option for you. You will be more competitive at DO schools given your GPAs. And since you are interested in more holistic, integrative medicine then the DO philosophy would fit your preferences more. One other thing to consider is that in 2020, all the DO and MD residencies will be combined so you will receive the same post graduate training as MDs, and when you are in your practice, then you will be practicing essentially the same as MDs. Finally, you can check out all good post-bacc options by looking at the AAMC post bacc search engines: https://apps.aamc.org/postbac/#/index This is a great place to start to see which programs would be the best fit for you!

Good luck!

Dr. Marinelli


Chances of getting into med school low GPA

Hi, I'm 23 and graduated last year 2017 from University of Colorado

GPA is 2.9 past MCAT was 488, taking it again in July. My GPA tanked because I could only get C's in Genchem, Ochem, Biochem Worked at a hospital in Africa for 3 months Volunteered 200 hrs in undergrad at a hospital Worked as an athletic trainer Latina woman Colorado resident

I was thinking of applying this year to medical schools, but I am not sure if I should since I am retaking my mcat in July. I am not sure if it is too late. Also I am not sure what would be suggested for my GPA. I am currently at a cross roads don't know if to apply for a masters or go back to school to get a better GPA.

Bottom line. Should I apply to medical school this year or wait another year (which would give me 3 gap years) by trying to increase my GPA.

mark-ER

I know it might be painful, but take a year, a few post-bac courses (& do well), study hard-core for MCAT. To have a chance at med school (even DO/osteopathic, and even as a URM), you will likely need to get to at least 3.1 GPA (3.3 would be ideal) and get your MCAT score to as close as possible if not above 500. No, it is not an absolute rule, there are always people on the far left-end of the curve as far as undergraduate stats get in every year, but your chances of getting into medical school (and frankly even getting interviews) right now are very low, <<10%. Also, masters is not going to help your undergrad GPA, though as I found out on SDN forums, there are MPH/posbac programs, where you can get you a useful masters in addition to being able to take undergrad courses & study for MCAT. Talk at length with multiple people who have insight into medical school admission and seek guidance from a pre-medical counselor at your undergrad, but you will have a very hard road ahead to make up for where you stand academically right now. Not impossible, but very, very hard. Good luck.

If you want another opinion, for somebody in a very similar situation, search these expert forums for the following post "Low cGPA (3.0), MCAT 484 chances of USA or CANADIAN Med School?" The individual received similar advice


How can I fix my current situation and get into a Medical School?

I am presently 24 years old and I have been out of college for approximately 18 months. I have worked different jobs and have not went back to school. I am looking to matriculate into any reputable medical school as soon as possible. I am open to Caribbean medical schools that are reputable that would give me the opportunity to start in the Fall or at the worst in the Spring semester. I have had some life events that have resulted in me having a horrible GPA and low MCAT score. My Gpa is a 3.11 and my MCAT is a 479. I don't know what to do at this point but I still would like to become a doctor. I am willing to give it one more chance.

I am open to special masters programs (that will take me and whose deadlines have not yet expired) that will allow me an opportunity to "link" into a medical school without wasting additional time. I am not getting any younger, I would really appreciate it if someone who has been in a similar situation could help me out or guide me in the right direction.

I presently have a job working at an Orthopedic surgeon's office, but it is a low-paying job and not nearly enough to sustain me. If someone could please reach out to me or reply to this thread and help me. I would greatly appreciate it. I really need help, I feel like I am wandering aimlessly and I want to fix the situation that I'm in right now. Please help! (I'm seeking the path of least resistance)

Mr.Smile12

I think that if you're going for the path of least resistance, Caribbean schools are going to be the option you would explore. That said, it's going to be a very dangerous path if you wind up failing medical school entirely, and the numbers you have shown wouldn't give many admissions staff (at least for MD or DO in the US) much confidence. I would do a lot of homework with your options with coursework, MCAT prep, and admissions chances.


Where do I go from here?

I am going into my senior year of college and currently have a 3.5 cGPA and a science GPA of 3.17. My first three semesters I did very well and kept a GPA around a 3.8. However, I began struggling with some mental health issues in the middle of my sophomore year and have performed poorly my last three semesters, leading to the decrease in my GPA. I am planning on retaking my three worst classes ( I got two C's and a D) this year and then completing a graduate program before applying to medical school. In addition, I have extensive research experience and extracurricular activities. My school offers a graduate program in physiology which would be ideal because they have many avenues to pay for it without loans. However, I am concerned this will not be enough and that I need to complete a SMP, which are quite expensive. In my scenario, do I need to complete a SMP or will another graduate program be viewed equally? In addition, should I disclose my struggle with mental health when I apply?

mark-ER

I think masters vs. postbacc (either stand-alone or SMP) are both a reasonable choice, as further discussed below, with the big caveat that masters level courses are NOT included in your undergrad GPA. Unless your MCAT is absolutely stellar (515 or above), and even with that MCAT score your GPA (especially sGPA) is your biggest hurdle to getting into medical school. So, I agree with the approach of re-taking the courses where you got a C or below. Retaking and getting an A or A- would show you can handle the work, and at the same time increase your GPA.

In terms of masters in physiology in your institution vs. posbacc/SMP, I would advocate for postbacc for folks who are non-traditional, have been out of college for some time, do not have pre-med prerequisites in place and/or already have another postgraduate degree (MS, MPH, PhD, MBA, JD, etc). It sounds like you do NOT fall into any of these categories. So perhaps a masters would be the best course of action, especially accounting for cost of education (it sounds like you have an option to get into your local masters programs with minimal additional expenses/ student loans), it would make you potentially more marketable for future jobs (plan B), and again show that you can handle upper level science classes. Lastly, I always think of a "third way", out of the box solution... as a concrete example, University of Michigan has a masters level physiology program, that's geared to help people figure out their career path and potentially apply to professional health degree programs (DPM, MD, DO, DNP, etc). In the evenings (1 course/semester) one could then take an additional science class at a cheaper (& easier) local university, such as Eastern Michigan, Concordia, or similar. It would be a very, very tough year but also a good prep for medical school.


Can I become a Doctor?

I am currently entering my sophomore year in college as a Electrical and Computer Engineering major. Meaning, I'm more of a math major than a science. I love my major and school, I have an academic scholarship that covers my 4 years, I have a 3.7 GPA, but I don't see any bio courses and barely any chem. I am not sure what all the requirements are to get into med school are either. What would be recommended in my situation?

mark-ER

You are only a sophomore, you have time. Indeed, there is a core curriculum required to apply to medical school -- that includes calculus, biology, chemistry, organic chemistry + biochemistry, statistics and physics. You likely already have physics under your belt, same for math and maybe stats. You have at least a year, maybe more to squeeze those other courses into your schedule. You could double major; alternatively, you can do a post-bac or take summer courses, though that's extra time & cost. As you will hear elsewhere, medical schools want students in all majors/concentrations, not just 'vanilla' biology or biochem. I think an individual with strong mathematics or engineering background, good GPA and (most importantly) strong logical problem solving skills and communications would make a great doctor. Good luck.


Can you take the MCAT in high school?

I am interested in applying for a BS/MD program. The school I would like to apply to requires you to take the MCAT. The website said that your MCAT score would be one of the things that they look at during the application process, and as it is a program that high school students are able to apply to, I am taking this to mean that you would have to take the test in high school. How would this be possible?

Edit- I am doing a full diploma IB program, if that would help me.

mark-ER

You could, but I would NOT recommend it. I think you could be OK in chemistry and physics with very good AP classes, but even with AP bio you are way behind in terms of breadth of background (others with 3-10 college biology courses) and not taking O-chem. Plus however you do, that test will stay on your record; yes you can explain it away, but why? But, at least back when I was prepping for MCAT almost 20 years ago there was an opportunity to take a full exam on a trial basis, with an actual score but not recorded and not validated. So if you are really insistent about trying it out, consider that possibility.


Withdrawing from a summer class - multiple “curveballs”

I need some advice. I just finished my junior year of pre-med, and decided to take my physics courses during the summer before my senior year. I am not doing well in the course - my first exam was a 49% and my second a 52%. He drops the lowest exam. I am considering withdrawing so as to avoid a low GPA. I am taking this class a different college than the one that I go to, so my overall GPA isn’t affected. However, when they analyze my scores on my AMCAS application it will show up. I did running start, and then went to a 4 year college. Over the course of these 5 years, I have had 4 withdrawals (two during my running start and two during my 4-year college). All of the classes were repeated with a fairly good result besides one that was not repeated. If I withdrew, I would graduate, but I wouldn’t get my degree until later. Contrarily, if I don’t withdraw, I do not have any time to study for my MCAT. I plan to take the MCAT during January of the next year, and this is my last chunck of time to study.

My college will only accept a C grade or higher, and I don’t know that I can pass it like that based on my trends so far. If I withdraw, I already spoke to the professor and he said that am I allowed to keep going to the classes (which I will do) but I won’t be there for a grade.

My GPA is a 3.689 currently, with a science GPA of 3.61. Will another withdrawal be very bad? What do you all think I should do?

mark-ER

If you strongly think you cannot get above a C (at least a C+), I would consider withdrawing. If it was an upper level physics class, especially graduate-level course, you could consider taking it pass/fail, but it sounds like this is a basic/preReq class. Having trouble in a basic pre-req physics class is not a good sign, and to be blunt 4 Ws (even with relatively good grades on repeats) is also a red flag. And unfortunately, yes your AMCAS application listed GPA will be affected -- no matter when/where you take your undergrad courses, they count toward your GPA. So even if you took an undergrad courses in high school, not in the college you eventually attend, yes these count. Ultimately, MCAT is the 'great equalizer' so if you do very, very well on MCAT, especially in the physical sciences section (assuming those are the courses where you had Ws), that will mitigate things a great deal. But again though not a dealbreaker, multiple (3+) withdrawals are a potentially serious red flag, so be ready for questions about this in your interviews. Bottom line: if you can bring up your grade, that should be your approach first (talk to your professor and/or TA about it & work your butt off).


Please help me with the school list, GPA 3.7, MCAT 511 (128/126/129/128), NYS resident.

GPA 3.7, MCAT 511 (128/126/129/128), average ECs, NYS Resident.

Hi, I’m trying to finalize my list. Plan to apply to 30ish schools. This is a list of 40 schools. Can you please help me narrow the list to 30? Is there any other school that I missed and should be added? Do I need to apply for DOs, too? Really appreciate your input.

1. Icahn 2. Weill Cornell 3. Pitt 4. Emory 5. Albert Einstein 6. Hofstra 7. Brown 8. University of Rochester 9. Ohio State University 10. SUNY Downstate 11. University of Maryland 12. Dartmouth 13. Thomas Jefferson 14. Georgetown 15. UMass 16. Tufts 17. Rutgers 18. Chapel Hill 19. University of Colorado 20. Temple University 21. SUNY Stony Brook 22. SUNY Buffalo 23. Robert Wood Johnson 24. New York Medical College 25. Drexel 26. Albany 27. SUNY Upstate 28. Tulane 29. George Washington 30. Rush 31. Virginia Tech 32. Virginia Commonwealth 33. University of Washington 34. University of Tennessee 35. Wayne State University 36. Chicago Medical School at Rosalind Franklin University 37. Loyola University Chicago Stritch 38. Toledo 39. Michigan State University 40. Howard

Mr.Smile12

This is definitely a lot of schools. What you're not doing is giving yourself a chance to see if you fit with the educational mission of each school. I suggest this article to help you with the process: https://students-residents.aamc.org/applying-medical-school/article/make-sure-you-fit-schools-mission/ .

mark-ER

I wanted to 'second' the other expert opinion. I have seen many applicants with your very solid, if not spectacular stats. Unless there are other major red flags you are not disclosing and/or you have issues with nerves while interviewing, you should get in somewhere. I would also recommend narrowing down the list to about 25 schools. If nothing else, it is a lot to pay upfront. I think the top of your list looks good for your geographic location/in-state status, you could flip some of the schools on the lower end of the spectrum for some mid-upper tier places (perhaps UChicago or Vandy)? Unless you have geographic ties to the area (such as close extended family, grew up there), exclude state schools that do not take too many applicants from out of state (Ohio State, U of Maryland, Michigan State, etc) and again put in a few mid-upper tier places. So bottom line is go down to 25, narrow down your list as recommended by me & the other expert and IF you get tepid response (1-2 interviews only), you can always expand the list later.


LORs with AAMC/AMCAS IDs being sent to AACOMAS

So I already applied to MD schools this cycle but am considering DO schools as a back up (my GPA isn't stellar). I have 9 letter writers that submitted LORs to Interfolio, but 7/9 put my AAMC and AMCAS ID numbers on their letters. I'm now in the process of completing my AACOMAS application but don't know whether it is a *terrible* decision to forward the letters with the IDs to DO schools.

I'm kinda in a sticky situation because some of my profs/physicians took *forever* to compose their letters and I'm sure it'll take another month for them to send a revised version. The AACOMAS application is well into the application cycle too. If I were on DO AdCom's, I'd be salty about reading something with an AMCAS ID on it.. but is it going to really reduce my chances? I'm an above average applicant for DO schools (cGPA 3.6, sGPA 3.5, MCAT 509), so I'm hoping that counts for something.

Bascially... to send or not to send, that is the question.

Mr.Smile12

I'm not sure there's any major disadvantage if you submit your application now. As for giving your ID numbers, your AAMC ID number is what is used to access your MCAT scores (I think), so it is fine if you had listed your AAMC ID. I'm not as familiar with whether you have different ID's for AMCAS vs. AACOMAS, but your AAMC ID should be fine.

That said, I'm not sure DO admissions committee members really have that much of a problem if you included your AMCAS ID. I'm sure many realize that applicants are casting a wide net, and the MD vs. DO stigma is not what it used to be.


Junior in undergrad, will I get into med school with bad grades freshman year

I'm a student at PSU and I'm at a 3.1. I'm certain that I'd like to go to medical school. My freshman year I withdrew from Biology. I also got an F in statistics (due to not spending time on this class and studying for calculus and getting an A). I retook the statistics class and also got an A. In both gen ed chemistries I got a C and retook both and got As. Every other class has been A or B. I suffered from depression my first semester at this school due to a suicide of one of my close friends during my childhood. Should I continue to pursue medicine or is my transcript already way too bad. I've done a lot of volunteer but I'm having some serious doubts.

Mr.Smile12

I'm sorry to hear about your friend. If you haven't actually shadowed physicians, you shouldn't stress out right now about applying to medical school. Make sure you know you're going to enjoy the lifestyle of a physician because it's going to be extremely stressful depending on what you wind up being attracted to in the field. Mental wellness is a big deal, and a lot of medical students and residents are saying that it's very important to keep your perspective and health while you're in school due to the stresses you'll experience. I'd talk with your prehealth advisor, and I'd envision your goal over a number of years beyond your graduation from college so that you are truly ready and mature for professional education.


HELP! Chance, or no? URM!

I've been wanting to type all of my info out for a long time, but finally have the energy and time. I've been majorly sick throughout my undergrad years (can provide medical documentation/proof), if needed. I am a psychology & addiction studies major. I've been wanting to be a doctor ever since I can remember (cliche, right?)

my gpa and mcat are HORRIBLE. I mean compared to my friends who have 4.0 gpas and such. To get offered an interview at my home state med school, you need a 496. My current gpa is 3.1. I do have an upward trend in my grades though. I have 7 years of healthcare work experience, amazing letters of recs, and volunteer hours. One big thing-I am from a reservation, and my state school is highly interested in Native American students and improving rural health care. Do I even try to apply to med school for 2019 or do a post-bacc program? I currently feel school burn-out, so I AM considering a gap year, or two.

I plan on retaking my MCAT, but do NOT see it improving. I think it was the hardest test of my life, and I hate how it determines how you'd do in med school. Any advice? Thoughts? Anything is helpful at this point. I'm smart, but not smart "enough" for med school I feel.

Mr.Smile12

I would talk with the representatives from your state school. Why are they interested in Native American students? What types of programs do they have where students and faculty help other Native Americans with health issues? Why do you think your impact in these problems is best suited if you are a physician and not any other health care professional?

I'm hopeful that you can do well on the MCAT, but know that it's the first of many additional high-stakes exams you will have to take in order to remain a physician. The MCAT is only the hardest test of your life until the next one comes along, and they will for medicine.


How will a dismissed drug charge affect my medical school application?

I got caught with lsd and the charge was dismissed through a first offender diversion program. I plan to be completely honest on my application about what happened and how I have learned from the experience. I am hoping that any adcoms or individuals that have been in a similar situation in the past can give me some insight on how much this hurts my application and if I should give up on medicine entirely.

Mr.Smile12

If you are totally honest in your disclosure, it shouldn't hurt you. You may have to answer a question or two on it in an application or an interview, but as long as you explain the situation and how you remediated, you should be fine.


What goes into my science gpa?

My major is exercise physiology and a few of the courses I have to take include: Exercise Physiology and Anatomy and Physiology 1&2 with labs. I was wondering if these go into my science GPA or not

mark-ER

Short answer -- they should count into sGPA. Longer answer -- I believe they should both cross-list within biology. Biology = science => sGPA.


Caribbean SGU or Post-Bacc SMP?

Here are my stats: 3.1+ cGPA, 2.9+ sGPA, 490+ MCAT (1st time), varied shadowing opps since I was young (ex. trauma, epidem, neuro, gen. surgery, etc.)

St George's Univ (SGU) seems to be one of the best Caribbean medical schools out there and it seems to be getting better...

An admissions counselor from SGU encouraged me to apply with my stats because they thought I would have a decent shot of getting into SGU, so I applied. I applied for SGU specifically because a family friend of mine went to SGU, completed their two year program there, and was able to match in the U.S. in Indiana for a residency program with family medicine. This individual is now finishing up their third year in the residency program in the U.S. and seems to be doing well.

I also applied for a U.S. Post-Bacc SMP program, which I have a good shot of getting into. The SMP program also provides MCAT-prep courses included with the curriculum. I could retake the MCAT at the end of the SMP, try for a higher score, and reapply to U.S. MD programs. This may be too preemptive, but I may reach a point where I have a choice to make: start the MD program right away at SGU or start the SMP program and retake the MCAT later for a higher score.

With a weak GPA (cGPA and sGPA) and low stats in general, I don't want to jump into the MD program too soon and unprepared, despite pressure from my parents to catch up with my HS peers who are already in the MD program. However, I have been reading a lot about the problems that seem to befall U.S. Med Residency programs as of late (not enough residency slots open, increasingly poor outlook for IMGs/FMGs w/ U.S. residency matching, etc) and I want to get into a neurology residency program, based on my past shadowing experience. Caribbean MD schools fail rates are typically abysmal and I don't want to become a statistic if I go to SGU before I'm ready.

The uncertainty of U.S. residency matching landscape in the next 2-5+ years plus the fact that U.S. Residency programs do not view students of Caribbean schools well leaves me concerned.

I know I want to become a physician. Given all this (lengthy) information that I presented, what would be best for me to do? Stick with SMP so I can retake my MCAT or go with MD in SGU? Or is there a third option you would recommend? Thanks.

Mr.Smile12

Among the Caribbean programs, SGU has a solid track record, but I am not sure what's going on with residencies in the future, since there are some significant changes that may make it challenging for most internationally-trained medical graduates. At least with most Caribbean programs, you have multiple entry dates, so you can go when it's convenient for you.

Now whether you should do SGU vs. a postbac, that's up to you. I don't know what your academic habits have been in succeeding well enough in medical school to get into neurology. Do you need resources to do medical student research to give yourself a strong shot?

You have a lot of questions you can ask on whether anyone from SGU made it into neurology, and what clinical rotations in the US in neurology SGU has agreements with. In the end it's your decision with your prehealth advisors and your evaluators (not your parents).


so what exactly *is* a post-bacc?

hello!! After messing around with my degree audit, I realized that if I took out all the pre-med reqs I have left, I can graduate from my university in a year, making me a senior as of right now. woo! buuut I still have eight pre-req classes left and a not too hot science GPA (3.2) (yikes, I know but as a liberal arts major that wasn't sure about medicine until a couple months ago, I'm not too mad about it). There's room for growth and I'd like to improve my GPA overall, so my advisor reccomended I look into post-bacc programs. I've been researching and they seem interesting but I'm still not sure if it's the right choice for me. I know I have a few options but right now I'm debating between finishing up my degree and finding a post bacc program/ taking my pre-reqs at my homeschool as a non-degree seeking student, OR go at a snails pace these next two years and graduate with my peers and with all my reqs done. I've found some interesting programs, like PRIME at Case Western Reserve, but I'm not sure if it's worth that giant price tag. Any information or advice helps

Mr.Smile12

The plain answer to your question in the header can be found online at https://students-residents.aamc.org/content/article/considering-postbaccalaureate-premedical-program/ . I hope you talk with your institution's prehealth advisors to help you with your choice.


Should I apply now or next cycle to better my ECs?

I graduated from my undergrad college in May of 2017. I took a gap year primarily to up my volunteer hours and work as much as possible to make sure the burden of my loans didn't fall upon my parents. As of right now I have everything filled out on my AMCAS except my ECs, and the more I reflect on it the more I feel I am lacking the correct amount of ECs to be considered a competitive applicant.

My ECs are as follows:

~24 hours of shadowing (planning on many more hours to come) ~177 hrs of non clinical volunteering (patient transport volunteer, feeding homeless, proctoring /grading science competition exams, etc). ~ 10 hours of clinical volunteering (participating in hospital emergency epidemic drills, participating in physician ultrasound training courses). ~2.5 years of D2 collegiate rugby ~ 7200 hrs paid nonclinical work

  • currently interviewing for CNA positions so factor in future paid hours of clinical work.

- 0 research - 0 significant leadership roles

Obviously my clinical volunteer hours are largely inferior and need some work. I applied to volunteer with my local hospice center so I should have future hours in that. But I don't want it to come off as shotgunning some hours into my clinical volunteering.

The other factors of my app would be my MCAT and GPA which are a 507 and 3.67c/3.69s respectively.

Should I take the time to apply this cycle or should I hold off and bolster my clinical hours? Any advice at all would be of great assistance.

mark-ER

I think your focus is a bit off -- you have plenty of activities, and a bit of extra volunteer work might help, but not very much (just a smidge, and just at the margins). Plus if you are interviewing for CNA positions, you will get plenty of patient experience so that's good enough to substitute. The bigger concern is your stats --- GPA is respectable, not stellar but OK (esp. if you went to a top-notch college and had a very hard major, think engineering). The bigger issue is the MCAT. 507 is on the left tail of the curve. What would BY FAR improve your application most is to retake the MCAT. Study hard, day in and day out for at least 2-3 months, do whatever you need to do to get at least 509-510 (above 512 would be ideal). You can still apply this cycle and take your chances, but it may be a tough road to plough. If you do plan to apply this cycle, apply to every single allopathic school where you have a chance (all in-state schools, all private schools at the lower end of the spectrum and even a few out of state schools that take substantial out of state applicants, esp. if you have geographic ties to the area). If you are interested in going Osteopathic, apply to all DO schools as well. Also, don't undersell yourself as having no significant leadership roles -- I am sure you did something that can be considered leadership.

Good luck.

Do I stand a chance?

Hello,

So I'm currently having a mental breakdown because I think I majorly screwed up my chances for med school. I graduated with a BA in Biochemistry and BS in Molecular & Cellular Biology. I graduated early, in 3 years instead of 4 (most people advised me to graduate rather than stay.) My cumulative GPA is 3.49 and my BCPM GPA is 3.36. I'm so disappointed and frustrated with myself because I let myself slip in my last semester and missed the cum laude and honors accreditation by dropping 0.01 points in my GPA. I wrote an honors thesis for the honors college at my school, which I thought would be a good addition to my app but now all my work for it is gone. I will be taking a gap year to gain more clinical experience/shadowing.

For my GPA, I feel like it could be attributed to many reasons. The number one main reason is that I should have worked harder. I'm hesitant to say this, but looking back, I think I had a lot of depressive episodes where I just felt demotivated and horrible, and let my grades slip as a result. I don't think the admissions board will accept this, which is understandable, but I truly think this did play a big part. Especially after my low score on the MCAT, I just felt inferior to everyone. Another reason is that my grandma's health has been declining with dementia and I took a couple short trips to see her. My father was also unemployed for a large part of my undergraduate years (I live at home), and the financial instability further contributed to my stress.

For more background info:

I took the MCAT once and my score was 503. Again, I'm angry with myself for this score. However, I only ran through a full practice one time, and an advisor told me that if I was able to get that score by practicing only once, I had a high probability of scoring higher with more practice exams. I plan on focusing and working hard to hopefully compensate somewhat for my GPA.

I have 2.5 years of research (I started the second semester of my freshman year and continued on to graduation). I will be listed as a primary author in a soon to be published paper. But because I did a lot of work for research, I feel like I didn't balance it well with my academics.

I was in the biochem club, pre-health club, biochem ambassador and mentor program at my school. I mentored new freshman and met with them monthly to provide advice.

I have 100 hours of clinical volunteer experience, as well as more social work volunteer experience (I counseled young children going through traumatic/emotional times after the loss of a loved one).

I'm planning on a volunteer abroad experience in Peru, where I will be shadowing a doctor. Plus it's been something I've always wanted to do. I will also be shadowing a pathologist.

Please let me know of any recommendations or if you think I still have the chance to turn my app around into a good one. Thank you!

Mr.Smile12

If you have a mindset that you are open to continuous improvement, then you always have a chance to improve your application. The problem is making sure you're focused on the right things, and it seems that right now you're not really focused on the obvious areas of improvement. Not knowing what other classes you had taken, you may have a chance to take some upper-level biomedical coursework either as a non-degree postbac or as a master-degree-seeking postbac. Your GPA's aren't that bad, and your MCAT may need more rigorous study.

That said, presuming you have regained some control over your mental health and your family situation, you need to show how you have been able to adjust and overcome these challenges and still do what you need to do to be the best physician you can be. Only then will one be able to give yourself a strong chance to getting in and staying in.


WAMC at Medical School and MCAT tips?

Hi everyone, I'm new to SDN and created my account specifically to get input on what you think my chances are for medical school! Here are my stats.

academics: I graduated from UVA with a BA Psychology degree and Sociology minor cGPA: 3.55, sGPA: ~3.21 -my sGPA somewhat inflated because I took a summer physics course at a 4 year university (that is "less prestigious") near home and got an A and A-. -sGPA is also low because I got a C in my first semester of chemistry, and a D+ in my first semester of orgo. I retook orgo and only got a B+ -also had a W my first semester of freshman year for calc 1 because I was naive thinking I could handle 17 credits the first semester -not a strong upward trend -did not take biochem so it puts me at a disadvantage for applying to one of my med schools (VCUSOM)

extracurriculars: -financial chair for premed organization for female students -asian student union for a year -peer advising group for a year -participated in a semester long leadership program for female students -undergraduate research assistant for a 1.5 years in the psychology department (no publications or presentations, but i did take an upper level psychology class where we were required to do a poster presentation for research we did during a semester) - volunteered my first 3 years of undergrad

-shadowed pediatrician for 5 hours and ob/gyn for 15 hours at the UVA hospital. General medicine in a private practice for 15 hours

my credentials aren't out of this world so I was wondering what your thoughts are. I am currently doing a gap year, thinking of taking on a scribing position since i don't have direct clinical experience. I also plan on taking the MCAT around March 2019 to apply for the 2019-2020 cycle, and any tips for studying and my time frame would be much appreciated.

also I plan to apply to mainly virginia MD and DO schools because I'm trying to get in state benefits - wanted to know what i should and shouldn't do.

THANK YOU SO MUCH!

Mr.Smile12

As an in-state applicant, I would think you may have a decent chance among the schools in your state depending on your MCAT performance, though your science GPA is a bit lower than desired. Network with the admissions officers at all the schools and get an idea of their treatment of your early grades. I would have taken biochemistry if you really wanted to have a shot at VCU if it is a requirement, especially if a significant amount of biochem content on the MCAT.


Medical School List: Request for Help!

Hi everyone!

I need some help with my school list. I am abroad right now, so it is not possible for me to have a one-on-one with my premed advisor. I came up with the list on my own, after doing a lot of research. What I want to do is study medicine so that I can participate in humanitarian efforts. Hence, I looked for school with strong global health tracks (that is the case for most of the schools anyway). Here is some information about me:

I am an NY resident. Female.

Academics: sGPA: 3.8 cGPA: 3.9 MCAT: 514. I was not too happy with my MCAT score, but considering the circumstances I took the exam in, I have been told to be satisfied hahaha! I have also been told not to retake. I have a Bachelor of Science from a top 20 school in Biology. I got some at least 3 research related scholarships.

Research: I have done research for two years in a cardiothoracic surgery lab. Unfortunately though, I have not been published. But I did present our poster a few times.

I have also done field research abroad for a summer. This is the experience I am most proud of. This was not published either, but I am currently working on getting it to be published.

Medical ECs: About 50 hours shadowing a pediatrician. About 80 hours volunteering in a cancer ward and ER. About 20 hours volunteering in a hospital abroad.

Nonmedical ECs: Multiple TA positions. Summer camp counselor for a year. I started a branch of ASBMB in our school. Multiple mentoring positions. Teaching abroad during my gap year, but doing so much more than teaching (volunteering, leading clubs, planning and leading camps, etc).

I hate making it sound like I am bragging, so if I am doing that, I apologize.

Anyway, here is my list.

Stanford ( I REALLY want to do something about the Cholera Crisis in Yemen, and Stanford seems to be the only school I can find that is involved in Syria. I know it is not the same, but I want to be involved in the program so that I can learn how I can get involved elsehwere). Hopkins Columbia Cornell Einstein Pritzker Duke Brown (There is a doctor here that I really, really want to work under. He has led humanitarian missions during the ebola crisis, which is simply incredible to me). Dartmouth Tulane George Washington Temple Chapel Hill Buffalo Stony Brook New York Medical College Drexel Albany

Any advice whatsoever, would be really appreciated! Thank you!

mark-ER

I think you have very solid stats (GPA, MCAT) and extracurriculars and you should be aiming to have a difficult choice -- acceptances at multiple schools. No need to worry about bragging -- your profile is VERY solid, but not absolutely top-notch stellar (i.e. not Rhodes scholar, etc), so you can and probably will get into one or more top medical schools, but unlikely to get into every single one. For instance, Stanford or almost anywhere in California will be a long-shot for anyone not from California, or without just absolutely amazing stats/story & extenuating circumstances (MD/PhD not withstanding). At the same time, that doesn't mean you shouldn't try. But overall, I am glad to see you have a range of prestige among medical schools you plan to apply to, with focus on ones you have geographic ties to (New York and Northeast). I would even encourage you to expand your list a bit, from 18 to maybe 25, so perhaps you have a bit more options. You seem to be interested in epidemiology/ID in developing countries (as your profile seems to suggest), so consider Emory (ties to CDC) and Vanderbilt as well. Also why not Sinai (due to your NYC ties), Pittsburgh (great hospital system, international outreach) and Mayo.

Lastly, do NOT completely focus on international/outreach work in your interviews and personal statement, to the exclusion of everything else. There is a still a LOT of need in the USA. As an example, Hopkins has an amazing school of public health with strong international outreach, but it is also in a rather 'rough' area of Baltimore, and wants to be a good player in the community. So they also look for medical students who aim to make their local communities better, practice in the USA, rather than necessarily far-flung yet poorly defined dreams of contributing to international missions. So be strong, express your passions/opinions, but not to the exclusion of everything else. Good luck.


Chances at med school?

Very worried about applying to med school and wonder if I have a strong enough application? About me: I'm currently still a California resident and moved to Reno for UNR Stayed close to Ca due to my father's diagnosis with dementia Grew up shadowing him as a prison doctor Worked in the entertainment business since the age of 7 till my junior year in college Last of 7 children and have had a hard time with my father diagnosis so a few bad grades are on my transcript: D first time around in calc repeated and got an A and a few Cs one in ochem2 Major: Biology Current GPA entering Senior year is 3.37 with hopes to come out around 3.4-3.5 Work as a full-time caregiver at a retirement home in lockdown/memory clinic Most recent work in the entertainment business was 5 billboards for Nugget Casino Hotel in Reno Volunteer as Big Brothers Big Sisters match to a little girl for last 2 years Shadowed in general surgery and internal medicine for a month in Trieste Italy Volunteer t Animal Shelter for 4 years

mark-ER

You have an interesting life story & some difficult challenges (seems ongoing, so best of luck). Your GPA will hold you back, but the biggest unknown is MCAT score. You need to get 510 or above to have a realistic chance at an allopathic (MD) school, 500-510 you can consider DO as well as a few MD schools (in-state, locations where you have geographic tie-ins). Otherwise, however painful, perhaps a year to re-group, post-Bac to improve your GPA (esp. sGPA), and really study up for the MCAT vs. year as a research tech at a university. Good luck.


Do osteopathic schools view scribing for a DO the same as shadowing a DO?

I am currently in the process of applying to osteopathic medical schools for the 2019 cycle. I know that DO schools highly value experience shadowing an osteopathic physician and submitting a letter of rec from a DO. I have extensive experience working as a medical scribe (1000+ hours) for family practice physicians who are DO's, and feel I can express my motivation for DO based on these experiences. I also have a great letter of rec from a DO that I scribe with . Am I at a disadvantage since I don't technically have any formal experience shadowing a DO? Or will they adcoms likely view the scribing as essentially "paid shadowing"? For secondaries that ask applicants about their experience shadowing osteopathic physicians (western U secondary, for example), would it be appropriate to discuss my scribing experience? In my opinion, scribing is simply a more active form of shadowing. I'm probably overthinking this, but just wanted to get some advice!

Mr.Smile12

I would need more information about how you are scribing for a DO. If this is an emergency department scribing experience, it won't matter what degree your physicians have... in all likelihood, osteopathic manipulation isn't likely to happen.

Nowadays shadowing a DO is a little more important for some schools than others, but most are looking for your understanding of osteopathic holistic practice and why it would be important to you as a physician. I doubt many application readers care if you scribe versus shadow, so do the best you can describing your experiences.


Future Doctor Needs Assistance

I’ve already decided that I want to go into the medical field someday. I've been seriously thinking about becoming a pediatrician. I absolutely love little kids However, 12+ years of school/training does seem discouraging... I'm not choosing this career simply for the salary, as I'm sure I want to have an occupation in which I can help others, especially if they're children.

So, is going into pediatrics a rewarding experience? Any pediatricians that can tell me how they love/hate their jobs? Also, any tips for a (maybe) future pediatrician would be much appreciated. I'm currently a high school student entering Junior year with a 2.9 GPA (Feel very insecure about it, I was slacking on my freshman year) and I know I should excel in science, math and English; but other than that, what else can I do now to prepare for the future? (ie, what colleges or majors should I be looking at, what SAT score should I aim for?) Any doctors that would like to share their high-school GPAs?

Thank you!

Mr.Smile12

High school GPA's are meaningless when it comes to entry to medical school (unless you're trying to get into a BA/MD program, which your GPA seems too low to do that). You need to be fully prepared for college science/math courses and the critical analysis that comes with becoming a physician. You should also excel in coursework that truly interests you, but be thoughtful about some of the classes you may take that may help you work with children, such as psychology and child development. So push yourself and see how you do with college-level AP/IB courses while you can.


Does chiropractic assistant count as clinical patient hours

I was recently offered a job as a chiropractic's assistant. Would working as his assistant count toward patient clinical hours?

mark-ER

Did you have direct interaction with patients? If so, I think it does count... I would include it, though I would downplay it a bit (list it lower on the list) and include another experience or two shadowing a physician (MD or DO), and volunteering in a hospital. Some in medicine, higher level academics in particular, may not think of chiropractors highly. On the flip side, there is also perhaps an overrepresentation of questionable advice coming from chiropractors, and as we all know a few bad apples can spoil the bunch (several personal anecdotes come to mind).


Personal Statement

In my PS, I talked about how I initially had self-confidence doubts about entering a medical caeer but through my essay I talk about how my experiences with healthcare have changed that. Is it bad to even mention those doubts, even if I feel they are important to my story?

mark-ER

I think you will get multiple point of view on this question. Some people don't ever have doubts and wanted to be a doctor since age 4; those are in a distinct minority (<20%). Having doubts is human, but therefore also NOT unique. In your PS you want to special/unique about your experiences, overcoming a challenge, perhaps relating to a specific anecdote/episode in your life. There are plenty of references/exemplars out there, including on SDN as well as books you can get from your library. Good Luck.


Will medical school care about two Ws on my transcript?

I have two Ws on my transcript. The was in physics lab during sophomore year. I was dealing with a medical condition at the time and wanted to reduce my course load from 17 credits to 16. The second W was in a computational biology class that I took during spring of my senior year. I got a concussion halfway through the semester, which caused me to get unbearable migraines whenever I looked at a computer screen for too long. This was obviously an issue in my comp bio class, where every assignment involved coding.

Will these two withdrawals negatively affect my medical school application? I have a 3.97 GPA and 524 MCAT score.

mark-ER

Short answer -- no, it shouldn't affect your application. But there is a good chance someone on the interview trail will ask. Short, clear-cut answer straight to the point, probably with only the most pertinent detail, if any, about personal health or stressor is best. I think your academic record otherwise speaks for itself, so you should be more than OK, with multiple acceptances. FYI -- while a junior in undergrad, I took a graduate class Pass/Fail and someone asked if I converted to P/F because "I couldn't cut it". Somewhat of a rude question, but: (a) there is very blunt people out there in every profession, including medicine & (b) sometimes the interviewer is just prodding to see how you answer a challenging question.


Questioning whether or not medical school is the right choice for me

I have been dealing with a tough decision. After graduating with my undergrad, I took a gap year to spend some time in a beautiful area loving life and to apply to focus on my MCAT and applying to medical school. I did well on my MCAT (512), and I have an sGPA of 4.0 and overall GPA of 3.9. I received admission to my dream medical school, but I decided to turn down the decision and give myself an extra year to decide what I want to do. So here I am.

I absolutely love the academic side of medicine, and learning about the different body systems etc., and I enjoyed all of my shadowing and internship experiences. My internship was in cardiology, but I did multiple rotations through surgeries, including heart surgery, and through the emergency room. While I loved my experiences, I never really felt that "Oh my god this is what I want to do the rest of my life" feeling. Being an EMT, I thought the emergency room would be the place for me, but I realized that it isn't the exciting action I was expecting. I love medicine, but I also love the adrenaline rush that being an EMT brings.

In my gap year, I've sen how much I love hiking and enjoying life, and I fear that becoming an MD will prevent me from continuing these adventures, and that my whole life will revolve around medicine. This brings me to the PA route, in which I could pursue what I am passionate about, while also having time for said activities. However, if one qualifies for medical school and has he potential to become an MD, am I being stupid not just going to medical school?

Then my last option, is to pursue the route of a paramedic/firefighter. During my ride-outs for EMT, I rode on an engine for a fire department, and the rush of excitement and happiness I felt every time I heard the tone and hopped on the truck was far greater than anything I had ever experienced in the hospital. The idea of being the first responder and the person who is there for the patient first on their worst day is so appealing to me. Being a college athlete, I love the physical aspect that being a firefighter entails, as well as the teamwork and camaraderie that come with the job. I worry that taking this route is a waste of a great MCAT score, and that in years to come I'll find it isn't as intellectually challenging and fulfilling. I am scheduled for a ride-out tomorrow with my local FD just to make sure that I still get that excitement and feeling I used to get.

I also know there is quite a pay difference between the three routes, but at this point in time I am focused on living my best life, loving my career, and doing the best I can for my community.

Renee_MD

Hi there,

I think if you are having doubt and there are alternative career paths that you are considering, then becoming a physician may not be the best route for you. Since it requires so much time (4 years of medical school, 3-7 for residency), I think you should be fully committed to this field before embarking on the journey. It sounds like you would thrive as a paramedic/firefighter, which does have a lot of cross over as a physician. Go check it out on your ride along and see how this goes and maybe you will find your calling here. Also, as you mentioned, I know that you want to have hobbies outside of your career, and in medical school and residency, you will not have a lot of free time for these other things. Once you are done with your training you will have more flexibility, but for the next 7+ years, you will be very limited.

I hope this helps weigh out your options somewhat! I think you are doing the right thing by thinking it through very carefully, and I'm sure the right decision will come your way!

Dr. Marinelli


I have recently gained admission to two Masters of Science in Biomedical Sciences and I do not know which to pick. Tufts MBS/MPH Program or Drexel University MBS

I am not sure what program to commit to because at Tufts University I will be getting 2 degrees and still get MCAT PREP, but no guarantees ( interviews, admissions, etc.), just going to get 2 degrees and hopefully do better on the MCAT. At Drexel, the program is also two years and the first year is all classes and an MCAT course and the second year is all med school classes, but I am promised interviews with several schools. My end goal is medical school, but along the way I have loved the idea of Public Health and doing something for the LGBT/HIV community and in general. I am very conflicted on what school.

mark-ER

Overall, either one is a reasonable choice. Based on your interests in community health, I think you would benefit from MPH & its strong emphasis on clinical research and statistics. You do NOT need a year dedicated to MCAT prep. With appropriate motivation, study tools and perhaps some equally motivated/gifted colleagues you should be fine. Another important aspect you did not mention that may be a decision maker is geography -- where would you prefer to live? Does your family and/or significant other play a role in the decision? Is there a difference in tuition/cost of living/financial aid?


MD reapplicant hoping to defer after being accepted

I'm reapplying but I will be doing a two year graduate program in the meantime. I want to apply now though because I don't want my MCAT to expire next cycle. I have thought of transferring those credits from the grad program to the medical school and doing a dual degree when matriculated but worst case scenario I would have to defer a year (meaning entering class of 2020). Worth still applying? If so, how should I report this on AMCAS/secondary applications? Thank you!

mark-ER

You have already been accepted into medical school, presumably at a great cost (monetary & effort), right? Why not go the route you suggested -- try to transfer into the grad program of your choice at the medical school and work with the admissions office there to get a deferment. Yes, you may loose some credits, but likely not entire year. Alternatively, you could do a year or two of medical school, then do a gap year and finish up your grad school. First year of grad school you generally just take classes. What I am trying to say, don't jeopardize your acceptance and go thru the process again unless you are really, really unhappy with the medical school where you were accepted, and/or are having serious second thoughts about your career.


Advice for medical school applications

Hello. I'm currently in the process of applying to medical school. Here is a summary of my experiences/academic info:

Biology B.S. w/ a neuroscience and french minor CumGPA: 3.5 SciGPA: 3.27 MCAT: I'm taking it in early August I have shadowed an MD for 20 hours, have completed 100 hours of community service, am a lead tutor for biology, do research in neuroscience/have developed a poster, and am currently completing a course to become an EMT. I also have been shadowing this summer.

I guess I'm wondering if anyone has any advice for which schools to apply to/ is there anything else I should think about doing this summer? I have a couple MD and DO schools in mind but was hoping to expand my list based on recommendations. Any advice or recommendations would be greatly appreciated!

mark-ER

The recommendation will very much depend on your MCAT score. You should be aiming for 512 and above. Unless you went to a top-notch Ivy league college (and even then), your GPA is holding you back. If you get 515 or above, I would recommend DO schools, along with all your in-state MD schools and a smattering of (at least 10) private schools that take out of state students, basically anywhere you have links to the area thru family, geography, mentors, etc. If you score 512-515, same strategy, but perhaps even more schools to give yourself a strong chance. Below 512, I would plan to re-take MCAT while making a contingency plan for a year -- MPH or a research. Good luck.


Chances of Early admission in KU-med

Hello, I am a 41 yrs-old medical student currently taking pre-requisite in Wichita state U,Kansas. I had 2 MS and one BS, of which the latest MS in Biology track Molecular Biology done in 2017 at Wichita st. I got my permanent residency in 2016 Dec which allowed me to start the pre-med UG classes in 2017. I am done with gen chem,org chem-1,phy-1, biochem,molecular biology, human phys,soc with sGPA 4.0 and currently taking org chem-2,phy-2,Pre-calc-2, psy and public speaking. My bio and english requirements are waived by Univ as I had MS. My MS GPA 3.887. My volunteering experience ~200 hrs till date, shadowing ~150 hrs, teaching experience 8 yrs ( more than 5000 hrs), research experience~7yrs and clinical experience as a CNA for 6 months (will be continued). I never served any executive board though. My plan to apply KU-Med wichita campus as an early decision applicant. I am curious to know what is my chance to get into the Med school and what are the areas I can improve on? I will be applying to Med school in 2019 summer. KU-Med is my only option as I am married with 4 yrs old kid.

mark-ER

Best answer would be: apply and keep an open channel with the admissions committee. A priori, based on your stats and, prior experience and medically-relevant background, your chances appear to be pretty good. Main element that is missing -- what about MCAT? Also, as you allude to -- do not put all your eggs in one KU basket, apply broadly. As for encouragement -- you can get in as a non-traditional student at age 41, having a residency status (green card) is a huge plus for a US born individual, and many medical schools will value the diversity you represent. Good luck!


MCAT Test Date & AMCAS application

I was hoping to get some advice on this plan. I would like to take the MCAT mid-July (meaning my scores come back mid-August). I have my AMCAS application ready to be submitted and I found out that you can add schools to your application even after submission. Should I submit the application with only a few schools attached and then add more after I get my scores back?. That way the application will already be verified by AMCAS. My quandary is whether to take the MCAT earlier or in mid-July (I think I could use the extra time to study), however I do not want to decrease my chances with time. Any advice is helpful. Thank you

Mr.Smile12

You should be able to submit an application with a minimum number of schools before you take the MCAT and then add once you have your scores. As the process is rolling admissions, getting your MCAT scores earlier is always more desirable especially when you need this information to decide whether you should submit to more or different schools.


AMCAS Application

I have my AMCAS application complete and could submit it in a few days (6/1), however I do not plan on taking the MCAT until mid-July. I know that it takes a while for them to verify your application, but should I submit it this early if I wouldn't get my scores until August if I take it in July? Also, do you think early-mid July is too late to take the MCAT?

mark-ER

So just to be clear, it seems like you have not yet taken MCAT (please correct me if I misunderstood). If so, there is a bit of a risk to submitting the application now, as you will have to pay now and you are now obligated to go thru the application regardless of your MCAT score (yes, you can pull out, but then you loose your fees). Conversely, some people find that 'externality' motivating, but according to the more traditional school of thought, perhaps if you are not motivated enough to study for MCAT regardless you should not be applying (??).

So in short, you could submit at or around June 1st, but it may be better to wait -- at the very least to see how your test prep is going and you are confident you will do well on the MCAT, or after you get your score. July is not too late, though you will have to step on it (no procrastination with finishing up the paperwork) as many schools have rolling admissions.


My chances for getting into the MD/DO progam

I'm in a very difficult position in my life. I want an honest expert opinion on my chances of getting into an MD/DO school. I have a 3.11 GPA sgpa of 3.3 without counting the D for bio and F I got for my genetics class. Yes, an F I was definitely was not counting on getting that F but unfortunately due to severe family issues I was not able to concentrate in class. I also changed my college that same year. These are not my excuses I know people who are in med school might have gone through the same things yet got good grades but every person is different. I made a mistake and I would like to improve it. I'm currently a senior about to take the MCAT but I'm not too confident on my application at this moment. I have clinical volunteer experience more than 100 hours, I did community volunteering, research and shadowing this summer. I am passionate about helping people that is the sole reason I really want to be a physician. If med school is not an opinion can someone give me any advice on what I should do like post-bac or master for improving my application and grades?

Mr.Smile12

You should make sure you talk with a prehealth advisor at your undergraduate program, but I would suspect that a master's granting postbac program is likely your best shot to show you can do the work for medical school. Ask your advisor and check the postbac program database on AAMC for help.


Parent with mental illness personal statement

Hi, My mother was diagnosed with bipolar disorder when I was very young, and her diagnosis and how it has shaped me growing up is one of the reasons I would like to become a doctor, so I would like to write about it in my personal statement. However, I know that adcom's may see this and think that perhaps I have inherited this condition, and it will affect my ability to complete med school. I obviously don't think this is the case but wanted to know your opinion. I have heard mixed opinions. Please give me your honest thoughts about this (I am okay with not writing about it but want to be sure).

Pathdocmd

I have seen this subject in an essay several times. In my experience, it has never been an issue.


AMCAS 1 Year Biology and Lab Requirement

I'll be graduating as a chemical engineer with a biotechnology concentration from UC Berkeley. They counted AP Bio and let me take a bunch of upper division biochemistry and bioengineering courses:

MCB 102 (Molecular and Cellular Biology) Nanoscience Engineering and Biotechnology BioEngineering 111

I'm wondering if I can count these courses as biology on AMCAS, or if I need to play it safe and take an entry bio class in college.

Also, I know you need one full year, two semesters of biology lab, which I don't explicitly have. I've done summer internships at biotech firms as a research associate, doing CRISPR, flow cytometry, PCR, etc, but I'm not sure if I can necessarily count that on the AMCAS as credit. Can you use work experience as a biology lab qualification?

Thanks!

tantacles

Most schools will accept upper level courses as substitutes for basic level courses. However, what schools will accept as substitutions for lab work differs by the school. I would suggest that you check the websites of some of the specific medical schools you are thinking of applying to to see how they count AP credit and try to adjust your plan accordingly.

mark-ER

I think you will be fine with MCB 102 to satisfy the requirements. If asked, you can show them AP bio credits on your transcript. I am sure you did well in college & on the MCAT (tough major & good school). But kudos to you for asking to double check, if you want to be extra-extra sure you can also ask the adcom office for your local medical school (presumably UCSF).


Low MCAT/High GPA, good experience should I apply to any MD schools or focus on DO?

I am a non-traditional applicant applying this cycle. Originally, I planned to apply to both MD and DO programs, but after receiving my MCAT score (500), I am unsure of my chances. I graduated Summa Cum Laude in 2017 from a public research university with a degree in Health Sciences. I have completed all of my premedical courses except biochemistry and have a cumulative GPA of 3.96 (Sci GPA of 3.9). I have over 600 hours of clinical exposure, 900 hours of research including a publication, served as president of my pre-health committee, and have received awards in leadership, community service, and athletics.

I was a two-time captain of my collegiate hockey team competing at the NCAA division-III level and a two-time academic All-American. As a student interested in orthopedics, I realize the MD track is a better way to go. In summary, my question is if (at all) I would get a look from allopathic schools with such a low score or if I am better suited to shift my focus to osteopathic programs. I appreciate any advice or suggestions regarding my situation. Thank you.


mark-ER

Do you have a good reason for wanting to go DO route? If you are only aiming DO because of your stats and thinking osteopathic schools are easier to get into, then instead really focus your application toward mid-lower tier MD schools and apply to EVERY SINGLE ONE that you think you have a shot of getting into. That means every one of your in-state schools, all schools where you have a geographic tie-in (family, lived there, attended college), and all lower tier private schools that take sufficient number of students from out of state. It will be expensive, but you can't win the lottery if you do not buy a ticket. Secondarily apply to a handful of DO schools, and again make sure you give them a reason why you want to do DO, instead of MD. Look and play with LizzyM score here on this site, and always remember, stats like GPA and MCAT are a bell curve. Yes, you want to be above mean, but plenty of people get in on the left end of the bell curve too, and if you apply to enough places you increase your chances. I think your high GPA, good (if not ivy league) college, and compelling story of being a high level college athelete and extensive volunteer experience are all a plus. You ought to get in somewhere with a broad enough application strategy, but even if does not happen, not all is lost. (apply again next year, perhaps while doing MPH and retaking the MCAT).


MCAT Prep (Again): Are Kaplan In Person or Princeton Review's MCAT Strategy worth the cost?

This will be my third time taking the MCAT (494 and 497). While being optimistic about the waitlist, I have not had studied so I plan on taking my scribe offer and taking it again in March-April 2019. I took a school offered prep course but I focus more on my GPA than my future. I finished with a 3.8 cGPA, and I tried self-paced Kaplan with the books and limited online. I'm trying to make the third time the charm. I know both the Kaplan and Princeton Review prep courses are $2,000+. Are they actually worth the cost? And which one has a higher success right? In need of advice before spending the cash.

tantacles

This question is difficult to answer. For many people, the Kaplan and Princeton Review prep courses are extremely worthwhile. Depending on your learning style and level of motivation, these courses may help you improve your score. That being said, some find that they do better with self-study. It sounds like self-study has not worked well, so these courses may be a worthwhile investment to give you a better framework with which to succeed.


I'm asking for a suggestion as to which program and medical school to apply to based on my credentials.

I am an undergraduate Biology major at the University of the District of Columbia with a 3.6 GPA. I have also done various internships at places like Georgetown University, Johns Hopkins University, and NIH. I would appreciate it if someone could suggest to me what program (MD/MD-PHD/DO) at which medical school would be best suited for me.

Mr.Smile12

Credentials can get you through the door in an admissions process, but you still have to pick your own doors to unlock. Do you want to stay close? Do you want more of a career in rural communities? Do you want an intensive research environment? I'm guessing AAMC and AACOM have resources with simple questions to ask yourself when it comes to identifying what are the key things you are looking for.


I have 3 C's (2 freshmen year and one sophomore) and grades have all been As and Bs from there with a GPA of 3.65. Do I have any chance on getting in to MD schools?

What is an MCAT score I should aim for to have a good chance at MD?

mark-ER

Clearly, you want to avoid Cs (or below), but what's done is done. You are showing improvement, plus your overall GPA is reasonable. If you can bump it up above 3.7 and your science GPA also in that range, together with a good MCAT score you should be OK. Those 2 aspects are key; anything interesting about yourself (personal background, leadership & extracurricular activities, research & publications) are certainly nice but more of a 'cherry on top'. In terms of MCAT score, don't think of a specific goal target, aim for the highest you can get. When you get the score back, you can use LizzyM calculator on this site (SDN) to figure out the number and prestige range as far as med school applications. Good luck!


Should I apply?

Hello,

This is my first time posting. I am very concerned mostly about my MCAT score. I completed my undergraduate as a double major in forensic science and biology premedical with a minor in chemistry. I graduated magma cum laude a year early. I then went on to my masters of science in cellular and molecular biology. My sGPA in undergrad was a 3.78 and my masters is a 3.97.

I have one publication and a lot of research. I should have 3 more publications by the end of 2019. I have presented at international conferences. I have shadowed 3 doctors and have been an emergency room medical scribe for a year. I worked in a forensic pathology morgue for 2 years. I would say my clinical hours are approximately 2500 hours. I have also been accepted for 2 competitive grants and have completed research with those. I am also now a research assistant at my university performing molecular research on Lyme Disease.

I have had multiple leadership positions in extra cirricular activities as president and various other chair positions. I am currently President of my graduating class. I was also in a sorority for women in technical studies where we did a ton of community service. I was in 5 honor societies in undergraduate.

I have (what I think) is a good PS, as it speaks about my clinical experiences and my journey and also into my sister, who is special needs. She is my main motivating force to become a physician. I also have great letters of recommendation from physicians and professors.

Here is where my downfall is: I got a 501 on the MCAT.

Should I even bother applying this year? I feel that everything is wonderful, except I have this HUGE flaw on my application, which will present itself first, most likely before medical schools see anything else.

Suggestions? Be nice.

Mr.Smile12

In my opinion, remediating a low MCAT score is easy; just retake it. You know better what may have distracted you from getting a better score. You may have done super well on practices but didn't do as well when you sat down to take it for the first time. The MCAT score you have may not be the downfall you predict for certain schools on your list, but certainly if you have high expectations of getting into a top prestigious program, then yes, you probably should take it again.


Should I void my June MCAT after taking it and immediately reschedule?

I'm currently scheduled to take the 6/1 MCAT.

Today I scored a 510 on my AAMC FL 2 (128/129/128/125).

My studying as a whole was admittedly poorly structured -- I took a Kaplan course over winter break with the original intention to take the exam at the end of March (my Kaplan FL scores were Diagnostic 499, FL 1 500, FL 2 504, FL 3 508, and FL 4 505), but was overwhelmed with other commitments in February and March so I pushed my date back to June. I probably studied like 30-40 hours total during the semester (I know, I'm the worst -- I just really wanted get a 4.0 this semester), and have been going ham (10-12 hours a day for the past 2.5 weeks) since I finished the semester.

I took the AAMC FL 1 a week into my going ham to gauge my progress and got a 506, breakdown 127/128/126/125. That freaked me the heck out and I've been going harder since...but today, still no improvement in Psych :'(

I'm a rising senior at an Ivy League institution who is planning on applying after one gap year (starting process senior spring). 3.79 GPA with a strong upward trend (had some personal and adjusting issues freshman year that really messed me up, but have carried a 4.0 per semester since. Hopefully will break 3.8 by end of senior fall). I'm going back to Cambridge for research this summer and will probably be working 40-50 hours/week.

I was hoping to take the MCAT around now because I have taken all of my premed courses like I would have applied directly (physics, ochem are more fresh in my brain). I also wanted to take it before I started my research so I could focus completely on that.

I am really hoping for a 518+ (in my dreams 520+ heh) so I can be competitive at top 20 medical schools. I've gotten As in most of the premed prereqs (save for the freshman year slip ups), and was admittedly overconfident in my abilities. During my content review, I did get the sense that if I spent more time reviewing, I could better applying the knowledge to practice problems/section bank/etc.

Haven't done the third AAMC FL 3 yet (was planning on taking it Saturday), but am now worried to take it and "waste" it if it is still not a good score.

I don't want to psych myself out but I do want to be realistic with myself. What should I do in my last ten days? Should I go into FL 3 this Saturday (reevaluate after) and the actual exam hoping that I'll get a 518+? Should I do as much section bank/content review as I can and take the actual exam and promise myself to void it, scheduling myself for a late July or August date after? Any advice would be appreciated.

Mr.Smile12

I have a different perspective that you have definitely been improving since your Diagnostic exam (Dx). I would consequently be very careful about adopting a mindset where you just take the exam and dump it afterwards. There are only specific instances where I would suggest you do that, and it usually involves an unexpected and perhaps sudden loss of confidence or focus during your exam. There have been many people I know who have done terribly on the practice exams only to completely nail their actual exam (with the old scoring system); there have been others that bomb it despite having done super well in the practices.

So, don't psych yourself out. Go in confident these last 10 days and take your practices. Focus on making sure you don't lose points on things you already know now. And make sure you give yourself time to relax soon before your exam day.


Advice for a Pre-Health student lost in europe?

I am a Pre-med student who has had a kind of weird college career so far and in need of some advice. long story short, I am currently in a BSN program in Norway chugging along as a B- student after completing my A.A and an EMT program stateside. I've recently decided I want to jump ship after finishing my BSN and go back to the states to complete my undergraduate degree in hopes of applying to medical school, but I want to get as much out of my time here short of a higher GPA.

So my question is, do EC's or MD/Professor LOR's from a different country (say, shadowing experience from Germany, Norway or Sweden?) actually help/matter if I'm applying to a US medical school? School related EC's dont really exist over here (clubs, frats/sorority, etc) and research isn't really available to students outside the science majors so I'm limited to volunteer work. Red Cross, Norwegian Peoples Aid (SAR/disaster relief based) or the Night Ravens (neighborhood watch group that patrols larger cities to help citizens at night) are pretty much the top 3 organizations outside of the churchy stuff this side of the pond.

I have 2 years working as a CNA for county home-nursing every 2nd weekend and 1 year as an LPN in a cardiology ward working every 2nd week. I was also a class representative to the student union for 1 year, 1 year as the student union treasurer and 1 year as the treasurer for the social activities group (plan events, apply for grants, organize people and overall keep track of money).

tantacles

Your extracurricular activities will count no matter where they come from and when they were done, but that being said, your most recent experiences are the most relevant. Your letters of recommendation from Norway will likely be helpful, but I would suggest that you at least get 1-2 from your new institution where you plan to finish your undergraduate degree.


GPA questions for med school?

I just completed my sophomore year, and I made C's in general chem. However I did well in other science and math courses. My science GPA is 3.2 and my overall is a 3.45. Do med schools round up? And also, is there hope for me to come back from this. Will they care if I retake a course?

Mr.Smile12

Every medical school will consider different calculations based on the AMCAS/AACOMAS calculations given to them. As far as I know, every calculation is taken to two decimal points, and each school may opt to recalculate GPA's if you retake courses (though most won't). Keep doing your best in your classes and MCAT, network with admissions staff and your prehealth advisors, and stay positive.


Was doing premed at Berkeley a bad decision?

Grade deflation, competition, and difficult personal events have made me question whether going to Berkeley was worth it tbh. My cGPA is around a 3.62 and my sGPA is not too good lmao. I am a freshman.

I wanted to transfer but my parents refused, saying that it'd "prepare me" for the rigors of med school and what not. Thoughts?

tantacles

A GPA of 3.62 is quite viable for medical school admission, and you are only at the end of your first year, meaning you have a chance to improve. It looks as if you're doing well, and if you're able to do well on the MCAT, you'll be in a great place to be accepted into medical school assuming your grades continue to be at a similar level.

That being said, it might be worthwhile to transfer, but transferring has downsides as well - you will have to create a different friend group and build new connections, which people sometimes find difficult. I would consider your decision carefully and ultimately try your best to do what is right for you and not for your parents.


Advice for low sGPA student: MCAT score? EC? Masters??? How important is undergrad major?

Hello,

I will be graduating undergrad in 2019 and plan on applying to med school that June. My stats are: 3.14 sGPA and will likely graduate with slightly over a 3.5 overall GPA. I will be taking the MCAT this August, and based on my preparation believe I could score about a 515.

I am a biomedical eningeering major and have research experience (one publication). I am also involved in a few student organizations (including orchestral emsables),, and am the president of one student chapter. I also volunteer at a hospital and will begin shadowing this summer.

My question is: does the fact that I am studying a difficult major compensate for my low GPA? Should I score a 515 on the MCAT, will that be enough to prove that I am capable? Basically, based on this, am I competative enough for MD programs?

Would getting a masters in my field help my application?

Thanks.

Mr.Smile12

Some schools may consider your lower GPA/major when reviewing your application, but they'll also find many engineers who do quite well in their coursework and have better GPA's, so your argument won't have much to stand on, depending on where you apply. A biomedical master's program is likely to help, especially if you want to have a broader selection of schools to apply to.


Boston University Pre-Med Undergraduate who really needs some advice...

Hey! Thank you to whoever can give me some guidance!

I am going to start my senior year at BU this fall as a Human Physiology Pre-Med major.

I really underestimated how much I would be challenged and as a result how much I would grow as a student and human during my time at BU.

Freshman year was the worst year of my life thus far, I got a D in Intro Bio and a C- in General Chemistry. These are the lowest grades I have received. Currently I have a 3.0 cGPA and I am confident it will rise in between a 3.2-3.3 by the time I graduate. I went from having a 2.3 GPA my first semester to a recent 3.75 GPA so my GPA has only been on an upward climb/trend.

Extracurriculars: - After school tutor for 1 1/2 years - Brigham & Women's Hospital Medical Career Exploration Program (180 hours) - Nationwide Children's Hospital Pediatric Intern/Research (3 months while at home for summer) - St. Vincent's Hospital Infectious Disease Intern (study abroad in Australia for a semester) - Boston Medical Center Research Assistant, currently working on 2 projects: 1. minority women who have been diagnosed with diabetes 2. writing a palliative care research paper (started about 3 months ago and will continue for at least a year) - Pediatric Shadowing Hours - Islamic Society Member 3 years, Pre-Medical & UNICEF Society 1 year

I have recently lost faith in myself when it comes to medical school admissions. I have made up my mind to take a gap year where I will continue research and work as an EMT (I am currently taking the class and will have my certification in July). It's been a tough pill to swallow when it comes to my GPA as I was a straight A student & valedictorian in high school. Freshman year was definitely a challenge but I only grew stronger from this and I really do understand how to study now (I could've gave up). I have not yet taken the MCAT but hypothetically if I get a high score, what do you think my chances are? I also have considered retaking those 2 classes but that is a lot of money. I am a very hard worker and I am so passionate about having the opportunity to add my contribution to pediatrics.

Mr.Smile12

Ultimately you'll have to address the fact that without showing consistent performance in challenging upper-level science classes, the overall/science GPA will still be an issue. Think of it this way: would you rather spend the money on retaking two undergrad classes or maybe doing a rigorous master's postbac program, or would you rather spend it on remediating a semester of med school courses considering how much higher tuition is? Challenges happen to everyone, but you can't think of shortcuts too much or else you shortchange yourself. To that end, work with your prehealth advising staff to come up with other options: MD is not the only way to be a physician.


Can you get in a medical school with D?

I am planning to go in a medical school. I just finished sophomore year of college. I started to doubt whether or not I will get accepted. My PGA is around 3.3. My chemistry grades are B+ and A-, for organic chemistry it's B-. physics C+ and (probably will end up with) D+. DO you think I have a chance of getting in a medical school? Any advice would be appreciated. good medical school that would at least consider me.

Mr.Smile12

Any D's made in prerequisite classes must be retaken, no exceptions. Otherwise, very few medical schools (I would probably say no MD or DO programs in the United States) will consider you seriously. There are too many very strong applicants in the applicant pool.


How to tie a personal life event into a personal statement

Me question is what is the best way (if at all) to incorporate a personal life event into a personal statement? I’ve been dating my fiancé for over 5 years now and will be getting married in less than two months. We just found out recently that her father has been having an affair with another women for over three years now. It was about as bad of a situation as you could imagine. There was a lot of lies and everything was going on behind our backs. It destroyed my fiancé’s family and they are having trouble grasping what all has happened. It has been especially hard for the kids especially since one is a little boy and I feel as though I am kind of acting like a father figure so-to-speak for the time being.I am having a hard time with it as well as I have grown to look up to this man as a father figure. This was something that nobody ever expected and they recently found all of the emails that have been sent back and forth over the years and it was heartbreaking to read. This has changed my life because it has helped shape me into a better person. To grow into a person who will be better because of this event. To be a better doctor someday because I know what my actions can do to others and how it can ruin their lives. How trust is so important especially in the aspect of patient-physician interaction. This has helped me to truly be a better person because of this mans infidelity. I would never wish this upon anyone. Is this somethings that I can use in a personal statement and how it changed my life for the better and how’s it’s been a life lesson to always true and caring for others? I have hundreds of hours of shadowing and volunteering in a hospital and other areas and things that I can connect to it describing why I want to be a doctor. The shadowing and volunteering experiences are why I originally wanted to be come a doctor. Interacting with doctors and seeing them make the lives of other better is what led to my passion for medicine. But will this experience help to portray how after going through this is will make me a better person and therefore a better doctor? Will it help me stand apart from other applicants/personal statements? Or will it be too off-topic. Any help would be appreciated.

Thank you

Mr.Smile12

A personal statement is your best chance at a first impression, and it's subject to any followup in interviews that you will have. I'm not sure I would want to be known as the applicant whose father-in-law has an infidelity issue. If you say you just found out about it, it may be too recent for you to be able to process and reflect in a way that you can tie it in to how you would interact with patients who you may encounter in a professional manner. Do you have an experience where a doctor truly broke confidentiality or a patient's trust in the same way? In general, I would advise against bringing this up.


DO Cycle 2018-2019: Is a June 30 MCAT re-take too late?

I took the MCAT for the first time this January and received a 502. I want to take it again to see if I can improve my score (nothing crazy, maybe a 504/505 or so), but I'm afraid that taking it June 30 will put me too late in the application cycle. I already have most of my AACOMAS Primary filled out, letters are completed, and I'm on track to have my primary submitted by early June. Of course they'll already have my first score to look at, but will having my second score not being released until July 31 significantly hurt my application?

Mr.Smile12

In all likelihood, retaking the MCAT in late June isn't going to be a problem with your application. Just make sure that the schools are notified as soon as your official scores are released.


How do med schools look at classes that you retook?

I ended up struggling greatly during my freshman year of college with many personal problems. I ended up getting C's in all of my science classes I took and ended with a 2.7 GPA. Obviously, I know I need to preform much better my next 3 years, but what if I retook them? How do med schools look at that? Should I keep my C's and just finish strong?

Mr.Smile12

You should ask the prehealth advisor on your campus about what to do. You may want to consider taking those courses again since you want to be sure you have a solid foundation for the upper-level classes you'll be taking, especially if your issue was related to coping with personal issues.


Is taking a Gap year a good idea postBac?

So right now I’m a Bio major going into my jr year. I’m in the honors college at my university and I did community college for 2 years. I’ve done this so far:

-a full summer as a research volunteer in general surgery for Trauma (150hrs) -hospice care (3months once a week) in HS -scribing for cardiology (recently started &planning to do 2 years) -bilingual -3.61 cumulative gpa (1 Withdrawal, 1 C in gen chem2)

I’m concerned I’m not in good standing to at least get into 3 medical schools. I’m hoping to get into atleast UM or UF. I haven’t taken my mcat but I’m planning to study as I finish my prerequisites(orgos and physics classes). Is it really necessary to take a gap year? I kind of don’t want to. Also my last two years of undergrad are going to be pretty overwhelming given that I’d be taking about 6 or 7 classes a semester. What are my chances and is a gap year the ideal route in my case? Thank you for the help!!!

Mr.Smile12

As a rising junior, it's too early to say whether you "need" to do a gap year. If you don't want to do a gap year, do well in your courses, network with students and admissions officers at the schools you want to get into, and develop a pre-application year plan with your prehealth advising team and your potential evaluators. If you do this and have a strong MCAT score, you may be well-positioned in your application coming out of undergraduate.

That said, you may not want to do a postbac, but if you want to become a physician, you'll have to understand that you need to embrace doing things you really don't want to do. There is a lot of that in medical school and residency, and in retrospect a postbac is going to seem like a very small inconvenience in comparison. To that end, you should determine for yourself how really passionate you are about medicine if you aren't willing to do whatever could be needed to prepare you for a health professional career.


Should I study for the MCAT this summer going into junior year even though I'm doubting the medical path?

I've been set on medicine for most of my life, but now I'm thinking about going into computer science/AI instead due to the creative freedom. This is hard for me because becoming a neurosurgeon has been my dream for a while, and I've also already completed almost all medical school prereqs. I also have a 4.0, love learning/taking classes (so don't mind the long medical trek), research experience (which is related to AI, so not a waste regardless), and am imbedded in medical "culture." I also find medicine extremely fascinating, but feel it has become too bureaucratic and also seems so specialized to the point that most medical positions seem monotonous (concluded this after shadowing many surgeons and some general physicians). I am also doubting my ability to think quickly and accurately under pressure. Even though I'm good with people and like being in control and the source of stability, I've recently questioned my own stability and also ability to actually be a good leader and take care of people properly in a medical context (albeit I've clearly not been trained yet). Furthermore, I really want to travel, and I'm not sure that will be possible as a doctor (especially a surgeon). Does it seem like I'm doubting things too much as an incoming junior in college? I do temporarily have lapses where I want to be something else, but this has been the worst one yet I believe. Usually I am very clear about entering the medical field, but lately I've forgotten why. What should I do? Continue with my plan to study for the MCAT or hold off until I decide to stick with medicine again? Perhaps shadowing another doctor might confirm my new career prospect or get me back on the medical track. Thanks in advance for any advice you may have.

mark-ER

Don't make any generalizations about medicine after shadowing generalists/hospitalists. No matter what, things can become routine but medicine provides a lot of room for creative people, especially computationally minded as you seem to be. Consider shadowing some specialists: medical genetics, molecular genetic pathology, or medical informatics (image informatics in pathology or radiology are particularly cool). Those are just some areas off the top of my head -- there are a lot of room for all types/personalities within medicine. Also consider MD/PhD, or a MD-masters pathway such as HST if you enjoy research. You can always go into iBanking or silicon valley/biotech after medicine. But it might be tough to do it the reverse route.


I wanted to know my chances to get into a direct bs/md or ba/md or bs/do program

Hi, i am currently a junior in high school and i am interested in getting into a direct medical program, but i feel as if i dont have the chances. My SAT score is 1430, my weighted gpa is 4.12. In addition to academics, im currently shadowing a doctor, i have been doing martial arts for 8 years, and i am going into a medical internship at Palomar Health in san diego. Plz tell me which direct programs i should apply to. My top md programs are gwu and uconn and my top do program is nova southeastern. thank you!

Mr.Smile12

Your first concern is to make sure you apply to the appropriate undergraduate institution who is a partner with the schools you eventually want to complete your medical education. Based on your metrics, I wish you luck in your chances to be part of those guaranteed admission tracks, so make sure you indicate your interest to the undergraduate schools in your application.


Will I get into med school?

hi I go to a pretty rigorous private university for undergrad and I made the mistake of taking biochem and organic chem II at the same time and I got a C- in organic and probably not much better in biochem. I have decided that I'm going to retake organic this summer. How will this affect my chances of getting into MD or DO school? I am a sophomore and haven't taken the MCAT yet, I am double majoring in biology and english, I've done research since my freshmen year and have one middle author publication and another coming out this fall, I am a writing tutor and voluteer at the hospital during the summer. cGPA 3.55, but all of the prereqs (except physics) are done.

Mr.Smile12

You're doing the right (and perhaps only) thing by retaking organic chemistry and any other course you got a C-minus in. It's too hard to say what your chances are until your MCAT is taken, but I would worry first about getting those grades in before you worry about the MCAT.


I am seriously torn about disclosing an alcohol related illness my freshman year on campus where I was not charged but was under university sanctions, attended alcohol counseling and stayed on probation for a year. There is NO record of this on my transcript and it is not discoverable. Are there secondary applications which will force me to release student code violations which are not documented on my transcript? There is no way for anyone outside of my university to know that I had one stupid alcohol/personal safety violation if I do not admit it. I get the honesty/integrity BS, but I also do not want this to quash my otherwise spotless and highly qualified 4.0 University President's award application. What is really the bottom line here?

I am a 4.0, scholastically rewarded, 20 year old applying to medical school for the first time with an alcohol related illness student code violation. I was transported from my dorm to the area hospital because the student health center was closed. I completed alcohol diversion education and was on probation for a year (university routine sanction). The rest of my record is spotless; my MCATS are high and everything else looks excellent. There is nothing on my transcript about this issue. It is not able to be discovered, nor is the fact that I even have a disciplinary record (much less what it says) unless I divulge it personally. Do secondary apps require releasing any/all records from the undergrad institution? Should I really cut my legs out from under me? Seems stupid. Or does it really not matter in the big picture for one stupid offense.

Mr.Smile12

You should talk with your prehealth advisor for specific issues with secondary applications, but in general, you should always side with self-disclosing. Work with your honor code administrator to review any such statements you give. There is normally a question on disclosing reasons why your education was interrupted in college (i.e., you were on probation for a year), so you have to say something because your transcript will likely show that gap.

That said, you are probably not the only one who has had alcohol issues while in college. If the offense was deemed not important to document on your record, that's better than the alternative. I would always advise on disclosure though, with appropriate notations on your remediation.


How do med school admissions look at five or fix years of undergrad?

Lets say you take more than four years to get an undergrad degree given that you do double majors and you did a fellowship/co-op for a year (gap year), how will med schools consider this?

mark-ER

You might be asked by a curious interviewer and need to have a good answer as to 'why', but overall no one will blink an eye. As long as you had a good reason to do it (many, many people take a gap year), you spent the time productively and maintained good grades/obtained good MCAT you will be fine. Many programs in the US (engineering comes to mind) are routinely 5 years. Should not be a worry in & of itself.


2.8 gpa senior year, taking a gap year. SMP or MCAT?

I am graduating in a couple of days with a low undergraduate gpa (2.8). It droped from 3.2 due to a D in biochemistry and calculus. I want to take a gap year to raise my gpa and also take the mcat/ do intervention but now I am unsure. I am moving to California and I want to apply there but idk how to go about that. Basically what do you think I should do. Should I take the MCAT first and then boost my gpa??

Mr.Smile12

Your priority at this point is to address your GPA first. You have a D in biochemistry that no medical school is going to accept unless you remediate that grade (calculus too for many schools). Go to your prehealth advisors and get some help in your questions about moving to California, registering for classes there, and applying as an in-state resident. You may have to fulfill a time requirement of residing in the state to qualify, but the medical schools there should be able to answer that question.


C's on Transcript

Hi,

I have 3 Cs on my transcript and 2 Qs (a withdraw without the grade penalty in Texas). Classes were Freshman and Sophomore yrs: 1st semester Chemistry lecture, 2nd semester Biology lecture&lab (one class), and Chemistry lab (2nd semester). I Q-dropped my statistics class freshman year and retook it sophomore year and got a B. I Q-dropped physics because I was changing majors.

Funny enough, I have a 3.73 CGPA and a 3.74 BCPM GPA. Drastic changes during Junior, Senior, and Super-Senior years - did not receive one B and got all As in OCHEM and BIOCHEM as well as some other upper-level biologies. I retook all my Cs and got As. I even tutored in chemistry and biology.

MCAT-523

I have 5 years of research in 1 lab with 2 papers, and 1 year of research at the University of Cambridge with 1 paper (all 2nd/3rd authors). I also have plenty of leadership experience and president of a club. I play on my university's soccer team too.

Did I truly bounce back? Do I still have a shot? (I need this more for peace of mind to convince me to apply this round of admissions)

Renee_MD

Hi,

Yes, you absolutely 'bounced back' and still have a shot. There is really not anymore you can do to improve those initial bad grades.

Medical schools understand that not every college transcript will be perfect, and they know that some students will have ups and downs. The point is to show them that you have grown from that, and have the academic ability to do well, and you certainly have shown that!

Great job! Good luck on this application cycle!

Dr. Marinelli


Gap Year Job

So currently I am in the process of applying for medical school. I was a computer science major and I have a offer from a tech company but I don't know if working at a tech company during the gap year might show a lack of commitment to the medical field. However, I don't have too many healthcare related activities during my 4 years in college. So would it be better if I did something medically relevant?

mark-ER

How about doing a gap year in something techy, yet medically relevant? There is a LOT of need for people with informatics/comp sci background in medicine. I could post some ideas, but they would basically be links to companies & (indirect) advertisements, and SDN frowns on that. Where are you located/what's your preferred geography? Feel free to PM me.


Extracurricular activities

I currently work as a respite worker (paid) for my autistic younger brother. I help him with various activities such as helping him take a bath, making him food, and helping him with homework. I am curious if this type of work would be considered a good extracurricular activity? I am also currently shadowing a few different doctors. I also am looking into doing an oversees missions trip with my church in the future. Are these other activities favorable for EC? I am just trying to see how these things can fit into my medical school application. I am currently a sophomore in college. Thanks for taking the time to answer my question!

Renee_MD

Hi,

Your work is definitely a significant extra-curricular activity in regards to medical school admissions. You are demonstrating true compassion and care with your work for your brother. In addition, I do recommend trying to get additional extra-curricular activities in. This includes shadowing as you mentioned, clinical volunteer work, non-clinical volunteer work, research and leadership. A missions trip can be a great activity, but to be competitive for medical school admissions, you want to try to have longitudinal experience in these different extra-curricular activity experiences as well.

Keep up the good work!

Dr. Marinelli


Two Full Time Jobs, Full Time Student, No Clinical Experience.

I grew up in a low-income family and always dreamed of becoming a doctor and help people. I am more financially stable now and was able to start college, and I realized how much closer I am to my dream. However, I am a Police Officer working two jobs averaging around 65 to 80 hours a week, I drive an hour and a half to school, and I am constantly on call with one of the two departments I work for.(I investigate crashes where a fatality or serious injury has occurred) I have been doing very well in school, and I am lined up to take the MCAT in August. I am unable to find the time to fit my clinical hours or volunteer hours in, will my Police career make up for that?

Mr.Smile12

Hello, and thank you for serving our community. It's hard to say whether one can replace your employment experience for a lack of clinical hours. (I'm more certain that many things you do off-duty could be argued for volunteer hours, but that may vary by school.) I would make sure to network with many of the schools you are looking at, presumably ones that are close to you and your family now.

mark-ER

First & most important is that your MCAT and GPA handle the hard cutoffs. Read the forums (though always with a grain of salt), adjust your application (# of schools and medical school tier) accordingly. After that, I think your life story and prior career will VERY much help. Yes, it would be helpful to do some shadowing, such as a 6 month stint doing volunteer work 1-2 hours twice a month for 6 months, just to show you are OK around sick people, but it is understandable if you absolutely cannot squeeze that in given your busy schedule. Are you by any chance interested in forensics, given your prior career track? One option to volunteer and to get a letter from your friendly pathology MD (CME at your local office), and maybe shadow there... There is usually at least 1 or 2 pathologists on AdComs; as a tangential connection and even if you wind up doing something completely different with your career, it might help with admissions.


choice of undergrad institution, choice of major

Hello, i am a 25 year old who has returned to college, i am in my sophomore year at a community college and currently majoring in Business, but 100 percent want to be a Physician.The reason i am getting my BS in Business is that i would love to use it to help hospitals run and with money being a crucial part of our healthcare system in the US figured it would work nicely. In particular when looking for positions such as chief resident. I am taking my pre recs for med school Biology Chemistry Physics and Math and have a 4.0 Gpa in all science courses as well as non science. Due to my good GPA i have the option to attend UC Berkeley as an undergrad. my question is since Berkeley does not have a med school should i choose a different school such as UC Davis or UCLA?that both have top 10 med schools? or will it ultimately not matter that Berkeley does not have a med school when i am applying to med school? would appreciate feedback thanks! just want the best advice to get into med school and fulfill my dream of being a Physician love this forum!

mark-ER

Go to the best school, in a place where you (& your family/significant other) wants to live, plus give some significant consideration to cost. So yeah, that being said if you are a california resident, UC schools are hard to beat. UC Berkley vs. UCLA is a toss-up in terms of education/exposure costs, you can suss out the advantages of one vs. the other. Your key goal is to do well in your undergrad courses maintaining GPA of at least 3.7 and to do well on the MCAT, sprinkling in activities with exposure to medicine as you go; you can do that regardless of whether there is an affiliated medical school, though it does make things easier.

Last random comment, don't mix up being a chief resident with anything to do with healthcare economics. Yes, it has some managerial components, but it is more along the lines of personnel management & HR (micro-micro economics).


Acceptance Rescinded?

I recently got accepted into an osteopathic medical school. The school states on the application prior to submission, that a student who is accepted is required to achieve a B or higher on any coursework completed after acceptance, although this was not stated in my acceptance letter. Recently, I received my grades back and had 2 of my 3 course grades (all 3 of which were pre-req courses) at a C level. I did not perform poorly due to senioritis.

My issues were 3. The first of which was significant health issues (cancer & Lupus) suffered by my mother that required attention. The second was full-time work during school hours in a city located 1.5 hours away from where I live and go to school, in order to provide for my family. The third was attending 5 interviews outside the country (I live in Canada and interviewed in the United States) in an 8 week span from early January to early March. Due to these 3 issues, I did not perform at my typical academic level.

I am afraid that when the school is informed of these grades, that they may rescind my acceptance. Do you believe the school will rescind my acceptance based on the information provided? If so, do you believe that the school will reverse rescinding my acceptance on appeal based on my extenuating circumstances? And do you have any advice for me on the matter (note: my professors are tough and did not give me the opportunity to improve my grade)?

Asking from a student who is having significant stress over the issue and does not want his hard work over the course of 5 years to go to waste based on the performance of 3 courses in 1 unusually difficult semester.

Thank you

Renee_MD

Congratulations on your acceptance! I am sorry to hear of your circumstances and some of these challenges that you faced.

I think the best thing to do would be to contact the school and explain to them the situation. I am not sure if they will make an exception or not, but if you are able to explain your circumstances and what happened, I would think they will be more than willing to work with you. This may be an exception and regular acceptance, a later start or deferment to next year so you can improve those grades. Really the answer will be based on the school and their own policies, however the best thing for you to do is to be open and honest with them.

I hope this helps. Getting an acceptance means they want you at the school! So I would be willing to bet they will work with you to make sure you matriculate!

Dr. Marinelli


Publications on the AMCAS

I am trying to list my publications in the activities section of the amcas and I have them cited in the AMA format. In this format they are over the 700 character length. What should I do to include all of them? Should I give two activities for them or not list all of the authors?

mark-ER

Plug in your Pubmed ID (PMID) and use Mick Schroeder's citation manager (free & useful online tool): https://mickschroeder.com/citation/. I double checked with one of those giant collaborative Gene Atlas publications & yes it will automatically shorten the author list to 3 authors. If you are middle author in a long string, consider listing first 2 authors, then ellipses (...) then your name, then ellipses again (...) and senior author. It is an honest way to represent your contribution. Good luck with your application. [also not sure by what you mean by 2 activities].


2.61 gpa and credit hours 141 should i proceed med school

My gpa is 2.8 and i suppose to graduate this may but i extend and i m taking two extra semester . In summer I am doing internship now what should i do I really want to be doc .. I am also planning to take mcat in summer. For extra curriculum i have 2 years work in ambulance and 4 years work ( 2 month /year ) in community center now what should i do

Renee_MD

Hello,

I think you should wait to apply and try to improve your GPA. Your extra-curricular activities are good, but unfortunately a lot of weight is placed on the GPA (and MCAT) and with a 2.8, it will be difficult to secure an acceptance. I would encourage you to consider post-bacc or extension courses and raise that GPA somewhere to around a 3.5 before you apply.

Good luck to you!

Dr. Marinelli


MD and DO personal statements

I’m going to be applying to both MD and DO programs and I have drafts for both my personal statements. I originally wrote my MD statement first and have whittled it down to fit the DO limit but I’m wondering if I submit two essays that have identical parts will they consider that plagiarism?

Mr.Smile12

By definition, you cannot plagiarize yourself. You can only plagiarize others and claim/co-opt their words as your own. To that end, I don't think you should worry about it. Good luck on your writing.


Letters of Rec

I am interested in applying MD/PhD. I have a fairly competitive application, but my biggest concern is my letter of recommendation from my senior year PI. I thought we had an amicable relationship, but she has consistently ignored all my emails requesting for a letter (4 emails, 2 calls to her office). I even reached out to the PhD student in the lab, and she told me the PI has been extremely busy and hasn't even been that responsive to her (she's currently not on campus because of medical issues or else she would've asked for me). As an MD/PhD applicant, I understand how critical it is to have a letter from every PI you've worked with. However, I am a post-bacc now and am doing far more intensive research at a top-tier school. In terms of a learning experience, my post-bacc has been incredible, and I have significantly more responsibility in lab now than I did in my entire undergrad lab experiences combined. I am getting a letter of rec from my current PI, my PI from sophomore year, and my summer PI, but I'm just not sure if I'll be able to get a letter from this senior year PI by June 5th (when application opens). I'm really worried this is going to be the reason I have a red flag on my application, and it would be really unfortunate because I've worked really hard and I'd hate to not get in just because my PI is too busy to check emails. Any suggestions? How should I prepare to explain this if I can't get the letter?

mark-ER

It won't be a big red flag if you don't have her letter. It sounds like you have plenty others. It's better to have fewer strong letters, than even one lukewarm/negative letter. Your former PI may be in the midst of a busy grant application season, family emergency, who knows... she just went dark-mode, as highlighted by the grad student's reply. And it sounds like you barraged her with a ton of emails/requests. Take a deep breath, wait for a month and check in again with that friendly grad student. See when that PI is back at normal schedule, and then either call or stop by, thank her for mentorship, and ask if she would be willing to write a STRONG recommendation letter (use those words). If you noticed hesitation, gently and casually back out of it.


I am reapplying to medical school but I know that parts of my application are not great. What are some things I can do to make myself more competitive?

I am reapplying to medical school but I know that parts of my application are not great. my GPA is 3.0 and my science GPA is a 2.67 my first MCAT was a 491 but I am retaking it. I took a prep course and studied differently. I’ve done a lot of volunteering and I also am published in research. my GPA suffered a lot because I commuted, worked 30+ hours a week during my first two years of college but I picked it up and made dean’s list for my last two years. The other reason it suffered is bc my family is not supportive in my educational goals BUT i am also the only one who is able to care for them health wise (which is why my MCAT suffered the first time - I had to take care of my family after surgery). How do I explain this in a more eloquent/acceptable way for my application? What are some things I can do to make myself more competitive?

Mr.Smile12

In my opinion, if you have access to a health professions advisor, you need to seek their advice and suggestions. You also should talk with your evaluators and see if they have any thoughts about what to do. In the end, you have to take care of the obvious problems with your GPA and your MCAT scores. Your application won't be part of any conversation in an admissions process without that. You probably should strongly consider a masters-degree postbac program, and that may come with a lot of resistance from your family and the stress that comes with taking care of them.


Strong leadership position, but below-average clinical hours. Damaged application profile?

Just to give my situation some context: I am ending my term as president of my University's student government. My GPA and MCAT scores (just around matriculant median) likely took a small hit due to this 30hr/wk time commitment, however this greatly impacted my involvement in clinical volunteering due to the more consistent time requirements. As of right now, I only have about 20 hours at the local hospital, with much more planned this summer. I got a late start to clinical volunteering because until 6 months ago, I was aiming to apply for an MD/PhD program, so I focused more on research (700+hrs, 3 poster, 1 pub pending). I also have 50 hours of clinical exposure through various shadowing experiences, and 200+ of non-clinical volunteering.

Due to my lack of clinical volunteering hours, some have advised me to take a gap year to boost my hours. How bad does this look in the grand scheme of things? Should I wait to apply next cycle? I plan to hit 150hrs by the end of the summer, but I am worried there is not much weight put into service hours completed just before or after application submissions. Thoughts? (Thank you in advance!)

tantacles

Like many others, I would suggest taking a gap year. Your application is good right now, but it could be great, and it sounds as if you've already assessed the weaknesses of your application. it is always best to apply when your application is completely ready, so take your time and get the experience you need to have the best application possible.


Bad First Year

I am currently a student a low tier state university. My high school career was beyond a wreck and so was my first few semesters at my community college due to suffering from mental illness, abusive home, and addiction (obviously not making excuses just wasn't mature enough to put the effort in to school) I left my community college with a 3.0 gpa and am currently excelling in all my pre reqs and my upper level science courses. However some of my courses from community college that have little to no meaning (mass communications, business) are Ds and there's about 2 or three of them and it's sincerely dragging down my cumGpa. I have very good letters of recommendation, from proffesor and physicians I shadow and continue to shadow. Also have a decent amount of volunteer hours and still have about 2 years left of undergrad that I will continue to do EC. Medical and Osteopathic schools are numbers games and I already know that. Best case scenario I'll be coming out with a 3.3-3.4 cum GPA and maybe a 3.5 science GPA. My question is what are my chances seriously as these are not competitive numbers and I know medical schools don't care for sob stories or excuses, just numbers. Does my upward trend hold any water? Or am I going to have to consider Carribean Schools? Any help and guidance would be amazing. Thanks

tantacles

If you've shown a great upward trend, even allopathic schools are likely within your reach, particularly if you have a great showing on your MCAT. I would suggest that you continue to do well and get your extracurricular activities and research in order so that you'll have the best shot possible.


What information do medical schools have about other acceptances

Can you explain what information medical schools have about the status of their applicants at other schools. I recently learned that once schools make an offer to an applicant, they can also see where else they have offers. This doesn't seem like a widely publicized fact and it really seems like it might hurt applicants. Does access change on May 1st such that schools can see where applicants are in even before making an offer? Do they see the schools that applicants turned down previously or just whatever current school they are holding.

Mr.Smile12

Suffice it to say, if you have multiple acceptances, you need to be fair to other applicants and give up your extra seats as soon as you can. It's not easy knowing that someone else's career hangs in the balance because you haven't decided which school to attend... but that is what is happening. Schools also need to know so they can fill their classes with people who are enthusiastic to start medical school.


Research, pressure, favoritism, and competition in undergrad

I am about to start summer research with my current organic chemistry and organic lab professor (I'm finishing up O chem 2). I don't feel qualified for this research position, since I applied almost a year ago and was accepted before this professor knew how I would do in his organic class (not too well due to some heavy personal issues that came up). Right now I'm doing just ok with a B-, but compared to my fellow straight-A researchers in my class I'm doing quite poorly. These students tend to be favorites of the professors and sometimes I feel we compete for his attention and approval, which I feel I have not earned. Anyway, recently my professor has been watching me like a hawk in lab and critiquing every point of error in my lab technique. I have made some silly mistakes in lab like spilling water all over my bench, likely due to the fact that I am often uptight and nervous for that very reason. While I appreciate the constructive criticism, I am getting stressed by the pressure and the feeling that I have disappointed my professor and that I shouldn't even have this research job. Has anyone else felt the same way? How did you deal with this competition and self-doubt? I truly want to succeed, but the doubt can be crippling.

tantacles

Every job has its difficulties, and it sounds like you are having a hard time adjusting to being directly critiqued by this individual. It is difficult to know whether your professor is critiquing you because he feels that he can improve your work and not because he is critical of you and does not want you in his lab. I would suggest simply doing your best and working hard to perfect your lab technique.


What are my chances for this application cycle?

31 brown female, Nontraditional, US resident, with a master in public health and an international master in Medical Genetics; 6+ years of research experience; 300+ leadership/ volunteer hours and 30 hrs clinical shadowing so far and will continue shadowing/scribe job this whole year. Reapplicant and Retaking MCAT in June for a better score.

Graduated from MPH program in 2015 and submitted primary to AAMC in 2016 with my international bachelor degree but was not successful in terms of course requirements and MCAT scores. So I took all the prereqs in a US university (1 year of each- physics, gen chem, O.chem, Biochem( graduate-level ), psych, Adv Anatomy and Physiology w/ lab plus graduate-level biostatistics, medical anthropology, health policy etc while in my public health program ). Post bacc cGPA= 3.9 and graduate cGPA= 3.8. Did not take general biology courses but getting them transfered.

What are my chances to get it if I applied to medical schools this year? What areas of the application should I work on to be successful? How does being a "reapplicant" affect my chances of getting in? Please help

Mr.Smile12

It's too hard to give any prediction in the absence of a solid MCAT score or specific feedback from admissions officers. I know that as a non-traditional student your access to prehealth advisors may be limited, but it is important to know what your overall GPA is when you include your graduate and postbac GPA's.


Career path for an Anesthesiologist

Currently I am a Junior in High School and I was wondering what would be the best career path to becoming an anesthesiologist. Im trying to get ahead anyway possible and I was wondering if I should take any certain classes to already be one step forward. Right now I am only one class away from finishing my A.A. and I plan on finishing it during this summer while taking a program at my local college to become a CNA. After that I'm not sure what I should do. Should I take an EMT or Phlebotomy class or should I try to become an LPN or an RN and then possibly an CRNA? Or should I just skip all that and just go to get my bachelors in pre-medical and then straight to medical school? Im so confused and unsure what to do any help or suggestions would be greatly appreciated. Thanks!

Mr.Smile12

I wanted to point out the website Explore Health Careers, which will occasionally spotlight a career area, such as anesthesiologist (https://explorehealthcareers.org/career-spotlight-anesthesiology/). One should also note there is a physician assistant career called anesthesiologist assistant that you may want to look into.

tantacles

In order to become an anesthesiologist your first order of business is to get into college. Your major in college is not important; it is simply important that you take the pre-medical requirements and get the highest GPA you possibly can. You can get your clinical experience however you want; being a CRNA may help you figure out if medicine is the right career for you.


Gap Year Plans

I'm a senior majoring in Biology and have a cGPA of 3.4 and a sGPA of 3.29. I am currently trying to figure out my gap year plans I am trying to decide between three options.

Option #1: Enrolling as a non-degree seeking student at a local university and just taking classes individually to boost my sGPA.

Option #2: Enrolling in an online post-bacc program for Medical Neuroscience

Option #3: I recently was accepted at CWRU for a Master's degree (2 year program but can be done in one year), so I would enroll here as another option.

The CWRU is quite expensive so I was hoping to stick with a cheaper option and work from home (hence option 1 and 2), but I don't want to make the wrong decision when it comes to applying for med school.

I also plan on applying this upcoming cycle to both MD and DO schools, taking these courses or doing a program would be a fallback if I do not get in this cycle to apply next cycle with a hopefully stronger GPA.

Thanks

mark-ER

Masters won't improve your uGPA. Post-bac will, and would be an asset, especially if you take at a top-notch reputable program (check forums). Stats, biochem, micro, things like that may be useful & interesting, and your plan for medical neuroscience isn't bad, especially if you have neuroscience research background and/or future career interest in neurology/neuropathology. What I would NOT do is just take community college science courses, just to improve your sGPA.

Glad you are being realistic and aiming for both MD and DO schools, and hopefully your application list is long (even with a rock-solid MCAT, unless you are from top notch ivy league university with engineering/physics major, 3.3 science GPA is quite low and a red flag). Good luck.


Cheaper Options To Raise GPA Instead of Post-Bac?

Help please!

I'll be completing my sophomore year of college in roughly a month. Currently, I have a low cGPA of 3.4 and sGPA of 2.8. My GPA dropped after making a C- in pre-calculus, and I'll most likely end up with a C in Calculus again this semester. Nothing else in my application is a concern (MCAT, extracurriculars, volunteering, etc).

I'm currently double majoring in Molecular Biology (B.S.) and Spanish (B.A.). With this being said, financially I'm not able to double major like I thought I could because it would put me graduating a semester late on top of taking full course work during every summer session until graduation.

I've been advised to choose a major and minor in the other. I love them both equally, but here are the pros and cons:

Spanish Major with Biology and Chemistry Minor: -Only have 7 more classes to graduate -More scheduling flexibility -Study abroad opportunities -Worried about not improving science GPA up (from taking less science credits) enough to be competitive and needing to do post-bac

Biology Major with Spanish: -More science classes to improve sGPA -If I do well enough I might not need a post-bac -Love the department -Behind in major, but will still graduate on time

My main priority is getting into my state medical school (ranked in top 25). I'm having such a difficult time deciding because I want to avoid doing a post-bac or masters at all costs, due to finances. Instead, I'd prefer to work in a clinical setting, do research, or volunteer abroad during my gap year(s). So, if worse comes to worse and I need to focus on improving my GPA, I'll do the post-bac if need be.

But, if there are other affordable options to focus on GPA, like retaking undergraduate classes and staying an extra semester to do that or something similar, I'd love to hear about them. I'd also love advice deciding between majors given my circumstances. 


I'd also appreciate any extra advice. Thank you!

Mr.Smile12

Having to choose your options based on finances is hard though the issue of overall debt is a serious concern. I always recommend talking to admissions officers about your situation with your C grades and whether that would be a real dealbreaker if everything is strong. That said, if you only limit yourself to opportunities due to cost, you need to have a real champion on your side (letter evaluator, admissions officer, prehealth advisor), especially if your GPA doesn't crack 3.5 to 3.6 (depending on the metrics of the school you are really focused on). There are plenty of people who don't properly appreciate the value of the GPA in the admissions process when so many other people have higher academic credentials.

The only person who will have to decide your ultimate major/minor combination or double-major combination is you. How passionate you are about both subjects and learning about both is the real important piece, not how it necessarily looks to others. You only live once and hopefully won't have to regret a road or option not taken.


Low GPA, taking MCAT soon, great resume.... Do I have a chance?

Renee_MD

Hello. Thank you for your question. I am sorry to hear that you had a hard time in college, but the most important thing is that you have really transformed yourself and have become stronger! As you mentioned, the MCAT will be a significant factor in your overall chances of acceptance. There is no 'minimum' score for admissions, but last year, the average matriculant score was a 510. So I usually advise people to shoot for a 510 or above. You can also apply to osteopathic schools whose average MCAT is about a 505, so even if you don't score as high, you definitely can still become a doctor.

In addition, don't let your history of depression discourage you. This is something you had to overcome and did, and is a great accomplishment. You will need this kind of resiliency as a physician, so developing skills to work through problems early on is incredibly valuable.

I wish you the very best! Keep up the hard work!


I'm in Pre-Med but intro to biology (at my university) is difficult for me?

I have a strong love for Chemistry and Biology, but Biology at my university is ridiculously hard. I heard that for Microbio and Animal Bio (I guess it's consider "Bio 2" or "Zoology" you name it) are not easy, however, doable I guess it is a way to put it? I'm taking those next semester. At any rate, I'm a biochem major so I would have to take some biology courses (along with microbio courses and physiology/anatomy which I took during high school, and it was doable as long as I studied by butt off for it), which I'm fine with. But with Intro to Biology it is just insanely hard... they design it to where it's inhumanely hard. I've taken MCAT Biology practice questions and I do well on those. But with the tests for Intro to Biology... it is insanely hard. Should I just give up my dream of being a doctor because intro to biology is hard? I might have a C in Biology affecting my overall GPA to go to a 3.5ish, so will med schools look down upon that? Because this spring semester of freshman year has been rough lol. Please help.

Renee_MD

Definitely don't quit on your dream of becoming a physician! Some courses will be more difficult than others and having the grit and determination to withstand that challenge is what medical schools want to see in their applicants! That being said, since biology is a pre-requisite course for medical school, I would recommend trying to get at least a B in the course. If you can not make a B now, you may want to retake it in the future. Good luck to you! Stay strong and keep working hard! This work is in preparation for the rigors of medical school!


Summer Organic Chemistry at Cal State

Hi! I'm a first-year engineering student at an Ivy League institution, and am planning to take organic chemistry I and II and the associated lab courses at a Cal State school this summer (since my major has a lot of requirements). Would going elsewhere to fulfill my Orgo requirements look bad if I decided to apply to med school?

mark-ER

It is not ideal, though the most important thing is that you do well in the class (i.e. at least a B or B+) regardless where you take it. Basically, O-chem is seen as a 'weed out' class by admin committees to see if you can cut it academically. If you are at a top-tier Ivy League level institution and you take O-chem at mid-tier or lower institution you may be asked the question 'why'. That will be particularly true if you do poorly on the physical sciences section of the MCAT. There are good reasons to do it -- having to be at a specific location for instance taking care of family member who is sick, or taking an extra class while doing an away research internship thru UROP -- and there are bad reasons to do it. Ultimately members of med school adcoms don't have time to tease things out and generally won't interrogate you, but in an otherwise pristine application it may raise an eyebrow, and again you may be asked if you have a good reason for making that choice.


If I got a B in Gen Chem, will I be ok in future sciences (ochem, physics, biochem??)

I'm a fourth year student that decided to pursue medicine after a long period of contemplation. I ultimately decided I should at least give it a shot instead of regretting it in the future. I like bio and am good at memorization but struggle with math and chemistry. I took gen chem this semester and worked the hardest I ever did throughout my four years in college. The material isn't necessarily difficult and I did all the practice problems and homework problems but the tests are really conceptual and nothing like the practice problems, which is why I haven't aced a single exam so far. Ultimately I'll be finishing the course with a B or B+ but I question whether this is the right path for me if I already struggle with general chemistry. Will I be able to do better in Physics and Biochem when I can't even get an A in gen chem? (I go to a top 20 school so I'm just hoping my sciences courses at my postbacc institution won't be as difficult). I ask because Postbacc is very expensive and I don't know if my will and effort alone will help me get into medical school if I struggle in the sciences.

Renee_MD

I absolutely think that you still have a good chance at the other courses! Switching educational paths can be difficult, and as your first chemistry course, you may struggle! You may also struggle in physics and ochem, as most pre-meds do, but the point is try as hard as you can and keep determined to succeed! That is what it takes to get into medical school! If you want to do medicine, then definitely stick to this path but you may want to consider taking only 1 of these science courses a semester so that you can fully concentrate your efforts, and do as well as possible!


Organic Chemistry (I & II) Below Average Performance

Hello. I just wanted to ask for your opinion as to how I should proceed with my upcoming application. My only area of concern on my application is my organic chemistry grades. I graduated from a Top 12 university, have an abundance of both clinical experience and community service experience, and have an above average MCAT grade both overall and within my demographic (82nd percentile, 127 in Chem/Phys, Hispanic). However, I am concerned about the C+ grades I received in both Orgo I and II. While I took those courses as an underclassman and have a significant increasing academic grade trend (finished with a 3.6 GPA and 3.45 cGPA), along with receiving higher grades in both Orgo Lab (B) and Biochemistry, amongst other advanced science courses, I am still concerned that admissions counselors may view these two grades as a major concern and thus disregard my application. Thus my question remains: am I overly concerned about these grades considering the strength of the rest of my application, or should I attempt to take a science course in the fall after submitting my secondaries (would take a course during the summer but would not be possible with my job)? Any response would be greatly appreciated, and I apologize for the lengthy paragraph!

Mr.Smile12

Have you asked anyone at the schools you have considered on how they would consider your organic chemistry grades? If you did rather well in biochemistry and on your MCAT, and other advanced science courses, I think those C+ grades may not be as critical.


International Shadowing of Doctors

As far as undergraduate and medical school admissions, how is a program viewed where students (undergrad) shadow doctors, but the program is international? I have heard international programs are not viewed well, but I am not sure? I wont be practicing any type of care, only shadowing licensed doctors. I will also be pairing this with volunteering in the United States, so my international shadowing wont be the only clinical experience I have. The program says the shadowing hours are coherent with standards from the AAMC, but I have seen writing from the AAMC saying colleges are wary of international experience. Any advice is appreciated.

Mr.Smile12

"Medical voluntourism" is a general concern, especially if you wind up having to do any sort of procedure to patients. That would be generally unethical, and you could get into a lot of trouble. If you just observe, there probably isn't as much of a problem, but you should know that the health care systems at other countries are remarkably different compared to the United States. Now, if you are able to do a proper compare/contrast and link the experience to your overall motivations to pursue medicine, you might be okay.


Can I apply to medical school with pre-med course work taken abroad?

I messed up and took Organic Chemistry I and General Physics I during my semesters abroad. When I returned to my home institution I took Orgo II and Physics II and got As in both. I have already graduated with degrees in biochemistry and math and am now serving in the Peace Corps, hoping to apply to medical school this cycle. But, is it even worth applying, given that these 2 pre-med courses were taken abroad? I have contacted the individual schools I am interested in, and all admissions offices say that pre-medical coursework must be completed in the U.S.

Given that these two courses transferred as equivalent courses to the ones at my home institution, and that I did very well in subsequent courses in these subjects, I kind of want to apply to medical school anyway. I am hoping that instead of automatically discounting my application, some schools might accept me on the pre-tense that I retake one or both of the pre-med courses taken abroad. Or do you think my application will be automatically discounted because Orgo I and Physics I were taken abroad, even though they transferred in as equivalent coursework?

tantacles

It is possible that medical schools will accept these credits. My suggestion is that you look on medical schools' websites and check to see if they accept these types of credits. If you took advanced courses in these subjects, medical schools might also be willing to use those as substitutes for the lower level courses you took.


Re-applicant Advice

I need advice on how to approach this next application cycle. This will be my third time applying. The first time I only submitted one secondary so no surprise I didn't get in. This last time I applied to 14 MD schools and did not get any interviews. I submitted my primary in the first week of June. I didn't submit secondaries till the end of August. I don't know if the long turn around time affected things. I am a white female and Colorado resident. I have a 3.96 GPA and a 509 on my MCAT (128/127/130/124). I graduated in three years from undergrad. I have worked as a CNA since my second year and have over 3000 hours of experience now. I was in a sorority and on an elected committee in the sorority for tow years. I was a TA. I did a senior thesis which was my only research experience. I was also a camp counselor, volunteered with my church, and volunteered with a community outreach program. I am planning on retaking the MCAT but would like to know what else I can do to improve my application. I just started volunteering in a hospital as well.

tantacles

On paper, your application actually looks great. It is hard without knowing all of the details, but there are many things that could be holding back your application. It is possible you did not apply broadly enough, and it is also possible that something in your personal statement or something that you wrote did not sit well with the admissions committee. It is also possible that one of your letter writers did not speak kindly of you. I would suggest that you find an adviser that you trust to divulge all of the details of your application and let them go through your materials to help figure out why you did not get any interviews.


Chiropractic student thinking of medical school

Hi there,

I am currently a chiropractic student at CMCC (Canadian Memorial Chiropractic College). I am thinking of applying to medical school in Canada however, I'm not sure how good my chances are. My cumulative GPA in undergrad was 3.29 (on 4.0 scale) and I haven't written the MCAT yet.

I plan on finishing my chiropractic degree before applying. I have also racked up quite a bit of research experience, extracurricular activities and 4 publications. My references would be strong also. Does anyone have some advice? Do I have a chance?

Thanks!

Mr.Smile12

I'm presuming you are applying to Canadian medical programs. I'm not sure how much they value grade point average, but I suspect they weigh it very heavily. There will also be a question on why you want to move to medicine, given that you chose to go to chiropractic medicine and haven't even given it a shot as a practitioner (i.e., what really is your commitment to medicine if you're going to quit a different health profession). You should talk with admissions staff at the schools you are interested in attending and see specifically what their perspectives would be.


I don't know how to start my Personal Statement

I come from a different cultural background. Being Asian, I was kinda pushed toward pursuing a path in medicine. After taking bio classes and shadowing and volunteering, I really want to pursue a career in medicine as a doctor and stay committed to it. My problem with the personal statement is that I don't know exactly why or what motivates me to become a doctor. It's just that I like science and anatomy and I can see myself becoming a doctor and helping others. I've never had a dream or a goal since I was young, I just know that if I work hard towards something, I excel at it. How do I go about finding the motivation and really just what do I want

tantacles

Some exploration into your own hobbies and goals is probably a good idea. Many people find that taking a year or two after medical school to explore interests outside of medicine helps them to cement their interest. There are many fields that have skills that are applicable to medicine, and it might be valuable to your future career and may help you figure out exactly how you want to proceed.


Lab Tech or Masters?

Hello!

I am currently a senior at BU with a cGPA of 3.2 as well as one year of coursework from UVM with cGPA of 3.63 (but I have had an upward trend for the past two semesters and maintained it at 3.6). Currently, I have 2 semesters left. This semester I am looking at a 3.8-4.0 semester GPA, and the next semester hopefully will be the same. I have yet to take my MCAT, and will plan to take it early next year.

I have recently been offered a lab/research tech job at MGH, however I was also looking at a masters program either in research or MAMS (med science).

I was talking to my current PI who is also on the board of admission of BU Med and she said that if I can maintain 3.8-4.0 for the next two semesters, my disastrous start at BU will not carry nearly as much weight as the last 4 semesters of course work. If the next two semesters go as planned, my new cGPA will be somewhere between a 3.45 - 3.6 (this is up from 2.44 my first semester at BU)

I was wondering which program would be most beneficial to me in terms of admission into an MD or MD/PhD program, trying to bring my cGPA up or working as a lab tech? There have been many variables in trying to formulate my decision and I am lost as to what I should do.

Thanks

tantacles

I would suggest working as a lab tech, particularly since if you are not able to gain admission to an MD/PhD program and you end up going to an MD program, which will likely be easier to gain admission to, you will not receive a stipend and will have to deal with more loans. Both will likely provide you with an equal chance of gaining admission to an MD/PhD program, but the research tech position may be better if only because you might be able to get a publication during your time in the lab, which would further bolster your chances of admission.


Seeking information about writing as a physician

I am an undergraduate student who is considering a career in medicine. However, I currently have limited experience working in a clinical setting. As someone who has struggled with writing in the past, I was hoping to gain some information regarding writing as a physician. Here are a few specific questions I was hoping to have answered:

1. What kind(s) of writing do physicians do on a daily basis? How does one train to do this sort of writing? 2. What kinds of information go into a patient's chart? What components do well-written charts have? Who is the audience you have in mind when writing a chart? 3. What are some of the most difficult aspects of writing for medical documentation?

Thank you for your time.

tantacles

1. and 2. Physicians regularly write notes that document the patient's story, their thought process regarding the patient, and the plan set forth by the medical team. This is what belongs in a chart. Depending on your specialty and level of training, the length, detail, and content of the individual note can change.

3. Medical documentation serves multiple purposes: It acts as a legal document, a billing document, and a reference for the people who will need to take care of the patient later. Fulfilling these three purposes altogether is difficult and the balance between them makes medical documentation an art.


Letter of Recommendation Advice--TA led class, Professor

So the non-science professor I was counting on to get a letter of recommendation suddenly is not available to write me a LOR anymore. I was wondering if a mainly TA-led class, with weekly professor office hours (that I would try to attend regularly this spring quarter) would be advisable for me to get a letter of recommendation from? The issue that I'm worried about is that the professor wouldn't really see how I do in lecture.

Renee_MD

Thank you for your question.

Obtaining letters of recommendation from professors can be difficult as they are very busy and usually receive many requests for letters from students. In some circumstances, it is appropriate to have a TA write the letter but it is crucial to have the professor of the course sign the letter as well. Most medical schools will not accept a letter signed only by the TA. In your situation, try asking the TA to draft the letter and have the professor sign it as well.

If the professor is unwilling to sign it, then I think it would be best to try to get one from another professor.

Good luck to you!


College Valedictorian

Hello,

I was hoping to get some clarity as far as what admissions committees think of college valedictorians. Does it matter at all? I was set to graduate this April two weeks before my last class and was asked to be class speaker, only to find out that I would have been valedictorian if I had all my classes finished. I could potentially wait until next year to walk, and unless someone else gets a 4.0 and has a similar extracurricular profile, I would have the valedictorian. I will not be applying to medical school until June of next year anyway, so I was wondering if it would be worth the wait. I will have my degree, though, just would not have walked.

Mr.Smile12

I think it may be personally impressive, but there are so many strong students applying for medical school it won't really matter. Furthermore, GPA doesn't predict your ability to be a good physician. Congratulations on being a student speaker, and enjoy the opportunity.


Non-traditional Student - Chances of getting in

I am currently wrapping up a Master's in Public Health this Spring. Going into medical school has always been on the back of my mind, but I have never felt prepared or ready to go until recently. I took all of the required pre-med courses in undergrad and had every intention of applying, but didn't feel ready and joined the Peace Corps instead.

After 2 and a half years in the Peace Corps, I returned home and served 2 and a half years with AmeriCorps, before finally deciding to pursue my Masters in Public Health. I'm incredibly happy with the choices I have made, and the path I have taken, but I still have the urge to give medical school a go. My grades aren't necessarily outrageously awesome, but they definitely aren't the worst. I have a lot of personal and professional experience to bolster my resume, and I have to retake my MCATs (my previous score was acceptable, but has since expired.)

Is it still worth it to apply, even though I'm getting older, and won't be starting my career until about age 40? I have also considered pursuing a Nurse Practitioner's license, but am not sure if that is of the same calibre that I am ultimately interested in. Any suggestions or help would be highly appreciated!

Mr.Smile12

With an appropriate academic record and MCAT's, I think you have a strong chance with a medical school program that values your experiences and knowledge in public health and public service. Your task is to find a program that would support students similar to your background and address whatever personal or family concerns you would have in making the transition through medical education and consider the financial support you need to make it happen.


International student from Australia applying to MD programs in the US

Just wondering if someone could give me some insight and maybe clear the road for me a little

I'm an Australian citizen wanting to study medicine in the US. I am an economics major graduating in December. I've taken single units for science pre reqs (chem bio physics and biochemistry) and in regards to extracurriculars I've done two months with the Kurdish red cross on site in Syria and Iraq and 350 hours of shadowing a physician. what do I need to improve my application in regards to extra curriculars? My top pick would be Vanderbilt as I've spoken to their medical admissions and they do accept 8-9 internationals a year so I know it's not impossible. I also messaged them and asked if I should take a year of coursework in the US before applying and they replied back saying they do not recommend it since the language of instruction was in English and I went to the University of Queensland which is ranked in the top 50. I also got a 519 on the MCAT :)

If anyone could give me any insight I would really appreciate it :)

Mr.Smile12

If you aren't interested in a research path (Ph.D.), which would be more friendly for international applicants, you will need to check with each of the medical schools you are interested in. The issue you also need to investigate is financial aid and citizenship requirements for internship/residency. I would network with other medical schools that you are interested in and find out their policies on accepting international students before beginning with the AMCAS application in May.


Is taking the MCAT mid june too late or leads to any consequences for TMDAS/Texas medical schools?

I had originally planned to take my MCAT early May, however after taking practice exams I am averaging around 500 when my goal is a 515+. I did some research and found the median acceptance score for texas medical schools is 508 to 514 depending on the school. I have 300 or so clinical shadowing/ medical internship experience hours 3.94 overall gpa, and a 3.86 science gpa. lacking in extracurriculars, and volunteering at clinics about 50 hrs. If that helps answers in any way. So due to my low scores I was considering of moving my MCAT date to mid to late June to better prepare, would there be any consequences in doing so? I will already have a gap year and I am trying to avoid having another one by not getting accepted this cycle. Also I have not been able to find whether or not and which Texas schools have rolling admissions, would taking the exam in mid june give me a large disadvantage? I plan to start working on my application as soon as I finish taking the exam and have it done by the time I get my scores back.

Mr.Smile12

You should have a lot of information networking with the Texas schools, so hopefully they can give you the answer about rescheduling your MCAT to later in June. I don't suspect it would make much of a difference since you're not moving it more than 2 months. It shouldn't affect your applications outside Texas through AMCAS or AACOMAS.


Concern about my gpa

So I might make my first C this semester in organic chemistry 2. My current gpa is a 3.82 and with a C my gpa will go down to a 3.80. I’m very concerned that the C will ruin my chances of getting into medical school. I am about to be a senior next semester, so time is running out. If anyone has any personal experiences and/or advice I would love to hear it.

Mr.Smile12

In general one C isn't going to sabotage your application, but a general pattern will raise attention. Move on as best as you can, learn from your mistakes, and excel in biochemistry.


Admission chance

Alright I'm going to be honest I heard SDN can be harsh and real and that's what I want, realism. Do not worry, I will apply no matter what is said but I also could use some motivation as I am doing well right now but not well enough, this is pretty much to give myself the feeling of being cornered so that way I perform better in a sense so please be as honest as possible. Ok so here goes... I am a junior in college right now, attending a university in Florida. Up until maybe a year ago I had no aspiration to be a doctor, mainly due to my social skills didn't want to burden anyone by being a doctor with poor bedside manners (there's enough of them already). I wanted to be a neuroscientist, I was pretty smart just no work ethic (slept through AP chemistry and got a C) and very little care for school. Basically I was a class clown in high school. I got to college same effort just more maturity, partied didn't care. Got chronic migraines and actually failed several classes, I think maybe 5-6 Fs over a period of 2 semesters (retook them got Bs and As which I know MD schools will take the Fs, that's fine no DO suggestions as I have researched into it and decided to just work harder for an MD). However somehow stars aligned in some odd way were I found myself in sales (improving my social skills) and then came down with a couple painful and scary bought of migraines with visual aura and I have always been a pretty empathetic person so I started to care for the other people who were suffering more than I was in the hospital. I then regretted my entire academic history. I grew up an orphan and I am also African American I never had stability so it kind of explains my care for school but no excuses of course. And the terms with Fs were primarily when the chronic migraines became uncontrollable. I have 2 true Cs on my transcript one in psychology and one in genetics. All my poor grades were prior to my decision to kill myself (metaphorically) in an attempt of the impossible (which I expect you to reinforce but I'm giving you a lot of ammo so won't be too hard). So here I am a Junior with a diagnostic of a 506 mcat before prepping (transferred from a community college where the Fs occurred to a university where my change occurred) and also I got mentally worn down due to my lack of discipline during the test and honestly just wanted it over 3 hours in. I plan to put myself on a tough training regiment for the mcat to push it north of 520 (my discipline from doing entrepreneurial things in cc kind of indicates that as no problem I would wake up 6:30 every day to work on business plans and stuff). Besides genetics which occurred during my first semester as I had to adjust to being a "good student" also got As in 4 other high level science classes that semester, and also the fact I have straight As in all my high level prerequisites (such as micro, molecular biology, clinical neuroscience (grad course), physiology, neuroanatomy, orgs 1 and 2, physics 1 and 2, biochem) I have been volunteering at the hospital (approx 200 hrs so far). I work as a scribe (I don't know how many hours but a bunch with 11 hour shifts AKA exhausting overnights). Also to throw out hypotheticals and to feed the fire I am about to set myself on I score lower than my diagnostic and get a 505 but lets be nice and run a hypothetical at like 513 something humble. Also I am in the process of publishing some radiology research but its slow because I am busy and worn down, and I am also planning a international trip where I will either help Haitian with like BP(tansyon wo/ba) and pulse (Batman ka) respirations (respirasyon) so basically vitals and vitamins and stuff, or i'll shadow some doctors in Columbia (undecided). What are my chances? I want to help people no matter what it takes I treat this as a life or death situation (hypothetically) don't hold back and be honest please. Sorry it's terribly long wanted to make this as comprehensive as possible. Also I am also applying for the masters in neuroscience as a back up then I will reapply, and I would prefer one of the schools in Florida.

Renee_MD

Thank you for your question.

It sounds like you have had quite a journey to get to this point of applying to medical school! And that's ok! I have worked with many applicants from different backgrounds before applying.

I do think that you have a chance for admission. I think the key to submitting a strong application will be to have a solid MCAT score (510+) and also completing the neuroscience master's program with a good GPA (3.5+). There is no exact formula to say whether these numbers will be sufficient or not to gain admission, but based on statistics and past students, you will be more competitive if you are able to have these achievements on your application.

In addition, I think you have good extra-curricular activities that you should continue your involvement in as much as possible. A radiology publication would look great on your application, and volunteering in the hospital and working as a scribe provide good clinical exposure and patient interaction.

I know the path to becoming a physician is long, but keep pushing! You are obviously doing great things and have a lot of potential, and if you want to be a doctor, then you are doing the right things to become one!

Please feel free to update me with your MCAT score in the future. I wish you the very best!


Necessity of volunteer hours for medical school?

Hi there,

I am currently a scribe in the ER at stony brook hospital. I was wondering if volunteering would be replaced by scribe hours due to the experience gained from the position? The learning curve for this job is pretty big, and I feel as if it will be something I can really speak about in an interview or personal statement. I am also currently trying to volunteer doing something I am interested in, but I am not sure if I will be able to volunteer there as they have not gotten back to me. Would scribing, research, leadership/TA experiences, solid GPA (~3.6 overall, 4.0 science GPA) and a high MCAT score trump volunteer hours? or is that something that is absolutely necessary for any applicant to get into medical school? I have some volunteer hours (~60 from a sports medicine class I took, and a club I was in). Any info is greatly appreciated.

tantacles

Certainly, given your medical experience with scribing, you need not do any medical volunteering. My suggestion is that you reach out to more organizations or non-medical volunteering that you are interested in in case the experience you are seeking does not work out.

In medical school applications, nothing "trumps" anything else. Every piece of your application is important, so I would suggest that you wait to apply until you have all of the components you need to have the best application possible.


Career change from DPT to MD: Pre-reqs and Post-Baccs as a disadvantaged student.

sGPA: ~ 2.5 cGPA: ~ 3.0 last 60 units cGPA: currently 3.4, plan to get to 3.6/3.7 by the end of the year.

I am a disadvantaged student seeking guidance in applying to medical school. Currently finishing up my last year of undergrad at a 4 year university. sGPA is low due to taking all my math and gen chem my freshman year when I was not ready to do so. Took Ochem brief 8a & 8b w/ lab instead of 118. I plan on retaking some math this summer however I am stumped on if I need to retake the entire ochem 118 series or just 118 a & c, also whether I should retake gen chem BEFORE applying to post bac programs. An advisor mentioned retaking these classes at an open campus and then applying to a post bacc would be the path to go.

While I do have clinical experience, I do not have any research experience. I am curious if I should consider research experience while retaking/doing a post bacc program or if my sole priority should be performing well in my post bacc program.

Mr.Smile12

It's not clear from your subject if you already have a DPT or were originally driving towards a career in physical therapy. Before I go into all these details of what you should do, I want to know why you changed and how that is affecting the courses already on your transcript plus others you are planning to take. Your specific questions on organic chemistry are best answered by your advisor since I wouldn't know what each of your organic chemistry classes are.

You should ask many of your questions to postbac programs you are interested in. Research experience is usually not the major issue for admission to postbac programs.


Grade Considerations for AMCAS application

My second semester of sophomore year I had a lot of family and medical issues going on, which caused my grades in two classes to suffer. I retook each of them at a later time and got significantly better grades. I'm wondering if the AMCAS will take these new grades into consideration when looking at my application/GPA because without those new grades my GPA is about a 3.0 and with them it's a 3.5. Is this something I should explain in a personal essay as well?

tantacles

Your new grades will count in your AMCAS application, and they will be averaged with your old grades in those classes.

If you are able to recover from this situation and do well, you may not need to explain two bad grades. The most important thing to do is to show a strong upward trend and do well in your classes in the future and do well on the MCAT.


Three year graduate

Hey So I'm a three year graduate, 3.87 total GPA, 3.9 science, science research experience from march of freshman year, started a club to help the underserved, about 100 hours of shadowing, volunteered two years in summer camp for disabled kids, humanities research from beginning of freshman year MCAT for July 24 should I apply this June? at the end of my second year

tantacles

If you are able to get a good score on the MCAT, you will be in a great place to apply to medical school. I would suggest you verify your application early and as long as your practice tests are going well, strongly consider applying during this cycle.


4 full semester withdrawals due to serious personal problems and applying to allopathic medical schools

Hi,

Thank you so much for reading and taking the time to answer my question! :)

I have FOUR full-semester withdrawals on my record due to serious personal problems.

On one of the withdrawals, my infant died.

The other three were because of my epilepsy...I wasn't a premed at the time and got the three withdrawals after I kept getting seriously hurt. I was having a LOC seizure a few times a week and with that kind of frequency, sometimes I would lose consciousness somewhere dangerous, hence the withdrawals after I would get really hurt, end up in the hospital, need surgery for fractures, etc.. I went to a big state university that really doesn't allow make-up work and would lose so many points after missing a few weeks of school that I would have to withdraw from my classes to prevent myself from getting a semester full of F's.

I went in for an epilepsy surgery eval after the third withdrawal where I got very hurt and was deemed to be the perfect candidate. So, I had surgery for it that same semester to get rid of the damn things and have since been fine seizure-wise.

I then took a long gap (3 years) from school for financial reasons.

1. Unless (God forbid) I were to have some sort of unforseeable tragedy, I will have two full years of full-time coursework before applying. I came up with two years/four semesters in a row because that is the time until I graduate.

1a. Will this be enough to allay any fears that I am at risk of leaving medical school? 1b. Would I need to do a post-bacc as well to be in a good position to get accepted to MD schools? (It would have to be a post-bacc and not an extra year of university for financial aid reasons. My stats are well within MD school range, which is why I asked specifically about MD-granting medical schools.)

2. Four withdrawals is ridiculous and I imagine I should say something on the "Is there anything else you would like to know?". How much should I divulge? Should I just say "Health problems" and "Death in the immediate family" and leave it at that? Or is four so many that it really does need more specifics? How specific should I really get?

tantacles

The most important piece of information for your application is your GPA. Thus, if taking the W's helped your GPA, that was a great choice.

In addition, your story is interesting, and the most important thing is to show that you are stable enough to complete medical school.

That being said, it is hard to say exactly how your application will be received. Your most important goal should be to do well in your future coursework to show you can succeed and to do well on the MCAT.


MCAT date

Hello,

I had my MCAT scheduled for May but I want to reschedule for June because I won't be ready to take it in May. I still plan to submit my primary in June without the MCAT scores so that I can at least get verified. My GPA is more on the low side cGPA 3.57 and cGPA 3.17 (Please see additional info at the end) so I am counting on a decent MCAT score to have a better chance. I am wondering if June 16 vs. June 29 would make a difference considering that the scores are released a month later which means that if I take it June 29 I wont't have a score until July 31st? Taking the MCAT in July 29 would give me almost 2 extra weeks for practice, if I take it June 16 I would have only 3 weeks to take practice exams as I would be finishing content review around May 24. Any advice will be appreciated.

Hispanic 24 y/o female with 1 year of research in a psychology lab for child anxiety and phobia at the university I attend, and 2 years of clinical experience (I work in a mental health community center doing psycho-social rehabilitation with the severely mentally ill population). My degree GPA was 3.76(Psychology BA) not sure if that will be relevant, my current cGPA is 3.57 and sGPA 3.17.

Thank you.

tantacles

Taking your MCAT June 29th is appropriate as long as your application is submitted as early as you hope. That being said, it is important that you make sure that you are completely ready to take the MCAT. Your GPA, while not unacceptable, is low for medical school, so make sure you take plenty of practice tests to make sure that your score falls in a range that will actually make you competitive for admission to medical school.


I volunteer at a hospital, how do I talk to patients?

I am a cope health scholar at a hospital. My scope of practice involves toileting, ambulating, feeding, and assisting nurses and physicians with other patient care tasks. I don’t know how to talk to the patients. I understand that they’re vulnerable and sick but all I can think of to say is “how are you?” or “how has your day been?” and they just seem like inappropriate and awkward questions to ask given their condition. I really just need some advice on how to speak to patients to make them comfortable and some topics that would make for good small talk.

tantacles

I think the approach that you are using is likely a good one. Talking with a patient can sometimes be awkward, and that's why the job that you are doing is relevant, particularly if you plant to be a physician, a career in which many patients can be difficult to interact with. Being cheerful and asking about a person's day without reflecting on their medical condition can be incredibly therapeutic, particularly since many patients spend the majority of their time thinking about the condition that brought them to the hospital. Anything appropriate topic you would talk about with a friend is fair game for these kinds of situations. Talking about tv shows the patient might watch is often a great in to a conversation as tv is often all there is to do in the hospital.


Would postponing the MCAT date affect my chances?

Hello,

I scheduled my exam for May 5 because I wanted to make sure that I get my application in as early as possible. I am a hispanic 24 y/o female with 1 year of research in a psychology lab for child anxiety and phobia at the university I attend, and 2 years of clinical experience (I work in a mental health community center doing psycho-social rehabilitation with the severely mentally ill population). My degree GPA was 3.76(Psychology BA) not sure if that will be relevant, my current cGPA is 3.57 and sGPA I think it is around 3.2 (not good I know) so I am relying on a good MCAT score to really improve my chances. I work full time and have had a few personal issues during these last couple months that have kept me from studying the amount of hours I had planned. I started in January and I am still half way through content review so I don't think(almost sure) I will be ready to take the MCAT in May. My question is; would rescheduling my MCAT for June or ealy July hurt my chances? How late can I take the MCAT considering that the scores take a month to be released and still be ok to apply this cycle? Also, I have read that I can submit my application without the scores, but how would that affect me? Should I just cancel and take it next year to allow more time for preparation?

Thanks in advance.

tantacles

Taking the MCAT in June or early July would be completely acceptable. My suggestion is that you have your application verified prior to your scores being released; that way, once your scores are released, you can immediately have your application fully submitted, and schools can evaluate you right away. Most schools do not start reviewing applications right when AMCAS opens.

That being said, if you do not feel you will be ready to take the MCAT in June or July and it will still be a rush, it is completely acceptable to postpone further and apply next year.


MCAT Score to Outweigh Low GPA?

Hello,

I am currently a junior undergrad student hoping to apply to medical school this summer. My overall GPA is currently a 3.3 and my sGPA a 3.1. I will be taking the MCAT for the first time in June and plan to send my applications in as soon as I get the score. My question is, is there an MCAT score that might "outweigh" my GPA? Also, I am a biomedical engineering major; will med schools consider that I studied a difficult major when looking at my GPA? Given a competative MCAT score, would I have a chance at acceptance or should I consider other backup plans for after graduation?

Thanks.

tantacles

It is possible that a very high MCAT score will outweigh your GPA; that being said, a great MCAT score will be necessary given your GPA.

While schools certainly do look at undergraduate majors when evaluating GPAs, the GPA itself is what truly counts as every student is able to choose what courses he or she takes and has the ability to choose courses in which he or she will do well in if courses prove too difficult.

With all that being said, all schools evaluate GPAs and MCATs differently, so I would suggest purchasing the MSAR (available online) to see just how high of an MCAT score you'd need to be competitive at your target schools. If your GPA for a particular school is below the 10th percentile, for example, I would expect that you'd need an MCAT score of that school's 90th percentile to consider yourself very competitive. However, the process of medical school application is unpredictable, so it is difficult to say exactly how things will turn out.


Biotechnology classes and SGPA

I took 2 biotechnology classes before. Intro into biotechnology and Regulations of biotechnology. Both classes have the code BTEC instead of BIOL. Would they both still be counted towards sgpa or just cumulative? AAMC has Biotechnology under the Biology list but im still not sure if every biotechnology class will be counted towards science gpa or just some of them or none at all.

tantacles

My suggestion is that when you list your courses, you do your best to simply say honestly which classes are science courses based on AMCAS' guidelines. If AMCAS disagrees with you, they may change the categorization of the course, but if the content of the course fits the bill for science, I would list the course as as a science course.


I'm taking calculus-based physics and I want to withdraw from the course to take non-calculus based physics next semester

Hello, I am a first year student with a 3.26 GPA and I am currently taking 17 credits. I am enrolled in General Physics I (calculus based), and I am having a really hard time in it. I spoke to my professor about it and he advised me to drop his course and take Physics I (non-calculus based) next semester. Add drop period is over and I would get a W in the course. Would it be better for me to take the non calc course and drop this one, knowing I would probably get an A in it compared to getting a C in this one? How bad will the W look on my record? The non calc course specifically says for pre-health majors in the description, could I make a case when applying to med school that I signed up for the wrong section?

tantacles

An A is always better than a C, and one W on your transcript will not look bad. I would suggest dropping the course and taking the non-calc based course if you believe you will do better in it.


Will this be considered an alarming downward trend and how do my metrics look?

I am getting ready to apply this upcoming cycle from a top 25 school in California.

I am Transfer applicant, and I did my first two years at a CC and my last two years at UC Berkeley. So my trend during each year of my undergrad is, 3.88-> 3.74-> 3.68-> 3.9.

-I know that during my first three years, I have a small decreasing trend but I hope it’s not too significant of a drop to be of concern. At the CC I had the 3.88 and the 3.74, and at UCLA I had the 3.68 and 3.9.

My overall gpa is a 3.82, and my Science GPA is a 3.74. My mcat is a 513. I also have two W’s on my transcript.

Thank you

tantacles

Your trend looks like you've shown steady work and done well during your undergraduate coursework. You are in a good place to apply. I would suggest you apply broadly to a mixture of reach schools, target schools, and safety schools.


How do ADCOMs look at Pos-Bac students?

Hello, Im currently finishing up my post bac year after graduating with a really low GPA (3.36).. I've managed to pull up my GPA to 3.39 solely based on last semesters work thanks to the 3.9 GPA I got last semester as a post bac. I still have one more semester to go (will be completed in May 2018)

I was just wondering how post bac students GPAs are looked at by admissions? Do they just look at your total GPA including undergrad and post-bac... and still frown upon it if its still low (3.4ish), or do they see that you got a significantly higher GPA as opposed to your undergrad GPA, even though you were taking more difficult classes as a post bac? What if as an undergrad you were finishing up a different major (chemistry), and only took the minimum medical related biology courses, but you clearly do well in the rest of your medically related courses as a post bac?

also if you were generally having an upward trend with the exception of one quarter as an undergrad, does it still count as an upward trend? I would think the one bad quarter would kill your chances of having an upward trend?

tantacles

Every medical school looks at post-baccalaureate study differently, but suffice it to say that an upward trend generally does not go unnoticed. It is hard to say exactly how your particular scenario will play out, but getting a 3.9 every semester in a post-baccalaureate course will look very good for medical school applications. Ultimately, your admission will still depend on myriad factors, like MCAT, GPA, your post bacc, and your extracurriculars, so it is hard to answer your question directly, but right now you're doing the right things to improve your application.


Are there any premed advisers who guide you step by step process in your journey into a medical school I have heard even if you have perfect GPA and MCAT score you are rejected. I know one student who has every single check mark but didn't get any interview.

How to increase your GPA by taking more Science classes ?

tantacles

The pre-medical adviser at your school is probably a good starting point. Truth be told there is no one who can guarantee your admission to medical school as there are myriad factors that go into the admissions process. I would suggest you speak to your pre-med adviser at your school as well as other students who have been previously accepted.


Bombed AAMC FL 2 help!

Okay so I've been studying off and on and my studying method is taking lots of full lengths as I believe best way to score well on the mcat is through lots of practice. I want to take me exam May 5th took NS1 and Ns2 and scored 498-500 and today decided to take AAMC full length 2 to see where I stand and my score was 489 and the weird thing is I got more questions correct in each section than on NS. SOO confused these are the number and the scoring is soo strange to me and seems so low Chem: 123: 28/59 CARS: 120 24/59 BB: 124 34/59 Psych:122: 28/59

On a EK fl I got 38/59 on CARS and got 126 and 127 on NS 1 and 2. I think i might have overanalyzed and not used to AAMC. I'm aiming for 515+. Analysis and tips please??

tantacles

My suggestion is that you continue to study and that you take more practice exams. Each practice exam is just one data point, and there is no way to know exactly where you will score based on one. It sounds as if based on your result you have work to do in every single subject.


I got rejected from the medical school that I sent a letter of intent to, so now can I send a letter of intent to a different medical school?

Hello! After I interviewed at my dream medical school, I sent them a letter of intent. Unfortunately, this dream school of mine recently rejected me. Now, if possible, I would like to send a letter of intent to another dream medical school of mine. Am I allowed to send this letter of intent to my second dream school since my first dream school rejected me? Thank you!

tantacles

Yes. You may send a letter of intent to another school now. Keep in mind that letters of intent do not guarantee acceptance, and it may not help you at the school you send your next letter of intent to.


2.7 G.P.A and about half way done with community college. Do I still have a chance at MD?

First off, let me begin by saying I am a non traditional student with a very unique story. I’m currently 26, but about 4 years ago I decided it was time to get educated so I enrolled in a C.C. At the time of enrollment I was a H.S. drop out, so I started by taking a class or two almost every quarter until I obtained a H.S. diploma. That was seriously tough for me as I never learned proper study habits and I was honestly just an all around dumb blue collar factory worker. On top of all that, I have been struggling with anxiety/depression for some years now which has certainly taken a toll on my grades. My transcript is rocky because of this, it goes up and down like a rollercoaster depending on how I’m feeling that quarter. The good news is that within the past year I have made great strides with this disability and am now starting to feel a sense of belonging. So my question is, if I can keep an upward trend from here on out, can I still be competitive? The only two science classes I have taken are general and o-chem. I still have another year until I finish my AA and transfer to university. Oh and I also left my job to completely focus on my studies. I’m just wondering if on paper I can still be a competitive candidate with an upward trend? I understand to get where I need to be it takes hard work and determination, I just want to be sure it’s still possible to get there. Anways, sorry for the short story.

tantacles

You absolutely have a chance, but you will have to do better in every single one of your courses. An upward trend will help you, but you have little room for error at this point. You will need to get almost all A's in your courses and do fantastically on the MCAT if you want to get into medical school.


Low GPA, Non-traditional Pre Med student, incomplete courses.

As of right now I am a senior Psychology student, and I currently have a 3.195 cGPA. I was a non-traditional pre med student in undergrad, meaning I took some of my pre med courses while pursuing a different major. I actually stopped taking those pre req courses after my sophomore year due to personal conflicts leading me to perform poorly in my classes. I had taken Gen Chem+ labs, Gen Bio, and Ochem+labs. As a result of that, I solely took psychology courses for the rest of my undergrad in order to raise my GPA to what it is now. I was interested in pursuing a post bacc program in order to raise my science grades, since I did poorly in those classes. However, I have not taken physics quite yet or some labs, and I know a lot of academic record enhancing post baccs require that I take physics and labs. Any thoughts?

tantacles

Currently, I agree that you need some GPA repair in order to be accepted to medical school. There are two options open to you once you've finished college:

1. You can do a formal post-baccalaureate program that will require you to take all of the science courses again.

2. You can do an "a la carte" post baccalaureate where you choose the courses one by one that you take.

I would suggest that you do whatever is less expensive and most accessible to you. That being said, if a program you want to go to requires physics, then you will need to take physics, so I would recommend you figure out how best to accomplish that goal.


MCAT Score to Outweigh Low GPA?

Hello,

I am currently in my spring semester of my junior year as an undergrad and my current GPA is a 3.3 (sGPA 3.1). I am a biomedical engineering major. I am planning on taking the MCAT in June and applying immediately after in July.

I have over 50 hours of volunteer hours at a hospital, worked as a research assistant (for credit) for one year (abstract published in journal), and have a leadership position in an academic organization. I have also been involved in my university's string ensemble.

I have no shadowing experience, but I did work in the reception in a medical office for many years.

Basically, my question is, will medical schools consider my difficult major when looking at my GPA? And how high should my MCAT be to even be considered given my GPA?

Thanks!

tantacles

Schools will consider your difficult major, but ultimately, the GPA itself is what matters. Presumably, you had a choice as to what major you wanted to choose and therefore could have chosen a major that you excelled in.

It is hard to say how high your MCAT should be to counterbalance your GPA; I would suggest you purchase the MSAR, available online, and use that to guide you as it will give you the best idea of what MCAT score will be good enough for each school you hope to apply to; each school has a different standard.


Gap Year Options

I graduated June 2017 GPA 3.7 I've applied and have one interview coming up. 2 years of undergraduate research Roughly 300 clinical volunteer hours Roughly 600 CNA hours (this was not on my application) MCAT 498 (I'm already planning on retaking this)

I'm hoping to reapply this Summer, but am not sure what to do while waiting on eventually getting accepted. I'm having a hard time deciding between:

CNA (I already have this certification and am working part-time) Medical Assisting (6 month program) Medical Scribe

I'm continuing volunteering so that's a given. I just don't know if it would be worth the time and money to pursue medical assisting since I'm not sure how long I'll have to work in that position.

Any and all advice is welcome!

tantacles

If I were you, I would not reapply this summer. Take an extra year. This will allow you to show significant improvement in your application before you reapply. It will also give you a chance to retake the MCAT.

If you do choose to reapply, I would suggest that you find a job that you enjoy in whatever field you want. If the job does not involve science or medicine, continue to volunteer and do medical service in the event you don't get accepted.


Bad Start in College, Complete 180 Academically But My GPA Will Stick With Me

Hi, I just joined SDN and have been working on my plan for getting into med school for a couple months, but want to take the opportunity to get expert advice.

I began college at a community college in Virginia in 2016. I was not ready for college at this point, I had no idea what I wanted to study, I wasn't prepared for the distractions of adult life, I had never developed good study skills in high school because I skated through classes easily. I ended the semester with a 1.8 GPA.

I took the next semester off to gain some work experience, settle into adult life a little more, and think about what career I might want to pursue. I'd always been interested in medicine so I shadowed an ER physician I know and from that point on I've definitely been hooked. I began volunteering with a rescue squad and spent the summer getting my EMT certification, as well as taking classes in technical rescue.

In fall of 2017 I resumed my studies with three classes as I finished up my training at the rescue squad. I retook one class I had failed because Virginia Community College System allows GPA forgiveness. I also took my second semester of English and first semester of human anatomy and physiology. I got an A in both English and A/P, and a B in the other class. This semester I am taking A/P part 2, general biology 1, intro to communications, and retaking the math class I failed in my first semester. As of now I have As in all classes and am on track to finish the semester with a 4.0.

I have also been accepted to transfer to Virginia Commonwealth University in fall of 2018, where I will be studying biology.

I have some questions about application considerations since my first semester will be following me for the rest of my academic career.

1. Using GPA calculators I have calculated that it might not even be possible to raise my GPA above a 3.4 (my minimum target) before I finish school. I will be working my ass off to get As, will med schools recognize my success in all semesters or should I expect to really be justifying myself to them considering my first semester?

2. I have heard stats get you to the door, ECs get you through the door. I have some excellent ECs and will be adding more once I move to Richmond, plus I have paid and volunteer experience as an EMT. Additionally I'm a professional drummer in a band that is financially self-sustaining. Will I be able to use that to diversify my application and show I have skills outside of academics and medicine?

3. I have found a real passion for biology. I know a lot of people recommend studying outside of natural sciences to set yourself apart but I truly enjoy biology and would like a degree in the field. What considerations should I make to make myself interesting even with the bread and butter premed degree? Are there biology specializations I can look into that will make my application interesting and still give me some good career options? (For example, a bachelor's in physiology may not be the most practical choice).

4. I plan to apply to all of the Virginia state schools. Am I going to be overshooting or are some of the schools achievable goals?

Thank you so much for your help, I really plan to make this happen and I want to be able to set myself up for success.

tantacles

If you are able to show a positive trend and get mostly A's, particularly in the science courses you take, it will look great on your application and may overshadow your previous difficulties.

You can absolutely include your drumming as piece of your application. Different readers will have different feelings, but the arts are generally viewed favorably.

If you love biology and will be able to get many A's in it, just study biology. It is very rare that a college major sets someone apart, so I would recommend you study what you want and do as well as you can as the quality of your performance will be more important than your actual major.

Applying to all o the virginia state schools is a good idea. I would also recommend you apply to a good variety of other medical schools to have the best chance of acceptance.


3.92 GPA, 517 MCAT but 126 in CARS. Should I still apply to top 20 schools?

Hi, there! I got my MCAT score back earlier this week: 129/126/131/131. I'm very happy with the overall score (517), but I'm a bit concerned about the 126 in CARS. I would like to apply to a couple of top 20 schools, but, browsing through MSAR, I see that a 126 in CARS is either at or below the 10th percentile for all of them (with the exception of Duke). My overall score is basically at the median of most of them though. I am definitely not retaking the exam, and I will apply broadly. Given the rest of my stats, should I even bother applying to top 20 schools? Or would it be a waste of time/money? Thank you so much!

sGPA: 3.95, cGPA: 3.92 ECs (pretty standard, nothing extraordinary): -A summer and two semesters of research (850 hours) -300 hours of clinical volunteering/patient care -120 hours of nonclinical volunteering -40 hours of shadowing -Executive Board member of two clubs

I also identify as a first-generation student and LGBTQ if that matters.

I have seen @Goro, @gyngyn, and @gonnif give awesome advice! Any help would be greatly appreciated!

tantacles

It is hard to say if you have any shot at top schools, but they may be willing to overlook one outlier on your MCAT score given the rest of your strong performance. I would suggest you invest the money to give the schools youre interested in a good shot.


Bioengineering major, GPA 3.3, MCAT 514 -- I know my stats are not great but what are my chances if I apply this cycle?

I have about 200 hrs volunteering at a hospital, I worked in a research lab for three years (wet lab, translational cardiology research), and I have strong leadership experience in student orgs. I know my GPA is very low, but do I have a chance of getting into MD programs in the US? Would a postbac/masters program be a good idea? Also, should I consider retaking the MCAT? I had scored higher on practice tests than I did on the actual day of.

tantacles

Right now you likely have a reasonable chance of acceptance. Your MCAT score is great and may actually be able to help you overcome your GPA. I would suggest that if you apply you apply both to MD and DO programs to have the best chance of acceptance.


Help in deciding next steps

I'm currently a junior at NYU studying neuroscience. I have a 3.515 cGPA and am planning on taking the MCAT in September. I'm getting worried about my GPA holding me back from getting into even mid-tier schools. It's been on an upward trend since freshman year, but still doesn't feel good enough. Should I consider a Master's program to try and boost this before I apply and broaden my chances? I have good extracurriculars and check off boxes in terms of other pre-med resume necessities (shadowing, volunteering, leadership, research, etc). Really conflicted about this and my family is playing a huge role in it too--finances are a concern and so is adding on additional years to the path for them. Any advice is appreciated!

tantacles

Your GPA at this point is acceptable for medical schools. Your MCAT score will ultimately determine your level of competitiveness.

Keep in mind that a Master's degree will NOT boost your GPA for medical school - only undergraduate level courses are counted in the calculation of your undergrad GPA.

It is completely acceptable to take several years off to work before you apply if that will make applying more financially reasonable. I would consider this as it would allow you to boost your application further, particularly if you work in clinical or basic science research and volunteer during that time. This way, your finances would be less of an issue. It is important to apply to medical school first and foremost when you are ready, so do not rush this process.


Should I give up? Low gpa/Bad academic record

Hello SDN experts, I have a 2.3 gpa. Currently, I'm taking some time off school to work. I'm technically a 7th yr now. My academic record has many holes/red flags. --Multiple course retakes with little successful attempts --Multiple academic probation, dismissals, retroactive withdrawals --Multiple bad quarters of below C grades No research/lab experience, few extracurriculars, no standout leadership/awards. I volunteered at a hospital - 813.5 hours. I'm aware of how bad my record is.. but my academic record does not reflect my true effort. I deeply considered my reasons to get into medicine, and I wish to find a way to "start over," if there even is such a thing. What are my options? Is there a chance, or should I just give up? What should/can I do at this point?

Thank you.

tantacles

While you certainly do not have to give up on your dream of working in medicine, a 2.3 GPA and your academic record will make very few medical schools, if any, interested. If you truly wish to be in medicine, you will likely need to take several years to repair your GPA and show that you are able to complete courses with good grades - show an "upward trend", if you will. Keep in mind that this is a difficult task. Medicine is not the only field of work in the world, and it would be prudent to look into other options, even if only as temporary lines of work, as you are extremely unlikely to gain entrance into medical school with your current grades.


Nitpicking Medical School Pre-Med Class Requirements/What are my chances?

Basic Information: - Graduated on time in 2016 at a private university with a BS: Biochemistry and a cGPA of 3.65 and an sGPA of 3.56 (both relatively uptrending) - MCAT: 497 (125/123/125/124); 498 (126/123/126/123); 511 (127/125/129/130) - Asian male

Clinical Experience:

- Emergency Department Medical Scribe: +1500 hours (2016-mid 2017)

Research Experience: - Chemistry research: +500 hours (2014-2016) - Poster and Powerpoint presenter at 2 separate symposiums - Pending publication

Teaching Experience: - General/Organic Chemistry Workshop Leader: +250 hours (2014-2016) - General/Organic Chemistry Tutor: +100 hours (2014-2015) - Organic Chemistry Private Tutor: +100 hours (2015-2016)

Other Experience: - Student Assistant/Supervisor of University Library: +1000 hours (combined); 2013-2016 (promoted in 2015)

Extracurricular Activities: - American Chemical Society Club Officer: +50 hours (2015-2016) - School Symphonic Orchestra Violist: +100 hours (2012-2014)

Hobbies: - Piano: Studied +9 years - Viola: Studied +6 years

Letters of Recommendation: - 3 science professors - 2 physicians (MD)

Notes: - mid-2017-2018 hiatus explained by recent house move and retaking the MCAT twice. Plan is to resume scribing and joining a community orchestra on spare time - Will plan on adding DO schools to application list

List of MD Schools (so far): - Cooper Medical School of Rowan University - Creighton University School of Medicine - Duke University School of Medicine (reach school) - Eastern Virginia Medical School - Geisinger Commonwealth School of Medicine - Howard University - Loyola University Chicago School of Medicine - Oregon Health and Science University (top choice/dream school) - Rosalind Franklin University of Medicine and Science - University of Toledo College of Medicine - University of Arizona College of Medicine - University of Minnesota Medical School (both campuses) - University of Washington School of Medicine - Wayne State University School of Medicine - Wright State University Boonshoft School of Medicine

Main Question: My goal is to get into OHSU. It's been my desire since high school and I think I have a decent shot considering my life-long Oregon residency and relatively solid application (gratefully improved with lots of help from others). Based on the MSAR, my results fit within their GPA and MCAT standard deviations and I believe that I would make a good fit for their school. My only problem is that I did not take a Genetics course in college and it is listed as a required (not recommended) course on their course list. Considering the rest of my application, I would hate for that requirement to be the only thing barring me from entering or even being considered at that school, since I don't know if I have to time to go back to school (even if it's from a community or public college). Will this requirement and my lack of taking the class really prove detrimental to my chances of getting accepted there, or am I simply just overthinking this? Thanks in advance.

Also, any recommendations regarding the list of schools are also greatly appreciated.

tantacles

At this point, you have a competitive application for medical school. That being said, if Oregon requires genetics, I would not make the assumption that they will make an exception for you. This may mean that you have to take a year off to apply to medical school. During this time, you can make yourself more competitive by getting a job in medicine or in research, and you can take just that one course to make sure that you have fulfilled the requirements.


High-School Classes to become an Anesthesiologist

From freshman to senior what would be the best classes to take for a job in the medical field as a anesthesiologist. Along with the best classes what would be the requirements to go into the field and become an anesthesiologist.

tantacles

The most important thing to do to become an anesthesiologist is to get into medical school first and foremost. To do this, you can take any classes in high school that you wish as long as you do well in them. When you are in college, you will need to fulfill all pre-medical requirements (which you can find on the website of the medical schools you wish to apply to) and take the MCAT as well as volunteer, find some experience in medicine, shadow physicians to establish your interest in medicine, and get experience in research. I would talk to your guidance counselor at school; they may be able to point you towards alumni who have chosen to go into medicine.


does my resume look competitive for DO medical schools?

gpa 3.58 no mcat yet, president of the anime club, president of the learning differently club where I tutored people with disabilities, president of the pre med social club, abroad to Nigeria for clinic volunteering, abroad to Kenya for helping people with disabilities, honors degree at my college, shadowed 200hrs, radio director at my church, participated in student leadership academy club, research for a year, clinical volunteering 250hrs

tantacles

Your resume looks competitive for both MD and DO schools. It is impossible to stratify you without an MCAT score, but if your MCAT goes well, you will be competitive for both.


I think I am good applicant, but am I missing something?

I am an older applicant. I am taking the MCAT in May and have completed the required coursework. I went to school right out of high school, but didn't get very good grades because I had no idea what I wanted to do and was just exploring different classes with no clear goal. So, I left after a couple semesters. I built a career, started my own small business, and spent a lot of time in various healthcare/mental healthcare settings. I obtained several certificates and eventually decided, at 27, that I felt called to go to med school. I returned to college and finished a degree. I feel I am a really well-rounded applicant with good experiences (academically and professionally) under my belt and surety that this is what I want and I will work hard for it. I have done a lot of really fun extracurricular things including volunteering and getting involved in my community. I have also been able to shadow a couple physicians and I have enjoyed that tremendously. But, my GPA is only a 3.6 because of the couple semesters I spent in school when I was was younger. Since I returned to school, though, I have had straight A's and achieved a 3.96 GPA in my minor, which is chemistry. I am really concerned about how these older semesters/grades look, and if the things I think add value to my application really do. I am concerned that it will look bad that I didn't finish my bachelor's degree as a younger student; I am 29 now. I think my MCAT scores will be relatively good. Am I being too hard on myself about not being younger or the "perfect" applicant? Will schools really be able to see how far I have come and how hard I have worked?

tantacles

It is difficult to say how schools will view your application. Much of that will depend on how you write it and which admissions committee member reads it.

That being said, your upward trend in your GPA will speak well to your current success and will likely be looked upon positively.

It is hard to stratify your chances without an MCAT score, but if you do well on your MCAT, you should have a great chance of being accepted to medical school.


I'm married and going to a caribbean medical school (AUC) as a female. Pros & cons?

I've been looking over these blogs and can't find exactly what I'm looking for, so I'm just going to layout what I'm going through. I'm 25 and married to a great guy. He is a mechanic and I work as an ascp certified microbiology lab technician. I applied to med school 4 years ago. I had a gpa of 3.8 and an mcat of 26. I had multiple interviews and waitlists, but didn't get in. I was crushed and tried to forget about that dream. Four years later, I realized that growing bacteria and obeying infectious disease docs wasn't going to satisfy my insatiable appetite for learning, I decided to re-apply to med school. The only problem was my MCAT expired after 3 years; I should just retake it, right? No, apparently the test has changed and added new subjects, so my $1500 of study materials are useless. Also, to reapply I'll be 1.5 years older by the time I find out if I "might" get in. I'm broke and find out Caribbean medical schools accept the MCAT for 5 years. I can't take out a loan for more MCAT study materials, but I can take out a loan for medical school. I applied to AUC and was accepted within a week. Every doc I work with is extremely supportive of this type of med school (even though the internet says something different). My husband is supportive and willing to quit his job and continue his education via online classes while I'm in school. Does anyone have any input regarding Caribbean med schools? Is it a lost cause or worth a shot? Also, what about marriage? Has anyone been married while at a Caribbean med school?

tantacles

Going to a Carribbean medical school is typically a bad idea. Carribbean medical schools inflate their match rates to make it seem as if all of their students are matching, whereas in reality, they have a high attrition rate and won't even allow many students to take Step 1. There are many posts about this available on SDN, and I would recommend you would review them.

Being married in medical school and being in a long distance relationship is difficult but not impossible. The most important piece of the puzzle is to speak to your husband about not only what he is willing to do but what he wants to do for his career.


MCAT Advice

I took my MCAT in January and just got my score back. I was not happy with the score that I got, so I think I am going to take a gap year and retake the MCAT next January. I really only studied for three weeks, so for the next time I take it, I plan on studying a ton. I want to start studying over the summer. However, I am a D1 athlete and I am in season in the fall. It's hectic enough being in season trying to manage my regular coursework and extracurriculars, so I feel like I won't be able to study much. Is it worth it to start studying over the summer or should I just take the MCAT at the end of the summer? How would you best recommend to stay fresh with the material while not studying super intensively? I also want to let you know that I will be doing research over the summer at my college, and after season ends, I will be done with college athletics and I will have a lot of extra time in November, December, and January to be able to study.

tantacles

Handling the MCAT is difficult, particularly when you have other commitments. My best advice is simply this: Make sure you have time to study, and do your best to only take the MCAT when you are ready, even if that means you need to delay your application for a year or two to be sure you'll do well.


Why do most of the pre req's for med school have little to do with medicine?

Some of the prerequisites have very little to do with medicine at all. So why not take a class like physics II or Ochem II and replace it with anatomy. I think that it is crazy that we require Ochem II and not anatomy or even crazier we require Physics II over anatomy? Can someone explain this to me?

tantacles

The pre-requisites for medical school are frustrating at times to complete. However, each does have some applicability to medicine, and each tests critical thinking skills. This is something that, assuming you become a physician and have some say over the requirements for medical school, you may wish to change as you continue your career.


Are professors all from sophomore year acceptable to ask for letters of recommendation from?

I am considering asking two of my chemistry professors from sophomore year and a professor from my major for sophomore year. I'm limited in which science professors I can ask as I am a non-science major so I have taken the bare minimum science requirements and they've all been very large. All three have agreed to write letters.

tantacles

There should be no problem asking recommenders all from one specific year as long as they can all write strong letters for you.


anyone is queue for a post bac program

I am looking for information about being the queue for a post bac program...what are my chances now since i was not invited for an interview but i was placed in the queue until May

tantacles

It is hard to say what your chances are as programs have individual selection criteria, and even a waitlist or queue varies by institution. The admissions committee is likely the only body that knows your chances.


Non-traditional applicant with PhD

I have a foreign Bachelors of Science degree (molecular biology and genetics) and just obtained a US PhD (genetics) degree and looking for a post-doc position. I have contributed to 11 peer-reviewed publications. I have close to 100 hours of volunteering and 50 hours of shadowing.

foreign BCPM GPA=2.64 (106 hours) foreign cumulative GPA=2.77 (144 hours) US Postbaccalaureate GPA=3.75 (12 hours) Cumulative undergarduate BCPM GPA=2.75 (118 hours) US PhD GPA=3.20 (41 hours)

1ST MCAT 495 2ND MCAT 505

I found out that some schools don't accept foreign courses or undergraduate degrees (even tough I had my transcript evaluated by WES and transferred courses to my current institution in US). I was also told that my GPA is very low. But the real problem is that I am determined about getting into medical school and becoming physician/scientist (even more determined after being told by a medical school admission committee chair that no matter how many A's I get after this point, I will never be able to get into medical school).

I am considering re-taking prerequisites courses here in the US to increase my overall GPA. I realize this might take a few years as I have to work and provide for myself.

What suggestions do you have for me to make myself a more competitive applicant? Best!

Mr.Smile12

Presuming that your citizenship status in the US is okay, your general GPA and MCAT scores are lower than ideal for certain MD programs. I'm not sure how much DO schools might be able to view your more recent GPA favorably. But I guess, the big question is why you want to switch from a Ph.D. genetics path to a medical pathway. Numbers and grades are important but you will at least get that question.


How bad is it to quit my pre-med job after 6 months?

I started working as a medical scribe in an elite ER 6 months ago. Although I loved it initially, working the required hours during the academic year takes too much time and energy away from my coursework and I might have to quit. How bad would it look to quit after 6 months?

I am a non-technical major and my resume is very light on science classes/traditional pre-med activities. My job as medical scribe was my one impressive pre-med activity. If I quit, does it look like I can't handle it or like I change my mind too quickly (I left 2 research positions after 2-3 months)?

Thanks for any feedback!

Mr.Smile12

Usually anyone doing a medical ER scribing job is advised that it is a full-time job, so I don't know if there are programs nowadays that allow for part-time hours. I don't think it's a great idea to be taking classes and doing scribing at the same time. I do think the pattern of you dropping something after a few months is a very concerning pattern in that it seems like you're not able to handle priority setting and be able to focus on doing a single job well. It's possible that the scribing opportunity while impressive isn't really something that really gets you up in the morning if it isn't inspiring you to reorganize your life so that you can excel in coursework and your scribing, but scribing is a really hard thing to do if it's the only thing you do.


Should I pursue medicine?

I am new here, and I can’t seem to find a post related to this question, so I apologize if it has been posted elsewhere. I will gladly take directions to that thread.

I am currently a 3rd year undergraduate student, and I am very interested in the field of medicine. I am also currently engaged, and I am looking forward to my marriage next year.

My issue stems from my own interests; I am someone who is very much wants a career that I am passionate about (and I love learning about medicine); however, I am also a very family-oriented individual, and my one major goal in life is to have a family, raise my kids, and spend time with my family. As most of you know, there are many times in the field of medicine where these two interests conflict, sometimes very intensely.

I have been looking for jobs that I could occupy once I graduate with my bachelor’s degree, just in case I cannot get into med school. My internal conflict is this: I feel as if I am not committing enough to med school to pursue it, yet I cannot see myself pursuing any other career. I want time with my family, but I would be willing to sacrifice some time for a job that I truly love.

Any advice or questions about the issue are more than welcome!

Mr.Smile12

Your topic comes up in some variation in the forums, so I would suggest that you review the feedback there. I will say that you have a lot on your plate and in your intended new married life. If your major goal in life is to have a family and spend time with them, you will be discouraged from professional education (medicine, law, etc.). There are women who do go through medical education (school, residency, etc.) and can balance starting a family, having children, and so on, but they do emphasize it is hard work, and you have to be mature about it. If you are admitting that you don't feel you're committing enough time to pursue a long-term career goal in medicine, as it seems you really cherish your future role as a family provider, you will probably want to take your time to think it through. Medicine is not the easiest to achieve work-life balance when you're just starting out, though it is possible if you have the resources and support. But those who I have talked to who have made it pretty much agree you have to have as much passion to be a health care professional as you have the passion to be an active parent. From what you wrote, I am not sure if you think you're there yet.


Will my age given my credentials and experience negatively impact my admission to medical school?

I am a Black premedical student and I will be 18 when applying for medical school and I want to know if the following would be enough to make me a qualified medical school applicant or if I would need additional experiences: - 3.68 science GPA and 3.8 overall GPA with 506-508 MCAT Score - Presenter at my university's undergraduate research symposium (2017) - A tutor for high school students in Math and History ~200 Hours - Volunteer at a hospital ~192 Hours - Member of a STEM club devoted to increasing women's representation in the field (3 years membership); we also visited visited the university's cadaver lab and simulation lab - Member of the university's Jewish club where I took part in organizing events for the community such as food drives (Leadership)

Please let me know if my age would play a significant role in negatively impacting my application or if it would at all given my credentials.

Mr.Smile12

I suggest connecting with any admissions officers and diversity officers at the schools you are most interested in. It does appear you have a lot of good items on your resume, but what cannot be determined here is your professionalism qualities when it comes to decision-making, listening and communicating, or working in very challenging situations ethically. (Not saying you don't have those qualities, but from what you have listed, one cannot really comment.) Why medicine if you have a lot of experiences that could also set you up well in a research-oriented path? What other community service experiences have you had that you have returned to over time?


Community college

I graduate in May with a BS in Psychology, I’ve taken all my prerequisites for medical school but my science gpa is trash, I’m studying for my MCATs right now, but once I’ve taken it i want to take a few courses at a CC that i have taken before like Genetics, Anatomy, etc. How will medical schools precieve that? I want to take them at a CC that’s like 5 minutes away from my house and it will be way cheaper. The bio department at my university is terrible so I refuse to take anymore classes there

tantacles

While community college courses will boost your GPA, they are not viewed as positively as courses taken at a four-year college. I would recommend that you take your pre-requisites for medical school at a four year college.


Re-applying only to number one choice before retaking MCAT

Hello, so I was wondering if I could get some advice about reapplying. I am a student from Maine and I am dedicated to the state and interested in only practicing in Maine. Because there are not any allopathic schools in Maine, the best option is Maine Track through Tufts. I applied this cycle to Maine Track and a handful of medical schools but I have not received any interview invites. This is most likely due to 67th percentile MCAT score. Otherwise, I have a great GPA, clinical experience in rural Maine, lots of research experience, leadership positions, and some volunteer work. Right now my plans are to not apply in the upcoming cycle but wait a year to take the MCAT (currently working full time plus a very long commute), and bolster my application with a publication and more clinical experience. My question is, should I apply this upcoming cycle but only to Tufts for the Maine Track option? New aspects would include a publication and more shadowing/volunteer work but I do not have enough time to improve my MCAT score. If I do decide to apply this cycle and don't get in, is it still feasible to apply a third time? I really want to do Maine track so I don't want to ruin my chances by reapplying prematurely. Thank you for any and all advice, I really appreciate it!

tantacles

I would recommend that you wait to apply again. You can absolutely apply to your top choice, but it is always best to wait until your application is fully ready. Taking your time and waiting to apply again is a good idea, and I would recommend not applying just to one school this cycle as it may hurt your chance of getting an interview during your third cycle.


When should I take the MCAT? I'm confused about the Early Decision Program (EDP).

The Early Decision Program deadline is August 1st, and I can only apply to one school. If I take the MCAT on June 29th with the scores released on July 31st is that cutting it too close?

Every week I have to study counts, since I just started my post-bacc program in July 2017 and spend ~40 hours per week studying rigorously on my own. The studying is going very very well, but the only thing is that I am banking on the MCAT since I have a finance degree with a low, 3.0 GPA due to health issues in undergrad.

This is why I feel that I need to apply to many schools, and choosing just one for the Early Decision Program is a complete and total shot in the dark. At the same time, I hear after the early decision, applicants are far less likely to get in to schools. I feel as though I will be ready by June, but 2 months is an extra 300+ hours to study. What would you do if you were in my shoes?

tantacles

I would recommend that you don't apply early decision unless there is literally only one school you could see yourself going to AND your scores are above what you would expect for a matriculant. Currently, your GPA is below average for all allopathic schools, and if you are not accepted to your early decision school you will be at a disadvantage when applying later.

Rather than applying early this year, I would suggest taking your MCAT when you are completely ready to do well and applying next year on the first possible day to multiple schools rather than pinning your dreams on an unlikely scenario that will put you in a bad position if you are not accepted.


MCAT and Application Advice

I am taking my MCAT on May 5th, one week before finals. My cGPA is 3.455, so a little low but with a good upward trend as well as decent ECs. I am currently very worried about getting a good enough MCAT score to make up for my GPA, and I am considering pushing back the date. My goal was to have applications finished and submitted by June 1st with an MCAT score release of June 5th. I wanted to get in early in the cycle so my application had more of a chance. The problem is, I work about 25 hours a week combined with volunteering, shadowing, research, and writing a thesis. I feel like I barely have time to breathe. I know that getting a good score the first time is crucial, however, so how much would my chances decrease if I moved my MCAT to May 24th (or later) to give myself a full 2 weeks with out school to give good focus on studying? (Obviously I am studying every week, but I only have about 3-4 hours to do so.) The release date would then be June 26th.

tantacles

I think the best thing you can do is take practice tests to help figure out where your score is likely to be. This will allow you to figure out the best study strategy. You are absolutely right that getting a good score is crucial, but only you can determine how much work you need to do to make that happen.


I am a US citizen, who completed her undergraduate degree in the UK in Economics, and am finishing my post-bacc in the US. Help with degree conversion?

Hello!

I am applying to medical school this spring, for admission to the 2019 intake and need some advice about degree conversion. About me:

1. I am a U.S. citizen but I finished my 4 year undergraduate degree in the UK , where I achieved a 2:1 in Economics. A 2:1 roughly equates to a 3.0-3.4 GPA. As the program was highly specialized, I finished with a Masters of Arts degree. 2. I then finished my MSc in the UK with merit. I did not take any science classes during my undergraduate or post-graduate degrees. 3. I moved back to the U.S. and started a post-bacc program. So far, I have completed Gen Bio (I and II), Gen Chem (I and II), Biochem, Organic Chemistry, Physics I, Physics II and have a 3.95 GPA.

How do I go about submitting my undergraduate and post-graduate transcripts to US medical schools? Will they acknowledge an international qualification and is this a disadvantage?

Pathdocmd

You do not send transcripts to the individual medical schools, but to AMCAS (https://students-residents.aamc.org/applying-medical-school/applying-medical-school-process/applying-medical-school-amcas/). The UK grades will not be converted in the AMCAS application, but your post bacc grades are excellent. I think the UK undergrad degree would be helpful by adding diversity to the class. A lot will be riding on your MCAT and the rest of your application (e.g., healthcare experience, volunteering, etc).


Choosing a Major Between Pathology and Virology

I am interested in the fields of virology and pathology. I aim to work at USAMRIID (United States Army Medical Research Institute of Infectious Disease). This institute focuses on protecting the warfighter as well as civilians from current and upcoming biological threats including bio-weapons and epidemics. They hire both virologists and pathologists and I was unsure of which to choose since both fields have similarities. From what I have read it seems like pathology would include a lot more than virology since it encompass parasitology, immunology, genetics, microbiology, and even autopsies (which I am actually quite interested in as well and had considered forensic pathology as a career). Due to the many fields pathology covers it seems like that would be the better option however with the goal of working with bio-weapons and epidemics would it not also be more viable to research viruses exclusively? Idk what are your thoughts?

Pathdocmd

There is no major in pathology. Pathology is a medical specialty that requires medical school, 4 years of residency, and most people do a 1 year fellowship after residency. If you are interested in infectious disease I would aim at a PhD in microbiology, which is research, or medical school, then internal medicine residency (3 years), then a fellowship in Infectious Disease (2 years).


PHD vs MD/PHD for Virology or Pathology

I am currently in my first year of college but am planning on majoring in either pathology or virology. I am unsure about weather to aim for the PHD or MD/PHD I have read a bit through the forums but still am unsure of which I should aim for. I intend to build my majors toward the goal of working at USAMRIID (United States Army Medical Research Institute of Infectious Disease). I do not much care for working with patients and I should not have to do that for the most part at USAMRIID however, from what I have read here the MD/PHD is much better anyways even if your only focus is research and lab work. Can anyone give me suggestions on which would be ideal for work at USAMRIID?

tantacles

If you do not care to work with patients, I would recommend, regardless of your future career path, not doing an MD. Doing an MD requires that you work with patients for at least two years, and you will likely be encouraged to do a clinical residency.


Low sGPA/cGPA. Crazy life story. TL;DR, what type of postbacc?

Hello SDN experts! As stated in my title, I have a low sGPA (2.7), and my expected cGPA after graduation this May 2018 will be 3.2. I've applied to a combination of SMP and course-work based post-baccalaureate programs. I've spent the majority of my sophomore-senior years in college commuting back home to deal with some intense issues with family. My story seems to carry some weight with pbacc admissions so far (got into Drexel BMS, UPenn SSP, and CWRU MS in Medical Physio). I have extensive research, volunteering, leadership, and shadowing experience as well. My application (IMO) is complete aside from taking the MCAT and nailing it, and bolstering my GPA's (A LOT). MAIN QUESTION: Which type of program will be looked upon more favorably in my case?

tantacles

If you want to raise your GPA, an SMP will not help at all; your GPA is not raised by master's courses. I would suggest that if you want to raise your GPA, you do a post-baccalaureate course to raise your science GPA before you apply.


Do I have a chance?

This is my first time posting on something like this, so please bear with me. I graduated in 2010 with a BA in psychology and achieved a modest 2.5 gpa. I transferred three times in my undergrad career and couldn’t seem to find a home during that stage of my life. Anyway, I worked for several years and now I’m back in school taking the science prerequisites. I’ve taken all of them at a community college, as money is a big factor. I’m almost done with the prereqs but I have a few upper level bio courses that I’ll take at a four year university. Never taken the mcat and so far I have about a 3.7-3.8 science gpa. I have no extracurriculars, no industry related work experience, however, I do plan to volunteer at a hospice while I study for the mcat. I suppose I should also say that I’m a minority and a Texas resident. What are my chances of getting into a medical school, preferably in Texas?

Thank you

Mr.Smile12

If you're in Texas you should connect with your admissions representatives from Texas schools who can give you better insight on how they may view your situation with undergraduate coursework. You really should try to find opportunities to observe clinical environments whether it's in hospitals or clinics to know what doctors are supposed to do. You should also find faculty or staff who advise prehealth students at your school. They should be able to give you appropriate advice or encouragement.


How can I increase my focus for the MCAT? It is so long.

So, my MCAT is going to be in May. I am applying to med school soon. I need to build endurance starting now. What is the best way that this can occr?

Mr.Smile12

Endurance is gained through practice, focus, and repetition. A runner doesn't get endurance without lots of training on a daily basis, and your mind doesn't work any differently. In general, you need to make sure you can be in a room devoid of distractions and be able to study or focus on your exam. I don't know how you have done already in classes where you needed this focus, but this should be a skill you've developed throughout your classes to date, and a computer exam won't be that much more challenging.

Among many tools would be meditation, which I think has some credibility in neuroscience research. Being able to calm yourself and focus on things for minutes to hours at a time takes practice. There are also quite a few forum posts that discuss techniques people have used to focus, especially with test preparation.

The other thing you need to have is a focused strategy when it comes to studying, and a dedication to that type of study for 90 minutes to 120 minutes (in my opinion). Focus on one or two domains of study at a time. I think the forums have suggested schedules which you can model for that.

Finally, look at the testimonials in Aspiring Docs that address test preparation: https://students-residents.aamc.org/applying-medical-school/taking-mcat-exam/how-i-prepared-mcat-exam/ .


Switched over to Biology/Pre-med, based on the stats provided, what are my chances for med school?

So as you read in the title, I have switched over to Bio/premed from a business major in the second semester of my sophomore year. I currently have a 3.3 cGPA (low I know). I have not completed any science courses yet so I believe that is a positive (so no existing sGPA, taking first science courses this semester). I am taking Bio 1 and Chem 1 this semester at my community college and then transferring to a 4 year university next fall where I will take the majority of my pre reqs for med school. I am ready to do whatever it takes in order to get into med school. I was quite lazy so far in college and did not apply myself much which shows in my GPA, but I am ready to get serious. Also because of my switch, I will not be graduating on time. So is it possible based on my situation? Any advice will help, thank you!

Mr.Smile12

With a clean slate when it comes to science courses, I think you know what you need to do: do real well. Get the science foundation and study skills that will help you with the topics that will ultimately help you in medicine and critical thinking, and gain test taking techniques to do well on high stakes exams like the MCAT and the boards. You can't be "lazy" with your learning or you will be easily overwhelmed. Your situation may be similar to those profiled in the AAMC's Aspiring Docs profiles, so I hope you can remain positive and resilient throughout the tough challenges ahead.


Options to improve application for 2nd Med Cycle

I am a 2016 UCLA graduate with a BS degree in Psychobiology.

I applied to medical school during the 2017-2018 cycle, and I submitted 16 secondary applications. I have not received any interviews. I am looking now to improve my application in hopes of reapplying, preferably in the summer of 2018. I am wondering if a better MCAT and more volunteering would be enough to make me competitive by June, even with a lower GPA profile. I would prefer not to delay another year by taking a post-bac program, but I understand I may not have a choice. I had poorer performances during my first 2 years of college, but I made improvements in my junior and senior year. My academic and extracurricular profile is below.

cGPA: 3.50 sGPA: 3.21 MCAT: 511 (April 2017)

2 Years full-time paid Research Assistant for UCLA Brain Injury Research Center (current) 1 Year volunteer Student Researcher for Spinal Cord Injury Lab at UCLA 1 Year volunteering program at the UCLA Ronald Reagan Medical Center 1 Research presentation at the annual UC Neurotrauma Conference 4 Years part-time employment as a Lifeguard at UCLA Recreation 30 Hours of physician shadowing

Thank you for your time.

Mr.Smile12

You should probably talk with many admissions counselors and your prehealth advising office, but I suspect if you think the problem is your lower GPA profile, then chances are you know what you have to improve unless you are willing to compromise a little. I don't know whether you applied to DO schools or why you wouldn't, but you should try to check whether the schools on your list are a little more selective than you would like or not. You might have to include a few MD schools that are newer and have very large class sizes on top of any DO programs you could probably fit with.


3.1 Overall gpa 510 MCAT

From Oklahoma. Have a 3.1 overall gpa and 510 mcat. Should I do a post bacc? Science gpa is 2.6 for AAcomas and 3.0 for amcas. Do I have a chance? My ecs and letters are already done.

Mr.Smile12

While I don't know where you are applying or what university you have attended is, in general terms, your chances would be better with a postbac master's degree program. I don't know if you just had a rough beginning or a bad ending, but strive for a better science GPA.


Chances at my dream?

Hello all. Ive been reading all the questions on here and I thought Id post my own considering my specific situation. I transferred schools the second semester of sophomore year from Seton Hall University to Montclair Sate University. I am now in my spring semester as a Junior. I wasn't happy at all at SHU, was underperforming there, and it was getting expensive. Here are my stats before and after transferring:

Seton Hall: Biology Major 3.0 GPA 2.Something (higher 2s) sGPA -> Haven't gotten the chance to calculate

Montclair State: Molecular Biology Major 3.9 GPA 3.7-3.8 sGPA (I haven't gotten below an A- on any course since transferring) Haven't yet taken MCAT but scheduled for June

Extra Curr: (with when completed) 300+ hours shadowing (internal medicine MD, pediatrics, internal medicine DO) - SHU+MSU 80 Hours geriatrics emergency department volunteering - MSU 120 Hours SMILE volunteering (food to the poor/homeless) - SHU+MSU Certified Phlebotomist (by time of application should have about 60 work hours) - MSU Certified Clinical Lab assistant (no hours worked yet) - MSU Summer internship cancer diagnostics lab - MSU Arabic Teacher for children - SHU

My main questions are:

1) Am I competitive for MD/DO schools? If so, can you list any? (I have a list but the more the merrier!)

2) How will my transfer affect how schools look at my application? (the averages of both GPAs aren't too flattering and its worrying me immensely!)

3) What can I do before application time to make me a more desirable applicant?

Thank you very much!

Mr.Smile12

There's really no way for anyone to determine your real chance of success in the application process until you take your MCAT. You do seem to have a lot of clinical experience and community service which should work in your favor, but what impact do those activities have on your career choice to medicine remains your responsibility to articulate on your application and in interviews. You need to do your own research on schools where you feel most comfortable and excited to learn as a student, and you can also think carefully about the individuals who will provide references for you. You should talk with your prehealth advisors before beginning the application process for sure.


Do I have a chance?

I'm a Junior at Swarthmore. I want to go to med school but the only thing holding me back is my GPA. I'm a psych major because I want to be a psychiatrist. As of now, my average is a B+, with an upward trend. The only grades in which I have below a B- are my chemistry courses. I'm going to take organic chemistry II and biochemistry somewhere else over the summer because they are so hard at Swarthmore. Hopefully I will do better prove that I am not actually that bad at chemistry. However, my biology grades are B's, which isn't great either. I have yet to take physics. Almost all of my other courses are B+'s or above. I have not taken the MCAT yet. I'm trying to weigh these disadvantages with my advantages. My college's website says since Swarthmore is so difficult, med schools bump up your GPA a little when considering your application. My psych grades are very high. I'm hoping they consider the specific field I want to go into when looking at my major. I was my high school's valedictorian and I'm in the National Honors Society. I got into a very competitive summer research program at Rockefeller University (SURF). I am listed as a co-author in 2 psychiatry journals from the work I've done in the program, have clinical volunteer experience, and might be published in a 3rd paper with a professor I'm working with in the clinical psychology department. I am also half Caribbean/Latina and my mother is an immigrant from Guyana, making me a second generation immigrant. I will take some time in between to gain clinical experience. Through connections I have, I will probably be able to get a job at some pharmaceutical company. Do I have a good chance?

Mr.Smile12

If you don't have a lot of clinical experience, including experiences outside being a psychiatrist, I think it would be very difficult to go through medical school and learn tasks that you probably won't really be using as a psychiatrist. You really should know what doctors/physicians do in as many specialties as possible, because that's what you will ultimately do in your clerkships in most of your training. Is there a reason why clinical psychology (PhD/PsyD) is not something you want to do?

You should also be sure to ask your prehealth and career advisors on campus. They can help you figure out your options.

tantacles

Most medical schools will look at your GPA in isolation without adding or subtracting points for difficulty. Thus, you still have a chance of getting into medical school. Unfortunately, the "mights' in your application will not be helpful until they are solidified. That being said, it sounds like you've done the pre-requisite things to get in but need to improve your GPA.

I can not, however, stratify your chances in any way until you have taken the MCAT. An excellent showing on the MCAT would make your application much more admissible for med schools, while a poor showing on the MCAT would seal your fate and make admission very difficult.


Road to Medical or PA school

Recent graduate with Bachelors in Biology GPA: 3.38 Science GPA: about 3.20

tantacles

It is still possible for you to get into medical school or PA school. However, you will either need to improve your grades with a post-baccalaureate degree or have a stellar showing on the MCAT to have a chance at MD school, though DO schools are still within reach.


How are my stats for med schools, and do I need more volunteering?

Here are some general stats about me:

-3.93 GPA and 3.89 sGPA at Rutgers New Brunswick -Molecular Biology & Biochemistry major and Psychology minor -100 hours volunteered at RWJ Emergency Department -80 hours shadowing an anesthesiologist/pain management specialist at Hospital for Special Surgery, NYC -70 hours of rotations shadowing in departments of oncology/hematology, OB/GYN, Surgery, and ER -I have worked around 15hrs/week in a gene silencing lab as a bench scientist since Fall 2015 -I've been awarded a research fellowship of $570 for writing a proposal for my research -I have yet to take my MCAT (additional question: what's a good score considering my other stats?)

Had a bad dream that 100 hours volunteering isn't enough-- Should I do more?

I plan on applying to med schools this June.

Mr.Smile12

I wouldn't worry too much about your 100 hours of volunteering considering you have another 150 hours in clinical observations or shadowing. You also have a lot of experience working in research labs, though I don't know if your preferred schools want more research. I do wonder what your non-clinical community service/volunteering experience consists of.

As for your MCAT's, you should make sure you have an idea from the schools you are targeting to apply to. They should have their published ranges in the Medical School Admissions Requirement resource or on their websites/recruitment materials.


I know some classes aren’t required for admission into Med School however if you had to pick the ideal schedule for freshman through senior year what classes would you take to help with the MCAT and Med school?

I am a High-School student with only a small amount of college experience and, I’m going to a private school with on a pretty large academic scholarship however, the school doesn’t have as much experience getting kids into med-school so I am trying to at least fix up a schedule that will help with the MCAT and Med-School.

Mr.Smile12

Every college you should be considering has a sample schedule made for those pursuing premedical/prehealth studies, but it also depends on the rigor of your college prep coursework and how much credit you may get before your start as a college freshman.

If you are taking about your high school classes, I think you need to be strong overall with plenty of rigorous college classes (math, science, history, psychology, arts, etc.). What is probably just as or more important is volunteering in the community whether in clinical or community service environments. You should do that volunteering consistently and over a long period of time, and perhaps continue it once you are in college. One should understand what it is like to care for someone with a chronic illness and what the current failings of our health care system are in helping patients (in my opinion).

tantacles

In college, there are pre-requisite courses for medical school: biology, organic chemistry, physics, and inorganic (general) chemistry. Many schools also require mathematics, humanities courses, and psychology courses. The best course schedule to get you into medical school is the course schedule that works for YOU; that is to say the course schedule that will allow you to get the maximum number of A's while still fulfilling requirements for medical school.


Undergraduate in US, Post-bac/Grad School in Foreign country

I am completing my pre-med requirements and my bachelors degree at a university in the US. Before going to Med-school I want to try to explore a little bit and I was thinking of applying to either a post-bac program or a grad-school in a foreign country (also possibly try doing some research there) where a lot of my family lives. I was born in the US and have done schooling up to now in the US. I am still trying to go to school in the US so I was wondering if this would look bad on the application.

tantacles

not going into medical school right away is a viable option, and many applicants explore other careers for many years before pursuing a career in medicine. This is absolutely acceptable. Similarly, doing more coursework or a graduate degree, provided you do well in that coursework, will not hurt your medical school application.


Extended undergraduate education or would a post baccalaureate program be beneficial?

I am a 21 yo currently finishing my 4th year of undergrad. I currently work full time as a scribe in the ED and a cardiology clinic. I have worked full time throughout my college career to pay as much of my way through. I have not had the smoothest undergraduate career but will be graduating next year with plans to began a post baccalaureate Pre-medical program (a.k.a “career changer”) to retake Gen. Chem. and finish the rest of my medical school pre-requisites with a much better GPA than my cGPA which will be roughly a 2.8-2.9 as a 5 year undergrad. My question to you is with now being a "non-traditional" applicant, with a lower cGPA. Would it be beneficial to take a "formal" post baccalaureate program or extend my undergraduate education an additional year to obtain these credits and potential raise my cGPA? I will have an Associate's in Psychology, BS in Individualized Studies (custom degree program at my university), and a minor in women's studies at the time of graduation. My primary reason for considering doing a post baccalaureate program is fear that an admissions committee would "look down" on such a prolonged undergraduate education.

tantacles

Either option that you mentioned is acceptable. Right now, your GPA is not great for medical school, and you definitely need GPA repair to be considered along with an excellent showing on the MCAT.

The length of your undergraduate degree is not too concerning, and should not impact your med school application significantly.


Is biology 1 and 2 required for medical school

I have taken microbiology, intro biology, anatomy and physiology, and planning on taking genetics. Will any of these courses qualify for the 8 hours of biology required for most schools?

Mr.Smile12

Recently, many medical schools have been rather explicit in noting the content that should be included in any biology requirements for medical school, so you need to be sure you read what each school says on the matter or consult each admissions office for schools you are interested in attending. To that end, you may want to focus more on the content you would need to for the MCAT and the kind of scientific or critical thinking skills you need to develop in taking the classes you are selecting as it would help you with higher-order thinking you would need for the MCAT and for medical school.


Medical School with Criminal Record?

I just graduated with Pre-med and now I'm planning on applying to medical school. I have a criminal record that has been EXPUNGED!! When I was 18 and stupid, I was involved in Grand theft which resulted in Felony. It happened in 2014. As my record is expunged, do I have chances of getting in medical school? Or should I just forget about med school?? Please help!! :(

Mr.Smile12

If you have an expunged item on your criminal background, you need to be honest and disclose it even if it is expunged from your record. A criminal background check is required for all health care workers, and failure to disclose would definitely jeopardize your chances. Be honest about your situation, and stay clean. Get your prehealth advisor or team to help you with that.


Statistics and English???

Hello everyone,

      I go to a liberal arts college that requires me to take so many useless core credits such as fine arts, philosophy, history, economics, 6 credits of religion, and this piece of work called "integrative core classes". They have the course title of "RCCs". It's terrible, because instead of taking a class that I really wanted to take like CANCER BIOLOGY; I am forced to take RCC Seminars-- which is just basically reading and writing. 
    MY QUESTION: Do med schools really, really want TWO semesters of English?

Also, I am a neuroscience major and have taken 2 semesters of upper-division PY400 classes for "research methods and behavioral statistics" with accompanying labs-- can these count towards my BCMP gpa? I recently just switched out of Statistics because I found that the class was too much review for me.


Thanks so much everyone!

Mr.Smile12

You will have to check with medical schools about their English requirements, but it is definitely important to note that usually the two semesters of English is part of your graduation requirements for your degree. If you have to take some interdisciplinary seminars, you'll have to take them regardless of your feelings about them. (I'm sure there will be classes and instructors you won't click with when you're in medical school.)

I may be mistaken but I think that your research methods class could count to BCMP if it is classified and verified as such by AMCAS. You'd need to check with them and your institutional prehealth advisors.


Sickness, grades, and MD application

I ended up getting mono & pneumonia finals week my first semester sophomore year. After making up my finals the beginning of the following semester, my grades dropped a bit (i.e, got a bunch of B's and had to retake organic chem2). Is there a place to mention my grades dropped due to illness on the AMCAS application/ if I have strong grades the following years should I be fine?

Mr.Smile12

On AMCAS, this is usually handled in the personal essay, and it may be a specific prompt that shows up in any of your school-specific/secondary applications.


Are my pursuits realistic?

I am a prior Navy corpsman with tons of hands on experience and knowledge. My primary role as a corpsman was a surgical technologist. Currently I still work as a surgical tech and just recently I decided to change my pursuits towards med school pre requisites. For the last several years I have been attending online college knocking out classes with no defined path. I thought maybe I'd be interested in cyber security. However, as it turns out I've been in healthcare too long and learning a whole different trade is just not possible for me at this point in my life. I've come the realization that I've invested so much time in healthcare and it would be foolish to abandon it. For the last 5 years I have worked side by side with surgeons of every specialty. GYN, Neuro, general, ortho, uro, ent. You name it. I have applied to more reputable schools to obtain my sciences and have stopped using my veteran education benefits for the time being. Currently I hold 95 credits from an accredited university. The only issue is as a result of my indecisiveness, my current GPA is not competitive at all. I've maintained a's b's and mostly c's with several d's and most recently a few f's. How problematic will this be seeing as none of these classes are pre requisites to medical school. Is there any work around for this? Assuming that I transfer to a reputable school, maintain a strong GPA, and score high on the MCAT, are my new found pursuits realistic? The one thing I do have going for me is my abundance of clinical experience. Please be as brutally honest with me as you can. Thank you

Mr.Smile12

Not knowing if you have connected with the prehealth advisor at the university where you have been taking classes, but definitely those grades are quite a bit lower than what most medical schools would consider, though if none of them are relevant to a biomedical science GPA, it is not clear exactly how much that would affect you except on a single school-by-school basis. To that extent, have you considered the USUHS Enlisted to Physician program track, and have you contacted them about your potential eligibility to participate? The website for this program is https://www.usuhs.edu/emdp2 .


Are my pursuits realistic?

I am a prior Navy corpsman with tons of hands on experience and knowledge. My primary role as a corpsman was a surgical technologist. Currently I still work as a surgical tech and just recently I decided to change my pursuits towards med school pre requisites. For the last several years I have been attending online college knocking out classes with no defined path. I thought maybe I'd be interested in cyber security. However, as it turns out I've been in healthcare too long and learning a whole different trade is just not possible for me at this point in my life. I've come the realization that I've invested so much time in healthcare and it would be foolish to abandon it. For the last 5 years I have worked side by side with surgeons of every specialty. GYN, Neuro, general, ortho, uro, ent. You name it. I have applied to more reputable schools to obtain my sciences and have stopped using my veteran education benefits for the time being. Currently I hold 95 credits from an accredited university. The only issue is as a result of my indecisiveness, my current GPA is not competitive at all. I've maintained a's b's and mostly c's with several d's and most recently a few f's. How problematic will this be seeing as none of these classes are pre requisites to medical school. Is there any work around for this? Assuming that I transfer to a reputable school, maintain a strong GPA, and score high on the MCAT, are my new found pursuits realistic? The one thing I do have going for me is my abundance of clinical experience. Please be as brutally honest with me as you can. Thank you

Mr.Smile12

Not knowing if you have connected with the prehealth advisor at the university where you have been taking classes, but definitely those grades are quite a bit lower than what most medical schools would consider, though if none of them are relevant to a biomedical science GPA, it is not clear exactly how much that would affect you except on a single school-by-school basis. To that extent, have you considered the USUHS Enlisted to Physician program track, and have you contacted them about your potential eligibility to participate? The website for this program is https://www.usuhs.edu/emdp2 .


Probability of Acceptance/ Ways to Improve

A little bit about myself.. I am 22 years old and in the process of completing my MPH @ George Washington University with approximately a GPA of 3.62. As for undergrad, I attended Dominican University where I earned a B.S. in Neuroscience with a concentration in cellular and molecular neurobiology with distinction, honors through coursework and a minor in chemistry at the age of 20. I was able to graduate early due to my ability to take 8 college courses each semester at once. My G.P.A. is 3.65, and it only went down due to a C in Orgo II.

As for extracurriculars, I have done about everything. Volunteered in Rush hospital for 1.5 years at a surgical unit assistant, Student Senator of student government and eventually treasurer for 2 years, committee member for educational policies at Dominican University for 1 year, clinical preceptorship internship for 1 year (volunteered with E.R. physicians, and shadowed surgeons), student ambassador for 2 years, etc. I also worked as a peer mentor where I acted as a teachers assistant tutoring students both inside and outside of the classroom (2.5 years) and as a research assistant for 1.5 years. This resulted in allowing me the ability to complete a distinction project, where I wrote a 60 page paper to defend my thesis and completed an oral defense. My paper was published in the Constellation and I won a research award. I also work as a wedding planner on the side, and through this event planning I created a organization that allows kids with chronic diseases in hospitals create interesting wedding decor (can't go into more detail).

As for MCAT, I think this is my weakest aspect. I took it the first time and got a 488, considering I did not study and sorta winged it. I am currently in the process of studying for my MCAT, this time for real. I am averaging 500 on the practice tests, and am about to hire a tutor to help me get my score up. I am planning to take it on April 20th, 2018.

I guess my question to you is, do you think I need to change or fix anything in my application? I applied before without my MPH degree and with the bad MCAT score and got denied. I think this occurred because I submitted my applications extremely late in the cycle. I am worried that I might get denied again. I applied in 2016 cycle, and have spent the last year completing my MPH, volunteering abroad in Peru (2x this year) and creating that small organization with the wedding decor. I still have time to add some more things to my application before I apply in June 2018, so what would you recommend? I really want to get in this time and can't afford to lose more time!

tantacles

Your MCAT is currently keeping you out of medical schools. It will be impossible for you to get in without improving it. the rest of your application is fantastic, but your MCAT is absolutely the reason you are not getting in. You must improve your MCAT to have a chance of getting in.


What are my chances into getting into med school and what should I do to increase my chances?

I am a senior getting a bachelor of science in biology with a overall gpa of 3.97 as of right now. I am working on my last semester graduating this May and applying this cycle. I have a Biology GPA of 4.0 but a science GPA of 3.94 due to taking general chemistry 2 in the summer and acquiring 2 B's. I have little volunteer hours , around 50 hrs total at a clinic, but i have around 400 total hours of shadowing and having an internship with a surgeon during the past 2 summers. I have no extracurriculars and little volunteering due to not having much free time working around 20-30hrs a week and caring for ailing family members. My internship included some research aspects as well as leadership with being a role model to the new and young interns. I have 1 withdrawal during the summer before starting university due to family issues occuring. however i took the class in the fall and got an A in it. I am taking the MCAT in exactly 12 weeks from today, (Is that enough? I am very nervous about it.)I just started studying as i am finally taking a lower course load to balance out responsibilities, work, school and studying for it. However I do not know what score I need to acquire I aim for a perfect score but I am not sure what score I should get and not worry about trying to retake the MCAT if I get at least that specific score that will get me into medical school. I am taking 2 difficult courses only this semester since they are all I need to graduate, I am not sure if hours taken affect my application as I have taken over 15 both my freshman and sophomore year but as my family's health declined and to save money for medical school and emergencies I clepped out of free electives, spanish, business, human development and started taking 12-14 hrs my junior and senior year and now 6. I am finishing in a total of 4 years. I do not know how clepping classes affects my application. During my gap year I intend to volunteer and work in medical related occupation. At the lowest my science gpa could drop to a 3.76 0r rise to a 3.95 at its highest while my overall could drop to its lowest at 3.86 or remain at 3.97. I have heard ethnicity and socioeconomic background plays a role as well but I do not know how being white and living in a middle class family affects it. I am knowledgeable in 3 languages fluent in english but knowledgeable to carry a conversation in the other 2. I have also shadowed a doctor for a year in high school before graduating if that aids in the application in anyway. I have tried to provide every possible detail to help. Thank you for all of your time, help, and feedback in advance it is much appreciated.

tantacles

Your application overall is great, and a GPA of 3.76 is very good for medical school applications. It is impossible to tell how you will do; your MCAT absolutely determines your success.

If you do not have much volunteering, I would suggest you make 2-3 hours per week to do some volunteering of some sort. It can be any kind of volunteering.

Unfortunately, I do not know what clepping is and I can not advise you on that topic.


Post bacc or classes at large?

I am a nurse with a passion to switch to medicine. My undergrad GPA was 3.48. I am wondering if I should do a post bacc premed program or take classes at large? What do medical schools prefer? If I did really well in classes on my own at a university as opposed to a full program would they overlook the 3.48?

tantacles

A GPA of 3.48 is likely to get you into medical school with an exceptional MCAT; no medical school will "overlook" your GPA as it will still be paired with your MCAT, but an excellent showing in a post-bacc will absolutely help your chances.. That being said, if you have not finished your pre-requisites, you will have to do those, and I would recommend doing them at any four year college. I'm not sure what you mean by "at large." I will say that there is little difference between doing a formal post-baccalaureate program vs. taking the courses a la carte from a four year university, so feel free to do either one.


What are my chances? Med school admission

Hello, I am a 24 y/o hispanic female who is only been in the states for 8 years. I got my bachelors in psychology about a year and a half ago. I graduated with honors and with a very decent GPA then I started to take pre med requirement courses and my GPA dropped down to a 3.54 which from what I have heard is not great for med school. My science GPA is even worse 3.1.

   I worked full time throughout all my undergrad and I continue to do so. I volunteered as a research assistant for a year at the anxiety and phobia program at the university I attend and by the time I apply for medical school I am going to have worked with psychiatric patients for two years which I would think is better than shadowing a doctor. I want to become a psychiatrist but I am getting a little nervous about my GPA and my chances to get into medical school.  I have mentioned what I have heard is relevant for medical school admission; ethnicity, gpa, and extracurriculars. I did not include MCAT scores because I have not taken it yet. What are my chances to get accepted? What schools should I be aiming to when the time to apply comes (I live in FL)?

Thanks a lot.

tantacles

It is extremely difficult to tell you your chances without an MCAT score. Your overall GPA is good for medical school, but what you get on your MCAT will also determine your chances.

You should apply to all of your state schools and then choose schools that fit your general stat profile if you want the best chance of getting in.


Nursing ( ADN) to MD, how ?, any advise or sugestion would be greatly appreciated

Little about me, I am 29 just graduated from Nursing school with the associate degree in nursing, with GPA 3.7. I always was interested in medicine but living in small town ( Nursing school was the only closest option) and working a full-time job to take care my parents. English is not my first language ( i speak another 3 ) but I didn't have any problem graduating from my college. I started working as LPN and was fascinated by the amount of knowledge our MD has, ( in the facility where I work.) To make this short, sometimes halfway through my nursing school I started to realize that I can do much more than nursing, not to offend anyone, my instructors always complimented me, and advised me many times that I should continue my education and become Nurse Practitioner and that was a goal. As days go by and the more I follow our amazing doctor ( i follow him on my days off just to watch him and learn from him for my own self ) my desire to become a doctor keeps growing bigger and bigger I can't stop thinking about it. Every one I have talked ( mostly nurses and NPs) suggesting take the NP route as this route would be a smarter way, but I am not sure if I will be satisfied when I do become NP, I am not as young as most med school students, I can't afford to make mistakes. Would it be wise to get my BSN in nursing and get few more additional science classes to meet pre req for med school or just switch and major in Biology or chemistry? And those who in med school, does anyone worked part-time ( to help out family) and combined school and studying? ( i know its probably not wise because you will spend most of your time studying but I just wonder ) . I would like to thank you all in advance, I just need to know if anyone els have been through this and how did you manage it to make it, what helped you, and what path you took?

tantacles

The main things you need to do to get into medical school are complete all of the pre-requisite courses and get a bachelor's degree (in any field; it does not necessarily need to be in nursing or a science field.). Followed by that, you will need to take the MCAT and do well. this will likely take several years, though many people pursue medical school later in their lives, even into their 40's.


Trying to make sure i have covered my biology pre requisites, having trouble with how my colleges general bio series

My community college offeres a bio series that is Bio1 - cell/molecular bio, Bio2-animal biology, and Bio3 - plant biology. Each of these courses is one semester. I have taken cell bio (Bio1) and am now enrolled in plant bio. i cannot take animal bio due to course conflicts. Will this cover me for one year of general bio? i have been looking here https://www.cc-seas.columbia.edu/sites/dsa/files/handbooks/Medical%20School%20Requirements%20FINAL.pdf and it seems only case western and Harvard explicitly state that botany will not count. but i have a fear that maybe the definition of general bio excludes plant bio across the board and i have heard some people here and other places say something like this. is this the case?

Mr.Smile12

The problem you will ultimately have is whether the biology you take is the foundation for the MCAT. In general, botany-based content will not be covered explicitly on the MCAT whereas the animal/systems biology would. You should check with prehealth advisors at your local universities that you may be interested in attending if you were to transfer coursework from your community college.


Advice "Mcat score vs course load"

This is my 5th year in college due to changing the plan on applying to PA program to MD. Thus I have had to take required classes that now count as electives for my major. I have been studying full time for the Mcat for the past three weeks and would like to continue doing this until April 21st to hopefully get a high score to boost my application due to my GPA being a 3.6. Though this semester I plan on taking physics 2, genetics and biology 2 while working as a CNA. Would dropping biology 2, graduating then going back and taking the class during the fall, while I applying look bad on my application. This is in hopes of focusing more time on the Mcats and getting a better score. I feel because I have a 3.6, I need to do extremely well on the Mcat and this out-weights dropping a general biology class that consumes alot of time and covers many subjects that won't be on the Mcat itself.

tantacles

Every student has a different load that works for them. That being said, if you are having trouble studying for and taking the MCAT on time, consider pushing it back. There is no race to get into medical school, and if you need another year, it might be wise to wait and prepare your application so that it is at its best when you apply.


SMP Curiosity

How does SMP work? 1. Once you get accepted to the SMP of school X and you finish are you automatically considered for going to X med school for your MD? 2. Does school X count the SMP if it was done from school Y? 3. What are some true SMPs with med schools? 4. I’m not looking to go for D.O. 5. Any other relevant information because I am absolutely clueless.

Thanks in advance

tantacles

Many SMPs offer an automatic interview to those in the program, but that is no guarantee of admission. It is completely possible to finish an SMP without being accepted to any medical school.

BU, Georgetown, SLU, Tufts, and Rosalind Franklin all have SMPs linked to their medical schools, and it might be a good idea to take a look at their programs.


Reapply immediately, or take a break (and what to improve)?

Hello,

I applied this 2018 cycle and completed my application at about 18 schools. I have received 4 rejections, and complete silence from 14 schools. Since it's almost January, I am assuming it's time to reassess and prepare to reapply.

A small snapshot: cGPA: 3.58 AMCAS sGPA: 3.36 MCAT 506: Percentile rank of score: 71% chem / phys: 124, 44% CARS: 127, 83% Bio: 128, 85% Psych: 127, 75%

Residence: California. Undergrad: UCLA: Neuroscience B.S.

I believe in undergrad, I have a significant amount of research / clinical experience, and have since been working full-time as a clinical research coordinator at Kaiser.

I have thought about improving in the following ways: 1. Retake MCAT 2. Try and get the professor letters required (I only have 1 science) 3. Pursue a SMP this coming year 4. Maybe some nonclinical volunteering

My question summarizes too: Should one reapply immediately in the following cycle or wait? Would an SMP be helpful for me? Do I really just need to up my MCAT?

Thank you for your time.

Mr.Smile12

This advice is contingent on other admissions officers and prehealth advisors giving you insight with your application, and it presumes you submitted an early application. You need to really be sure you read the specific preferences for each school you apply to when it comes to their letters. If you only had one science professor letter, you were probably excluded because you didn't have the required number of science letters (presuming 2 or more). You might check if the MCAT or sGPA is lower than par for the schools you have applied to and get advice on retaking the MCAT or pursuing an SMP. They should also help give you advice whether a reapplication is worthwhile in the next cycle or later.


help deciding what to do this upcoming semester..

hi there! i need help deciding what to do this upcoming spring semester. I am transferring from a community college to the university of miami.. I still have yet to meet with an advisor to help me in deciding my track and i am growing anxious thinking of how i can prepare for the upcoming semester.. As of now, the classes i know i will forsure be taking are bio, chem 2 and lab, (no labs available in the spring for bio). what other classes should i take to fulfill atleast 12 credits? i want to also participate in extracurriculars and maybe get a part time job and dont want to burn myself out.. any suggestion on both classes and building my resume? Much appreciated!

Mr.Smile12

I hope your meeting with an advisor will help you with your answers. The prehealth advisors will usually have a list of courses and sequence that will help you keep your coursework with other responsibilities in mind. You also may wind up meeting others who also transferred from community college who share your interests in a health professional career who can give you advice.


What can I do now as a freshman to help my app later on?

So I just finished my first quarter. I am curious to know as to what I can do now, mid-way through school year to help my app. Thank you!

tantacles

I would suggest that you start doing some volunteering once per week (a few hours is fine), continue to do well in your courses, and think about getting involved in some research.


28, clinical research scientist, stellar(daily) clinical/OR exposure/extracurr, avg MCAT (501), low grades, 4x applicant (late apps due to low $), no interviews; how should i best improve my app and do I have a shot

I am 28 years old (URM nontraditional applicant) and I currently a clinical research scientist. I work both in the lab and in the clinic on basic, translational, and clinical research projects. I love it but I know the best manifestation of my work is as a MD. It will help me fulfill my best potential in this realm.

However, Im a current 4x applicant, with no interviews. I have applied late because life circumstances have forced me to divert my money to other emergencies at the time of applications; I dont come from wealth and I fund these very expensive applications on my own. i have an average mcat (501) but my clinical/OR exposure/experience and extracurriculars are at this point, better than most students gaining admissions. Its just that my overall GPA (3.08) stinks. I am confident in saying that as I have had a better CV than my sister who is a current 4th year. She had better grades.

Do I give up? I have been working towards this for the past 10+ years and im so far in that turning around now seems fruitless. I know tons of people that turn around to go get PhDs, RNs, JDs, MPHs, etc after a failed attempt or two. Im too far in and quite frankly, more years in have validated my love and knowledge in this field. I believe I am best fit to train in this profession.

What should I do? My 2015 MCAT runs up after next cycle.

tantacles

Your MCAT and GPA together do not make you a competitive applicant for medical school. Frankly, without an enormous improvement in your MCAT and a great improvement in your GPA, you are unlikely to have success getting into medical school. I recommend that you do not apply next cycle and that instead you take time to do a post-baccalaureate program, if possible, to improve your GPA (this can be done alognside a job) and study for the MCAT. The process of getting into medical school may take several years, but please wait to apply again until your MCAT and GPA are repaired. If they are never repaired, you are simply wasting money.


Retake or...?

I just finished my first term. C- English, C+ Calc 2, and B+ Gen Chem, and A- GE. I know it's not the best, but I am definitely gonna be working my tail off from now on. I have the option to take an F on my English class and take it next quarter and that will boost my GPA .6 points. Should I? Also, during finals week there was a death of a family friend that affected my family and I. What should I do?

tantacles

An F in English will count as a 0.00 on your AMCAS GPA. Taking an F and retaking the course will not be good for your GPA. I would suggest you simply do your best to improve your grade.


Should I give up on my lifelong dream?

Hey! My name is Sou (short for Soukaina). I am Moroccan born, but raised in the USA. I am a Berber (amazigh tribe). I was raised Culturally as a Moroccan. I can speak two languages fluently- Arabic and English. I am also intermediate in French (four years). I am also an artist - painting and drawing is my forte. I painted a painting which was given to the chancellor of PSU Harrisburg and gave a speech to the board of directors based on it. I work part-time as a pharmacy technician at Rite Aid. I have been working there for 3 years as of today. I’m 21 and faced with a dilemma that has been haunting me for a long time. Ever since my 3.0 gpa my first semester I have been depressed. Therefore, that gpa continued for a while. Recently, I picked myself up and started getting 3.6+ semester gpa. I currently have a science gpa of 3.2 and a cumulative gpa of 3.1. I still have around four semesters of school still and was wondering if I should consider another field of work or keep going. I don’t see myself doing anything else. I didn’t take my Mcats yet. I am currently studying for it.

tantacles

You can still do medicine. You need to make sure that you continue to get as many A's as possible in your courses. You also need to do well on the MCAT. It is impossible to stratify your chances without your having an MCAT score and a final GPA. I would suggest that you apply to both MD and DO schools as this will give you the best chance of being a physician.


I am a sophomore undergrad and have 5 c's in my transcript.

I have 5 C's one is a C in US Government i got during high school it was a concurrent enrollment class. 2 of them were C+ In phycology and statistics. I also got a C in math 1050 and chemistry 1210. I will be retaking gen chem but i am worried if this is it for my medical school dream. How can i overcome this and how are my odds looking on getting accepted to a medical school. Im feeling extremely discouraged and sometimes i just want to give up or drop out.

tantacles

The most important thing is your overall undergraduate GPA. Aim to do your best in upcoming courses, and you can absolutely make up for these grades. You will also have to do well on the MCAT to be competitive for medical school.


Starting off my first year of pre-med with a not so perfect GPA...

This is way early, but I just completed my first semester of pre-med and my GPA is a 2.85. I know I need to work very hard for the rest of my years, but do I still have a chance for applying to medical school?

tantacles

You have a chance; you will just have to do better in the coming years and do well on your MCAT.


Near Graduation jitters.. HELP Please!

Graduating in the spring with a B.S. in biology looking to take a gap year! As I near my last winter break as an undergrad I’d love some insight to see where I’m at as an applicant for the 2018 cycle: URM Black male 3.62 cum gpa 508 mcat Clinical research in hospital setting 250 hours Health disparities research on colon cancer and prostate cancer in AA men (hopeful to get at least one publication out before June 2018) Bench research (plant bio 1 year and pharmacology 1 semester) Volunteer at local high school In a competitive stem leadership program on campus Work in a sim center where I assist in setting up class for med students at my campus’s medical school

Would also love some potential schools as to where I would have a good chance to get in. I’m lost as to where to apply to!

Thank you

Mr.Smile12

If you work this spring semester on getting your application together with references that confirm your involvement in clinic, research, coursework, and volunteering, and are on track to submit your application by May/June, you should have a good chance at some schools that you may be reasonably interested in. Work hard this spring to talk to school contacts at recruitment fairs to give you a better idea of what schools would be supportive of you as a medical student. Connect with AMSA and SNMA student members to get a better idea of what schools would fit with your goals and mentor you to success.


Finishing Ms looking to continue education but is med school an option for me? Brutal honesty appreciated

Hi all,

I was wondering if those who are more informed than I could give me some tough love advise on my chances of getting accepted. I decided to do a ms out of college to see what I wanted to pursue and now I'm possibly thinking the M.D. application route if it's a possibility.

Undergrad: 3.0 gpa bio/Neuro double majors with plenty of research. 2 years of garbage 2 years of 3.4 work.

Ms: out of a solidly ranked Neuro program with 3.45 GPA including gross anatomy with full dissection, histology, and medical neuroscience with M1s and a research thesis as well a certificate in teaching biomedical sciences.

Did not take orgo or biochem in undergrad so I took it while in grad school and did well. I'm currently shadowing and studying for the mcat as I finish up the ms at the end of this academic year. 24 yo Hispanic male from Illinois.

Assuming I do at least average (big assumption I'm aware), what is the realistic chance of being accepted somewhere?

Thanks for your time and honesty

tantacles

If you want to go to medical school, you must do well on the MCAT. Assuming that goes well, you have a chance. If you want to go to an MD school, I would suggest GPA repair which will need to happen in undergraduate and not graduate courses. Many colleges offer a la carte post-baccalaureate courses, and those would likely help your application.


Post-bacc options for Canadians?

Hi all,

I live in Canada but completed my undergraduate degree in the U.S at Penn State University (on athletic scholarship) My overall GPA was a 3.48 which is not competitive enough to get accepted to medical schools in Ontario. Additionally, I need some more pre-req courses such as biochemistry, physics 2, and organic chemistry 2 (which will also help me with the MCAT).

There are plenty of post-bacc options in the States but they seem to be really hard to come by in Canada.

Would you recommend doing a masters to improve my GPA and then also taking some of those courses to help me with the MCAT? Or do you or anyone know of programs in Canada that exist for pre-medical students trying to boost their GPA and get their pre-reqs?

I did consider going back to the U.S to complete a post-bacc, but tuition as an international student is extremely expensive.

Thanks so much, any information will be extremely helpful as I feel stuck.

tantacles

A master's degree will not improve your GPA at all; graduate level courses do not affect your AMCAS GPA.

You do not actually have to do a formal post-baccalaureate program to improve your GPA. Taking courses "a la carte" at a college or university near you (not a community college, ideally) is completely appropriate. The courses may help you with the MCAT, but you will likely still require a significant amount of study to do well.


No interview invites

I am a Asian women with 80 + lizzi M. I applied to mix of top 20 and mid tier schools . My secondaries were spread between July and late August. I have had 2 interviews ( 1 already rejected). I applied to 22 schools and nearing chirstmas have no II. I am yet to hear from 10 mostly selective schools. I am preparing myself for gap year. I have 100 + hours of volunteering in India, 2 plus years of research experience ( no publication) and 40 hours of shadowing. Any suggestions on what area I should focus on to strengthen my application next year?

tantacles

It is hard to say where you need to focus for a better application cycle without knowing your entire application. That being said, I would recommend taking 2 years; you have just interviewed for medical school and would likely benefit from a full year to improve your application before putting in a new application.

It sounds like you lack volunteering experience. I assume your 100+ hours was not over the course of a year but rather over the course of a mission trip. I would suggest you find a longitudinal experience to participate in for a few hours a week and do that over the course of the next year while continuing to improve your application with a job, either medically-related or not.


What is my best plan of action?

What SMP or Post bacc programs give you a degree that can actually help you get a job if med/D.O. school does not work out?

tantacles

Most SMP and post-bacc programs are not meant to help with finding a job and are strictly meant to help you get into medical schools. I would suggest doing a MPH or another master's degree if you are not planning to go to medical school.


Best option for non traditional student

I'm hoping to get some advice about next steps I should take in order to be admitted to a MD or DO program.

I am a 2016 UD Davis Graduate with degrees in Spanish and Biology. During my time there I had in my mind that I may want to pursue medicine, but didn't take it too seriously. -__- Regretting that now, of course. I did complete all required pre med courses with a 3.3 science avg.

      Q1. My counselor does not recommend a post bacc where I retake classes because I got all As and Bs. Essentially it would have been better if I did worse, apparently. Do you agree?
     Q2. I have 1 quarter (4 units or 2.67 semester units) of English because I was exempt from taking an upper division class. Can I take 1 semester at my local CC to fulfill requirements? Should I take two online quarter classes instead? 
     Q3.  Will a MSMHS post bacc(assuming I do exceptionally well) be helpful in applying?
     Q4. Taking into account my low GPA, is there a score I can shoot for on the MCAT that will be alluring enough to schools that they will give me a break? I feel confident I can reach into the 90+ percentile. 

Essentially I just need a lot of advice and I am having a difficult time figuring this all out on my own. Thanks in advance for the help!

tantacles

1. A post bacc is still viable to improve your GPA. If you are able to get all As in those courses, it will raise your GPA considerably. You don't have to do a formal post-bacc; you can just do an a la carte post bacc where you take the courses you want.

2. It depends on how the courses are recorded in AMCAS and how many credits the courses give you as well as how the individual schools you apply to look at courses.

3. I'm not sure what an MSMHS is, but I assume it is a master's degree. That being said, master's degrees do not increase your GPA, but it is possible that medical schools will look upon them favorably.

4. Above 510 would be a good start.


Is transferring twice going to hurt me badly?

Hello. I am a freshman at a liberal arts college. I wish to transfer from my current college into a cc, which has a guaranteed transfer to my in-state flagship, after the end of my first semester of financial and family reasons. Is this going to hurt me badly in medical school admission?

Thank you!

tantacles

The most important thing is that you receive an excellent GPA. If you are able to maintain an excellent GPA, you will be in good shape for medical school admission no matter which school you attend.


should I retake pre-PA prerequisites?

Im currently a Junior in college intending to apply for PA school this summer. I am trying to rush my pre-requisits because i need them done prior to application and I need to apply this summer to be accepted fall 2019. Unfortunately rushing isn't working out so well. Im looking at getting B's in almost all my prerequisites this semester. next semester might even be worse. im thinking I should retake my current classes next semester at the community college (its cheaper) because i know i can get an A and I still remember the material. my overall GPA now is a 3.75 at community college and 3.83 at university. I also will be applying with the minimum DPC hours, barely over 1000 by this summer so i will not be competitive there. I just dont want too many Bs on my prerequisite courses. i will list all my grades below.


Biology 1 and 2: 3.0 and 4.0 Chemistry 1 and 2: 4.0 and 3.5 physics 1 and 2: 3.0 and 4.0 nutrition: 3.7 anatomy: 4.0 psychology: 4.0 english 1 and 2: 3.0 and 4.0 (plan to retake eng 1) developmental psych: (current) looking at 4.0 Organic chemistry 1: (current) looking at 3.0 microbiology: (current) looking at 3.0 biochem: next semester physiology: next semester

im considering moving biochem and physiology to next year and retake orgo 1, microbiology, and english 1 next semester. this would be pushing back my application date by a year meaning i can not go until fall 2020. please give me advise on how to be the best applicant. would PA schools rather have somebody who has As by retaking a few or someone with Bs first attempt?

thanks!

tantacles

A GPA of 3.7 is not prohibitive. I would suggest you just continue to do well in the courses you are taking and accept the fact that your grades are not perfect. You will likely still be able to gain an acceptance to PA school with these current grades.


PA school HCE experience question

Okay, so here it is. I have close to 1600 hours working as a medical assistant in a pain management clinic. However, I wasn't a certified medical assistant. In Arkansas you can work without the certification. My first question is, do you think this will look less credible to a board if they see that I wasn't certified when working? Would you advise going back and getting certified as a phlebotomist, CNA, MA, etc and go through the whole process of getting hce experience again? Also, the clinic I worked at recently closed their doors. I most likely will not be able to get a recommendation from the clinic's staff or the doctor. How bad is this? Will I be able to list the healthcare experience without needing a recommendation from them? Thank you in advance for answering.

tantacles

I think it's most important to represent your health care experience accurately. If you worked without a certification and your employment was legal, your work was thus a valid experience.

As far as getting a recommendation, this will depend on the specific program. If no recommendation is available, then you will have to apply without one regardless.


entry requirements

Am a permanent reaident and I have a BSc degree in human biology but was obtained from africa. How are my chances of getting admission into a medical school. Am yet to take the mcat though

tantacles

As a permanent resident of the USA, your chances are entirely dependent on your MCAT score and GPA. Some medical schools also want applicants to have a degree from a US undergraduate. I would check on individual schools' websites to see what their requirements are.


Getting into DO school with ultrasound BA?

I am an echo tech and I have my BA in diagnostic ultrasound. I recently learned that this is not an academic degree (i.e. microbiology, biochem, etc) but rather a vocational degree (i.e. nursing), I am only able to go after DO rather than MD, which I more than happy with. Yet I still have a lot of pre requisites to take like the ochem/ gen chem series and others. I realized that the only way for me to take those at a university is if I enroll in another BA program and spend another 4 years getting a second degree. I was taking the pre reqs at a community college could save me some time, but it probably wouldn't better my chances of getting into med school. If anyone has suggestions on what the best route from here would be or if there are other options that I may not be aware of please let me know! Thanks :)

tantacles

If you have a bachelor's degree, you are eligible for medical school. Any major is all right. I would recommend that you take your pre-requisites at an undergraduate institution. Many places will allow you to take courses a la carte, and this is a completely appropriate way to save money so that you do not have to pay for a second degree.


Any dyslexic doctors out there?

Hello! I am a freshman pre-med student at Texas A&M university. I have struggled with severe dyslexia for quite some time, however, out of sheer determination and utilization of different tactics I have managed to compensate for the extra time that it takes me to study. I have been doing well in my classes so far and am very excited about pursuing a career in medicine. I was just wondering if there are any doctors out there that have dyslexia and have any advice on different approaches they took to getting in?

(Dyslexia does not inhibit intellectual abilities, it affects the language processing center meaning it takes me longer to read- that's it .)

Thanks and Gig'em!

tantacles

There are many physicians with dyslexia; it is a common disorder that does not inhibit one's ability to practice medicine. Many physicians with learning disabilities request extra time on standardized exams.

It might be worthwhile posting on the general pre-medical forums on SDN for more insight and thoughts. Our moderators on this site are very few and are more equipped for answering questions about medicine in general than providing advice based on our personal experiences as we represent a very small sliver of medicine.


How much time should i study for the mcat if i have only background in physics?

Hello ,

how much time do you think i need to invest in studying for the mcat to get a score above 498? I have background only in physics. so i need to study the rest from scratch..

which online course would you recommend and how would you recommend to study?

Thanks in advanced

tantacles

It is hard to say how much you should study for the MCAT; each person has a different threshhold for how much they will succeed. In addition, many sites offer courses for the MCAT, and which one is best will depend on your personal learning style. I would suggest you post in the MCAT section of the general SDN forums for more recommendations as the specifics of which course you should use are very personal.

Mr.Smile12

It doesn't really make sense if you haven't had any coursework in biology, chemistry, or social sciences to try to take a high-stakes exam like the MCAT where that content and its application/analysis are tested. You are underestimating the amount of time and understanding you need in order to be a higher-order thinker to do well. You can probably be okay with some of the online resources to get some basics, but true understanding of a topic doesn't come from just memorizing equations or organic chemistry reactions.


What are my chances of getting into medical school and what schools should I apply to?

Last year when I was about to finish my bachelors in psychology I decided I wanted to go for psychiatry instead so I finished my BA( 3.8 GPA) and I added a second bachelors in Biology just so that I could take the classes that I needed to fulfill prerequisites for med school. I am now taking the last class I need, and my GPA has gone down to 3.5. I did a year as a research assistant when I was doing my psych BA in a program for Child Anxiety and phobia at the university I attend, I did some volunteering at a hospital but it wasn't for too long as I have been working full time all along. I currently work at a mental health center as a psychosocial rehabilitation facilitator, I have daily interaction with mental health patients so I have some hands on experience. I plan to take the MCAT in May but I am worried about my GPA because I have read everywhere that a competent GPA is somewhere between 3.6 and up. What are my chances getting into medical school and which schools should I be targeting at the time of application? I live in FL and am a hispanic female(Not sure, but I've heard this has some influence too).

tantacles

Your GPA is still acceptable for medical school. I would suggest that after you take your MCAT, you follow these steps:

1. Purchase the MSAR, located here: https://students-residents.aamc.org/applying-medical-school/applying-medical-school-process/deciding-where-apply/medical-school-admission-requirements/

2. Make a list of potential schools you will apply to based on your GPA and MCAT combination.

3. Post your list along with your stats and extracurriculars in the "what are my chances" forum on SDN.

It is impossible to advise you right now based on just a GPA.


What should I do next?

I have recently graduated over the summer with a 3.0 cGPA. I have not taken the MCAT nor the pre reqs required to apply to Med school. My hope is to get all this completed now and finally get the acceptance into a program. After searching and browsing these forums for awhile, I have learned some useful information, but nothing that really feels relevant to my situation especially since many of the threads are a bit outdated. I want to get into a post bacc program with conditional acceptance to Med school, somewhere in the a state such as PA, DE, MD. I currently live in Washington state and really need to be closer to my family again. At this point, I kind of feel like I don't know what I should be looking for. I've seen suggestions for pursuing a post bacc program, a DIY post bacc, or an SMP in order to get the desired acceptance. I really just need a clear picture of where I should focus my energy to get the results that I need. Thx for taking the time to help.

Mr.Smile12

I would look for a postbac program that can provide you access to your prerequisite coursework and some upper-level biomedical courses. Some postbac programs may also include MCAT preparation so make sure you look into that.


Do I have a chance at Med School?

I have a low gpa (3.2) but excellent leadership and clinical experience and a goof mcat. will this gpa bring me down? I want to go to med school on the west coast if possible

tantacles

With your GPA and a "goof" MCAT, you have very little chance of medical school. I would suggest a post-baccalaureate program to raise your GPA.


Chemical engineering to pre-med

I am a sophomore with a chemical engineering undergrad major with an emphasis in biochemistry. I have decided that I want to go to medical school after my undergraduate, however I have a lot of questions, and need advice from people who have experience in this!

1. My GPA is 3.49/4, which isn’t horrible for chemical engineering. However I have heard that GPA is very important on the applications for medical school. How will this be taken into account? Are my chances lowered with this GPA?

2. Chemical engineering is considered one of the hardest majors at my university, and it definitely isn’t a rumor. It’s a very difficult major and it takes up a lot of my time. I am stressed a lot about my classes often, which isn’t anything unusual for me, but I know that I need to be getting in a lot of volunteering and shadowing for medical school applications. Would it benefit me to change my major to biochemistry?? (which I assume would increase my GPA and give me more time to devote to extracurriculars for applications) or is it better to have a lower (but not low) GPA in ChemE?

3. I work as a pharmaceutical technician, and just out of curiosity I was wondering how this would be viewed on a med school application?

4. What are your general “most important things to know” for pre-med students and for the med school application?

Thank you for any help! I truly appreciate your time!

tantacles

1. and 2. A 3.49 is not a great GPA for medical school. Keep in mind that your major does not matter to medical schools as long as you do the pre-requisite courses. If you will do better in a different major, it would be beneficial for medical school applications to change your major, whether you change to biochemistry, English, music, or French.

3. It will be viewed as work experience and will help your application.

4. Use SDN's pre-medical section judiciously; it can help you figure out your extracurricular activities, which are quite important.


Will I be setting myself up for bad grades with taking Anatomy, Pre Calculus, Chemistry 2, and Genetics in one semester?

Hello, I am a Sophomore/Junior in Undergrad, so far I've taken Biology, Chemistry 1, College Algebra, Nutrition and all other basic courses. I am a Pre Optometry Student and I work a Casual Part time job. I am aiming to possibly take Anatomy and lab, Pre Calculus, Chemistry 2 and lab, with Genetic and lab. I am the most worried about taking Genetics with only have taken Biology for majors. Please give me advise on the pros and cons of this schedule.

Thank you

tantacles

Every person has a different threshold for what they can handle. A normal courseload is absolutely acceptable for medical school, so if this represents more than you can handle, I would drop 1-2 courses and see if it's manageable.


Pre-Med? Nursing? Please Help!

Hi, I am currently a freshman in college. I am an Art History major on the pre med track. I have always wanted to become a surgeon. It is my dream job. But recently, I am heavily struggling with Chem 1. I have a D in the class and I am thinking about dropping the class (I will). If I can't even get through Chem 1, how am I going to get through Chem 2, Orgo, Biochem etc?... I've never been good at science and my family thinks that I shouldn't be a STEM major. But I just really want to be a doctor.

    So recently, I got interested with the nursing program at my school. I really want to work in the medical field. I talked to the advisor and all but I am not sure if I should do pre-med, nursing or just Art History. I love Art History and it's so fun but I know I'll regret it if I don't even attempt for a career within the medical field. I'm thinking maybe I should not be a STEM major. Some people told me that sometimes you can't do what you really want to do because you are not gifted in that particular area. 
    Honestly, I want to be a doctor but I know I'm not smart enough and that I'll doubt myself every moment as I start taking harder science classes. I like nursing too but I know nursing isn't easy and I may not get accepted to the nursing school at my university.  If I prepare to become a nurse, I'd like to graduate with my BSN, experience being a nurse and go to graduate school to become a PA. If I want to become a surgical nurse, what do I have to study and do?
    This is so hard because if I decide to do nursing, I'll have to give up Art History and Pre-med and totally shift gears. I really don't know who to ask because no one around me is in the medical field and my advisor only tells me what classes I need to take and all.. Please please help!
   Please excuse me if I said any wrong information or if I post this in the wrong place.

tantacles

If you feel that you are not able to do science well, you do not have to be a STEM major. I always suggest that people do what they're good at. Sometimes, we love things that we are not good at, and it creates conflict in our lives. If you want to be a nurse, all you need to do is finish a nursing degree. If you want to be a PA, you will have to finish the same pre-requisites as a pre-med student, which may get in your way. That being said, it may be that if you get the proper help you will be able to succeed. This decision is exceedingly personal, which makes it hard to give real advice. You have to figure out what you want to do and what you are able to achieve and look closely at where those goals match and where they differ. Therein is one of the hardest parts of life, but it will make your decision for you once you have the right information.


Does dropping organic chemistry alter chances of admission?

Alright, right now i am in my sophomore year 1st semester. I have organic chemistry 1, intro to psyc, genetics, med terms B and precalc. I am thinking I will have to drop organic chemistry, as I am currently at a C and seem to be trending downwards. I don't have enough time to donate to organic chemistry, as I just got a job as a laboratory tech that requires 20 hours a week without weekends. I am struggling to keep up in the class because of my job ad the amount of credits I have, so I am thinking about taking it over the summer. The only thing I am worried about is that this would be my second class that I drop, and I am worried that my application to medical school will be affected by the drop. Otherwise, my GPA is 3.58 and I have had my CNA since highschool with 1 years worth of hours, I worked with children in an afterschool program for a year, and I currently work as a lab tech. I also have a summer temp job at 3M and am in biology club. How does my application currently look, and will dropping organic chemistry affect me in the long run in regards to acceptance to medical school?

tantacles

Dropping organic chemistry, provided you take it again later, should not negatively impact your chances at medical school. You will have to do well in this course later, though, just as you will have to do well in your other courses, to get into medical school.


I have been dismissed from a previous university. I have taken a year off to reassess and better myself. I want to apply to Case Western... what are my chances?

I went to Miami University. I first had an academic dismissal for one semester. I tried my best to appeal it to the court, however, it did not work out. I came back to school and was again filled for an academic dishonesty (all because I did not site a rough draft paper). Again, I tried to appeal it, but failed. So, I took the year off. I began to work as a scribe and bettered myself. I want to finish my medical studies. Note, the dishonesties I had gotten were in Humanity/Arts classes, nothing relating to any of the science courses I took. Anyway, I am from Cleveland. I would like to return to school again as a transfer student and finish my degree. What are the chances I can get into Case or like Ohio State?

tantacles

Academic dishonesty is often a career ender for medicine. At the very least, I would recommend taking several years off to improve your application before applying to medical school. When you apply, you must report these charges of dishonesty, and I would recommend that if you apply you talk about what you learned from this experience and express your regret at having cheated.


Classes after undergrad degree

Can I take classes after I graduate with my bachelor's degree? What will happen to my undergraduate gpa? I want to go on to med school but I have to get ny gpa up and work on research. If i take classes at my university after recieving my degree will those classes be added to my undergrad classes and help raise that gpa?

tantacles

If you take undergraduate courses, those courses will be added to your GPA and help your med school application if they raise it. Graduate level courses will not count. Talk to the registrar to find out how those courses are classified.


What are my chances getting accepted to Ku med 2018?

To keep the information short and to the point. I have a 3.8 GPA, Exercise Science Bachelors with a minor in Chemistry. The rest of my application is well such as work actives, letters, etc. The only thing that was bad was my MCAT, I got a 487, but scores 125 Chem/Phys, 124 Biochem sec, 119 CARS and 121 on Psychology. I know I should have prepared more and im a Kansas resident. I am planning on becoming a Psychiatrist in Kansas, so I can help give back to my community. I would love any helpful feedback.

tantacles

It is hard to say what your chances are as our advisers do not work at KU. Your GPA is excellent, but I would recommend repeating your MCAT. This will give you the best chances of gaining acceptance.


What are my chances of receiving admission to DO (or even MD) schools here in the States?

- Will be 29 years old during expected Fall admittance 2019. I am a Black - African/American applicant. Current Peace Corps Volunteer (Community Health Extension - Paraguay) Close of service date May 2018 - Fluent in Spanish and French

Highlights - 3.43 non-science GPA / 3.14 science GPA - 3.25 overall - Most recent MCAT 506 (4th attempt - previous attempts 19 24 23 - all older than 3 years) - 150 volunteer hours in Hospital Setting (ER / hospice) - 1 year patient advocate - paid position (Hospital Setting) - Child Family Health International volunteer experience in Ecuador and Non-Profit Children 2 Children donation of clothes - 6 weeks - 250hrs + shadowing multiple MD/DO physicians - Connecticut Resident

Mr.Smile12

I would think that your service time in the Peace Corps on top of your accumulated experience shadowing or working in a health care setting is going to help you. I would hope that in-state/in-region schools would at least take a look. The only concern I may have is whether you have had any recent coursework in the sciences that you have taken; if so, how well you did in those classes could help your application.


What are my chances of getting into medical school in Canada? Ireland?

I am currently in a pre-med undergraduate degree in my 3rd year. I want to apply to medical school in the near future. Over the course of my undergrad, my marks have not been as high as I'd hoped due to some mental health issues that I am on my way to overcoming. I'm currently sitting at a 3.2 GPA. My current year is going ALOT better and I'm hoping it'll go up.

I have tried to keep my extra circulars active and I'm thinking of doing my MCAT in the summer this year. I am willing to do a few extra years after undergrad. Mainly, I'm feeling a bit discouraged with my marks and even my extra circulars seem to need some work. Is it too late in the process for med school in Canada to be an option?

tantacles

With a GPA of 3.2, it will be very difficult to get into medical school in Canada. With a great MCAT score, it is possible, but improbable. You would need to improve your GPA significantly, and many Canadian medical schools prefer Canadians. You have not stated your nationality, so it is hard for me to advise you.

I am not familiar with Ireland's requirements for medical school.


What are my chances of getting into medical school?

I wanted to know what are my chances of getting into top medical schools like John Hopkins, Indiana University Medical School and Eastern Virginia Medical School or any other schools in the US? I am a current pre- med student. I just started my third year of undergrad. I am a global health with concentration in clinical science major. My overall gpa (from 1st and 2nd year) is 3.77. But my science gpa is 3.61. This will hopefully increase to 3.8 (overall) by the end of this semester. I work at a nursing home and volunteer at a local hospital. My ethnicity is black. I have not taken my MCAT yet . It would be great if you guys can advise me/ tell me the minimum MCAT score that I would need to be competitive.

tantacles

Your chances will depend on how well you do on your MCAT. With a high score, you have a chance of getting into top schools, and with a good score, you have a good chance of getting into the other schools you have listed. I would suggest taking the MCAT and exploring your options at that point.


Post bac help

I am reaching out in regards to receiving some advice about post-bac admission. I am currently a senior who will graduate with a degree in Neuroscience in spring with a current GPA is 3.1 hoping to graduate with overall undergraduate GPA will be a 3.20. (I would have to receive all A's three quarters to boost my GPA to a 3.30, although I am trying to do that I am also realistic). I have finished Biochemistry (B+) and I am retaking a general chemistry class that I did poorly in freshmen year (C- grade). Throughout senior year I am taking upper level biology classes to prove I can handle rigorous coursework within my major, (Cellular Neurobiology, etc.) My whole entire goal for my senior year is to improve my GPA. I believe a post bac program would be a wonderful opportunity to strengthen my skills, continue with volunteer work, and improve my science GPA, which is extremely low (2.7). My largest mistake was overloading my class load (physics, orgo, cell bio, etc all in the same quarter) and receiving C's. I am a really hard worker and I try to do everything right (go see my professors every week, attend TA office hours, read the textbook, spend hours studying, never miss a day of class, etc). I plan on taking the MCAT when I feel I am ready to take it (most likely March) and applying to post-bac programs fairly early. I have around 600 hours of volunteer experience in the PACU as well as the Hospice unit of a hospital. I have authorship on a paper and involved in many academia clubs. However, I have put all of those on the back burner to really focus on strengthening my GPA. I have met with my academic advisor multiple times on my future because I feel uncertain if I will gain entrance into a post-bac programs. Furthermore, I am reaching out to see if there is any advice you can give me in regards to strengthening my overall application to receive admission into post-bac programs, I am an Illinois resident who is really interested in the Loyola MAMs program, Midwestern University masters program, Rosalind Franklin, Rush Biotech, and SLU Anatomy. I feel really lost on where to go from having such a bad performance in undergrad. I am more interested in DO rather than MD just for reference. I have thought about other career options such as quitting my dream of being a DO or MD doctor all together and perhaps become a podiatrist, pharmacist or Nurse Practitioner. Thank you kindly for taking the time to read this message.

tantacles

Improving your GPA is very important. It is hard to say, without the MCAT, what your chances will be, but with a GPA of 3.1 and a low science GPA, your chances would be poor for medical school without a stellar MCAT performance. It is worthwhile to reevaluate your goals now, as getting into medical school may not be feasible.


Low GPA

I studied abroad for a year and failed both semesters. I had a 3.18 before I left for study abroad. 4 C's in upper level science courses were dragging me down. Now I'm at a 2.57 because of the study abroad grades.

I returned to my University and retook the classes. We have a academic forgiveness policy. I got all As and Bs in the classes once I retook them but my school said the academic policy does not replace study abroad grades so they are still being calculated into my GPA.

What should I do? This is my senior year. I will be finished in December with all required courses besides Spanish 202 so I'm going to take it in the spring along with 3/4 of the classes I got a C in.

However I still don't think my gpa will be above a 2 something before graduation. && it's too low for post bacc..... is there anything else I can do to raise it or better my chances for getting into a post bacc program. I already retook the classes I failed abroad and I'm retaking the classes I have Cs in which will help some if of course I do better which I plan to, but I still don't think it will help much. Any advice????

Mr.Smile12

Did you ever get connected with your prehealth advising office or your career services advisors? It seems that you have arrived too late to seek their help, so it may take you a while to get to a medical school if that is your desired goal. You haven't even mentioned trying to take the MCAT, and I think you are wise to determine that your overall academic record needs to show more preparation for the real challenges of a medical education. You should seek the advice of your career services office and your health professions advisors to get you to determine a realistic goal, and after a few years, you may want to return to preparing for a health professional career after you have gained better insight on your academic preparation for those specific careers.

Now there are plenty of health care careers that you can also consider, and that's why seeking the insight of those advisors can help you construct a plan for what your immediate next goals should be.


Dyslexia on Adversity Essay?

I was wondering if I should mention dyslexia in the adversity/challenge essays or if it would be looked down upon. I have maintained a GPA above 3.5 in college so shouldn't it be seen as having overcome it or have figured out how to deal with it and persevere?

I'm sorry if this question was already asked. When SDN showed me some related questions that were already asked, they were general "adversity" for the headline and when I clicked on 'view more' to see what they were specifically asking, I got a 'Page Not Found'

Mr.Smile12

It's too hard to predict how any one school would view your dyslexia as an adversity or challenge, but I would think most would acknowledge any learning disability to be. That said, only admissions officers could give you specific advice how their school would view that disclosure.


Pass Failing a non science Course

I know this may seem like a very odd question...I go to a private libral arts school which requires you to take a semester of fine arts. To be completely honest im not the artistic type and im worried that this course may put a dent on my steady GPA since its a 3 credit class. My school offers a Pass/Fail option for once class every year so that it wont affect my GPA in the long run, and I recieve the credit it just shows up as a P on my transcripts. However ive been told different things about med school admissions with the Pass/Fail option. Keep in mind ive never used one of these before, any thoughts?

Mr.Smile12

Why are you interested in taking a course pass/fail? I don't think it would count towards your GPA calculation, but you shoulfd check with admissions staff.


Lab course work as lab experience?

Hi! I'm currently applying for research assistant, lab tech, and lab assistant jobs at the hospital I'm currently volunteering at. I don't really have any formal experience working under a professor or PI. So I was wondering if the labs I took throughout my undergrad coursework would count as laboratory experience? In one class we were allowed to take the techniques we learned the first half of the semester and create and analyze our own research project. So maybe that counts but regular lab course work does not? Any clarification would help and be much appreciated! I know some hospitals/labs will consider different things to count as experience as others.

tantacles

Your coursework does not count as work experience, but it does count as laboratory experience as you have done lab work.


Should I still pursue a medical school admission?

I was dismissed from my university a year ago for poor grades. My GPA was 0.96 I retook my general Chem at a local Community college along with Physics and general ed. courses My advisor is telling me that I've ruined my chances for a US medical school; even with a post bacc or a Master's, my undergrad will be a red flag. Is she right? Can I get an upward trend going from this point on to my upper div courses and hopefully redeem myself? If I can get an upward trend going, can I land a medical school admission without a SMP or post-bacc?

tantacles

It will be very difficult to get into medical school after being dismissed from college. I would suggest reevaluating your career choices as it's important to be realistic. You also have to pass undergraduate to get into US medical school.

It is possible that with a stellar record in the future, you could gain admission, but right now there is a black mark on your record that must be remedied, and this will take time.


Will a B or C in Calc 2 kill my chances for medschool?

I did a lot of dual credit and ap classes in high school so when I got to college I got thrown into upper-level classes I wasn't completely ready for. I am retaking Principals of Chem despite my AP credit because I was hoping an A would look good on my transcript. Yet my main issue is I was told I wasn't able to retake Calc 1 for this benefit. I went into college with 67 hours and at the time I thought I was helping myself by taking these classes early but being a freshman/junior at the same time has been a strain on my adjustment period. So I am struggling pretty harshly in Calc 2 and I got a C on my first exam. I am obviously going to strive for better but I honestly don't know if I can achieve higher than a B at this point. I am taking Stats next semester and I am way more confident about being able to get an A there. I'm just wondering how it looks coming from being very successful in high school to having trouble in my first semester and how to maybe use this experience to my advantage and/or recover from this grade. Thank you!

tantacles

A B or C in calculus will not kill your chances for medical school as long as you do well in your other courses.


pre-med course work from accelerated school?

I have a BS in accounting, but am now considering medical school as a viable opportunity. I am a 23yo, single mother with a disabled child and just do not have it in me to go a traditional brick mortar. Besides most of the other implications, I am wondering if anybody has any advice on nontraditional/online/accelerated schools that I could attain my premed sciences from?

Any comments or information would greatly be appreciated!

tantacles

If you wish to pursue medical school, you need not go back and do a full degree, but rather you can take your courses a la carte. However, the majority of your courses should come from a 4-year college as the rigor from those courses is higher than those from online courses. That being said, if you are able to excel and get A's in all of your courses, it is possible that medical schools will overlook the fact that your school is an online one, provided this is not denoted in the name of the school.


How good are my chances of getting into a PA program?

Hello! I am planning on applying to PA programs next summer 2018. My current overall GPA is a 3.7, but my science GPA is only a 3.4. The thing that worries me is that I'm having so much trouble in my chemistry class and I'm afraid that my science GPA will drop. I did the math and if I make a B or C in this class, my new science GPA will be at a 3.3 or 3.2. I'm wondering if it is still possible to get into a PA program with my GPA being that low? I plan on shadowing a PA this semester and I worked as an optometry technician for almost 2 years, equaling about 1500 hours of direct patient contact. It's not the traditional patient contact (CNA. EMT, MA, etc.), but I've emailed some schools and they accept that form of HCE. Does anyone know if my application seems weak at this point? What else can I do to improve my chances? Any input is greatly appreciated!

tantacles

It is hard to say what your chances are. Each program is different, and your entire application will likely be looked at. I imagine you will still have a reasonable chance of admission as long as you have a strong application otherwise.


Will student background screening/check show expunged misdemeanor?

A couple years ago I was charged with a misdemeanor. I have turned my life around since then and haven't gotten in anymore legal trouble. But I was wondering if this would show up on my student background screening when applying for medical schools if I got the misdemeanor expunged? And if so what are the chances of still getting accepted into medical school? The misdemeanor was falsification.

tantacles

It is unclear as to whether this will show up on your record. Regardless, read the AMCAS rules carefully. You may be required to disclose this misdemeanor even if it it not mentioned based on AMCAS' rules. Whether it will impact your chances of admission to medical school depend on the misdemeanor itself and how you speak about it on your application.


Please Help! Lost my pre-med advisor and have no guidance

So, this past June my Pre-Med advisor started teaching and left his position. They have yet to replace him, and as a result me and many other pre-med students are rather lost. Start of fall semester I was instructed to make an appointment with him so he could basically get me into volunteer organizations and under a physician to shadow. With him being gone, I ended up screwed over. I was told to hold off on volunteering as I have been taking 5-6 courses with labs during spring and fall semesters and 4 courses during the summer. I have been desperately trying and contacting multiple hospitals, filling out volunteer applications, going in, in attempt to talk to someone, trying to get into research opportunities, etc. Either no one responds or I get an automated response email wise. I am in my last year of pre-med. I take my MCAT May of 2018. Because of the course load I don't have very much volunteer work. I have no idea how to get under a physician for shadowing. Everyone keeps saying that, hey you have two bachelor degrees, a high gpa, and will get a good MCAT score so that's enough, but I know admissions don't necessarily care about that as most students have that. So what do I do??? Will I even get into med school at this rate? As doing the bulk of volunteer work in your last year doesn't exactly look good... Any advice is greatly appreciated. Thank you.

tantacles

I would suggest that you work on finding some shadowing or volunteering experience before you apply to medical school. This may mean delaying your application, but you should wait until your application is ready before you apply. If you are getting an automated response, keep asking around as to who the right person to contact is; there is likely someone who will respond to you, but you may need to wait as many hospitals have waiting lists for volunteering.


What are my chances?

I am in my Junior year of my Undergraduate Career at the University of Iowa. I have a University of Iowa cGPA of 3.31 and an overall GPA including some college credits from high school of 3.55. I have 4 years experience as an EMT and 1-year experience as a Paramedic. I am involved on campus as being the President of a student organization, I will have 2-3 years of research under my belt, and I also volunteer regularly. I have not taken my MCAT yet, but I will take it this May. I am wondering what my chances of getting accepted into an MD program are and I would also like to narrow my search for Medical schools. I am starting to worry about my GPA, and just would like some general advice. I have done lots for my resume over the years, but my GPA has dropped and I am concerned. Any help is appreciated!!

tantacles

It is absolutely impossible to stratify your chances without knowing your MCAT score. Your GPA itself will not stop you from getting into medical school, but you need to do well on the MCAT.


State school to ivy and now unhappy, should I go back?

So I've looked at a lot of posts and my situation seems a little unique and I'd appreciate any feedback that can be offered.

I decided to go to the University of Vermont when I graduated high school but I also had a guaranteed transfer offer to go to Cornell University my sophomore year and beyond. I had a great year at UVM, made amazing friends, was on the perfect dance team, great neuroscience club, thinking about doing the school EMT service, joined a sorority etc. But by the end of the year I decided to go to Cornell because I figured I would also have a great time there AND get an Ivy league degree, hopefully giving me an edge when it comes to applying for med school. Now it's been a month or two and I really don't like it at Cornell. I can't seem to make a good group of friends, everything is so competitive I can't get into anything to make me stand out, and my old sorority and dancing is completely different here. I'm really struggling to enjoy anything except my classes and am sorta home sick for my old school.

So all in all, I was much happier at UVM than I am currently at Cornell and I'm wondering if going back to Vermont for the next two years will lower my chances of getting into medical school or should i stick it out and potentially be unhappy for three years at Cornell but have an ivy degree?

Mr.Smile12

I don't know what advice you had when you first thought about transferring away from your state school. I'm sure the guarantee to transfer was very enticing, but I would have cautioned anyone who is thinking about transferring to understand whether there is enough support at the new university as it might be a different culture altogether. What was your goal when it came to transferring in the first place? If it was to be in a more challenging but rewarding situation , then understand that this is all to help you develop focus for your ultimate professional pursuit, to be surrounded by equally motivated students, and to get more support from people who want to go to medicine as you... then it was probably a rational decision.

Unfortunately just having a degree from an Ivy doesn't make you that much more desirable in the admissions process, and I hope many people were also telling you this as well. I do think that in your new environment, you need to challenge yourself to seek the resources, communities, and activities that can really help you through these difficulties. It is a myth that medical schools really want practically perfect people (though it always seems like those are the ones that are your ideal premeds). And there is something to be said about making adjustments because you'll make them again in medical school.

You do have freedom to choose, so I hope you can decide what location best puts you in a better situation academically and emotionally.


Please help! Currently freaking out about my grades!!

I am currently a first semester sophomore and I am very worried with how my grades are. My first semester of my freshman year was pretty good (all A or A-), but my grades dropped a bit second semester (a couple B) because of a couple stupid classes. My GPA as of now is around 3.65, mainly because of those Bs from last semester. Now, I am in organic chemistry and I am finding it very difficult. I am getting help, but my grades are not reflecting that and I am beginning to worry that I won't be able to make an A in this class. I know that organic chem is supposed to be hard and whatnot, but it is honestly making me hate being premed, even though I definitely want to become a doctor and practice medicine one day. Can someone please advise on what I can do? I am so worried and stressed about this right now and I don't know what to do.

tantacles

A 3.65 GPA is respectable, and with a good MCAT score, you will still be able to get into medical school provided all of the other parts of your application are solid. It is ok not to get an A in every course. I would suggest you do your best going forward, but cut yourself a little bit of slack; not everything in your application has to be perfect.


Do I have a chance?

I am a non-traditional student that attended university 10 years ago. In the 1 year i spent there I acted like an idiot and had a 0.8 GPA. I spent 6 years in the military after that and then went back to a different university. I now have a 3.96 GPA after 3 years. I received a 513 on the MCAT with decent shadowing experience. Will my 1 year of awful GPA ruin my chances?

Pathdocmd

Absolutely not. There are a lot of applicants. medical students, and physicians that had a rough start in college then grew up and did very well. I wouldn't worry.


Do I have to do extra curricular activities my freshman year of college?

I am majoring in Biomedical Engineering as my pre-med major and I was wondering if I have to join EC's my freshman year to get accepted into a top medical school like Harvard? I am adjusting to college right now and I do not think I can handle a club right now.

tantacles

It is ok not to do extracurricular activities in your first year of college if you do not have time. I would suggest that you expand your view of medical schools; every US MD school will train you to be a physician. Do your best in college and do well on the MCAT and you will be in a good place to get into a medical school, though it is impossible to predict where you will stand in terms of specific schools at this point.


Does it matter what kind of a Biochemistry course counts as a pre-requisite for medical school? Trying to decide between two biochem courses that are offered at my university (Biochemical Basis of Health & Disease vs Introduction to Molecular Biology)

There are some differences between the two courses, one is more biology-based biochem course versus the other, which is more biochemical concepts focused on physio, health, disease.

Mr.Smile12

I would want to know if you have a health professions advisor who had "official" word from the biochemistry course instructor or department hosting the class if either one would be rigorous for preparing for medical school. Usually the prehealth advising office collects the information on "acceptable" prerequisite classes for prospective applicants.


Medical School

Hai, I`m currently doing my first year of undergraduate degree. I`m aiming to become a psychiatrist, but not clear what class should i take to enter medical school. I`m doing a AA that has only chemistry, biology and psychology ( also abnormal psychology), just one math( statistics ) but no physics. Is that recommended that i only do few of these? Or should i change my courses to AS that has physics and lots of maths in it?

tantacles

If you wish to get into medical school, you will need a full year (two semesters) of inorganic chemistry, chemistry, physics, and biology at the bare minimum, but every school has its own requirements beyond those courses. I would look at the specific schools to figure out the requirements. Your major or concentration of study is not important as long as you do well and fulfill the pre-requisites for medical school, so study whatever interests you.


Do medical schools care where you get your undergraduate degree from?

I am currently at North Dakota State University, but I am considering switching to a smaller college, Bemidji State University. Should I stay at my current, larger, more well-known school or switch to a smaller school? I am a biochemistry major and both schools have this major, as well as a pre-medicine program.

tantacles

Some medical schools do care about where you did your undergraduate degree. Keep in mind, though, that the most important thing is to do well in all of your courses, fulfilling the pre-requisites for medical school. Your major itself is unimportant, and you can major in anything you wish as long as you do well in it, whether it be math, english, the sciences, the social sciences, the arts, or something else completely different.


Is calculus required for pre med / accelerated or combined bs/md or bs/do programs

I am a junior in highschool and was wondering if I have to take calculus as a pre med. i am applying to multiple bs/md and bs/do 8,7, and 6 year programs and am debating to take pre calculus this year or ap statistics. Thanks again for your help.

tantacles

Many programs require calculus, and many don't. I would look at the websites for individual programs to determine whether calculus is necessary. Some medical schools require calculus and some do not. My personal advice is that pre-calculus is a useful course, and if you think you may take calculus in college, you should study it.


Institutional Action AMCAS Application

Hi, I am a pre-med applicant with a conduct violation and want to know if I'm totally screwed. It was a really difficult time from me and I have gone through extensive therapy and learned from my mistakes.

I was involved in a Title IX investigation where someone I had a casual relationship with falsely accused me of awful things because his girlfriend found out and forced him to tell the university. I had a 2 year casual relationship with this person, but it got worse when I got pregnant and had to have an abortion, which really messed with me emotionally. I had two interactions with his girlfriend (once was before she became his girlfriend, when we found about each other and I apologized to her over FB because she thought they were exclusive and she told me to never talk to her again and blocked me). I kept hooking up with him because I was distraught about the abortion, he was the only one who knew, and hooking up with him was the only way I could see him. Fast forward 6 months, he raped me (I was drunk in my dorm room and said no) and I was so disoriented after the fact that I contacted her and told her again by text (yes i realize how stupid this was but i was just raped, wasn't thinking straight, and was intoxicated). He got angry I told his girlfriend and we cut off contact.

The guy and his girlfriend then proceeded to accuse me of sexual assault, stalking, and harassment. I was about to file a report myself about the rape but he and his girlfriend basically beat me to it with their lies. The guy lied to protect his relationship with his girlfriend and said we had only hooked up once but I was able to prove that he was lying through his teeth with text messages of him asking to hook up multiple times, etc. I was able to disprove stalking because, point blank, I was really afraid of seeing them on campus (conjured up a lot of emotions about the abortion) and absolutely did not stalk him or her. I was able to disprove sexual assault because, point blank, I never engaged in that. But, because I texted her after she told me on Facebook 6 months prior not to contact her, I was found guilty of harassment (not sexual harassment).

Bottom line, I was removed from housing because of alcohol (underage, and I was a resident assistant at the time), and found guilty of harassing the girlfriend (texted her once after she told me not to). I was not able to prove my rape. Sanctions: No contact letters and mandatory counseling to help me get over my grief over the abortion and my rape.

Am I screwed for med school? How deeply do they look into it if I check the box that says I have a conduct violation? I am an otherwise strong applicant. I know it reflects poorly, but I have learned a lot from that experience including to cut off unhealthy relationships and to look for help in the right places (counseling)

tantacles

This will absolutely be school dependent. My suggestion is that on your application you don't tell the full story. Simply admit to the parts that you were found guilty of and state what you have learned from the experience and how you will change your conduct in the future. Talking about finding counseling as a coping strategy is a great thing to mention as medical schools only want to accept people who have healthy ways of coping with their issues.


Would I significantly benefit from a postbac/SMP?

I'll be a senior and was wondering if, given my GPA and GPA history, it would be advisable to enroll in a postbacc/SMP program. Currently, my overall GPA is 3.48 and is projected to finish at a 3.55-3.60 assuming things go as planned. My MCAT score is also a 521 (taken earlier this summer).

My GPA history, separated by year and fall/spring semesters, is as follows:

Fr: 3.83 / 3.83 So: 3.17 / 2.81 Jr: 2.89 / 3.88 Sr: Projected around 3.8/3.8

After the fall of my junior year, I decided to see a psychiatrist and was diagnosed with persistent depressive disorder, and was prescribed medication for it, to which I attribute the jump in grades. Would med school adcoms be willing to give less weight to the 3 semesters in which I've had an extremely low GPA, given the explanation for it? Or should I enroll in a postbac program to raise it and improve my chances?

tantacles

You might benefit from a post-bacc. but given your stellar MCAT score when combined with your good GPA and known recovery from mental illness, I don't think you would need one to be accepted.


Secondary application essay asking about shadowing experience, can my answer be about scribing?

I have little volunteer shadowing experience (under 10 hours) but I have over 200 scribing hours. Can I answer an essay prompt about shadowing by describing my scribe experience?

tantacles

Your scribing experience is a bit different from shadowing as your job was not just to observe while you scribed. I would suggest you ask the physician you are working with if you can spend an extra day shadowing as that will count.


Chances of getting into med school after transferring

Hello! I am currently an undergraduate majoring in microbiology. My current GPA at my current school is a 3.3 but I expect it to go up to at least a 3.6 by the time I graduate in a year and a half. However, this school is my third, as I have transferred twice before. I transferred from the first school because I had trouble adjusting and left w 2.0 GPA. I have since retaken all the classes I did bad in and have gotten mostly A's and some B's in them. I left the second school with a GPA of 3.5. I am now at my third school. After my first year of college, I tutored chemistry for one semester at my second school. And currently I am working as a research assistant with plans to volunteer at a hospital pretty soon. My grade trend is upward. However, If I take all my classes from all three schools into account and calculate the cumulative GPA, I would be at about 3.1-3.2 by the time I graduate. With all this, and assuming I get a good MCAT Score, what would be my realistic chances of getting into medical school?

tantacles

Transferring to another school is all right in terms of admission. The bigger issue is your GPA. The following table illustrates percentage of accepted applicants by GPA and MCAT:

https://www.aamc.org/data/facts/applicantmatriculant/157998/factstablea24.html

Improving your GPA is important unless you get an absolutely stellar MCAT score as your GPA from all three schools will be counted by AMCAS.


3.2 Science, 3.1 cum chances of getting into med school

I'm worried about what my chances are of getting into med school with a 3.2 science and 3.1 cumulative GPA, assuming I get a high MCAT score. I had a tough beginning to college but have been recovering since. I am also a volunteer research assistant at my school.

Pathdocmd

You will have to improve your GPA regardless of what your MCAT turns out to be. With that GPA your chances of an allopathic school are diminished. However, if it was just a bad Freshman year then a dramatic increase, that is different. If the only medical/ volunteer experience is being a research assistant, then your chances are slim to none.


What are things you did to get into medical school besides academics?

I am a second year premedical student and I want to know what are things you guys have done to get into medical school. Any shadowing, outside clinic volunteering, etc.?

tantacles

Just about any extracurricular activity under the sun will increase your chances of getting into medical school. Of particular importance are showing your commitment to service by performing volunteering, performing research, and getting clinical experience in a hospital or clinic setting. Personally, I worked as a volunteer in an emergency department, recorded books on tape for the blind, and performed cardiology research on mice.


Should I become a 5th year senior to officially declare a pre-med concentration?

I’m a senior psychology major who will graduate this fall. I have not declared a pre-med concentration, but I am taking/have taken pre-med classes--i.e., general chemistry, organic chemistry, general biology, general physics, and biochemistry. I would need to take molecular biology, calculus, and either genetics or microbiology to meet the requirements for the pre-med concentration at my university. I did not want to delay my graduation any further, so that's why I have not taken those classes. My current dilemma is whether I should become a 5th year senior and declare the pre-med concentration or just graduate this semester. Additionally, I am unsure whether I should take an upper-level biology course this semester, or keep my course load light to preserve my GPA. I'm currently enrolled in biochemistry, physics 2, a psychology course, and applied medical physiology with a lab (which is a seriously intense course).

My current GPA is a 4.05--my university uses the plus/minus system. I've only earned a B+ in an art history class taken my freshman year. I have research experience, and I volunteer regularly. I work part-time as well. My plan was to study for the MCAT and shadow after my graduation in December until I take the MCAT in the summer of 2018.

Pathdocmd

A "pre-med" concentration on your diploma or transcript will make little, if any, difference to admissions committees. The additional science course (except calculus) would make you a slightly stronger candidate (depending on the school) and help you when you do get to medical school.


Retaking the MCAT with a good GPA?

So I have a double major and double minor with a 3.8 GPA. I just got my MCAT score back and it was a 496. Please don't ask if I even studied for the MCAT. I spent around 9 months with it and did a lot of practice tests (12) and was scoring well (around 510). I guess I had an MCAT I wasn't prepared for and psyched myself out. I have my primary application in and have gotten a secondary already. My question is do I need to withdraw my application or try applying DO? I would love to be a DO but I unfortunately don't know much about their schools. I also have great extracurriculars and references from doctors. I don't know what happened and I'm very discouraged and really debating withdrawing completely. I also know it's a little late in the game to even retake and work full time which I think was also my problem with why I did so poorly.

tantacles

Trying DO is a good idea, but I doubt that you will be successful with a 496 MCAT. My suggestion is that you withdraw this cycle and plan to apply the next cycle. Applying your first time when you are truly ready is extremely important.


pre-physician assistant advice?

I am applying to physician assistant schools this fall. I am wondering if i should attempt, or wait until next year. Currently CASPA has calculated my overall GPA as 2.8 and my science as 2.5. I have over 1500 hours of direct patient care. Additionally, i have shadowed and am actively volunteering. I feel like i meet prerequisites for some schools, but i may not get in. Should i not waste my money on more applications?

tantacles

Most of the advisers on this forum are not PAs, but my general suggestion is that if your GPA is below that of the average PA, work to improve your GPA and apply next year.


Can I major in child physiatrist as a pre med student?

I'm a pred med student and I'm a little conflicted as to what I would like to major in. I know I have a passion for kids and working with kids as well as helping others. I was wondering if as a pre med student if I can major in as a child physicshiatrist and minor in psychology? Would that be a good idea? I would like some advice and some guide in the right direction and perhaps some ideas in which route I should go in regarding my passions. Thank you in advice.

tantacles

As a pre-medical student, you can choose any major as long as you fulfill the pre-medical requirements in addition to that major. What is important is that you do as well as you can in your classes as you possibly can. When you apply to medical school, you will be applying for a general medical education, and you will specialize after.


Doing a post bacc vs second bachelors

I plan on applying next cycle in May 2018. Currently it is almost September 2017. During this time what do you recommend I do to improve my application?

I have low undergrad GPA in B.S Biology of 2.58 but a better M.S GPA in Biomedical Sciences of 3.5. My MCAT is a 505. I know I should retake the MCAT and if I score better around510 will this suffice to be competitive by May 2018 cycle?

It is too late to do a post bacc as the time to apply for this has passed. Do you recommend I just focus on the MCAT or consider starting a second bachelors in Spring semester?

Thank you.

tantacles

With a GPA of 2.58, it is unlikely that you will be competitive for ANY MD or DO programs without some kind of post-baccalaureate program regardless of your MCAT score. I would suggest that you hold off on applying until at the very least the 2019 cycle and do a post-baccalaureate program, retake the MCAT, and improve your chances maximally before you even consider an application.


Committee Letter by only academic professors

I realize that I only have my academic professor to write me a letter of recommendation as part of my committee letter, is that a bad thing? I did some volunteering, but they were for a short period of times (3 months tops) for each one and after a while, I stop because I was commuting from home.

Mr.Smile12

Admissions committees usually are much more flexible when it comes to institutional letters, as long as your letter conforms to the requirements set by the prehealth advising office. So it's only a problem if your prehealth advisors/committee requires specific letters that they can include for you, but usually the admissions committees will defer to the advisors.


Need expert advise deciding DO vs SMP/MD pathway

I am a senior in college with a GPA and sGPA of 3.2 and an MCAT of 512. I have leadership/ mentoring activities for a year, team building activities, 200+ hours of shadowing, some research experience and an upward trend in GPA.

I was contemplating either going to a DO school with my stats or try for an SMP into an MD school pathway. I want to go to a DO school because it saves time and money (no SMP) but I have heard that its hard for DO's to get competitive residencies and thus mostly are in primary care. However, I also want to go to an MD school because it is the traditional pathway and will provide broader avenues for specialties.

What do you suggest for me to do?

Mr.Smile12

Only you have an idea of what you want to do as a physician and whether you want to spend the money to get to that goal. You should check out resources from AACOM about residency matching, especially since a unified match is going to happen (if it hasn't already). DO's can get into specialties.


Post bacc?

Hi everyone,

I want to apply to med school but I don't know if i should do a post bacc first. I am a rising senior.. My GPA is 3.2 and MCAT is 506. I am a research intern at a hospital. I've shadowed doctors in two of the top hospitals in India. I've volunteered with a local non profit for 3 years now.

I'm not sure if my stats are good enough to get in. I was thinking of doing a post bacc, but some of them require you not to have taken the MCAT. And since they're expensive, i wanted to apply to a good one like Bryn Mawr, Goucher, Drexel... I also don't know if my chances of getting into those are good? any suggestions??

tantacles

Your stats right now are acceptable for DO school but not great for MD. I would absolutely do a post-bacc to improve your chances. It is hard to say what the chances are of your getting into a particular post-bacc program, and I would suggest e-mailing those programs or looking at their websites to get a better idea of your chances.


Would it be a very wise idea to take all the pre requisite course work for the Carver college of medicine at a four year university?

I am attending a community college in Iowa to get all of my general education requirements for my major (Biology) out of the way! I've heard that many medical schools want students to take pre requisites at a four year university. My plan was to wait until I transferred to a four year university to start my pre requisite courses needed for medical school since courses at a four year institution are more rigorous. Also during these years I plan on job shadowing and volunteering at a local hospital. Do I have a good plan so far? What else can I do? I really want to attend medical school at the Carver college of medicine.

tantacles

My suggestion is that you take a maximum of 1 course at a community college and take the rest (if not all) at a four year university. The courses will be more rigorous at a four year college, and medical schools are aware of this.


Late First Time Applicant-Wait or Apply Now?

So I am a rising senior in at a 4- year undergraduate program (business management major with chemistry minor). I am part of the honors program and will defend a thesis before graduation next semester (Spring 2018). I've hovered around a 3.65-3.7 GPA during the first three years, have decent (not great) EC's, and 100+ hours of shadowing and clinical experience. I am taking the MCAT on August 18th. The score release date for me would be September 19th, which I have realized is very late in the app cycle. Would it be better to sit on my MCAT score (and retake if the score isn't great) and apply next cycle, or just go ahead and give it a shot now? Any advice would be appreciated.

tantacles

If you haven't submitted your primary application yet, I would suggest that you wait until next year. Being complete September 19th is pretty late, and it would put you at a significant disadvantage. Waiting a year would allow you to retake the MCAT if necessary and continue to prepare your application.


Picking College for Pre-Med

So I just recently moved to Arkansas, from Texas and found myself in a depressing situation. I am currently in high school, and still deciding what college to go to. In Texas, I am probably going to find myself paying 4x the college cost to go to a decent college (University of Houston). Or I could go to state school over here for alot cheaper of a price (University of Arkansas). So here is the dilemna, Arkansas only has one medical school, which includes pharmacy and nursing. Meanwhile, in Texas there is alot more medical school as well as dental and programs like JAMP. Both school accept 90% instate students. My family is currently in a tough finicial position and I have 2 years left to decide where to go. Anybody have any advices?

tantacles

If you go to college and do extremely well and do well on the MCAT, it will put you in a great position to go to any medical school and potentially get a scholarship. I would suggest that you go to a school that makes sense for you financially and makes it possible for you to do your best in undergraduate. It is hard to say what the best choice would be, but it is an intensely personal decision.


Class Retake A Negative On My App??

Finished my undergrad with an overall GPA of 3.7. At the beginning of my senior year it was at 3.43 or something close to that. I retook a couple of classes that I got Cs in to get As (Ecology, Biochem, Physics 2, and Nutrition for Health Professionals) and one class to get an A after getting a B the first time (which I realize retaking it now was a mistake). On my transcript from my school it shows that those original class grades were dropped and my GPA is not the 3.7. HOWEVER, my new concern is that I was told that those Cs that I got ARE going to be used when med schools look at my overall GPA, and that my overall GPA is going to take a substantial hit. While this is a over site of my own fault, wouldn't adcoms want to see significant improvement in grades for core classes in regards to my major, and an improvement in GPA? Or is this something that is seriously going to hinder my application? I know I should have done better my first time around but I had some personal things seriously hindering my performance, and now I'm afraid that one bad year of undergrad, and trying to correct for that year has ruined my chances. Any and all help would be appreciated.

tantacles

Medical schools have very different ways of viewing retakes, but in terms of the calculation of your GPA for MD school, both classes will be used for the calculation. This does not mean your chances are gone of getting into medical school. However, I would suggest you do the best you can on your MCAT and apply broadly to increase your chances of acceptance.


Should I buy MCAT books freshman year of undergrad to read?

I am a freshman pre-med and I want to buy mcat prep books to read before I start each of my pre-med classes. I plan on learning the information in the books before I start studying for the mcat around Junior or Senior year so it is easier on me. I want to score in the 100th percentile so I can get into top medical schools for surgery. Is it a good idea to do this or should I wait to study?

tantacles

My suggestion is that you do as well as you can in your science courses, as those are the basis for the MCAT, and study for the MCAT during a dedicated period of time after you have finished your courses.


Can I talk about how having the Eating Disorder, Bulimia Nervosa, has encouraged and solidified my interest in becoming a medical doctor?

Hello, guys. I am applying to medical school this cycle, but I am yet to turn in my primary application; I haven't written my personal statement.

I was wondering if I could talk about how suffering from the ED, Bulimia Nervosa, for 4 years has shaped me and piqued my interest in pursuing a career in the medical field. Would talking about my ED be an absolute NO NO?

Thanks.

tantacles

Your experience with an eating disorder, when presented in the right light, could be very beneficial to your application and provide great insight into your motivation for pursuing medicine.

More importantly, though, we are approaching a late time to apply in this cycle. If you have not submitted your primary application, your application will be late. If it will be more than 10-12 days before your application, which must be fully complete with a fully realized personal statement, it submitted, I would suggest waiting until next year to apply.


Will my GPA from ten years ago haunt me when I apply, even if I have turned things around completely?

I am an extremely non-traditional student who originally went to undergrad in 2006. I was very immature, had no guidance from family, and came from an underperforming high school. That being said, I stuck around and floundered for 3 years, and I eventually left with a final GPA of 1.85. I did not graduate, but it should be noted that these were all business classes and I attempted no science courses.

After coming home and reevaluating my life, I became an EMT. I went to work full-time for a busy urban EMS agency, enrolled in the local community college and graduated with an Associate's degree in Emergency Medical Science. I continued to work as a paramedic, did very well, and eventually moved up the ranks to become a critical care flight paramedic with a large research based teaching hospital. In this position I have trained staff, written protocols, participated in research, and I have also taught ACLS, PALS, and BLS/CPR in the hospital's life support education department.

I made the decision to return to school and get a bachelor's degree two years ago. I finished my B.S. with a 3.6 and have a science GPA of 3.8 including all of the medical school pre-reqs, all while continuing to work full time. My question is this: is it worth a shot to apply to med-school now, given my extreme character change? Or will my mistakes as a 20 year old keep medical school out of reach?

tantacles

Your GPA from 10 years ago will be on your AMCAS application and may get you screened out of some medical schools, but given the amount of time you've spent in between undergraduate #1 and #2, it's likely that many will willfully overlook your GPA as a 20-year-old. Schools often award reinvention, and you've done a great job of redeeming yourself.


Getting a second bachelors.

I have a very low undergrad GPA in B.S Biology of 2.58 but a better M.S GPA in Biomedical sciences of 3.5. My MCAT is a 505. I know I should retake the MCAT but would it also help to do a second bachelors perhaps in biochemistry or something along those lines? I understand that some schools will like the improved masters GPA but they may not appreciate or reject me based upon my low undergrad gpa?

tantacles

Your undergraduate GPA along with your MCAT will get you screened out at many schools. I would suggest doing an a la carte post-baccalaureate program to re-do your pre-med pre-reqs and get better grades to prove to medical schools that you can handle the material.


Should i study medicine in Uk?

I have completed my A level and i am 20 years old. Can somebody please tell me if i should apply in a UK university for medicine. It is very expensive to study there as an international students, i dont know if i will be able to pay all the loans once i become a doctor.

tantacles

If you wish to practice medicine in the UK, studying medicine in the UK is your best option. If you wish to practice medicine outside of the UK, I would suggest going to medical school in the country you wish to practice. Insofar as financial issues are concerned, it would be worthwhile to compare the tuition of UK medical schools with the tuition of the medical schools in the country you are also considering practicing in. It is quite difficult to advise you without knowing anything else about where you wish to apply.


Which Kind of Post-Bacc Program?

I'm currently a rising senior biology major at a top 10 college looking to enroll in a post-bacc program after I graduate. My dream program is Scripps but I'm not sure if I should be looking at career-changer programs because I was a biology major. Over the course of my major, I took one year of general chemistry with lab, one year of biology without lab and one general physics class. Am I eligible for career changer because I didn't complete the pre-med coursework? Or am I ineligible because I have taken some of the reqs? Thank you so much!

tantacles

If you are able to complete the pre-medical coursework with excellent grades, the number of pre-requisites you took during undergraduate will not be important. As far as individual programs and whether they will accept you, I would reach out to the programs themselves and find out what they are looking for in terms of applicants as it is impossible to predict what individual programs are looking for.


Decent GPA, but downwards trend, can I still get into medical school?

I had a really strong start then had a few falls. Year-by-year my stats look like this FR 4.0 cGPA, 4.0 BCPM SO 3.72 cGPA 3.63 BCPM (C in Orgo I) JR 3.3 cGPA, 3.2 BCPM (W in Orgo II and C in PhysII) so overall... 3.7 cGPA, 3.6 BCPM

Some other aspects of my application are the following: -I have several scholarships including a full ride merit scholarship -I've been a CNA for 2 years, and will be transferring to a highly respected hospital soon -40 hours of shadowing -1 year of Clinical research where I hold a leadership position and my name was on a poster presentation at a medical conference -300 hours of non-cinical volunteering -I was a lab aide/student researcher at a biology lab for 1 year -I am in the Honors program at my school where I will be doing a senior year research project/presentation I am a Latina/Hispanic girl

My senior year I hope to: get straight A's and go on a week-long medical missionary trip. I have NOT yet taken the MCAT. I will take a gap year after undergrad...any suggestions on what I should do? I am really worried about my downward trend in GPA. What are my chances?

casedentalmed

It is a bit challenging to look at someone with just numbers without additional context, and such advice is best given by someone such as an academic or prehealth advisor at your school. I don't know if you have taken a lot of strong upper-level biomedical science classes or if your struggles are with coursework related to having to work. I will admit that general downward GPA trends with increasing rigor is a concern because it doesn't get easier in medical school. You'll need to ask advisors at your school and in various admissions offices on suggestions.


Am I in over my head?

Hello, so I wanted to get some advice on a difficult situation I have found myself in. I graduated this last year with my bio degree with the hope of taking my MCAT at the end of the summer after full-time studying to get it over with before taking my 2 gap years. I had a lot of hope after making a studying plan, purchasing all my AAMC material, and signing up for the July 28th MCAT. I had 2 hitches though, I hadn't taken a year of physics yet and I'm a terrible procrastinator. I thought though that since I'd blocked out my whole summer, then I'd be more responsible and have no excuses plus I really needed to do well with my 3.2 sciGPA weighing on me.

Well, I failed myself. I couldn't stay on track and spent most days too lazy or overwhelmed to either start studying or reviewing what little I did accomplish for the day. I ended up rescheduling my MCAT for August 24th and redoing my study schedule. I am 3 and half weeks out from my MCAT and I still haven't gone past the first 2 chapters of most of my books and I don't know what to do. I have been in a depression this summer as I felt like I graduated with no sense of true calling into medicine or any field, and I feel like that has killed my motivation for the MCAT even though I want to try, I just can't get myself to. I thought I wanted to be a doctor but I constantly saw others around me with a passion and discipline for their chosen paths that I didn't have. Although I've done the whole pre-med classes, shadowing, and experiences that qualify me to apply next year, I didn't do it with the best grades or same level of reassurance for pursuing medicine, but I also I feel like by giving up now, I am not giving my 100% and would be walking away with a lot of regrets.

At this point though, I don't know whether to try to cram and take it in August or even early September or reschedule it for next January. I've wanted advice all summer as I saw myself slowing slipping farther and farther behind, but I felt like the more the summer passed, the deeper my shame and guilt grew as I had been telling everyone that I was studying when really I was paralyzed by my own fears and doubts. Now I feel trapped because everyone(family, friends, mentors, pre-med advisor) expects me to take it and do well since I was supposedly studying all summer and if I change my date to January, the whole summer and my registration money feels like a waste and I wouldn't know what to tell people. Do you see my dilemma? And to be honest, I don't know if I need MCAT advice or psychological help more :(

tantacles

I would suggest that you reschedule your exam for when you are truly ready. You will ideally not take the MCAT more than once. It does not matter what your family thinks; if you do not do well on the MCAT you will be unable to get into medical school. Change your plans if necessary and let the people you love know that your plan has changed.


Became Premed Student Beginning of Sophomore Year

My initial question is: should I have a gap year between college and medical school?

I am currently a sophomore in college. I changed my major from Nursing to Community Health with a premed track. My question about having the gap year between college and medical school is because of the prereqs that I have not yet taken.

I still need to take: one year of general chem and one year of organic chem, one of physics, microbiology, biochemistry, statistics, and genetics. Would it be possible for me to take these classes before I take the mcat next year (2018-2019 school year)? In addition to these classes, I have to take all of the classes that I need for my major.

Do you think that it would be possible for me to complete this entire list of classes before I take the mcat next year? If not, would it be helpful for me to take a gap year?

Another question that I have is, would it be wise for me to take the mcat before i have completed all of my prereqs?


casedentalmed

Your question would be best answered by an academic advisor and a health professions advisor at your school. Note that the MCAT may cover some items you might have to learn in the courses you plan to take, and at least the health professions advisor can be able to determine this to help you make decisions on sequencing your courses. I would not take the MCAT before you know you have coverage most of the topics that you would need a solid foundation for the exam.


Can I apply to US medical schools after my second year of undergraduate studies?

I have finished my second year of undergraduate (in Canada) and I am starting my third year in September. Many Canadian medical schools accept students after their third year. Can I apply this application cycle to US medical schools?

Some info about me: GPA 3.95, MCAT 515, lots of extracurricular and research


casedentalmed

This is a question that requires you to contact the schools that you specifically want to focus on. There are some schools that may explicitly state how many completed class hours you must have at minimum to be considered. The more critical issue though is whether the schools will consider you if you are not a US citizen (you don't say that you are a US citizen going to college in Canada).


I am a senior Health Science major and I'm wondering if my goal of medical school is unattainable at this point in my career?

I have a science GPA of around 3.6, however, I have had to re-take Organic Chemistry twice (because I had a baby in the middle of the semester) and my GPA is only around a 3.2 right now. I do have an internship on my resume as well. What are more ways I can improve my chances of being accepted into medical school? My undergraduate degree was more difficult than I initially thought going in, but I love it. I love medicine, and I have been working towards becoming a doctor my entire life. Does anyone have any tips to improve my chances of medical school? I'm taking the MCAT this fall, and I graduate in May 2018 with a bachelors in Health Science (pre-professional) and minor in chemistry. And if you could cater your tips to a full-time student mother, that would be great. I'm blessed with my husband being able to work for the both of us at this time, but I am at school for around 30 hours a week already. Thank you so much for reading this far! And I appreciate all recommendations. I've gotten some negative responses in the past because I have a 1 year old, and apparently 21 year-old mothers don't "make it" in medical school, and it is so discouraging.

casedentalmed

For advice on how to approach your application being a full-time student mother, you may be able to find someone within the American Medical Women's Association or American Medical Students Association (http://www.amsa.org/members/benefits/amwa/). I am hopeful you will find someone who can be more help than those that have responded to you negatively.


Acceptance rate with online degree

I am finishing my undergrad with a degree from Penn State World Campus (online degree) studying Biotechnology. I understand I will most likely have to take post bac classes to fulfill pre-med course requirements. On average how many students are accepted to medical schools with undergrad degrees from online universities? I am 32, I have been a firefighter/paramedic for 6 years and I am currently working as a paramedic for a remote community in Tahoe National Forest; I love being a paramedic but I have always wanted more knowledge and authority to treat patients. I am hoping my background will help balance the fact that I have an online degree.

casedentalmed

I hope you have been able to make contact with admissions officers at the schools you are looking at seriously. They should be able to let you know if any students with online degrees similar to yours have ever applied or been admitted. That said, if you know you have to take postbac classes, I would focus on making sure those are NOT done online and presented to you in an intense science-oriented schedule similar to what you would experience in medical school.


Can one go into surgery while having sjogren's- as in punctal plug treatment--> watery eyes--> the need to wipe watery eyes ? I just realized this could be a problem with the whole goggles/vision thing.

So sjogren's can be treated with punctal plugs, which cause excessive tearing. This hasn't been a problem working in a lab/ with wearing glasses, but it is necessary to wipe away excess tears to maintain vision. Would this be a problem in pursuing surgery?

tantacles

If you are able to do the work, you will be able to go into surgery. I would suggest that you look into this while you are on your surgery rotation as this will give you insight into whether surgery as a career will be possible.


Multiple research positions vs continuing in one

Do top research schools prefer students who have done like 3 years of research at one lab over someone who did 3 years of research over 3 different labs? Or would that be not that big of a concern?

Also, how are summer research projects looked upon? Is doing 3 summers of intense research and none during the school year considered bad?

casedentalmed

I'm not sure if you have talked to anyone in admissions or students at what you rank as a "top research school". I would suspect that there are many applicants with significant research experience applying to those schools. I'm sure there are plenty of applicants that have been in only one lab over two years and others in multiple lab rotations over a similar period. If it were me, that wouldn't be the key issue as opposed to being able to talk about what you did in each lab and how you were involved.


Schools/programs (MD/PhD) to apply (3.67 cGPA, 3.57 sGPA, 525 on MCAT)

I would be incredibly happy going to any medical school but I'm curious if I stand a chance at the top tier schools where my GPA is in the 10th percentile of the accepted pool. And if so, is there a way to know which schools would be more lenient on below average GPAs.

I'm really interested in pursuing research and definitely considering the MD/PhD path. But I see most MD/PhD candidates have incredibly high GPAs. So would there be a chance of acceptance if I chose the MD route instead.

I do realize that schools have a holistic view but I feel like that occurs after getting to the interview.

To give you some basic info.

I'm a computer science major. Struggled a bit in my freshman and sophomore year (mostly math courses). But my junior and senior year, I have mostly A's and 2 B's. I've taken 20 credit hours every semester since sophomore year (which I don't necessarily think makes all that much of a difference).

Extracurricular Activities/Etc: Research - 1.5 years of computational biology research, half a year of theoretical CS research (I also will be doing full-time research for the next year) Volunteering - 2 years volunteering at a hospital (~150 hours) Regular community service (~250 hours) Leadership - 3 officer positions and founded one healthcare related organization Founded a non-profit (very small) Shadowing (a few weeks) 2 computer science internships IM tennis

tantacles

You have a chance of acceptance going both the MD/PhD routes, but I would suggest that you apply to both, particularly given your GPA. You likely have some chance at the top tier schools, but it's extremely hard to say, and I would recommend you consider those schools "extra" schools that you add on after the fact.

Wedgedawg, a prominent SDN user and moderator, has created a system to determine where you're most likely to have a chance at acceptance. See below:

https://forums.studentdoctor.net/threads/wedgedawgs-applicant-rating-system-updated-jan-2017.1131149/

Once you've come up with a list of schools to apply to based on that system (I would choose 15 based on your GPA and MCAT score), add on some reach schools and then post in the What Are My Chances section of SDN with your full list so that we can give you some guidance about what schools to add and subtract.


Low gpa 2.7 Community College . Will I still have a chance for medical school?

When I started community college, I had family struggles which impacted my gpa. My parent aren't supportive and I was struggling from lack of resources to navigate college and poverty.

My current gpa is 2.7 from community college and looking forward to transfer in Fall 2018 for undergraduate college.

Will I still have a chance to go to medical school when I transfer to an undergraduates college to reset gpa or will my community gpa still count against me?

tantacles

You still have a chance to get into medical school. Your community college courses will still count, but many medical schools reward an upward trend in grades. You will have to improve your grades and do your best to get all A's in the future.


Retaking Gen Chem II Three times...?

I took Gen Chem II the first time during the spring semester earlier this year and withdrew because I wasn't doing well. I'm currently retaking gen chem 2 the second time this summer and i've had my past two exams given back to me... they've both been terrible grades. I think I won't be able to make it to a C. (We only have 2 more exams until the final grade is out.) So i'm reconsidering on retaking gen chem 2 again next spring semester if I fail this summer. Not to put any blame on anyone, but I came from a really bad high school so my math foundation wasn't so strong. I now realize that the reason why I'm not doing well is because I suck at math. I I feel so bad because I still haven't made it to organic chemistry yet. Is it even possible to become a doctor anymore? I'm thinking about physician assistant school as well... I don't know what else to do with my life.

tantacles

My suggestion is that if you are failing, withdraw from the course before you get a grade. It is still possible to become a doctor, but you have to assess your abilities and figure out if you're capable of completing the requirements given that general chemistry is the first of many.


Optional essay on secondary apps

How important is it to write an optional essay on secondary applications when you have no terrible things you need to explain to the admissions committee (ex. bad grades, poor MCAT, time off, etc.)

tantacles

If your GPA and MCAT are appropriate for the school and your essays for mandatory sections are solid, there is often no need to fill in optional portions of the applications. Those portions are optional, and it is not a trap.


Would an Emergency Medical Care undergrad degree put me at a disadvantage when it comes to med school admissions?

I live in a very rural area of my state. At most of the colleges I have toured, I have been interested in a degree in Exercise Science/Kinesiology. These schools have been either out-of-state or at the other side of the state. However, at the closest state school, they do not offer this degree. If I pursued a degree in Emergency Medical Care would that be looked down upon by medical school admissions?

tantacles

Any major is fine for medical school admissions. The most important thing is that you get the best grades possible and do well on the MCAT.


adding new schools after you have been verified by AMCAS

Hey guys. I have been verified by AMCAS for the past two weeks and i am still working on some of them. If i send out my primaries to new schools will i get secondaries right away or will it come in waves. So is it better to wait until i have gotten through some of these secondaries first or just send my primaries now?

tantacles

It depends on the school and how they screen. Many schools will send their secondaries right away. There is no reason to wait if you know you will be applying to more schools.


Help with aspiring Dermatologist?

I am currently about to start my senior year of highschool (2017-2018) class of 2018, and I sorta messed up my GPA. Science isn't my favorite subject, I'm better in History or English but I don't like careers dealing with those studies. Everything I want to do is medical which includes science. Science isn't my favorite but if I pay real good attention I can pass the class with an A- or B+. I currently plan to retake and make up science courses that I messed up by failing by doing Credit Recovery while taking my regular senior classes. Can I still get into a decent college? I want to get into dermatology and I know it's a hard career to get into, I've heard it. I want to go to a decent college and earn my bachelors majoring in Science. This probably sounds silly since it's not the strong suit of a subject for me but I'm so determined to be a dermatologist that people say I probably won't make it though medical school but thinking into the future and picturing myself as a dermatologist, puts so much motivation and focus into the subject of Science. Is this possible? Can I get into a decent college? Get into medical school? Or even get a spot in dermatology residency? I'm so motivated but also scared that I won't be able too due to my current highschool grades (that I am currently fixing up asap) taking the SAT, college admissions and all. I want to try and maybe be a dental assistant to get by outside of school since in my state I only need months of training. Can anyone give me advice? What courses to take in college? How I can fulfill a career as a dermatologist? Get into a good medical school? Also, I want to move into cosmetic dermatology working with ageing skin, chemical peels, microdermabrasion and such, work with acne and stuff. I'm also okay with warts and such and skin diseases and all the other stuff as well but more into cosmetic dermatology. Please please please help me out, I tend to overthink and think far into the future and stress myself out.

tantacles

The most important thing is that you get into a four year college and do your best in your pre-medical courses. There is no need to be a science major unless you wish to, as your major itself is unimportant. Your high school record disappears once you enter college and you will have a clean slate. It might be useful also to open your mind to other fields of medicine as you will have to do rotations in multiple medical fields and a year of internal medicine residency to be able to pursue a career as a dermatologist.

Your first step, though, is getting into college, and after that you need to get into medical school. I would suggest that you focus on those two pieces of the puzzle for now.


Emergency Physician vs. Flight nurse

I'm currently a Pre-Med student AEMT and I'm actually doing a bachelors degree in which I will obtain my paramedic cert. I love EMS and Emergency Medicine. Lately I've met a lot of awesome flight nurses and I've just kind of been thinking of that as an option but I don't know if it would fulfill my lust for knowledge in Emergency Medicine and I've always wanted to be an Emergency Physician. I'm just looking for Pros and Cons as well as words of wisdom on this subject.

tantacles

I would suggest that you seek out people in each of these fields. The big differences are that in an emergency medicine residency, you learn to treat patients of all sorts in a hospital. Similarly, in general nursing training, your training will be similarly broad. The main difference is that as the physician, you will be the boss of any interaction, while as a nurse, you will always be tethered to a physician.


I'm a Pre-med student EMT that's trying to decide between EM physician or flight nurse, I love emergency medicine, and I want to know if anyone has any words of wisdom on this topic?

I'm currently a Pre-Med student and I'm actually doing a bachelors degree in which I will obtain my paramedic cert. I love EMS and Emergency Medicine. Lately I've met a lot of awesome flight nurses and I've just kind of been thinking of that as an option but I don't know if it would fulfill my lust for knowledge in Emergency Medicine and I've always wanted to be an Emergency Physician. I'm just looking for Pros and Cons as well as words of wisdom on this subject.

tantacles

I would strongly suggest that you find some emergency physicians to talk with as this will likely inform your decision. Emergency medicine physicians and flight nurses have very different roles, and an emergency medicine physician could potentially work as a flight physician, so I would suggest that you seek advice directly from sources who have pursued these fields.


Should I do an SMP?

I have cGPA 3.50 and sGPA 3.41 with MCAT score of 516 (1 year old). I got into Tufts MBS program but I am still uncertain on whether this would help me. I would like any advise on whether this SMP is worth it for me. I personally want to get into mid-tier school (like George Washington). Is that possible? I didn't submit my application yet, but I'm also considering whether I should still apply. Pretty much I want to know what is the best way to spend this one year.

tantacles

You have a good GPA and an excellent MCAT score. I would suggest you use the Wedgedawg Applicant Rating System (below) to help you choose schools. You would likely not benefit from an SMP as you are situated well to get into medical school given your stats. I would suggest you get a research or medically related job and make money rather than wasting your money on a special masters' program.

https://forums.studentdoctor.net/threads/wedgedawgs-applicant-rating-system-updated-jan-2017.1131149/


How will a lopsided MCAT score affect MD/PhD admissions?

I got a 520 on the MCAT with a 125 in CARS. The were (132/125/132/131). Does anyone know how important the CARS score is in MD/PhD admissions? I don't know that low CARS score will be heavily frowned on...

casedentalmed

There are a lot of factors with the admissions process for the MD/PhD programs, and certainly having high MCAT scores is part of the review. That said, review will likely revolve around multiple items, and just having a low subscore on the MCAT in-and-of-itself won't be a problem if there are other items in your application that could address your critical thinking and reading skills more positively.


Will my MCAT Accommodation Documentation be Reconsidered by the AAMC in Time?

So I complained and appealed the AAMC accommodation department's original response rejecting my application for double time on the MCAT. I subsequently received time and a half. This was upsetting because I had to then decide if I wanted to spend $1250 out of pocket for a re-evaluation with my psychologist. I am an online MS student studying medical microbio/biochem. I am completely running on federal aid at this point. My psychologist performed my original evaluation seven years ago and I felt it was best to go back to her. The only problem is that she doesn't accept medicaid. I started my re-evaluation appointments simply when I started them and decided to spend the money. They began in early July. My documentation would have been ready for mailing on 7-19-17. However, in my excitement with "finishing" my appts. I looked over the AAMC approval letter once more and found that I completely forgot to bring the Request for Reconsideration, Evaluator Checklist and What Does My Evaluator Need to Do? forms to the attention of my psychologist. I can tell that my psychologist didn't look over the approval letter I printed out for her when I asked if she read it. I have been so thorough in reading everything but this one time I got my arse kicked by missing these forms. Thus, I have to schedule one last test to be completed on 8-2-17 and my documentation will be ready for mailing on 8-7-17. My psychologist is a booked up specialist on vacation this week. I rescheduled my test again from 8-24-17 --> 8-25-17 to 9-8-17 --> 9-9-17. The Silver Zone deadline by which a decision on my documentation must be reached is 8-25-17. I will FedEx overnight the documentation so that it is received on 8-8-17. This gives the AAMC 17 days out of a normal 30 re-review cycle to approve my documentation. The AAMC cannot state if this is enough time. Is this enough time? For the initial 60 day review, it took 37 days to get an answer. This sounds nuts but if we multiply this ratio toward 30 it equals ~19 days. This occurred during a less busy time. I submitted additional requested documents on 1-11-17 and was approved on 2-17-17. July-September are peak testing months and accommodation applications are reviewed in the order they are received. I cannot take the MCAT without double time. I am not going to allow schools to filter me out based on MCAT score when I was unable to perform my absolute best. I won't let AAMC win this ridiculous fiasco and I don't want to delay my application to 2018.

As a side note -- I'm quite annoyed by ETS and the AAMC. ETS gave me a horrible testing experience on the GRE at one of their testing locations. I had to fight a credit card dispute for a test fee refund. The AAMC ignores most of my questions in email responses and responds in a very vague, general and robotish manner. There is a form for EVERYTHING that you try to do with them. You have to submit a form just to get permission to do something else that requires a form to be filled out! All this takes time -- often up to 30 days per form. It is so darn hard to get accommodations on this exam. I understand they are making sure that only those who deserve them get them. Still, I have received accommodations for nearly a decade for college exams at two different universities and the GRE without hassle the first time around. The AAMC is going overkill big time. This is a deterrent from applying but biomedical research and clinical work in infectious diseases with an MD is my goal. I am not competitive enough for an MD/PhD program and I feel I'd be better off with an education in medicine over research skills lost without a PhD at this point. I want an education in medicine.

tantacles

I would suggest that you take the time that you need to have the best application possible. If that means delaying your application for a year, I would suggest that you do this. Getting extra time on your MCAT will be important, and you want to do your best.


Secondaries Question

I'm a premed in my freshman year of undergrad, and I'm just looking into the application process early. And I was wondering if there is a way to look at the secondary applications of primary schools that I want to apply to? And I mean an actual copy of it I can look at on my laptop. Or will they only be available in secondary threads on the premed forum?

casedentalmed

In general, most of the secondary prompts probably have been pretty stable over a set number of years except for a couple of schools. Regardless, many medical schools have their own application portals that they only send out on a limited basis, so I am not sure you will have much success accessing the secondary applications themselves unless those links are somehow public.


What should my plan be as a transfer and pre med?

I am doing an associates in Biology at a community college, starting the fall of 2017 and will complete it the spring of 2020. I am planning to transfer to a four year university thr Fall of 2020. When should I apply to trasnfer? When should I begin studying for the MCAT and when should I take the MCAT? When should I be working on a clinical/medical experience? I had a plan for all of this but now I am confused because I thought I would finish my associates earlier. Please help! I need a plan!!

casedentalmed

Success in the route to medical school relies on assistance from your staff advisors and faculty mentors. It's just as important to rely on their expertise in your journey as it is to get great grades. There should be many counselors who can give you insight into your coursework and your plans to transfer to a four-year institution, and to that effect, they should be able to help you with your plan.


What are examples of academic honors/awards that most secondaries ask for, and what advice would you give to a pre-medical student to enhance this section of the application?

I'm an incoming Sophomore in undergrad, and I'm just skimming through secondary apps for some of my prospective medical schools. One of the questions that is common to most secondaries (from the ones that I've read through) is a list of academic achievements and scholarships. This question just kind of threw me off guard and made me realize that I hadn't really received any awards/scholarships during my freshman year of college. I was just wondering if you could possibly list a few academic honors/types of awards that you received during undergrad and put on your secondary (if this applies to you).

tantacles

Academic achievements can be anything from presenting a poster at a conference to graduating with honors from college. There is no need to fill this section up with extra material. If you have not received any awards, typically you will not be hurt by this; many students are accepted to medical school without any awards.


AP credits when creating undergrad schedule?

Hi,

I am an incoming college freshman, starting undergrad. I recently received my AP scores and I am so happy I got a 5 on AP English Lit and Comp. I would love to totally transfer these credits and not have to deal with comp in college. However, I am a bit worried if medical schools will accept this? Do medical schools accept the AP cred from high school or do they require that you retake the course at your college? idk what the smartest thing to do is? Thank you

tantacles

Many medical schools may require you to take an advanced level course to cover your english requirement. I would suggest you check the websites of medical schools you plant to apply to to see if you should take 1-2 more English courses to fulfill their requirements. Every school is different, so this will be on a case by case basis.


Route change from PhD to MD late in the game. NEED HELP ASAP

Hello all ! So I was originally planning on doing a PhD program (microbiology) once I graduated. I have since graduated and while I was volunteering in a hospital in my down time, I had the privilege of shadowing a doctor that has both his MD and PhD. So I was able to go and see his lab work and also shadow him clinically with patients. I have since decided to change things up and pursue an MD instead, and couldn't be more excited about this transition! I have quite a bit to ask so bear with me!!

Hence forth, I need insight on the process. Does the volunteer work I'm doing in the hospital (patient transport, mailing work, paper work, news paper/flower delivery, and other delivery services) count as clinical or community service experience? Should I look for something more?

I graduated from my Biology Human Physiology emphasis program with overall GPA of 3.65, I'm currently in line to shadow some more doctors across multiple specialties, and take the MCAT in August. Since I wouldn't be applying for the start of this fall, I would be applying for next fall. Would a basic EMT course in between help or hinder me? Or was looking to do a masters of physiology or masters of medical science at Loyola.

Also, how does the cycle process work? like if I applied for next fall semester, would I have to apply sooner than later? or would applying next spring be enough? I'm new to all this so any and all input would be helpful!!

tantacles

Your volunteer experience is both volunteering and clinical experience.

It sounds like you are preparing yourself well for medical school. I would not suggest an EMT course; unless you plan to practice as an EMT, this course will not be helpful to you; EMT training does not prepare you for medical school and will not put you in a better place. Rather, I would suggest that you start getting research together, take the MCAT, and find a job where you will be involved in the medical field. That could be as an EMT, but it might also be in a doctor's office or research lab. There is no reason to do an extra master's degree when you could simply work for a year or two in a relevant field and make money while you do it.

It is almost certainly too late to apply for the fall semester of this year, so what I would suggest is that you take a year or two to strengthen your application, find an adviser, and look into what it will take you to apply to medical school. You may have missed some prerequisites in college, or you may need more relevant extracurriculars, and you definitely need to study for and take the MCAT, and all of these things take time and money. Take your time and only apply once.


Can I see still improve my grades?

I completed my second year of uni with a gpa of 2.0. I'm pretty disturbed as my grades went downhill. I got a C in gen chem 1/2, a D but it's a pass on my transcript in calc 2. I also didn't do well in retaking gen bio and organic 1 in my first two years. The first time I took gen bio I got a D and the second an F. I failed organic chem two times. I feel like idk what's the trick to get through my last two years of college by sailing through, but I don't know if I can make it to organic chem 2. However, I can make it to physics and mammalian biology. I feel like my transcript is ruined. Please share some insight.

tantacles

It is possible to improve your grades, and an upward trend will be looked upon well. Just keep in mind, though, that you will have to do exceptionally from this point on to have a chance of getting into medical school, and your MCAT score must be stellar.


Do schools look into institutional action

Do Medical schools look request to the the disciplinary record of all of the students they send offers to? In order to verify that a student was not lying on their application?

Could a student get away with not disclosing their institutional actions and not get caught? Or is the risk too high?

Please let me know, thanks!

Pathdocmd

95% of institutional actions are minor alcohol infractions. One is no big deal. You need to disclose any and all institutional actions and any criminal record, so matter how small you think they are or old they are. The schools WILL find out when they get transcripts and/or committee letters. AMCAS sends out reports to all medical schools if they find a problem. It is much to risky to try to hide it. If you get caught your chances of getting accepted, let alone an interview, are zero. If they find out after you are accepted, they will rescind your acceptance or if you somehow matriculate, kick you out.


Physics confusion!

I have the option to take a calculus based physics even though my program (molecular Biology B.A to Biomolecular Science M.S.) only requires algebra based. I heard from one of my friends that upper level Medical school will not consider applicants that haven't taken Calculus based physics. I was wondering if this was true or not?

tantacles

Whether a medical school will take someone who hasn't taken calc-based physics is extremely variable. I would suggest that you look on the websites of individual schools you are interested in to see if they specify.


Continue taking second series ochem?

I am in a weird situation. I have taken my ochem class at a community college but have heard many times that its looked down upon. I have graduated from a university with a BA in English and have already completed my Bio and Gen chem series and because of money situation decided to finish my prerequisites at a community college. My question is should I invest in going to a state university and dropping this class (class starts next month but resisted already) or continue the second series in this college? (also, I do plan on going to a post-bacc or doing a masters) Also, non-related but I'm also taking a calculus course and was wondering if it mattered where its taken since it's not really part of the prerequisites.

tantacles

Many schools may not take pre-requisites that come from a community college, but most are ok with other courses being taken at a community college. I would strongly suggest that any pre-requisites you take be at a four year college.


Overlooked Disciplinary Action

So during my first semester of freshman year I was caught in a room with alcohol and received a written warning for alcohol underage. I was not drinking or intoxicated but was in the room where the alcohol was. I was told that this would not appear on my record by the hall director since it was a warning. I am now receiving secondary applications from schools and have just become aware that the alcohol charge is reported on my disciplinary action record but not my transcript. I am wondering if you think I should just report it in secondary applications? Go back and call AMCAS since I said no to IA on the primary? Or not mention it at all? Any input would be appreciated.

tantacles

I would not report this in your secondary applications but rather e-mail schools to have them put a note in your file. Most schools will look over an alcohol infraction, but if it appears on your transcript or record and you have not been transparent, they may care more. Thus, I would contact the schools individually instead of writing anything in your secondary.


If I am going against an undergraduate school's anti-gay policies, how honest can/should I be about it with medical school I am applying too? How will it be seen?

I currently attend Brigham Young University. I came to terms with my sexuality during my junior year and had a change of religious belief away from the Mormon religion which runs the school. The school prohibits any kind of homosexual behavior including dating, holding hands, hugging etc... Not wanting to give up my research and take longer to graduate by transferring schools, I have been dating guys, but keeping it quiet. I have been open about my sexuality in my applications to medical school, and my transition to accepting myself and my sexuality is a big part of who I am and something I would like to include in my application. Not to mention schools ask about sexual orientation in secondaries. I am worried about how to approach this issue in secondaries and interviews. I would love to just be completely honest, but I am worried that violating the schools policies will be seen very negatively even if the policies aren't agreeable.

tantacles

About 99% of schools will mostly be on your side. I would recommend that you do whatever is most comfortable for you in terms of interviews and translate that to your primary. If this is something you'll want to talk about, it will most likely be well received. On the other hand, if this is something you're uncomfortable speaking frankly about, it would be poor form to mention something on your application that you didn't want to speak about as the application gives many interviewers a chance to get to know you before you come to visit their school, and they will likely want to ask about questions in your applications.


low gpa post bacc

im a junior with a very low gpa. most probably would go for a post bacc or smp after bachelors. i was researching on post bac/smp programs that has linkage with med school. what i was able to see was that most post bacs require you to be a career changer(no or few science courses completed). if you have completed all prerequisites you are not even eligible to apply to those programs. since i haven't completed all of my science prerequisites should i change my major to a non science / non premed major? thank you.

tantacles

Many places allow you do to do a la carte post baccs, where you take the courses and are not part of a formal linkage program. I would suggest this if you are too far along in your major to switch course. You will need to repair your GPA, and a linkage with a medical school is not necessary, if you can do very well in your postbacc.


Disclosure of Sealed Criminal Record on Secondary

Hello,

I was interested in applying to medical school, however, I have a sealed criminal record that will be 5 years old by the time I apply (2019-20). This won't show up on a state background check or most commercial (in the event it does I challenge it with the court order for sealing). The incident involved a childish, immature argument over shoes with my sister that escalated to calling the police. I was ultimately convicted of disorderly conduct (a fourth-degree misdemeanor) in 2014, before I formally started college.

Otherwise, I'm hoping to craft an exemplary application to show that I have since matured exponentially and grown into a better person after this turning point in my life. Hopefully, my credentials offset this black mark in my past that I fully own.

How should I approach answering the questions on secondaries with a sealed conviction?

Follow up accordingly, and thanks for taking the time to read!

casedentalmed

Full disclosure is always your best strategy, especially if the misdemeanor will ultimately get expunged from your record. Read the instructions carefully and be prepared to tell them the satisfactory minimum required for your disclosure.


AMCAS hours mistake in activities.

I messed up my hours on my activities sheet. I had a job that I miscalculated hours for saying I worked 1400 hours, but I really worked around 800. My question is how big of a deal is this? I want to be honest and I did NOT mean to falsify those hours. I didn't update it before I submitted. I called AMCAS and they told me I can't do anything about it and I'll possibly be able to adjust it with secondary applications. I'm worried because that is a lot of hours and I would want to check that if I was reviewing. My application is also not verified yet.

tantacles

I would not mention it unless it is brought up by an interviewer. If someone asks you about it, be honest and say you miscalculated. If not, you need not say anything.


Canadian, 3.83 GPA, 510 MCAT, help!!

~100h shadowing, one publication, Started Charity Organization Own a small transcription company and I do transcriptions for a family doc. Worked/Volunteered at a clinic and pharmacy for 4+ years

I would really like to go to a top tier American University or something like Albert Einstein. Aside from retaking the MCAT next year is there anything else I can do? (Special Masters Program etc.?) What should I do next year if I don't get into any schools?? (Masters etc.?) I am also Hispanic/Latina, Female. Struggled a bit in first year but consistently got higher GPA: 3.73 in first year to 3.83 in second and 3.93 in third year. The reason I'd like to apply to American schools is because I am Cuban so I would be able to become an American resident in a year of living there. Would this give me an advantage or is it based on me having a Canadian Undergraduate degree?? Thanks!! I'm a bit lost when it comes to American Universities

tantacles

I would suggest that if you are absolutely intent on going to a top tier university, you retake the MCAT. However, I would strongly suggest that you apply broadly and if you are accepted to ANY US MD school, you take the acceptance. All of them will grant you a fantastic degree and make you able to apply for residency. If you do well in medical school, you will set yourself in to go into any specialty.

You can still add schools at this point of your application, and if you are worried about getting in, you may wish to add some lower tier MD schools to prevent yourself from having to apply again; it is still early in the application cycle so you have a great chance of getting an acceptance, particularly given your stats.


Pre med req situation

I enrolled into a gen chem 1 course while I was at my previous school, and I had filled out a transfer into anther school that has prestigious biology program and made it. Before I received the acceptance letter from my new school, I enrolled at a CC to take gen chem to get ahead for the semester. While I was at orientation two weeks ago, I was told by my advisor they wouldn't accept my credits. So now I'm left with an option to withdraw and get my money back but at the visiting school I will have a W grade for a pre med req. is it possible that I won't ever have to mention this grade of a W to anyone ? Like med schools. I'm a stellar student with a 3.9 gpa. And this is so unfortunate, I literally feel my life coming to an end. Please get back to me I would love to hear your advice. Thank you

tantacles

Even if your school won't accept your credit, the credits may still be worthwhile. Your grade, while not accepted by your school, will definitely be accepted by AMCAS, and if you get an A, it will positively impact your GPA. I would avoid dramatics as your life definitely will not end over one course. If you get a W, your life still will not end, and you won't be heavily impacted. You are over-thinking the situation, and you don't need to worry about either scenario.


Will this ruin my chances of med school?

I am currently going into sophomore year with outstanding grades 3.82 gpa. During this past year at my college I wasn't happy with the biology department so I decided to fill out a transfer to a very well known biology program at another school. Since I didn't take chemistry at the college I attended as a freshman I decided to take it as a summer course at a CC. When the course started I received a letter saying I was accepted into the other school(yay), so I went to orientation and told my pre med advisor I'm taking chem at another school as a visiting student and she tells me they won't take my credits for the course. I'm doing well at the CC but I contacted their financial office and their willing to refund me 50% of tuition and leave with a W grade. I'm in a messed up situation and I don't know what to do. I am scared that this W will cost me med school.

tantacles

One W will not cost you medical school, and if you take this course, it will still be accepted by AMCAS.


What does it take to become a plastic surgeon? still considering what to specialize on.

I still don't know what to go for, I will be becoming a high school senior this august and after fixing some grades in the community college I will go into pre medical to then get started in medical school. Also how long would this take?

tantacles

I would suggest that you focus on getting into medical school first. That will be the most important step. I would suggest you do your best to get as close to an A as you can in all of your courses, and then study for the MCAT and do well. Along the way, make sure that you do plenty of volunteering. I would use SDN judiciously, particularly the forums, as there are many skilled and wise advisers there who are physicians and admissions committee representatives who I'm sure would be happy to advise you.


2 Gap Years or 1?

Currently debating whether it would be wise of me to take another gap year and would greatly appreciate any input.

Just graduated from a top 12 university, average cumulative GPA (3.6), below average science GPA (3.45), solid research experience, worked part-time during the entirety of school, solid ecs. However, my clinical experience is lacking (shadowed around 45 hours total 3 different physicians). I plan to work full-time in a hospital during my gap year after re-taking the MCAT (originally received a 29).

My question: given my stats aren't eye-opening and I lack clinical experience, do you recommend I take an extra year to apply so that I can include my full-time medical employment on my primary AMCAS next year, or will doing it for a few months prior to interviews (and continuing afterwards) be sufficient? Would of course rather apply this cycle, but am open to delaying if absolutely necessary! Thank you!

tantacles

I would recommend that you take the full year off. You want to have the best application possible and only apply once, and it sounds like you need an extra year to brush things up and be ready.


I took 2 years of community college classes while still in high school but ended up with an unsatisfactory GPA?

I took roughly 80 credits through a dual enrollment program at a local community college but I ended up getting a 3.0 (!) GPA. I wasn't really sure what career i wanted to go into until the end of my senior year in hs, but the damage has already been done. Now I am a college freshman starting fall 2017 with a direct acceptance into their biochem (bs) and performance music (bm) majors. How much will this impact my medical school application/acceptance? How can I compensate for the mishap?

tantacles

Medical schools understand that classes taken during high school might not go well. My suggestion is that you look forward and do your best in the courses you have going forward and do your best on the MCAT as well.


Looking for any medical programs across the U.S. Would extremely like help!

I see myself in the future doing emergency medicien. I'm currently trying to set my path towards becoming a E.R physcian (Trama doctor), and i'm hitting a couple of road blocks. First of which is that due to my age(17yrs), the programs in my area(Homestead FL), all pretty much require you too be 18. Which is fustrating to me because i really wanted to dedicate this summer to advancing my carrer and im two weeks in with absolutley nothing. I even have skipped partys and didnt hang out with friends just as a caution too not get tied up and prevent me missing any calls. So as a result of getting fed up with the situation of programs near home so i decided to branch out more. So i decided that i'll seek programs abroad. One caught my eye, the program is called "Gap medicien", and it was pretty much perfect in terms of what i was looking for, it had a emergecy medicien class, i would've been able to travel to republic dominica, and apprently this program allowed you too actually preform care under the guidence of the doctor, which i thought was really awsome, it was really hands on which i really appreciated, so being excitied i showed my parents. Who are definitely supportive and encouraging of what i want, but i ended up with a lecture from my dad about how dangerous dominica or any 3rd world country can be????. And that he wants me too be a more worldy person but it'll have to basicly be at his pace. So whatever i didnt fight him on it. We ended the discussion on the basis that it has to be in America, So being that im dissopointed with the results of my search and summer so far, i ask preferably doctors or anyone whom truley knowledgeable on the subject, would see this and be so nice and caring to donate their time too point me in the right direction being; if theirs any programs in your area that would fit my progress, it doesnt even have to be strictly emergency medicien but something hands on preferably. And that also goes for any internships or shadowings, i just want to get out there and take my first step already. I'm very eager to get started and thank you for the consideration too reply it is defiantly appreciated????. If anyone wants to talk more in depth about it then this post allows ill leave my snapchat too thoes who would ask. Thanks again.

casedentalmed

You have a lot of time before you should consider opportunities outside the US. Since you haven't even hit 18, you will still face some restrictions on what you are able to participate in, and even when you get into college, hands-on experience in a clinical setting is highly frowned upon until you actually get into medical school. I'm sure there are many outside-of-US mission trips organized by various faith-based groups you can be part of (as long as your adult guardian or parent goes with you).

Realize that health is just one of many needs that most people in a destitute position face. The more you learn what struggles they go through with their lives, the easier you can understand what you are able to do as a physician and what you cannot do. ER is not the glamorous life you may see in TV shows or movies. Watch a few documentaries like "The Waiting Room" (you can stream this) to really understand.


Burn out vs second guessing med school

Ever since I was 12, I've always wanted to become a physician. I truly felt it was my calling in life. I couldn't imagine doing anything else (not in a stubborn way; I always kept my Options open). Sadly as I grew older, becoming a doctor became part of who I was. Everyone would associate me with medicine. My dreams of being a physician were always met with sincere encouragement and support even from established doctors.


I graduated in 2016 and decided to take some time off. I took a job as a PCA in my college's cancer hospital which is really prestigious (HR placed me there I had no say).

I always knew oncology wasn't "my thing" but I decided to give it my all.However, as time went on, I became worn down. The HR assigned me to pre op so I had to be there at 5 Am every shift. I had to work 40 hours/week, 10 hour shifts, with one 45 min break. Bear in mind this was my first real job, and I went from being a full time student to working full time. I became extremely exhausted. I'd do my job well and would receive compliments from patients and nurses but inside I disliked it. Also like i said earlier I hated anything oncology. Although I had great empathy for the patients, I resented being in a cancer related environment where all the patients had cancer and nothing but it. I felt like if I had been placed in the heart hospital (I love anything cardiology) I would have enjoyed it better.

Basically I'm wondering if what I'm feeling is just burn out or if it's really second guessing. I grew up with an OB/GYN father And a RN mother so it's not like I had idealized views of medicine. I know what it takes. However I'm wondering if this is an accurate assessment bc my pca job is more along the lines of nursing than medicine. I basically did all the duties a nurse does (make beds, transport patients, clean) except for administering drugs and talking to the Dr. I always knew I wasn't cut out for nursing and this job confirmed this but I'm wondering if it's accurate to transfer this and conclude that I'm not cut out for medicine. On one hand I wonder if I can handle even more rigorous schooling (it would be the rigor that would stress me out not the actual content), but on the other side, I know plenty of physicians who hated medical school and residency who now love their jobs as attendings.


I also realized in my job that if I did become a physician I wouldn't want to do 24/7 patient care, meaning that I wouldn't always want to be around patients in the hospital. I've been considering the idea of conducting research so that my time would be split instead of constantly in the hospital all day. In wondering if i should consider that road.

I apologize for the length but I'm very torn here Ana would appreciate some advice. In case you were wondering I have all the research, volunteering,shadowing (all which I enjoyed) etc so I definitely don't need to do anything else like that.

Thank you!

tantacles

You are currently not doing the job of a physician. There are many fields in medicine where you are not always required to provide patient care, and you might consider one of these: Radiology and pathology are two examples. There are also positions in medicine where you can do substantial research and not spend all of your time in clinical practice. The options are as varied as you can imagine.

It sounds as if your job just isn't ideal and that's why you're second guessing yourself. Consider finding a new clinical position that suits you better. Consider a position outside of oncology.


Is June 25th still early to submit applications?

tantacles

Yes. June 25th is an early time to submit medical school applications.


My Cars score is lower than I would like.

My MCAT score is 132/128/131/128. Do you think my CARS score being unbalanced will negatively affect my chances of getting into a top tier school? My GPA is 3.94 and extracurriculars and research are pretty good to go.

tantacles

It seems like you have a great chance of getting into medical school. it is hard to stratify your chances without other information, but your GPA and MCAT score put you in the running for many fantastic schools.


I am a new graduate and need advise on how to maximize my gap year for mcat preparation and medical school admissions.

I just graduated with a BS in Chemistry (Biochemistry focus) from the United States. My cumulative GPA is 3.54 and my in -major GPA is 3.48. During my undergrad, I volunteered at a children's museum (1st year only) , was a club officer for two years , and have 1 year research experience in computational chemistry and 1 year in biochemistry. I was unable to gain clinical experience or do more volunteering in my senior year to due extenuating family circumstances which I plan to explain in my personal statement. I plan to take the MCAT between Jan-Mar 2018 and apply for the 2019 cycle. I am applying to MD and DO schools as well Canadian schools.

So, my question is what should my priorities be before the application opens to get into med school? At the moment, I am somewhat overwhelmed and would really appreciate any advise.

So far, I plan to prepare for the MCAT fully from the Start of September and begin shadowing/ volunteering at a hospital with it. I also lack work experience but do not want to overwhelm myself. Should I volunteer, work part time & prepare for MCATs? or Just do MCAT preparation & gain clinical experience until MCAT is over.

tantacles

My suggestion to you is that you do not try to explain away your lack of clinical experience; rather, you should take extra time to help you feel less overwhelmed and get the appropriate experience. This way, you can spend the time you need to take the MCAT and then work while doing some light but continuous volunteering for a year to prepare for medical school.

Medical school is a marathon and not a race, and I would suggest that you take the time you need to have the best application possible. That might mean applying in the 2020 cycle instead of the 2019 cycle.


Chances of getting into a PA program

Hello, i was wondering if i can get help regarding my chances of being accepted into a PA program. I messed up my first few semesters in college but stepped it up before i graduated. I really want to become a physician assistant. i planning on taking the GRE after July and will also be taking microbiology in the month of july (confidant ill get an A). I know i can score really good in the GRE, I dont care which PA program i get into, as long as i can get into one. Ill have my personal statement reviewed by professionals .

CHEM 1: W, retake B+ CHEM 2: C+ BIO 1: W, retake C BIO 2: D, retake B- Physics 1: W, retake A Physics 2: A- Organic Chemistry: F, W, D+, A Organic chemistry 2: B Biochemistry: C+ Anatomy and physiology: B Anatomy and physiology 2: B+

Resume: TA for organic chemistry 1 and 2 for a year Research associate for chemistry department for about 400 hours Volunteered in a hospital for about 350 hours (shadowed a nurse) Worked (currently) as a Medical assistant and scribe in a private practice for about 300 hours ( i work with a NP and Doctor)

tantacles

It is hard to determine admissions chances, particularly without a GRE score and a full GPA as many schools have GPA minimums. I would look at the specific requirements from schools to figure out your chances and speak to applicants from SDN so that you understand your chances better.

Consider also having people from SDN review your personal statements; professional personal statement reviewers are great, but may not necessarily be worth your money.


What else can I do to improve my chances of getting accepted? From a current Masters student

I received my B.S. in Biology (concentrating in Molecular and Cell Biology) at Cal Poly San Luis Obispo with just a 3.3 GPA. I was an RA (Resident Advisor) for two years, which kind of affected my grades more than it should have, but I also had some bad winter blues. I only did two semesters of undergrad research. I have just one more semester to complete of my Masters in Biomedical Sciences at Hebrew University in Israel, which is considered a very good university and research school. I will most likely finish with a 3.76 GPA, and there is a chance that my work will be published, but probably not before I finish my degree. However, at least I will have a nice thesis from work with a great PI. I also currently work part time for an Israeli startup that is a database for online research tools. Does my lack of a lot of things (such as a lot of undergrad research, a good GPA, a lot of volunteer hours and shadowing) completely ruin my chance of getting accepted into a medical school in the US? Or, if I do well on the MCAT, could I possible get in? I just want to know if I should do more things before I start studying for the MCAT or while I study for it?

tantacles

I would suggest that you work on remedying your lack of research, volunteer hours, and shadowing before you apply to medical school. I would still take the MCAT and do your best, but just realize that you may need to spend a year or two figuring out your extracurriculars before you apply. If you do well on the MCAT, you still have a good chance of getting in provided everything else is in order.


Already sent in application, but want to retake MCAT and resubmit score this cycle

I have already submitted my AMCAS application into schools (submitted early on June 2) and I just received me MCAT score back. However, it is not strong enough nor where I need to be. I plan to retake it early August... I am doing serious work in preparing differently (CARS killed my score!!!!!!!) Is it possible to let schools I am retaking it and submit my second score in time for this application cycle's consideration?

tantacles

It is possible to do this. Many schools will wait until they receive the second score if you tell them you are retaking. However, many schools may just evaluate you with what is on your application. This will likely be on a school-specific basis.


Research Experience on CASPA Application

On the CASPA Application (PA School), what exactly can you include as Research Experience?

The CASPA website doesn't clarify other than "preferrably outside of classroom work".

I'm wondering whether to include a Sociology research piece I did for my undergrad Senior Thesis. This was a project that was part of my classroom coursework, but I executed the research myself, and it was granted Distinction and published.

The reason I want to include it somewhere on my application is because it was related to the medical field (it was Medical Sociology research).

tantacles

If you did research, it is absolutely relevant to include on an application.


Chances of MD or DO w/ cGPA 4.1 sGPA 3.9 MCAT TBD

Hi Guys, I'm really having trouble deciding a couple of things. I hope you guys can provide some guidance. 1. My practice MCAT exams are pretty low. My real exam is coming close, june 17th. Should I just take it and risk the fact that I may have a low MCAT score on record? 2. Should I apply this cycle or wait until next cycle? I really wanted to make it on time for this cycle, but how are my chances looking... especially since my MCAT exam will not go so well.

NM Resident Vietnamese; Female

cGPA: 4.13 sGPA: 3.9 MCAT: 495 on practice exams. Real exam to be taken June 17, 2017

Medical Volunteering: - over 120 hours

Non-Medical Volunteering: - over 300 hours

Shadowing: - over 230 hrs MD/DO Radiology - 120 hrs MD Ortho - 15 hrs MD Emergency

Research (3 projects) - 2 Paid and One on my own -Consistency of MRI during right ventricular function -Cancer Center Overview -The incidence of Injury to the Brachial Plexus During Traditional vs Hawk Tackling.

Employment: - Radiology/Faculty Assistant at UNM Hospital - Teleradiology specialist

tantacles

Please wait to take the MCAT until you are ready. An MCAT score of 495 will tank your chances of getting into school. If you need more time, take it, do more studying, and do your best. It is impossible to stratify your chances without that score, but at a glance, your extracurriculars look good.


Which medical schools are known to send out secondaries before primary application is verified?

I have heard of some medical schools sending secondary applications to students before their primary application is even verified. Do you know schools, or have a list of schools that are known to do this?

tantacles

Many schools do this. That being said, until your primary application is verified, submitting the secondary application will serve little purpose given that almost no school will offer an interview without having a verified AMCAS application.

I think the best source for this information would be your colleagues on the SDN forums who are applying to these schools; most schools do not release this data.


Can I become an orthopedic plastic surgeon

Hi I was having trouble picking between ortho and plastics I really love them both so I saw something online about a orthopedic plastic surgeon and was wondering if this was true.

tantacles

If you wish to do both, you may be able to do both residencies. I would suggest that you speak with your adviser in medical school. If you are not in medical school yet, I would focus on gaining an acceptance to medical school.


What should my objective be on my resume as a student?

I want to gain clinical experience as well as research experience or a job related to what I like, I am Interested in anything related to neurology, neuroscience, Neuroradiology and diseases such as Alzheimer's

tantacles

You do not necessarily need to have an objective on your resume. If you feel you must put something, you can customize your resume based on the job you are applying to. For example, if you are applying to a neuroscience job, you can write "To gain experience in neuroscience research." You can use this same strategy for every job. be sure to customize your cover letter as well!


I was averaging 504 on my practice MCAT exams, I won't know my score until July 5th 2017, should I just submit now, or wait until I get my score? I felt good on all sections except for CARS.

I felt better than usual on CP, BB, and PSs, but I did not feel good on CARS, my average on CARS was a 124, and I am mostly applying to DO schools.

tantacles

I would suggest submitting with just one school, and then you can customize your school list based on how you do or choose to wait to apply until next year if your scores are too low to gain acceptance.


512 MCAT (124, 130, 127, 131). Will that C/P kill my app?

GPA 3.7, sGPA 3.5. I got As in Chemistry and Physics in college, but just bombed the MCAT section.

tantacles

It is possible that it will hurt your application, but if you apply broadly to many schools that fall within your MCAT range, you stand a good chance of getting interviews. I would suggest you purchase the MSAR to help you choose schools appropriately.


Institutional Action

I had made a prank as a sophomore (last year) where I posted a staff member's picture and phone number on a public intimacy website. I didn't know that this was an offense until I found out from the school that the staff member received nudes, and this caused her emotional distress. I was charged by the university with sexual harassment. I didn't know that I could get in trouble for such a stupid prank, and when I found out, I was shocked and just apologetic. I never meant to cause a person emotional distress. I thought it was merely a prank. I'm about to apply to medical school. How bad are my chances with this on my record? I understand I have to report it to AMCAs and a reflection on how this has influenced me for the better. I just want to know whether it's worth applying still or not.

tantacles

It is hard to say how much this will affect your record, but it is likely that it will negatively impact your chances of receiving an acceptance. I would suggest that if you apply this year, you focus on what you learned from the experience and how you grew, and express how sorry you are that you did this.


Coursework AMCAS

In the AMCAS handbook, it states:

"The most common reasons for a returned application include: failure to list coursework in chronological order." and "Enter courses in chronological order exactly as the appear on the official transcript of the school where they were originally attempted."

I have completed inputting my coursework, and I did it in the exact order that is on my transcript, but it still did not save in chronological order. I made sure to do so. The semesters themselves are organized, but the courses are not in exact chronological order. Will this be an issue, and if so, how can I fix it?

Thank you

tantacles

I would submit your AMCAS application and allow AMCAS to determine if your application is appropriate. If they return it, you will have your answer.


Non-science letter of recommendation

I participated in an elective program through the University of Tennessee College of Medicine that I was able to shadow physicians of many different specialties and I gained one hour of credit for each semester. The associate dean of the medical school who was over the program wrote me a letter of recommendation for my application. Will this count as my non-science since I received credit for it? Or do I need to ask a professor who I had a lecture with?

tantacles

I would try to find a professor with whom you actually took a course for your non-science letter. This letter, while it may be strong, will not count for much in terms of medical school applications because it shows little about your academic potential; shadowing is an activity where you do little but watch, and this letter, while it won't hurt your application, likely won't count towards that requirement.


Really, how important is undergraduate research?

So I have heard people say "do undergraduate research if you want to, if not, it's ok not to". But then I have also heard that it is an unwritten requirement at most schools these days. The MSAR indicates that at most schools, 80-90% of students have done some research, and this is even higher at the top ranked schools.

So basically, I have been working in a lab as a volunteer for a few weeks now, and I honestly don't enjoy it. My question is, would I be better off continuing with research this summer just to have a few months of it on my application, or would it be better for me to use this time to gain additional shadowing experience/volunteering (things that I do actually enjoy but already have a fair amount of). I find that the two options are mutually exclusive, as the research is taking up a large portion of my time along with class. So assuming I am not interested in going to a top 50 research ranked university, how would not having research experience affect my app? I have a solid GPA (~3.8) and assume I will get a solid MCAT score as well.

tantacles

At many institutions, research is not valued strongly. Clinical experience and service may be valued more. I would suggest that you look for schools whose missions are more focused on service to apply to as those schools would be more likely to review your application positively, particularly if you have an appropriate GPA and MCAT score. In addition, a few months of research will likely not be a significant positive factor in your application due to the short duration; it will likely count for little.

I will also recommend, though, that if you don't enjoy scientific research that you participate in scholarly work elsewhere; if you are able to find a position doing humanities or social sciences research, it may hold just as much weight.


Graduated Pharmacy student

I have completed my 6 year Pharmacy degree at Rutgers University and have graduate with a doctor of pharmacy. I want to continue to enhance my knowledge within the healthcare profession and apply for medical school (MD or DO). I graduated with a GPA of 3.066 from Rutgers University.

I am not sure how to calculate my science GPA but If i count my GPA for BIO, CHEM,ORGO, AND pHYSICS it is only 2.5.... I struggled my first 2 years of school. I have many other science classes in graduate years but i am not sure if they count.

I just wanted to know what path I should take. I have a lot of clinical and research experience of working in multidisciplinary teams in the hospital.

I still have to take the MCAT and I understand I have to score high.

What are my chances for a Post-bacc program for DO schools or even MD schools. any help would be appreciated.

tantacles

Mathematics courses apply as well for your science GPA. There are many calculators available online.

The competitiveness for post-bacc programs is hard to gauge, but you can also do a post-bacc a la carte at your state school if you are not accepted to a formal program, and you will likely need to participate in one for GPA repair. If you can correct your cumulative GPA and science GPA and do well on the MCAT, you will have a chance at medical school, though DO will likely be a safer bet.


More suited for MD/DO or PA?

Hi, 34 yr old male here that just graduated from Berkeley MCB program with a degree in cell and developmental biology with an emphasis in physiology and medical biology. I was a transfer from San Jose City College and previously attended North Idaho College for a year. I have calculated my BCPM (3.49), AO (3.27) and Cum (3.42) GPA as best possible using the AMCAS guidelines. I have not yet taken MCAT because I'm not sure if I am competitive with these scores. I did not do research at Cal, but I do have lots of clinical exp. working in hospital settings as a CNA (2000+ hrs) on the nursing resource team (AKA Float Pool) so I've worked in all departments: ED CCU PCU ICU GEN MED SURG REHAB ect. I have completed technical training and obtained certificates in Nurses Aid, Phlebotomy, and EKG tech. Can someone in the know tell me if I would be a candidate, if got a decent MCAT score? I'm also considering PA school. Would I be a better candidate for PA school vs Med school given my current background?

Best to all! and Thanks!

tantacles

Without an MCAT score, it is impossible to stratify you, but I would estimate that if you were to receive an MCAT score of 512 or above you would be competitive for MD and DO schools and 508 or above you would be competitive for DO schools. You would also be a candidate for PA school. I would suggest that you take the MCAT and purchase the MSAR, which will tell you the MCAT and GPA combinations for students at all US medical schools. Then, I would make a post in WAMC AFTER you have received your MCAT score for help building an appropriate school list.


Academic probation do I still have a chance to attend med school?

My story: my freshman year first semester, I started off really well I was doing on my work, handing it in on time, was getting the grades I wanted . Then I had got into a really bad car accident. This car accident caused me to stress and kind of lose focus on all of my work. I was not trying to lose focus on my schoolwork but it was something that just kind a happen the stress just took over my whole body I had to go to court it was just a big mess. During finals week of my first semester I thought I did good but then again I know I didn't because of how much stress I was in. I ended up failing I believe three classes and that caused my GPA to go down to a 1.0 when it was originally a 3.2. When I came back for the second semester of my freshman year I was placed on academic probation and I met all the requirements but I still had to back out of two of my classes because my GPA was so low I ended up going from a full-time student and taking six classes to going to a part-time student taking three classes. This semester my academic advisor had changed and he asked me what I wanted to be a major in? and I explained to him that I wanted to go to med school to be an anesthesiologist and things of that nature. He was just kind of putting me down telling me I couldn't be a doctor and I would have to work really hard which I already knew . I have never failed any class in my whole entire life and this was kind of depressing for me but I knew if I wanted to get where I wanted to go I would have to work my butt off. I made sure that I didn't let what he told me put me down so much because I know if I wanted to do something I could and it wasn't going to stop me. so I made sure that this semester I was working the hardest trying to get all A's and that unfortunately happen for one of my classes but I got a B in my other class and a C+ and my third class this raised my GPA to a 1.9. I'm not sure how other schools work but for my university in order to get off of academic probation you have to have a 2.0 and higher. I'm kind of upset about that I have a 1.9 because I'm literally like one point away from a 2.0. I registered for classes like biology and applied pre-Cal because I knew in my gut that I was going to get this 2.0. So I could take these classes but now that I am a continued academic probation student and going into my sophomore first semester I will not be able to be taking these classes which I'm kind upset about but I know that I can take other classes so that something that I am going to have to do until my sophomore second semester. I am planning and I am going to be working really really hard this semester and I will be taking winter classes after my sophomore for semester aswell because I really do want to get caught up and I am planning to transfer at the end of my sophomore year. so I just really really really want this for myself and I am determined to get into med school. I just was wondering if it would still be possible I know it's going to be really hard but is it going to be possible is basically my question ?

tantacles

If you are able to get all A's from now on and do extremely well on the MCAT, you may have a chance at medical school. Your academic record as it stands is troubling and you will not have success unless you show significant improvement. Right now, it will be an uphill battle. Do your best, and take time off to improve yourself if necessary.


Low GPA welp

Here are my stats!

Cumulative GPA: 2.9 Idk my science GPA yet

MCAT-- aiming for a high score

Founder and President of a non-profit organization to help developing countries with medical equipment

Held various leadership positions on cultural boards

On Dance team for 1 year

Research for 2.5-3 yrs working on my own project

Volunteered at hospital for 5 semesters

On APO for service hours besides the hospital hours

Do I have a chance at MD or DO or Post bacc?

tantacles

I would strongly suggest that you do a post-baccalaureate program (either formal or a la carte through a local university) to improve your GPA. Right now, you will have very little chance at admission to any MD or DO school regardless of your MCAT score.


Do schools screen based on unfulfilled prerequisite coursework?

I plan on fulfilling my biology requirement and possibly other courses that may help my application in the fall. Although I am enrolled at a community college, course registration for the fall has not opened yet, so I cannot guarantee I will be taking the class in August.

Should I still enter the class I intend to take on the AMCAS?

tantacles

If you are going to be taking the course in your next semester, you should enter the course on your AMCAS. If your pre-requisite coursework is not complete, some schools may interview you solely based on your prior coursework.


Low GPA First 2 years of Undergrad, should I retake courses or what else can i do?

Hi,

I'm a student at Brock University in Canada. I just recently completed my third year in Medical Sciences program at Brock. Honestly, the first two years were not so great academically for me. My GPA was really low the first two years and personally, I really did not take things seriously till my third year. I really did not know if I really wanted to become a doctor till the end of my third year and I started to take things seriously.Now just completing my 3rd year, I have been able to significantly increase my GPA to a 3.7 with still one more year more to go, but looking at it now and all the requirements for all the Medical Schools, I feel like my chances have really been slimmed down significantly.

Now my question is that since a lot of Medical school (at least here in Canada) do look at cumulative GPA, what should I do to increase it. I do have 1 year still left but should I add another year and retake some of the courses to increase my overall GPA. Also to mention, I have not been too much involved during my 3 years as well and am just starting to get out now.

I would really appreciate if anyone could really help me try to figure out where to go from here. I feel like I've already wasted some really critical years and would really like to know what to do from here to increase my chances.

Thank you.

tantacles

The best thing you can do for US medical schools is to continue to do as well as possible during the rest of your undergraduate degree and do very well on the MCAT. You can not change your old grades. If you feel you still wish to increase your GPA after school is over, you can take on a post-baccalaureate program to increase your undergraduate GPA further.

Most importantly, do not put in your application until your application is optimized. Even if that means waiting 2-3 years, I would suggest that you take the time you need to get your application truly ready and do not rush.


Primary application submission before MCAT score?

Hi everyone! I'm re-taking my MCAT June 19th (scores released July 18) and would like to submit my primary application beforehand. Will this affect my chances of any secondary applications or is it possible to fill out secondary applications if possible?

I'm applying to both MD and DO

tantacles

You can and should still fill out secondary applications while you wait for your MCAT score to return; schools will wait to offer you an interview until your score comes back.


3.1 cGPA & 3.0sGPA

So, I have a pretty low GPA compared to the average applicant. I have a 3.1 cGPA and a 3.0 sGPA. Freshman year killed me, I had just lost my father to cancer and college was my first time being away from home; I was in a really dark place. However, since then I have improved. Freshman Year: cGPA 2.34, sGPA 2.39 Soph. Year: GPA for Soph. year was 3.20; my cGPA became 2.81, sGPA became 2.75. Jr. Year: GPA for Jr. year was 3.57; cGPA became 3.10, sGPA became 3.0. Granted, if I do well Sr. year those numbers will go up slightly. I have 200+ hours of shadowing, and around 70 hours of volunteering. I have been published once and have been doing cancer-biology research for the past 2 years, which I hope to continue until I graduate. Obviously my pre-health advisor encouraged me not to apply this cycle but other than that, he's been pretty useless. I plan on studying heavily for the MCAT this summer & during fall semester of next year & taking it over winter break. However, I'm unsure what I should do in my gap year but I'm leaning towards SMP. If anyone can give me insight or advice on how to proceed from my current position it would be greatly appreciated. Also, which SMPs would be best for someone in my position? Overall, I'm trying to get a feeling about where I stand and where I should go from here. Am I on the right track? Is there anything else I should do? If I do well in an SMP what are my chances of gaining acceptance to an MD school? DO? Is SMP the right option or is there something else I should do? Thank you in advance to everyone who answers.

tantacles

My strong suggestion is that rather than jumping into an SMP, you address the factors that make you a poor applicant right now. Your science GPA and cumulative GPA is poor right now. A formal post-baccalaureate program would be a good way to show that you are able to handle the material of medical school. In addition, if you get all A's, it will improve your GPA significantly and show an upward trend. Doing this will take 2-3 years but will put you in a great position for medical school.

I would also suggest that you continue your extracurriculars to the extent that you are able and do as well as possible on the MCAT.

Most of all, take the time you need to make your application great before you apply.


Should I get an associates in Nursing so I can be able to pay my way through getting my Bachelors in Biology and then continuing to Med School?

I am about to be a senior in high school, I have the privilege to attend multiple medical volunteer programs and surgery is really what sets my soul and ambition on fire. However, I am torn on wether or not I should just receive an associates of nursing degree so that way I can work and receive a steady income while receiving my bachelors in biology and then hopefully continuing on to med school. I just need some advice on wether this is a good or bad idea, because a lot of people tell me it is crazy not to get an associates degree in case something happens while I am trying to obtain my bachelors.

tantacles

If working as a nurse is appealing to you, it is a good idea to have nursing as a backup plan; and yes, this can also help pay your way through your bachelors degree. Keep in mind, however, that your main priority must be doing well in your courses. If you are able to do well in both you'll be in a great place. The clinical experience will serve you well in your application for medical school if you can frame it the right way.


Should you include the experiences listed in the work/activities section in your personal statement?

Should you include the experiences listed in the work/activities section of your application in your personal statement? I feel that they are important enough that I should talk about them in the personal statement, but don't want to make the admissions committees think I am repeating myself.

tantacles

Your personal statement should talk about you. It should not repeat what is in your activities, but should build on your activities in a way that allows you to express your motivation for pursuing medicine. If you've mentioned something in your activities and you wish to elaborate on a specific experience, your personal statement is a great place to do that.

Keep in mind, however, that while the AAMC does not say this, in my experience, the best personal statements tend to be short (around 500 words or 1-2 pages double spaced), so if you are writing a fantastic personal statement, you should not have much space to repeat yourself.


Post Bac.

This is somewhat of a follow up question. My quick background info is I have finished all my prerequsites with a 2.96 cGPA and 2.5 BCP GPA. At this point, my far off future M.D chances are looking bleak, but I have been told I have a chance for future D.O matriculation if I buckle down. I understand that means making the most informed and very situational based decisions. I have been accepted to an SMP, but have been receiving mixed decisions on whether actually enrolling in the SMP or doing other post bac program that would actually affect my undergrad GPA.

My question is, are there any post bacs out there , I'm looking specific names, that offer both a mix of prereqs. and upper elective BCP courses that would help me repair my undergrad GPA as well as prepare me for an MCAT retake? Also, would these post bac be willing to take a chance with me such as the SMP programs, due to current Low MCAT and Low GPA, but good ECs, Rec letters, and shadowing experience . I have already been denied by Upenn's specialized post bac program. Also considering it is mid May, would it be too late in to the cycle to apply to these post bacs?

My only alternate choice would be to work as a part time pharm tech or surgical technician for a year, retake my MCAT and wait to enroll in a formal bac next instead of attending the SMP program that I have been accepted to for fall 2017.

tantacles

I am not familiar with individual post-baccalaureate programs that offer these services. However, many universities, in particular public universities, may allow you to do an "a la carte" post-bacc in which you can take individual undergraduate courses without going for a degree; this will allow you to raise your GPA in preparation of applying to medical school without incurring the expense of doing a full degree. Doing well in courses this way can ultimately lead to GPA repair which can put you on the path to medical school.

If you MCAT is low, you definitely will need to retake it; it is hard to determine when you should. By taking the MCAT earlier, you may be able to get into some SMPs that have automatic interviews at medical schools. Ultimately, the best path for you is somewhat unclear because of your poor GPA; no matter what, you will need to do better on the MCAT and do something to make up for your GPA. The best option is not immediately clear to me.


CPP vs UCR (Chemical Engineering Pre-Med)

Hello, I've been accepted to a few different school for chemical engineering. Ive completed most of the undergrad prerequisites at Napa Valley College and am now ready to transfer to a larger university. I'm between Cal Poly Pomona and UC Riverside. I know both hold a reputable title, but I somehow feel UCR would be a better option as I am looking in to medical school after my B.S. I know that CPP is the better engineering school, but if I think about my true end goal of becoming a doctor, UCR seems like the better choice because it has its own school of medicine. However, if i don't get in to med school, the engineering degree from CPP would open a lot of opportunities for me. I thought I had it all figured out, reading through several posts on this website and also at college confidential. I guess any last minute advice before I bite the bullet would be very helpful. Thank you for reading and thank you for your time!

tantacles

I would suggest that you go to medical school at the place you feel you will be able to do the best academically. Your major and school are not incredibly important. Just make sure you keep a high gpa.


Formal Post bac or SMP ( Graduated with a B.S in Kinesiology & Finished all Prereqs)

I have recently been accepted to the two year medicine track for Masters in Biomedical Sciences at , in fact about two days ago, and honestly have very stressed out rather than excited ever since.

My cumulative GPA is 2.96, but it will increase a tad bit since I will most likely be taking Organic Chemistry I this summer (last minute registration process in the works). My biggest worry is due to my undergraduate science bcpm gpa, it is very low at 2.35 and my BCP gpa at 2.5. I was somewhat surprised that even with this science GPA, I was accepted into the BioMed program a have applied to plenty of Special Master's Program (SMP). I have not applied to any formal (taking upper level undergrad classes) post bacs, but I am heavily considering it as well. I have finished all the prereqs and excelled in numerous Kinesiology upper electives. I have only taken 2 BCP upper electives. My biggest concern (you may have heard this plenty of times, apologies in advance), is that even with a stellar 2 year SMP GPA and MCAT, my undergrad science GPA will hinder my admission chances.

All in all, my question is , would a good academic record at Master programs at a known SMP combined with a future good MCAT (which I am confident that I can focus and put in my best efforts to achieve) realistically help me recover over my undergrad science GPA? Logically a 2 year upward trend should carry some weight, but I know medical admissions can be very tricky at times. I would really appreciate your honest opinion so that I can make a well informed decision on whether to attend the SMP


Sorry for the lengthy message, I tend to write a lot when I am stressed.

Thanks a lot for the taking your time out to read this!

tantacles

A stellar mcat and smp performance would help you get into medical school. However, the most important thing is GPA repair. The best solution may be a postbaccalaureate program as this will allow you to improve your amcas gpa and will help prepare you for the mcat.


Will medical schools care about 1 C+ in a biology class?

I'm a freshman in college and so far I have a 3.58 GPA and a 3.56 sGPA.

First Semester Freshman Grades Biology: A- Chemistry I: A Calculus A: A Middle East Class: B+

Second Semester Freshman Grades Cell Biology: C+ Honors Chemistry II: B+ Statistics: A Gender Studies: A

Will the C+ in cell bio come back to haunt me later even if i work hard and try to get good grades in my upper level biology classes? Will medical schools see that one C+ and not even consider my application? I'm just really worried about the grade in my bio class hurting me in the future. Any responses are helpful. Thank you in advance!

tantacles

Medical schools will not take too seriously one C+. Do your best in classes going forward and you can make up for this one weakness with future strong performance.


Post - bacc options

To make it short, I am a RN embarking on the pre-med journey.

I have been accepted to one of the local state universities, but I am having a hard time swallowing the extra debt I will accumulate from the classes (mainly because there is no guarantee of acceptance on the other end). I have a 3.90 cGPA and a 3.94 sGPA from my BSN.

I'm considering taking the needed prereqs (BIO, CHM, OCHM) at a local CC instead in an effort to pay cash for the classes. I have read that adcoms may see it as the easy way out to revert back to a CC. If I can explain my reasoning to save money, will it hurt me to to go the CC route? I'm not trying to weasel my way through, simply trying to contain debt...

Thank you.

tantacles

If you wish to go to medical school, it would be wise to take these classes at your state school rather than at community college. Rigor of the classes is, in fact, different and will matter to med schools.


Do I have a chance at M.D.? I made a D in a composition course and a C- in a geography course.

The composition class I had while in high school along with the geography course my senior year, through a program that a local community college offered at my high school. I would like to apply to many medical schools in the future after I receive my undergraduate degree. However, I am concerned that these two grades will have an affect on my admission chances to an allopathic school. Also I am considering the D.O. route!

tantacles

Allopathic schools will not disqualify you based on two grades. Do as well as you can in all other classes and your pre-requisites as well as the mcat and you will have a good chance of acceptance to med school.


Post Bacc Masters Program GPA

I was in a 1 year post bacc masters program (non thesis) and have all the hours to finish however i am not satisfied with my GPA of 3.625. I could take 7 hours this summer to increase it by 0.065 points to a 3.69; however, I also am planning on taking the MCAT early August (in 3 1/2 months). Would I take the classes and risk not being able to study for the MCAT properly or suck it up, accept my GPA and focus my energy on the MCAT this summer. My underlying question is how will admissions look at my masters GPA and at a "decent" GPA will the MCAT likely be the deciding factor?

Another option I have would be postponing applying, taking the MCAT in August, and take more classes in the fall and write a thesis.

tantacles

With your gpa, your mcat will be the deciding factor. Keep in mind that graduate level courses do not contribute to your amcas gpa.

No matter what, doing well on the mcat needs to be your top priority. If taking classes alongside it will make this impossible, I would suggest taking extra time to take the mcat and postponing your application by a year.


3.4 GPA???

I finished freshman year of college with a 3.4 GPA. Should I give up my dream of becoming a doctor or do I have a chance to bounce back from this and possibly get into med school?

tantacles

If you are able to do well in future semesters and do well on the MCAT, you will be in a great position for medical school. If, on the other hand, you do worse, you will be unlikely to do well in medical school.


Can I take the MCAT in July and still be likely to get in?

Bachelor of Science Major: Biomedical Science Major: Behavioral Neuroscience Minor: Biology GPA: 3.585 BCPM: 3.4706 4 years experience as a nurse assistant. 1 year of experience as a home-care aide. Only volunteered one hour one time, only shadowed one doctor one time, in pre-med clubs but not actively participating much, only research experience from classes; research methods in psychology, advanced research methods in psychology, drosophila genomics research (a research based course), and some research within other courses like animal behavior as well.

How likely am I to get in?

My advisor previously told me to take the MCAT in late May to early June but will taking the MCAT in July hurt my chances of getting in if I submit AMCAS beforehand?

I am not ready to take the MCAT now; I had to take 19 credits last semester so I had no time to study. If I wait til July I'll have 2 months to study around working 36 hours a week although one month will also include a summer course. Is this enough time?

tantacles

It is impossible to stratify your chances without an MCAT score. I would suggest that you take the MCAT and then return to ask for advice. If you must apply this year, submit your application on June 1 (or as early as possible) to one school without an MCAT score and then reevaluate and submit to more schools once your MCAT is back. Alternatively, you can delay your application to next year, which may be a better idea as you will be able to retake the MCAT with less pressure if this attempt does not go well and improve your extracurricular activities.


Chances of Getting in

I currently am the president of the Pre-med club at my university, have a lot of research experience and I am published. I also have a lot of shadowing and volunteer experience. I have shadowed both DO's and MDs and even Volunteered helping the poor in India. I also have great recommendation letters and have an MCAT score of 514. However, due to being unaware of my interest in medicine till later in my college education my GPA is a 2.75 due to not focusing on my grades till later in my college career. I am currently applying to post-bac programs to show I can handle the high-level course work required in medical school. I am also applying to a few DO medical schools. My question is should I even try to apply to medical schools this year due to my GPA? Also, even though I have a weak GPA is it possible for me to make it into medical schools due to my other credentials.

tantacles

Because you need the boost from the post-baccalaureate program to have an acceptable GPA and to show you can handle the coursework, I would suggest that you wait until next year to apply to medical school. This will also give you more time to study for the MCAT, which you must have a stellar performance on to get into medicals school.


Falsey accused of cheating. IA on school record

Hi all. I'm about to graduate from a 4 year public institution and about two years ago i was false accused of cheating on a portion of a homework assignment for a bio lab. I got a zero on this assignment but still passed the class. I have IA on my schools records that med schools will be able to have access to if they request it but not on my transcript. I was collaborating with my lab partner on our assignment (which is allowed in this class) and my TA thought our answers were too similar because of our indentation and short hand that we used. She reported it to my schools student conduct office and I was sanctioned because the advisor believed it was "more likely than not" that I had cheated. My gpa already is not that good and I plan to do a masters program after taking a gap year to raise it (currently 3.15) I am currently trying to get this expunged from my record but my school has no specific process for doing so and no one seems to want to help me. Basically everyone I've talked to has turned me away. I want to know what you all think I should do. I feel as though denying this even though I'm being honest is hurting me because no one seems to believe me. Is my dream of getting into med school over? Do you have any advice to help me get this expunged or what I should do/how i should explain the situation if i cant get it expunged? Should i continue to be honest or should i pretend I did this and be very apologetic? Do you know anyone who has been admitted with something like this on their record? It is going to be on my record for 5 more years. Should i wait 5 years before i apply? I have always been so against cheating and I am always seeing people around me cheat and even get caught and still have no action against them. I feel like i have been treated really unfairly but still i have learned and grown a lot from this situation. All I want is to get into a medical school and it's eating away at me that this incident could kill any hope of that. Any advice?

casedentalmed

I don't know what due process you had when it came to appealing your IA, but what is done is done. Any objections you had should have been raised then, and hopefully a judicial board of your peers would have considered your appeal. Trying to overturn it after this long is only going to distract you from the application process, and if there is no such process for appeal in your student conduct office, then you are going to be out of luck unless you want to take a very arduous detour in your life trying to sue your university. That won't make anything easier for a medical school application.

You should go to your student conduct office and talk to the dean about your situation and how you should address it in any future application or interview. The questions you ask are relevant for a conversation with that person and your prehealth advising office. I'm sure there may be some people who have had situations similar to yours who have been admitted to medical school because they were very forthright with the situation.


Should I apply now, take a gap year, really consider DO?

I am having a lot of trouble finding guidance towards deciding on my application path. PLEASE HELP!

Here are my credentials:

Binghamton University: Integrative Neuro Major, Spanish Minor, cGPA: 3.48 sGPA: 3.13 800 research hours as Research Assistant in several labs 70 shadowing hours Studied Abroad in Spain 6 weeks VP of Kickline Dance team 2 jobs simultaneously (psychoeducational tutoring and customer service job) About to be EMT certified and plan to spend summer, senior year doing patient care hours Spending summer studying for MCAT and applying in August after MCAT if I apply this year Volunteering in ER this summer

I am unsure if I should apply late August 2017, see what happens and, or take a gap year. I also had always intended to do MD but am I begin pompous by not considering DO more seriously? I have higher chances of getting into a DO program and would not have to wait to attend medical school then, but would I regret not being patient now and becoming a better MD applicant to pursue that?

casedentalmed

I hope you have had a chance to talk with your prehealth advising office (https://www.binghamton.edu/pre-health/) because they would be your best resource to help you with your decision regarding choosing between MD or DO. Once you let them know of the schools you are seriously considering (both MD and DO), they may better help you with the decision to take a gap year, to apply in August, and so on. As for your chances being better for admission into a DO program versus a likely longer wait to be admitted to an MD program, their expertise can help you really understand what things DO schools value compared to MD schools. In the end you also will need to communicate with the admissions offices of the schools you are strongly interested in.


Applying with an unfinished low GPA

Hi, I'm a junior with a summer and a year left of undergrad school and have a low (2.6 ) GPA. The average GPA at KU medical school is 3.7 and I plan to work my way up to a 3.0 which I calculated as possible but hard. Is it possible to applying before raising my GPA? I know I can do well in the MCAT as I am more of a study and retain than a study and throw away student. I have already started showin improvements as my courses were all upper level science courses such as immunology and cell biology

casedentalmed

You should have some meetings with your prehealth advisors and your faculty in your major. It's always possible to apply before raising your GPA, but you need a realistic view of the schools that would likely consider you. If your heart is set to go to KU, have you talked with anyone in the admissions office there? How about current students? You may be more than just your low GPA, but there are hundreds if not thousands of applicants who have shown they have a more solid grasp of the foundational knowledge needed to do well in medical school. You should probably be looking very closely into postbac programs and consider masters-granting programs if you are really dedicated to becoming a physician.


Will weak spots in application hold me back?

Im worried my GPA's might prevent me from getting an MD acceptance. My cGPA is 3.58 and sGPA is 3.38. I think my EC's are about average with 200 hours of shadowing, clinical and non clinical volunteering and over 500 hours research experience. Also hold greek organization leadership positions and multiple semesters of TA experience. I have a 507 MCAT score and am URM that is fluent in spanish. What would you say about my chances this cycle? Thanks in advance

casedentalmed

It's really challenging to give anyone "odds" because every medical school looks for different things about an applicant's background, experiences, and preparation for a career in medicine. I would hope you would have a strong chance at a spot in any generic medical program (MD or DO). Good luck!


MCAT date

Hey!

So I applied to take my MCAT May 18 and its a week before and I still am not scoring where I would like to be. My AAMC practice score was 500 but I would like a 510. My GPA is 3.96 and I hope to apply soon for admissions to the 2018 class. If I push off my test until June 29, this will delay my application process, but I am sure I will score significantly better with extra time to prep correctly. Do you think its terribly late to wait for my scores on Aug 1st to apply? Or should I apply ASAP even without my scores?

Any thoughts??

casedentalmed

If this is your first MCAT and you are confident that you will achieve your scoring goal, go ahead and send in an application and let the scores come later. A lot of schools will have begun selecting people to interview by late July so I would not wait until August to apply unless there is a real reason. Now, if you really are not confident you'll get the 510, you're going to have to make some hard decisions on whether you should focus so much on applying soon versus getting a good score. Kicking things down the road now is a sign you're likely to do the same when you're in medical school, and that usually sets you up for problems. You'd rather be the person who takes time and does things right the first time than someone who has to keep trying and failing. (The analogy I want you to think about is being able to insert an IV: you want to be the one who has practiced enough to be able to do it consistently as few times as possible rather than the person who has to keep fishing and fishing.)


A Second Thought

I just finished my first semester at Utah State University and currently am attending a summer session. I am taking Human Anatomy class and right now I a feeling it. What I mean is I'm seeing how stressful a science class like this can be and I have a feeling this is how a typical class might make you feel in Medical School your first year. I am in a discouragement kind of phase right now. I would never have thought I'd fall into this kind of feeling. Truly I always wanted to become a surgeon due to my experience as a patient, helping other patients make their quality of living better, and throughout high school it os all I ever wanted to be. There was no "Plan B" in my life. Still, I do not know what else to do with my life if this doesn't workout and not go to Medical School. It has been a good 7 years since I started college, I'm still in the habit of procrastinating on my studies and daily readings even though I say I am going to do it, there's something holding me back. I need help and what ways can I get the motivation in reaching my goal?

casedentalmed

Utah State has some great career advisors and prehealth counselors who can help you properly reflect and assess your lack of a Plan B. There's a lot to being a surgeon than what you describe, so I'm not sure how much shadowing you've done and research on the team that surrounds a surgeon (anesthetist, surgeon assistant, surgery nurse, etc.). Take the time to really find out; otherwise you may start but figure out the career is not as fulfilling as you hoped and lose motivation after committing to the debt of your education.


Choosing between University of Northern Colorado or Regis?

I have been accepted to both schools for Masters in Biomedical Sciences program. There is little to no information on how successful these programs are. The reason I am applying to only in-state colleges is for the cheaper tutions and to avoid drastic changes to my life right now. I feel that Regis being a private school will have a more supportive environment as compared to University of Northern Colorado. I know that a whole lot depends on individual effort to do extremely well on the program but it would also help knowing which school would be better in terms of helping me succeed in being a stronger applicant for med school. What do you think?

tantacles

My suggestion, with regards to these types of programs in any case, is to prioritize going to the program that will grant you an automatic interview to medical school. Barring that, it's probably most important to go to a place that is inexpensive. However, if you think you will succeed more at one program than another, then it may be worth the money to you to go to a different program. This is not a decision we can make for you.


I want to be a psychiatrist, but I'm not sure if I can get through med school?

I have fainted in the past at doctor appointments when getting shots and blood taken. I can't stand vomit, I can't smell it, hear someone throwing up, or see it or I nearly get sick every time. I I really want to be a psychiatrist but from my understanding I don't think I can get through medical school? Will I have to do these things to patients or dummy's? Will I have to during my residency?

Thank you, your answers are appreciated!

tantacles

You will have to be able to tolerate bad smells during medical school and during your intern year, when you will be forced to do rotations in specialties besides psychiatry. I would suggest getting some clinical experience volunteering or working in a hospital which may help you overcome fainting when seeing bodily fluids.


Scared about my future medical journey

Hi guys,

So I am first year college. I failed my first college course (US History) when I was 16 years old, doing a dual credit program while still a Junior and Senior in High school.

I retook that course and made a B. After graduation from high school with almost a year of college done, I failed my second college class (Introductory Biology2), in the spring.

Introductory biology is a medical school requirement and I am freaking out about not reaching my dream of becoming an MD. I'm gonna retake this class and try my best at it. I have a 2 C's (one of then in the first introductory bio) and the rest of my grades are A's and B's. My university's medical school is not that selective but competitive, I am really scared about my past mistakes. HELP me please do I still have a chance?!?

tantacles

The most important part of your application will be your cumulative GPA. A few bad grades will not hurt you. and it sounds like you have made improvements. You also need to do well on the MCAT.

I would suggest you invest in the MSAR to find out which schools you would be competitive at. I also suggest you post specifics in the What Are My Chances forum on SDN once you have a better idea of your stats.

casedentalmed

To add to advice given: I don't think that the dual-credit class is going to be an issue, but the fact you hadn't done so well in introductory biology should give you a reason to take a step back and figure out what the issues were with those grades. Were you overwhelmed with the lectures? What type of help did you get in preparing for exams or labs?

I also want to forewarn you that the AMCAS and AACOMAS applications will calculate GPA's that will include all repeated coursework attempts, so those C's will count on your application though your registrar/transcript for the school may not. What each individual school does when viewing recent repeated coursework is its discretion.


Should I volunteer at a hospital or travel Europe?

Hello everyone. I just finished my freshman year at college, and now I'm trying to figure out my summer. I have a volunteer job I applied and received, but I can back out if I want. I am taking dance classes and two instrument lessons as well. I also plan on getting a job to pay for my classes/lessons. My problem, however, is that my friend just offered to go on a month long european trip in July, and I'm not sure I could keep the volunteer job if I want to do that. The reason I'm considering it is because I've lately had this urge to go out into the world and live. I hate that my life is dictated by academics and always preparing for my career. I really want to be a doctor, but I want to live while I'm young too. Any advice? If it changes anything, I'm about to do a medical mission trip for a week, so if I don't volunteer this summer, I'll at least have that, and I plan on shadowing too.

tantacles

If you feel that going to Europe is important to you, I would consider going. You can find another volunteer job, but if this particular volunteer job is important to you, I would stay and perform the job.

In addition, please note that your medical mission trip will not be helpful in terms of the medical school application process; these one-week trips may as well not even be reported on your application as medical schools tend to look for longitudinal experiences. If the medical mission trip in any way strains your finances, I would cancel it if you can still receive a refund.


UCLA transfer or stay one more year?

I am a current transfer student who has been accepted into pretty much all of the UCs. I am leaning toward UCLA. I have a 4.0 GPA. The question I have, is do I stay at community college for another year to finish up Organic Chemistry 1/2 and Physics 1/2? This will save money and I am sure that these courses will be easier at my school now than at UCLA. Furthermore it will save money. So, my questions to ask, are should I transfer? I've been at community college for two years. Will it look bad if I stay, as it might look like GPA padding? I have a friend who is transferring who I do everything with, and it would be nice to transfer with him to take O Chem/Physics together at UCLA. Also, if I stay, the first semester would be O Chem 1/Physics 1 and the second semester would be O Chem 2/Physics 2. What should I do with the rest of my time? I am currently an EMT. Finally, I can take the MCAT immediately after my time at community college, as I would be finished up with all of my major prep. Also, the upper division courses at UCLA are much easier and I believe it'd be much smoother sailing after physics and O chem. Thanks!

casedentalmed

I would check with prehealth advisors and transfer student advisors at the UC schools you are interested in regarding their experience. The difficulty answering you is that I'm not certain how well you would be prepared for the MCAT after taking just prerequisites at a community college. You also need to be sure you plan on taking some biochemistry and psychology/sociology coursework, and your answer doesn't give any insight into your academic plan to take those courses (if you have already taken them).

Furthermore, your goal shouldn't be able just taking the MCAT but being truly prepared for medicine. Upper-level biomedical coursework is always going to be encouraged for you, and those would be easier to take at a UC since I don't know what is available for you at a community college.

I'll admit taking prerequisites at large universities is challenging because of the size of the lecture halls and accessibility to professors and TA's if you need help. Cost is certainly a factor, and I wouldn't discourage anyone from choosing a cheaper option. I would presume for you, your preferred medical schools will be on the west coast, so the schools are used to reviewing and admitting students with community college coursework. It seems like you have digested some of these considerations in leaning towards your transfer decision.


Downard grade trend in senior year?

For my undergrad gpa (pharmaceutical sciences): Freshman: ~3.84 sophmore: 3.9 (2 jobs) junior: 4.0 ( 2 jobs) senior: ~3.7 (3 jobs + 1 TA position)

90% of the courses taken in all my undergrad years are science courses, as i cleared GE's in high school AP's

my research in junior was based in neurobiology and now i'm getting a B- for more neurology class in senior upper div I've only gotten 2 b's before my life in undergrad

i don't know how this will factor into my admissions, how hurtful it would be? i went from working two jobs since sophmore and junior year to 3 jobs and a TA position in my senior year (this year) would this help soften the blow on my downward trend in GPA?

I need an outsiders opinion, i'm too burnt out to make objective analysis of the situation myself

thank you!

tantacles

A 3.7 GPA in your senior year does not constitute a downward trend. I would be more concerned if your GPA dropped below a 3.5. Going from an A to an A- average is not a big deal.


MCAT Expiration and Retake

Hello, I took the old MCAT in January of 2015 (scored well) with the intention of applying to medical school in the 2016 cycle. Ultimately I decided to take two gap years once I finished undergrad and am now preparing for the 2018 cycle. Seeing the number of schools that will no longer accept the old MCAT, I am in a bit of a dilemma. Since my old score is technically still valid, my undergrad is willing to write me a committee letter using it and I should be able to submit my primary very early in the cycle. However, there are many schools I wanted to apply for that will only take the new style MCAT. I am wondering if I should indicate on my application that I plan to take the new MCAT later this summer (August) and if these schools will still consider me once I send them an updated score instead of outright rejecting me. Will not submitting my new score until September then make me a late applicant? The reason I have not taken the new MCAT yet is because I spent my gap year building my extracurriculars (researching, volunteering, and shadowing), which I felt was a weak point in my application.

Thanks for taking the time to look, I really appreciate any advice here.

tantacles

If you are very interested in going to a school that requires the new MCAT, you should take the new MCAT. You can inform schools of your new score after it comes back, and there is no need to tell them beforehand.


Organic Chemistry, Retake, & Medical School

Hi guys, so I am currently in my second semester of my freshman year of college. Currently, I am in organic chemistry 1 lecture, however it is not going as well as I expected. As of right now, I have a C in the class. I took the final yesterday, and I did not feel too confident about it. I am worried that I could possibly drop down to a D+, which I know would look significantly worse on my transcript when applying to medical school. I had already planned on retaking the course anyways, and I am already enrolled for organic 1 over the summer. My question is, how would this hurt my chances of being accepted into medical school if I do drop to a D+ in the class? I am determined to do much better this summer when I retake the course, but will this bad grade destroy my chances of acceptance? I am really worried about how this will look on applications :(

Also, I received a 4.0 GPA my first semester, and until this class I have had a 4.0 GPA in science courses. Additionally, I have all A's and one B in my other courses this semester.

Thank you so much for any help you give!

tantacles

It's very likely that with a D in organic chemistry, you would have to retake to be accepted to medical schools. Many do not accept grades of less than C in pre-requisites. The D will hurt your chances, but only insofar as it drops your GPA. Retaking the course and getting an A is your best option at this point.


Masters in biomedical science help!

After extensive research, I have decided to go for Masters in biomedical science to boost my application. I will be starting soon this year but haven't decided as to which college would be better in terms of helping me navigate. It's kind of late for me to take post-bacc classes as I already have my pre-med classes down. The problem is that my undergraduate GPA is not even close to being competitive, MD or DO. I know that the masters program wouldn't count towards my overall GPA during med school application but I can't do a post-bacc either. Am I making the right move? Doing master's in biomedical science doesn't come with a guarantee. What should I do? How do I repair my GPA or redeem myself for med school application?

tantacles

If your pre-med classes did not go well, a post-baccalaureate degree still may be a good option - it will boost your undergraduate GPA, and if you do very well, the upward trend can look great for medical schools. Repairing GPA is always the best option if you want to enter medical school, and a post-bacc would absolutely allow you to do that. Keep in mind that you also need to have a fantastic performance on the MCAT.

Barring the option of repairing your GPA, no SMP program is a guarantee. I would suggest one where you receive an automatic interview as those programs are more likely to accept you, particularly if you get all A's.


What are my chances with 11 withdrawals and 1 incomplete?

the first 2 years of college I had 6 withdrawals due to depression. 3rd year I had a retroactive withdrawal due to depression and major family issues. Because of that, I decided to take a semester off, so I can regroup myself. After coming back to college I had an upward trend, with all A's, now my GPA is 3.6. I took the MCAT and I scored 524. What are my chances to get accepted into any medical schools? Preferably my top choice is NYU.

Thanks!!

tantacles

Your chances seem good provided you can express how you learned from your situation and improved. Your upward trend is impressive. Please calculate your full AMCAS GPA and purchase the MSAR to gauge your chances and apply broadly. You can also post a potential school list on the SDN forums for continued feedback from admissions professionals so they can trim down your list. Your situation is unique and probably deserves more attention.


RN to DO?

Hey there. I'm currently a registered nurse with the goal of becoming a physician. My GPA is on the lower end (3.0), which is why I'm attempting the DO route as opposed to MD. If I wish to enhance my GPA, should I just take non degree classes at the city university ? I work at a top city hospital, which will pay for the tuition to an extent. I wasn't sure if there is a program that I can apply for. I don't have the basic science courses Required, so I as unsure which program I would even qualify for.

Any input or advice would be great. Thank you!

tantacles

You may absolutely take classes at the city university to boost your GPA. Many universities have formal post-baccalaureate programs, and these usually have the courses you would need to be accepted into medical school.

Please keep in mind that if you do well on the MCAT and and in your post-baccalaureate program, you have a chance at MD schools. Once you have completed your pre-requisites, I would recommend investing in the MSAR, an online resource that can give you an idea of where you would be competitive. This will allow you to gauge your chances and apply appropriately.


URGENT what are my chances?

I want to apply to med school this summer, but unfortunately I had many problems during my undergrad years (had t o move multiple times, had to work full time) that really hurt my gpa. So what are my chances?

3.1 cum GPA and 3.1 science Gpa.

I have three years of clinical experience total, 1 year medical assistant, 1 year hospital clerk, 1 year clinical technician. Hispanic late 20's Senior at Rutgers Im planning to take the mcat within the next couple of months and I'm hoping for an above average score. (I scored high in my premed classes) I'm trying really hard and I'm getting a few excellent recommendation letters. Do I have a chance to at least 1 allopathic med school? I can not take another year off due to multiple reasons. Thank you in advance!!!!!

tantacles

You have a chance if you absolutely kill the MCAT. However, I would not suggest applying for medical school right now. Your GPA needs repair. You may not be able to take another year off, but if you don't get in when you apply, you will be forced to take another year off, so make sure you prepare and make the best application before you apply.


Do I need to take science courses if not taking during college to go to medical school?

Hello,,

Degree: Bachelor Major: Economics College: Art and Sciences GPA: 3.8 Graduation year: 2016 - The issue: The only science courses that has been taken: Mathematics / nutrition / public health and earth science - Question: Do I need to take the science courses to apply for medical school? Or MCAT exam is enough?

Thank you for taking the time to help put.

Please feel free to share your wisdom about med-school as much as your time allows.

Best wishes

tantacles

Most medical schools require a full year of general chemistry, organic chemistry, biology, and physics for admission. You will likely need to do a post-baccalaureate degree if you are near the end of your university education in order to get the pre-requisite courses in.


college gpa?

Does the gpa for classes taken at a different College while undergrad, count toward the overall gpa of med school application? What about the gpa for dual enrollment classes?

tantacles

The GPA of all undergraduate courses, regardless of where they were taken and the level of the course, counts towards your AMCAS GPA.


low gpa

CURRENT SITUATION: Im a community college student transferring to a four year college this fall. my cGPA according to AAMCAS is 2.3 with 81 credits. i will be able to bring it up to about 3.0 by the time I graduate. i'm confused about what to major in the four year college. thinking about majoring in bioengineering or biology at the 4 year college and do a post bacc or SMP. another option is doing associates/bachelors in radiology technology and repair my gpa while doing radiology tech job. MY BACKGROUND:

Came to the U.S 7 years ago with family, started high school as junior in middle of the academic year. went to community college because thought that was the best option for me at the time(regrets). Didn't do well in the first college, dismissed from there for low gpa(never appealed dismissal because i hated that school). but now have above 3.0 at the new college.  

MY QUESTION: what could be the best option for me right now? I'm 24 and without a degree and no debt.

tantacles

If you are able to get all A's in the rest of your undergraduate courses, it would look fantastic for your medical school application. Medical schools often award reinvention, particularly if you are able to show an upward trend. Ideally, you should try to have above a 3.8 from this point on to show that you've truly improved to the point where you will not be an academic liability in medical school.


DO rec letters to MD schools?

I am applying this cycle (to begin med school August 2018) and am planning to apply to MD and DO schools to give myself more chances of getting in and keeping my options open. I know most DO schools require a letter of recommendation from a DO. I was wondering if I should include this DO letter in my application to MD schools as well? And visa versa.

tantacles

If you have a strong letter from a DO that knows you and knows your work, absolutely use it. However, if it is just a shadowing letter, it will not be helpful to you. Stick with letters from people who have seen your work (i.e. professors who graded you) or have worked with you in a professional setting for an extended period.


Post- Bac versus 5th year Pre-med

HI all,

I am currently a senior with a low cumulative GPA (3.3-3.4). I've always been interested in health care and medicine, so I cast a pretty wide net in my experiences. As a result, I have a lower GPA, but I am a double major in psychology and human physiology and I have quite a bit of extracurricular, anatomy & physiology, and clinic experience. I decided later on in my studies that I wanted to be a pre-med and apply to medical school, so I also haven't taken all of my required courses. I still am missing my general chemistry and organic chemistry labs, and a calculus course. I am scheduled to graduate in June.

I'm curious what the recommended way would be to complete these courses? My goal is to raise my GPA while finishing these, so that my application will be stronger. I don't know if a post-bacc or a 5th year in my undergrad would be more appropriate. Will post-bacc count for my undergrad GPA? Or will it be considered the same way as an undergrad GPA? I am interested in SMP (special master's programs) or a master's in public health as well, but I have heard that both post-bacc and graduate level courses are not considered to be as strongly as undergraduate GPA.

I'm having difficulty weighing my options. What are your recommendations?

tantacles

A post bacc will count for your undergraduate GPA and is probably the way to go, especially if it's cheaper. If you get all A's in a post-bacc, you will be in great shape for medical schools. Keep in mind that you'll also have to do well on the MCAT, and post-bacc courses will likely help with that. Your post-bacc courses will be given the same weight as your undergraduate courses, but if you were to do a master's degree your courses would not be factored into your GPA.


Low BCPM, not sure what to do next

So I am a sophomore and I am almost done with my prereqs for med school (only need physics and biochem) and so far I have a lot of C's on my transcript followed by several B's and A's. My BCPM and cumulative gpa are on the low side and I havent taken the MCAT yet. For someone in my position what would should I do so that I still have a chance to get accepted to an allopathic med school?

tantacles

If you have many c's tarnishing your application, my suggestion is that you complete a post baccalaureate program to show that you can handle the rigor of medical school.


What should I improve on?

Good evening,

I am currently a junior at the University of South Florida as a Biomedical sciences major with a minor in public health. I plan to take the MCAT in the summer of 2018 and apply that cycle. I wanted to ask for some advice on what I could improve on while I still have time before I graduate in the fall of 2019. My goal is to be accepted to MD, but I am open to DO as well. I would love and greatly appreciate any advice or recommendations that you all have to offer.

GPA: 3.6; sGPA: 3.42 (both upward trend) Lowest grade ever received so far is B- Pre-med course grades are primarily B+s, with various Bs (No As yet. Biochem is only pre-req remaining. Had to Withdraw from gen chem 2 and lab my sophomore year. Retook and earned a B+ in lecture and A+ in lab) (Usually take 10-12 credits of science with 17-18 credits a semester if it matters) Honors college- will complete thesis

Volunteering: 128 hours at Moffitt Cancer Center (Recently quit, should I return?) ~25 hours various other small volunteering activities Planning for 200-300 hours as a Camp Boggy Creek Counselor over the course of the next year

Shadowing: 15 hours Internal Medicine 20 hours Immunology 5 hours Cardiovascular 140 hours Shadowing internship consisting of Pediatrics/Surgery/Oncology/GYN led by USF Morsani doctor.

Leadership: Social Chair of Camp Boggy Creek USF 2017-2018

Research: Microbiology based- 2 years by the time I apply. Will complete thesis.

Work: Ulta employee 2014-2015

Thank you!

Pathdocmd

You look like a OK candidate so far. I would try to get even more of an upward trend in your sGPA, do the best you possibly do on the MCAT (take it when you are ready, not to "practice"), and get some volunteer experience with patient contact. You have enough shadowing in my opinion (I can't speak for any other school). The key to any volunteer, patient, or research experience is experience is communicating the impact it had on you and how it will impact your future classmates and patients. Good luck!


MCAT expiration and retaking the MCAT

I took the MCAT in the summer of 2014 with full intentions of applying in the 2015 application cycle, but life happened and here I am gearing up for the 2018 cycle. I have some general questions regarding retaking the MCAT and how it will look to medical schools considering that my old scores have expired, so any information on the topic would be greatly appreciated!

A little about myself: I took the MCAT twice in 2014, and ended up with a 5 point increase from my first to my second score - so my testing went well as far as retaking can go. Unfortunately there were some family circumstances that came up, and I've had to put off applying until this year for the 2018 application cycle. I’ve checked the MSAR and the AAMC MCAT School Preference pdf, and considering a large majority of all medical schools will only accept the 2015 MCAT, including all of the schools I’m applying to, I decided to retake the exam. I retook the MCAT in March and I’m currently waiting to hear back about my score. My questions are as follows:

- Considering that it’s almost considered taboo to take the MCAT 3 times, will this impact how medical schools look at my application considering that two of my scores are expired?

- Will schools still be able to see my expired scores? I haven’t really been able to find any information on this one, but I’m assuming the answer is yes.

- If my MCAT ends up not going as well as I hope it did (god forbid), would it be feasible for me to retake it considering my two earlier MCAT scores are expired?

I’m not sure if anyone will be able to answer these questions, but thank you for any input in advance!

tantacles

Taking the mcat because your mcat expired overrides the "taboo" of not taking the mcat 3 times. Schools may be able to see your expired scores, but your most recent scores will count most.

If this administration goes badly, you will have no choice but to retake. But be judicious and do not retake if your score is just ok and will get you into school.


Rutgers vs. TCNJ vs. UPitt

I'm going into pre-med as a biology major and I am currently choosing between the NYU Presidential Honors Scholars Program, the Rutgers Honors College, the TCNJ Honors Program, and the UPitt Honors College. Which college do you suggest would be the best in terms of medical school matriculation rates, research opportunities, academic rigor, competition and community, and the best overall for a biology major?

tantacles

All of those colleges are great. The most important thing is that you choose an undergraduate school you will do well at and that you will be happy at. I would also think about which school costs less and which will be least expensive. These can all play into your decision process in different ways, but ultimately, your priorities need to determine how to field this decision.


How much difficulty will a spurious psych history cause to future licensing?

I am interested in changing careers from public health research to a clinical field. I would like to apply to a MD program, but I am worried that I may be unable to get a license. This is an unusual situation, so I am hoping that these problems can be overcome.

I had a B-12 deficiency, Postural Orthostatic Tachycardia Syndrome and abnormal neurological reactions to anticholinergic medications. These conditions are all treated or well controlled now. I have what would appear to be a history of psychiatric illness due to these problems, though. My understanding is that I must report this. Will a medical problem that could cause potential anxiety, emotional lability or memory loss make me unlicensable? What sort of proof is required of my mental stability?

tantacles

As long as your conditions don't impair your ability to practice medicine, your conditions will not make you unlicensable. Physicians do not have to prove mental stability.


Did poorly in my first semester biology course...Can I not apply to medical school anymore?

In my first semester of my freshman year, I had the typical story: first time away from home, too much drinking and partying and not enough studying. This caused my grades to plummet and I ended up receiving a D+ in my introduction to biology 1 course (i know, it hurts to look at my transcript.) After that, I really got my act together and went on to take many more upper level biology courses including developmental biology, molecular genetics, biochemistry, neurobiology, cell biology, just to name a few. I received a B in some and an A in the rest (I received an A in biochemistry, which is arguably the hardest class at my college.) The pre med requirements need a year of biology and now that my medical school application is coming up, I am worried that my D+ will disqualify me for my 1 year of biology requirement. Would medical schools look at my upper level courses and count those toward my 1 year requirement? Or is this unheard of? Please help me if you have any info. I really don't want to hear that I have to retake intro to biology 1 in my future. Thank you.

tantacles

If you are able to improve, you still have a great chance of being accepted to medical school. Just note that you may not be able to use this D+ for one of your pre-requisite courses as most schools require a C. Do your best going forward and try to get all A's.


Undergraduate Major

Hello firstly I know I saw a similar post on this question, but whenever I try to click on it I get a 401 error, so I decided to re ask. I am wondering simply what is the most optimal major for a pre med track? I personally have always loved the sciences being top of the class through bio, chem, physics, and math, and never found any difficulties in one area, so now I do not know what major to choose. I am leaning towards biochemistry seeing that it may be the most helpful for the MCATS, and as I learned many med schools even require courses to be taken in biochemistry. Though I know there are other options such as chemistry, biology, neuroscience, biomedical engineering, etc. Based on preference they all equally call to me, and choosing one seems impossible. That is why I am wondering which is the most optimal to choose, and if I should spend more time taking classes for those majors or in internships and research? Thank you!

tantacles

The best major for a medical school applicant is the one you can do best in, whether it is English, history, biology, Scientology, theatre, music, chemistry, or anything else you can think of. As long as you complete all of your pre-requisites and do well in your courses, medical schools do not care about your major.


ACTUAL waiting time for AAMC to verify application?

Background: today is March 21st -Taking the MCAT May 19th -Graduating undergrad in August (Yes I'm ready for MCAT) -Working on my aamc application to enter early decision program by August 1st

Questions: Can I submit my aamc application for review before I take my MCAT/get my MCAT score? if not, How long is everyone's average wait time for application review? I know the aamc website says about 6 weeks. But I am wondering if it's any sooner than that because I want to be able to apply by Aug 1st

tantacles

Yes, you may submit your AAMC application before you get your MCAT score. I would submit to one school in case your MCAT goes badly so you are only a reapplicant at one place if you need to wait and reapply. If you choose to simply wait, the later you submit, the longer your lead time.


Advice on Waitlist

So I have been placed on the waitlist at my top choice school. This school is one with a reputation for really liking interest letters/updates/LOI. I am wondering, would it be a good idea to have my PI who already wrote me a letter of rec send the admissions dean an email? I have been working in her lab full time since the beginning of the application cycle and feel that she knows me much better now than when she wrote the letter. It would be kind of an updated LOR. It would be a very positive email and I feel it could give me an edge if this sort of thing is acceptable. What is the general consensus on this? Too much? Or does it speak highly of me that she would be wanting to write another letter?

tantacles

If your PI is in strong support of you entering this medical school, it is unlikely to hurt your application. It is also unlikely that your PI's letter will provide substantive help.

What I would do is give a phone call to the admissions office and ask if extra letters of recommendation are helpful to waitlisted applicants and if they will even accept that information from you. If they say they will, then and only then have your PI write the letter.


Which STAT class for Med school?

Which STAT class is recommended for Med school, 200 (elementary statistics) or 250 (biostatistic)?

tantacles

All statistics classes are appropriate for medical school. I would take the statistics class that you feel you are most likely to do well in.


Pre-med currently in Army ROTC

I am currently a Bio major in Army ROTC with a year to go before I graduate. I'm taking 5 years as opposed to 4 since I enlisted in the national guard during my freshman year of college. My undergrad GPA is currently a 3.41 and I take my MCAT this summer in May, in addition to going to camp this summer for rotc ( I am contracted and currrently in the process of picking my branch for graduation ). I would like to go to med school but I already know I will not be accepted due to not having time to conduct extracurriculars such as clinical experience. I decided later on during my undergrad years that I wanted to study medicine and focused on the military during my early years when I had more time to do things. Upon commissioning what I can I do to strengthen my application during my gap years, I will have a service obligation of 4-6 years depending on what job I get. I am predicting that I can graduate with a 3.5 GPA next year and I am planning on branching medical services corp ( would like to be an aeromedical evacuations pilot 67J during this time). While on active duty I am also planning on pursuing a MPH in biostatistics and epidemiology; and or a masters in biochem while searching for clinical/research experience towards the end of my service obligation if time permits. Is this a good plan that will get me into medical school once I am done with my 4-6 years? After med school I would like to be a flight sergeon and plan on taking the HPSP scholarship if accepted.

tantacles

Your strategy is reasonable. My suggestion, rather than doing an MPH, is to do undergraduate courses to raise your GPA; Master's level courses do not count towards your AMCAS GPA. If you are enthusiastic about getting an MPH, then by all means pursue that, but it will not have a substantial effect on your AMCAS application.

Otherwise, your plan sounds great.


So I'm currently a junior in highschool, So I am an NRI, and I need so information from u guys. Basically these are the courses i took and am taking currently:

So I'm currently a junior in high school in America, So I am an NRI, and I need so information from u guys. Basically these are the courses I took and am taking currently: AP refers to college courses honors refers to advanced course

9th grade: English hns world history hns geometry honors (hns) biology hns debate latin 2

1oth grade: English 10 hns AP world history 2 Algebra 2 honors Chemistry hns AP computer science Latin 3

11th; AP lang (English) AP us history Pre-calc hns AP physics 1 AP Psychology Human Anatomy Latin 4

12th I'm going to take: AP Literature AP comparative government AP calc AP biology AP macro-micro economics Medical assistant 1 Medical assistant 2

So I want to get into good colleges in Hyderabad. I have straight A's and B's for academic. And my SAT score was good as well. I also did some after school like: Model United nations and debate tournament team, got selected in science fair placed 1st 2nd and 3rd, in Virginia. How are my chances for getting into these colleges in Hyderabad? I can talk telugu, but I can't read. will I have problems? I want to go into institute of medical science. I am going to stay in hostels, so are they clean and do they AC, will I have any problems? Is there ragging? Will I be able to understand professor's accent and lecture? Will I be able to pick up? how many years does it take to finish college?

casedentalmed

I am interested in why you want to go from the US to India for college. The entry process for getting into college in India is so vastly different. Your list of accomplishments and AP coursework aligns yourself better to going to a public university within the US. If your intention is to become a physician, and you desire to practice in the United States, I would encourage you to stay in the US for college. It is much more difficult if you wind up getting your medical education outside the US/Canada and try to re-enter the US to practice.

I understand the challenges for financing an education in the US, and being a non-resident you're not eligible for federal aid. I don't know if you have any intention of getting a green card. If you do, a community college route may be your next reasonable step to transfer to a traditional undergraduate program. But if you have established residency in your state, you may qualify already for in-state tuition for universities.


Withdrawal from History

Hi! I currently have As or A- in all my classes as a psych major and my pre med requisites from last semester (except for a B in chem lab) and this semester and left with a 3.72 gpa. This semester I had family emergencies and I withdrew from Latin American History (a world history requirement for my college), as there were only two tests and I would have failed the first one. Will this withdrawl impact my opportunities if I decide to transfer to a better college? More importantly will this impact my chances of med school? Thanks!

tantacles

One withdrawal will not have a significant negative impact on your medical school applications.


Non-traditional student with low GPA

I'm a 31 yr old, non-traditional student, getting my BS in public health education. I really want to pursue my dream of being a doctor. However, my GPA is low due to old coursework from early 20s. I took a 7 year hiatus from school and when I returned made straight As. My cGPA is 3.11 and my sGPA will most likely be around a 3.4 to 3.45. If I get a decent MCAT score, do you think I have a chance of getting into med school or will I be wasting my time. I have about 8 prereqs left to take and 6 classes until I receive BS degree. I am doing volunteer work, shadowing physicians and will have a public health internship. Any advice would be greatly appreciated. I wish I could go back in time and tell my 21 year old self to study and go to class. I am afraid my past mistakes are going to hinder me getting accepted to med school.

tantacles

Most medical schools understand that in the early years of college, immaturity leads to some difficulty studying. If you truly went back to school 7 years later, your reinvention will play into your application positively; some medical schools may obsess over your total GPA, but many will also look at the full story, particularly the fact that you've managed straight A's, and look upon your application favorably. Continue to get straight A's, and do the best that you can on the MCAT. When you apply, apply broadly, and make sure you don't only apply to top tier schools.


How Competitive am I to Allopathic programs?

I am a senior at a small private school. My GPA as it stands now is a 3.98 cumulative with a 4.00 sGPA. This will be my second time applying and I got one interview (out of 12) the first time, but no acceptance yet. I feel as if my MCAT hurt me the first time at 505 (128/123/129/125), so I retook it and got a 508 (126/125/129/128). Other than this here are some miscellaneous credentials; 130 + hours of nonmedical volunteering (swim instructing, merit badge counselor, and humanitarian efforts) 940 hours paid clinical work 100 + hours of shadowing Served as head lifeguard of a camp for 4 summers. Worked as a retail associate for close to 500 hours Club president for Pre-med club, Vice-president for Sigma Zeta Honor society. ~200 hours of organic chemistry and biochemistry research ~500 hours of work in a molecular biology lab as a lab assistant

To be honest, I really would be happy just getting into an allopathic program. As a Missouri resident, I will apply to University Missouri-Columbia in addition to several private schools (Creighton, Wake Forest, New York Medical College, Geisinger Commonwealth, Pennsylvania State University) and I thought I might as well try for these but it would be a stretch (Warren Alpert (I lived in Rhode island) and Saint Louis University).

Any advice is appreciated!

tantacles

You will be very competitive for allopathic medical schools provided you apply broadly. I would suggest you purchase the MSAR and apply to 10-15 schools with MCAT medians around your MCAT score. Any other schools you apply to will be reaches.


How strong of an applicant am I for Medical School, What are my chances of acceptance, and where should I apply.

I am a URM and come from low socioeconomic background (parents supported a family of 5 on $30,000 annual salary), First Generation college student I graduated HS with 96 college credits ( earning an A.A in Pre-medical Science from Community College) 3.8 GPA I graduated from JHU B.S in Neuroscience. 3.02 GPA (upward trend, took straight upper level science courses since freshmen year)

Was accepted into JHU MHS program, and the Boston Special Masters Program chose Hopkins based on financial reasons

Currently getting an MHS in Biochemistry and Molecular Biology concentrating in Cancer & Reproductive Biology 3.8GPA - Have taken science heavy courses such as Immunology I & II, Molecular Toxicology, Cell Biology, Molecular Biology, Cancer Biology, Reproductive Physiology, Virology, Nutrition, Genetics & Gene Therapy, Genome Integrity, Nucleic Acid Chemistry . . . so on)

Overall: undergrad GPA = 3.27 Overall: undergrad Science GPA = 3.17

MCAT First Time: 501 MCAT Second Time: 520

I have Shadowed 4 doctors/or surgeons = around 260 hrs total (200 are from 1 doctor who I have also been working on a patient database that will be part of a publication soon)

Health Related Volunteering = 110 hrs total (mostly at JHU Children's Center as well as a Free Health clinic for the homeless)

Non Clinical Volunteering = around 202 hrs Community Service Fraternity - mostly focused on tutoring inner city kids ECHO Seed Farm - generates agricultural techniques to teach those in other countries to help them grow food more efficiently Several other Volunteer events spread out through my 4 years at JHU

Research Experience = 1800 hrs (worked 10hrs a week during school and full time during breaks) 3 different labs (1 lab for the past year) 1 Publication in review and contributing to 3 other projects that will be submitted for review in a couple months (fingers crossed)

I founded a Cultural Dance Group at Johns Hopkins

Have 3 Solid Recommendation letters (Science Professor [took a course with him every year at JHU], PI from lab I've been at for the past year, Doctor Shadowed (the one that I've shadowed over 200 hours and will publish a paper with)


Also I will be publishing my Masters Thesis which is a Scientific literature review on Immunotherapies and Colorectal Cancer

What are my chances what schools should I apply to.

tantacles

You are a strong applicant for medical school provided you apply to the right schools. My suggestion is that you purchase the MSAR, and apply to schools that have median MCATs below yours. Your MCAT will make up for your lower-end (although not too low for admission) GPA.

In addition, you can use the following tool made by an SDN member to help you choose schools:

https://forums.studentdoctor.net/threads/wedgedawgs-applicant-rating-system-updated-jan-2017.1131149/


6 year pharmacy program vs premed

I have to make a tough choice and I would really appreciate any help. I got accepted into Long Island Pre-pharm program which is 2 year of undergrad and then 4 years of pharm school. I got into the honors program and have been awarded the presidential scholarship with will cover full tuition for 4 years which means that I am going to have to only take loans for the last 2 years which mean that by the time I am going to graduate I would be in $80-100k in debt but I would make a 100k salary after just years. However, on the other hand as a doctor I would be able to make a bigger difference and more money, prestige, etc. but it is not guaranteed that I would make it to med school unlike the guaranteed option of the pharmacy. So if you were in my position what option would you take.

tantacles

This is an intensely personal decision. My suggestion is that you figure out what you want to do with your career, and that will guide your decision. If you are undecided, keep in mind that you can still go to pharmacy after completing your undergraduate degree, and you can still go to medical school after completing pharmacy school. This decision is a big one, but it's not one that anyone else can answer for you.


Time and prerequisites

Hello,

I have a Bachelor of Science in nursing; therefore, I lack most of the hard science prerequisite course work needed for the MCAT and most medical school admission requirements.

I need to take: BIO 1 & 2 w/labs CHM 1 & 2 w/labs PHY 1 & 2 w/labs Ochm 1 & 2 w/labs Biochemistry

I do work full-time as a RN and have a family. My current plan is to take these classes over a 3 year period starting Fall 2017 and ending Spring 2020. I plan on taking biochemistry in Spring 2020 and taking the MCAT so I will be ready to apply by June. Overall, will me taking a slower approach in completing these courses be looked at poorly by adcom's? I don't like the idea of stacking 3+ science courses with labs in one semester. Plus, I figure the extended time will allow me to add up more volunteer and EC hours. What do you think?

Thanks.

tantacles

This plan is just fine. Just keep your GPA as high as possible, and only apply when you're truly ready; if your MCAT score isn't great, don't rush it.


Biomedical Sciences or Chemistry Major

  • PLEASE HELP*

Would it be better to major in Biomedical Sciences at Texas A&M University (College Station) or Chemistry BS at UT Austin for medical school? I'm worried about the biomedical science major because the college it is in is literally called College Veterinary Medicine & Biomedical Science, with a huge emphasis on Veterinary Medicine. I checked out the clubs they have and most of them are for vet medicine...I have nothing against veterinary medicine, but I'm just afriad that I wouldn't get sufficient help and support for medical school going there. In contrast, I love the biomedical sciences major, and I feel like it's going to be easier in terms of getting a higher GPA.

I also like the chemistry major at UT. I absolutely love chemistry so much because it is so interesting to me, but I heard that college chem is super hard. This really worries me because a high GPA is really important for medical school. ( And also UT has a higher college ranking...But I'm not sure if that actually matters)

So in terms of the whole package...When adding volunteering hours, shadowing, studying for MCAT, research, etc...It will definitely make both majors stressful too.

I really want to know y'all's opinion...because everyone that I've talked too keeps telling me to pick the one I like...Which makes it super hard on me cause I love both, yet both of them have there cons too.

  • Also I'm fine with either school setting, sometimes I'm in the mood for a city-like surrounding, and sometimes a college town.
    • And if y'all know anything about each school's research program, please tell me because I think that's significant?

I know that medical schools don't care about what majors people choose, but I would really appreciate an answer because I'm really stuck.

Thank you so much in advance:)

tantacles

As you know, medical schools do not care about your major. Choose the major you are most interested in and that you will do the best in. If that means choosing a major that you feel is easier, do that.

In terms of which school to pick, you need to do whatever will make you happy. The most important thing is getting a high GPA, and if you are unhappy with where you live, that may translate to lack of interest in your courses, though some people can simply power through.


Should I wait to apply until the next cycle?

It's 2/18/17 and I'm deciding whether to apply to some DO schools or wait until the next cycle.

Here are the details:

Application stats: 514 MCAT, total GPA 3.3, BCPM GPA 3.2, over 800 volunteer hours, over 1000 hours working as a medical technician with direct interaction with patients, award for undergraduate research, volunteer of the year award, socioeconomically disadvantaged status, 28 yo white male.

An SMP would not be financially feasible. I could not get a cosigner due to my family's financial status and my estrangement after coming out as gay. I also don't know how I would "improve" my application other than racking up more volunteer and clinical hours.

I'm applying to DO schools because I want to go SOMEWHERE. My application is verified to NOVA and PCOM but I wanted to ask for guidance before I submit the secondaries. Would it be unwise to send PCOM and NOVA secondaries at this point or should I submit ASAP? If I didn't submit, wouldn't I be considered a re-applicant in the next cycle? Wouldn't it hurt my chances to NOVA and PCOM to not submit the secondary since they already "know" that I applied this cycle?

And should I risk applying to extra DO schools at this point such as LECOM? I don't think I can "improve" my application between now and the summer, so I'm deciding between applying now or submitting everything to additional DO schools at the very beginning of the next cycle.

I submitted secondaries to MD schools in September/October, I interviewed at USF in late January and got waitlisted. I was rejected from two MD schools but have not heard back from the rest.

I know I'm far from an ideal situation, so your guidance is supremely appreciated.

tantacles

If you have already applied this year to medical schools, applying again next year would likely be a poor choice - medical schools want to see significant improvement in your application, and it is very hard to do that in the year that you applied to medical school. Take this next year to do more volunteering and get a job so that you have money for your application, and reapply. In addition, applying to DO schools right now might not be a good idea - the schools are likely wrapping up interviews, and your chances would be poor.

As you probably know, your GPA is probably what is keeping you out of medical school. If there is any way to take some more undergraduate classes to raise it, that would be the best thing to improve your application.


What clinical experiences have shaped your interest in medicine?

Hi friends! I'm new here, and I was hoping to get some feedback on my volunteer experiences. During my undergrad, I was heavily involved with Habitat for Humanity and Lion's Club. Right now, I'm in a gap year. I am working as a unpaid intern (aka volunteering) with a charity organization for my local hospital 5hrs/week. I help out with fundraisers that raise a lot of money for the hospital's causes, such as providing free mammograms to women in our community, transportation for people who need medical care, hospice care, etc..

Would you say this is non-clinical as well? I am indirectly involved in providing medical care to these individuals.

That being said, I would love to hear what clinical experience have reconfirmed your interest in medicine.

I am just getting my feet wet with the Remote Area Medical Clinic in my community, which provides medical/vision/dental services to the uninsured/underinsured people in my area. It should be an awesome turnout next month!!

tantacles

The unpaid intern position is a non-clinical position - Helping out with fundraisers and transportation for people in need is fantastic volunteer opportunity, but clinical experiences generally refers to interacting with patients while they are receiving health care and not just while they are on their way to receive it.

Personally, my clinical experiences did not confirm my interest in medicine but were useful in helping me see how important health care is to vulnerable populations. Volunteering with patients in an MS clinic was a useful experience for me.


The ultimate grey area MCAT retake or not to retake

I received a 511 overall on the MCAT (CARS 130:, Psych/Soc: 128, Bio/Biochem: 126, Phys/Chem: 127). I was very disappointed with this score and really wanted to break into above the 90th percentile range. Historically on practice tests I had performed much better on both Psych/Soc and Bio, and I am a biology major and consider that to be much more my strong suit than Chem, which I have been historically weaker in. At the time of taking the exam I had not yet taken Biochem. I’m very very torn on whether I should retake or not. I am scheduled for the April 22 date as this was the last time before graduation that I could take the tests and not have the scores delay my AMCAS application this cycle. Any advice would be so appreciated I have been going back and forth for months.

GPA: 3.80, BCPM GPA: 3.68

tantacles

You are not in a grey area. I would suggest you do not take this exam again. You received an admirable score, and with your GPA and MCAT, you stand a great chance of getting into an allopathic medical school provided all of your extracurriculars in order, you write a good personal statement, and you interview well. I would strongly suggest you cancel your test for April 22nd unless taking this exam causes you less anxiety than not taking it. You do not need to retake the MCAT.


Nurse to MD/DO

I have heard a variety of opinions and advice on whether or not being a BSN prepared RN will fair well with ADCOM's. By the time I apply, I will have just under 4 years of RN experience in the emergency department. Of course, I will have completed all of the necessary pre-med course work by then as well.

Does having the BSN,RN hurt me or help me?

Thanks

tantacles

Many RNs have become physicians. Your RN may hurt you, and it may help you. If you can cogently elaborate how your RN has shaped your perspective and explain your transition in a way that is favorable, does not speak negatively of either profession, and shows your insight into medicine through the nursing lens, it will be a great asset. If you are not able to do this, your nursing experience will likely not help you or hurt you. If your nursing experience has made you bitter, jaded, or hateful of physicians and that shines through, it will hurt you.

In other words, it depends on how you're able to frame your nursing experience in your application. it certainly counts as clinical experience, which is a positive.


Can I get into medical school or physician assistant school with a general studies degree form a horrible university ?

so I am attending a sub par university with a national ranking that has tanked substantially in the past 4 years (it has dropped from 150th in the nation to 184th in the nation according to us-news ) I had intended on majoring in cell and molecular biology but since they told me I had to take 16hrs of foreign language credits, 12 more hours of English credits, and god knows how many more humanities I changed my major to general studies. Sitting at a GPA of around 3.4. I have been chronically ill for almost a year and a half now (I did get a letter from my physicians*), my grades have taken a hit because of it (used to be a 3.9) I had to retake ochem and genetics because of how horrible my professors were (seriously tenure is a concept that has outlived its usefulness) but I did get A's in them both on the second try.

Pros:

I did work as an EMT-intermediate for a very brief period of time (like 6-months tops)

I did well in microbiology, anatomy, ochemII, biochemistry, immunology, statistics, psychology and did undergraduate research with my biochemistry professor. I also worked as a tutor and made an educational website/blog.

Should I consider getting a masters degree in biochemistry or molecular pathology from my university before I apply for PA school or Medical School ?

Any advice would be appreciated

thank you


      • EDIT: the letters are just explaining that I was chronically ill during this time and that it took a significant amount of time away from studies ***

tantacles

Your major is wholly unimportant to the medical school admissions process, and your school ranking, while holding some weight, also isn't very important. The absolute most important thing you can do is to get the best grades possible. I see that your GPA has declined due to your illness, but if you want the best possible chance of getting into medical school, you need to get as many A's as possible from here on out.

Just to repeat: You do not need to major in the sciences to get into medical school. Major in something you're interested in, and major in a field in which you will be able to get A's.

Your letter from your physicians will not help you in the medical school admissions process.


Code of Academic Integrity Violation

So I recently got caught cheating on my exam. After owning up to my mistake, my professor was kind enough to let me off with a written warning and a 0 on my exam. I am wondering if this has to be reported on my med school application even though it is not on my transcript?

tantacles

If your professor reported this instance to the University and the warning was official, you must report it. If it was not reported to the University officially, you do not need to report it. The only person who can answer this question is your professor. It does not matter if nothing appears on your transcript; what matters is if you were sanctioned by the University.


How is psychology research looked at?

How do admissions committee's view research experience in psychology? I am a psych major and the only research experience that I have is from working as a research assistant in a professor's lab. Does this type of research experience hold less weight or value to an admissions committee?

tantacles

Psychology research is definitely counted as research. The more involved you are in the work, the more it will contribute to your application, and if you are able to get a publication, that will be even better.


When I started college I wasn't sure I wanted to be pre-med, took bio and one gen chem but then became a public health major and didn't have time to take my pre-med requirements. I am about to graduate and need to decide if I should stay at my institution for an extra year or do a post bac program. Problem is that most post bacs require that you've never taken a pre-med class (career change) or retake pre-med classes (GPA boosters). What should I do?

I am a junior- rising senior at a small research university. I have completed biology I and II, Gen Chemistry I, Stats, and Calculus. I cannot complete the rest of my pre-med requirements before I graduate because my public health major gets in the way. I could stay an extra year to take Gen Chem II, Orgo I and II, Physics I and Physics II, and Biochem, or I could do a post-bac. However, most post-bacs require people to have completed all/most of their pre-med requirements or none at all (career changer). I am not sure how to finish my pre-med requirements/ I feel stuck in between. Ps: I do not like my undergraduate institution and would be happier to go somewhere else...anywhere else. Thanks!

Pathdocmd

You do not have to take these classes in a formal post bacc program. It sounds easier for you to take the additional chemistry physics at a different college (I would not do this at a community college) and the take the MCAT and apply. There is nothing wrong doing it this way. I would also get more biology (cell/ molecular, micro, physiology) under your belt while you are at it.


Im older non traditional RN

Hello, are there any RN's that are pre-med out there? I have a unique set of circumstances my husband and I are both pre-med any advice. Thanks in advance

tantacles

There are many RN's who are pre-med and many who have completed medical school. The clinical experience of being an RN will absolutely serve you well in medical school and beyond.

This thread from the student doctor forums has a few nurses who became MD's, and you might consider contacting them:

https://forums.studentdoctor.net/threads/thoughts-on-nurses-turned-md.1151511/


Would it be alright to take the MCATs on May 13?

Hi guys! I am sorry if this question has been asked already. I did find one thread but it originated in 2009. So I wanted to ask it in the context of the present. I am a senior in college now, and I plan to apply to med school this summer. I was wondering if it would be OK for me to take the MCATs on May 13? I know I get my scores back before the second week of June. Do you think taking it on May is appropriately timed? I cannot afford two years of gap as it stands now. Thanks in advance for all your advice.

tantacles

That's fine. You can still submit your AMCAS on the day it opens, so be sure you have everything ready before then, including your letters of recommendations and transcripts.

I would submit to one school along with transcripts to start the process of getting your AMCAS verified. When you get your MCAT score back in June, add schools according to how competitive your MCAT makes you.

If you do not do well enough on the MCAT, please take the time off that you need to get in. There is nothing worse than applying more than once to medical school, and it is always better to wait and retake if you receive a score that will put you out of contention for a spot.


Gap Year Plans

I tried viewing the questions already asked and, every time I did, the website gave me an error message. So, I apologize if I am asking a similar question to ones already asked.

I am 32 years old and a senior at the University of Michigan. I am a biochemistry major with a 3.97 GPA who got a 515 on the MCAT. I have no research experience and only a few hours of shadowing experience. I have already been rejected by several schools and am expecting to be rejected by all of them at some point because of my lack of extracurriculars. However, I did get into an SMP program.

I thought about doing the SMP program to try and get more clinical and research experience. But, I am hearing that this program is not designed for students like me and that it would be a waste of time and money.

My concern is that I am unable to go back home to live with my parents. So, I can't spend all of my time volunteering and shadowing to fill in the gaps in my application because I will have no money on which to live.

Do you have any suggestions for someone in my situation? I am unsure what to do.

tantacles

With your stats, an SMP doesn't make sense. You seem to have figured out the problem with your application, namely your lack of extracurricular activities. You may also not have applied to the right schools. These can be remedied reasonably easily.

I would suggest that you find a full time job with a two year commitment that is either clinical or research based (clinical research would likely be best) so that you can build your application and stay afloat financially. In the meantime, I would start volunteering. There are organizations that will let you volunteer 4-6 hours per week, and if you did this for the next year, you would have enough volunteering as schools like to see a longitudinal experience. Ideally, you will have medical and non-medical volunteering, but if your job gives you clinical experience, you can get by with just non-medical volunteering.

If those don't appeal to you, you can also find a job that interests you in any field that will support you and then subsequently find clinical experience and non-clinical experience over the next year. Research will be more difficult to find, however, without a full time job after college.

In short, an SMP is not good for you; SMPs are for students who need GPA repair generally, and you do not need that; you need to improve your extracurriculars.


Have You Ever Applied to Medical School w/ Your Wife or Husband?

Hello:

My wife and I are applying to medical school in 2017 with the intent of gaining acceptance to the same school. We recognized the competitiveness of the process and likelihood of being accepted at the same university. We are both non-traditional medical applicants with very competitive and attractive backgrounds.

Have you ever applied to medical school with your wife or husband? We appreciate any insight and recommendations! Thank you!

tantacles

It will be difficult, but possible, to get accepted together. The main issue is that if your stats are very dissimilar, you may find that one of you is unable to get an interview at one school while the other is interviewed. I suggest you both apply broadly to schools you are both competitive for. If one of you gets an interview, it would be a good idea to politely reach out to the admissions committee and mention that the other spouse is also applying for medical school and that if there is any way they could offer him an interview as well you would be very appreciative.

I would also write about this in any "additional information" section available on your application so that schools will already know your situation.


Do I send an update?

I interviewed at a school in September. They are non-rolling and release all admissions decisions in March. I am in my gap year and in addition to being a research assistant at a hospital, I started a new job and two new volunteer positions in October. On my initial application, only the research position is listed. Are these new activities worth an update? I do not want to reduce my admission chances by submitting one that is unnecessary. Thanks!

tantacles

Because the school does not have rolling admissions, it would probably be a good idea to hold off on sending an update; your update will likely not help your application a tremendous amount unless you have received multiple A's in classes or published a paper. Starting a new job and two new volunteer positions is, unfortunately, not a tremendous update. If you are not accepted to medical school, definitely add these positions to your next application.


Multiple MCAT scores consideration for schools in New York

So when I first took the MCAT I received a score of 484, and then I retook it and received a score of 520. Will medical schools like SUNY Upstate, downstate, stony brook, albany consider the upward trend or will they downright just average the two scores? If they average the scores, then the score becomes lower. So I was wondering if they will even consider the upward trend. Also have you ever called the schools to find out such as Stony brook, upstate, downstate, buffalo etc? Thanks for the help!

tantacles

What schools do about multiple MCAT scores is school dependent. I would suggest that since you are in state for these schools (I imagine you are) that you apply to them regardless. It is possible that they will not take the 484 incredibly seriously given your high later score, and it's also possible that your low score followed by a high score will give them pause.


Help!!! Need advice, opinions, facts, all welcomed!!

This may seem a little dumb lol but I have dreams to become a doctor. I'm only 16 but I want to be know so when the time has come, I'm good to go. My dilemma is that I don't know what school I should go to or any of that good jazz. I have great grades, strictly on the A/B honor role, and I have this 200 medical book my aunt lended down to me when she was practicing to become a nurse. I was looking in the Emergency Medical field but I am keeping my options open. Any ideas or thoughts??

tantacles

The first step is to get into an undergraduate college. There, you need to take all of the required pre-medical courses. An SDN user has written fantastic advice for pre-medical undergraduates which would be worth a read at the following link: https://forums.studentdoctor.net/threads/advice-to-end-all-advice-for-undergrads-your-critique-please.783462/#post-10418099


Should I change my major?

I am currently a sophomore student aspiring to major in Biology and Spanish. I have already taken every class that is needed for admission into medical school. Initially I was wanting to double major because I felt that it would set me apart in the application process when applying to medical school, but as I look at what I need to take to graduate with a Biology degree at my University it is all very daunting. I realize that if I do not keep my grades up above at least a 3.5 then no medical school will probably even look at me. So I wonder if it is better, now that I have all the classes that I need for medical school, to go and just fluff my GPA and make sure that I am good on that end. Maybe it would be more advantageous for me to go ahead and complete my major in Spanish, maybe get another major in English or History and then minor in Biology or Chemistry? I realize that it is very hard to judge my situation based on the information I have given you, but any words of advice would be greatly appreciated.

tantacles

Your major is completely unimportant for medical school. The most important thing is to be certain that you have a high GPA.

I must repeat this: Your major does not matter. Raise your GPA as much as possible to have the best chance of getting into medical school.


non traditional student very confused about the MCAT

Please help me, I have no pre-med friends and no pre-med adviser access.

Background Info: finished my undergrad with a very low gpa many years ago (was not taking school seriously, majored in psych, never took any real science classes). I came to my senses and took nothing but science classes at my local community college (a very good one) and got a 3.6 gpa. As far as prerequisites are concerned I need one more semester of physics and 3 more semesters of chemistry (I took 3 semesters of chemistry but two of them were introductory and don't count). I applied to the sf state post-bacc program and was told i was too high risk to be admitted even though they acknowledged that i had potential and had really turned around academically. After being denied admission i ambitiously decided to just study for the mcat on my own and apply again with an mcat score.

Here is where i am losing my mind: I studied the kaplan 7 book series front to back (literally flashcarded every single concept/definition of every page, I probably have about 3 thousand flashcards) and am now realizing that i misunderstood how the mcat works when i started taking the online practice questions provided by kaplan.

I am okay with the fact that I wasted a lot of time going into too much detail with the material, but my problem/anxiety lies with the physics and chemistry sections. I am convinced i wasted my time buying sterling's physics and chemistry questions book because the actual mcat seems to be much less concerned with calculations.

I have anxiety especially with organic chemistry as I lack any formal education with reaction mechanisms etc. I learned with Kahn acadamy and understand the Kaplan material, I just dont understand where/how i can practice discrete practice questions that only stay within the realm of mcat material (which i appreciate is superficial)

I have 3 full length practice exams from Kaplan that was provided online but I dont want to waste them until I feel fully confident with my abilities.

My question: should I keep practicing in depth practice questions on chemistry and physics (which seem to be few and far between on the real mcat) or should I just bite the bullet, start really tackling the test format and hope i dont have an anxiety attack when I get discrete physical sciences questions (just from lack of experience)

Thank you, and Im really sorry for rambling.

tantacles

Start tackling the test format. You can buy more practice tests. If anxiety is an issue, I would buy as many practice tests as you can and do as many as you can so that the test format seems old hat to you on the day of the exam. Push back your test by a month or two if you're not ready.


MCAT

I have never taken the MCAT. I am preparing for the MCAT right now. I have 4 months to study. I am not sure on how to study for the MCAT. Also for the psych/soc. section is there alot of anatomy/biology questions. I am using TPR books to study from and it has a lot of biology/anatomy integrated into it. I am a psych major and I didn't go through biology to understand the basic conepts in psychology. I am just lost on how to study and need guidance. This is my first time here and I am not sure how this works. But I hope I get some answers back.

Thank You :)

tantacles

Using review books is a good way to start, but taking many practice tests is key - this will give you a chance to answer questions under real test conditions and see how you would do on the real test.

I would also suggest you look on the SDN forums; there are countless strategies that have been suggested, and many of them have resulted in very high scores. Ultimately, everyone studies differently, and you have to find a way of studying that works for you.


Gap year(s) advice

Hell Experts!

Please don't be scared by the long post, i've just thought long and hard and really need your feedback on what I should do next in my pre-med journey. So I am currently a senior entering my last semester as a cell & molecular biology major. Right now, I have a 3.4 cGPA and 3.1 scGPA so far with not much of a trend, but still have calculus and another biology class left this semester as well as a year of physics that I need to take over the summer. I have not taken the MCAT yet, but plan to this summer(timing depends on your feedback).

-Clinical experiences aren't particularly strong(length of time-wise) with 60 hrs of shadowing, summer internship in mental health nursing, and respite care for a child with autism(3 & 1/2 semesters). -Extracurriculars include peer mentoring(2 semesters), treasurer & now president of science & math national honor society(3 years of participation), chemistry & biology tutoring(2 semesters) and helping out with the youth at my church(6 years), traveling(i've visited over 11 countries as part of mission trips, school trips, and study abroad programs).

What should I do next? -Just Apply? My family wants me to apply this cycle and use my gap year to strengthen my clinical ECs. They're thinking I should just give it a try even though I would feel rushed and i'm not sure if schools will let me apply without finishing physics beforehand. If I did apply, I would have to apply later(july) even though I know earlier is better because I need more time to study for the MCAT(which I would have to take by June at the latest) which I haven't even started to study for besides gathering materials.

-Extra courses vs. Post-Bacc vs. SMP? I know my GPA sucks for MD and probably DO schools and i'm thinking if I wait until the 2018 cycle, I can do one of these to improve it. I'm not sure though which is better for my case as I hate the idea of huge loans for an smp and i've already completed most of the courses for a post-bacc.

-Masters program? This won't fix my undergrad GPA, but a good grad GPA in a hard science masters is better than nothing and can be a back up plan in case I don't get in according to my advisor. Although posts about grade inflation in grad schools, especially MPH programs, have largely deterred me from using it as a GPA saver.

-Work? I like the idea of focusing on working as a medical scribe for more clinical experience and studying hard to get a strong MCAT in my gap year(or two if I don't apply this cycle) instead of spending money taking classes. I already have an ED volunteer research assistant position in a program lined up after graduation, but I definitely need that time to find community service work i'm interested in.

-A combination of any of the above?

For the longest time I didn't like the idea of taking 2 gap years for times sake, but I know now that the reality of my decreased competitiveness is forcing me to consider it unless you guys think I still have a chance at MD/DO schools this cycle? My dream has always been MD, but I heard DO is great for primary care which is an interest of mine. Thank you so much!

tantacles

It is impossible to tell you what your chances are without an MCAT score.

That being said, your GPA could definitely use an improvement, and I would recommend a post-baccalaureate program to improve your AMCAS GPA. If you apply now, your extracurriculars would not appear on your application and your GPA would work against you.

You should only apply to medical school when you are 100% ready. Right now, it looks as if your application still needs work and you are not ready to apply in earnest.


What would be a valid reason to argue a grade change? C- in Orgo Chem Columbia PostBac

I am a disabled combat veteran. I have multiple issues with my medication management this past semester. Additionally, I scored above the mean on my first midterm, dropped the second (as an option) and score just below the mean on the third midterm. My final was three hours long and worth 60% of the grade. Just prior to my final I was having many issues with the VA and can even supply a letter from my Physician. Because I did poorly on my Final I received a C- in Orgo, a required premed course. I go to Columbia and am in the Postbac premedical program. This C- will make in ineligible for committee support and will most likely cause me to be dismissed from the program. My Dean told me he needed to speak with me early next week. I am scared. this is my last semester and have never received a grade this low in my life. Please advise. I dont know if I should focus on the mishap of the VA or if I should argue the unfairly amount of weight the final exam was on my final grade and argue for a grade change. Even a C would keep me in good standing. Please help me.

tantacles

If you did not score high enough on your exam to receive above a C, then it is unlikely that your dean will change your grade. I would suggest that if you need the grade changed that you pull out all of the stops; perhaps you can ask for a retake of the exam. Currently, you have not earned a grade above a C-, and you need to accept that and either fight for your career path as a physician or move on to something else.


Is business calculus and finite mathematics counted towards BCPM?

Hi all, I took business calculus and "finite mathematics" my freshman year back when I was a business major. These are both 100 level math department classes at a small private university I'm attending. Will these count towards my science GPA on my AMCAS?

Pathdocmd

It depends if it was designated as a business course or a math course on your transcript (e.g., bus 202 vs math 101).


Graduate or undergraduate level Post-Bacc

Which is better for someone with a GPA on the lower side? Somewhere between a 3.3 and a 3.5? There are a lot of opinions on this in the threads but very little of that is from adcom members so I would like your opinion. How much do adcoms put stock in graduate level courses?

tantacles

Graduate level courses do not affect your AMCAS GPA. Undergraduate courses, on the other hand, are factored into your AMCAS GPA. If your goal is to get into medical school, I would take undergraduate level courses.


Will a medical school value my study abroad research experience?

Hello! I am currently a junior studying biology with a chemistry minor. I will be entering the medical school application cycle for the 2016-17 year and I wanted feedback on an experience of mine. I am currently involved in an academic year-long biomedical research program in England. It is a clinically and medically based program that has allowed me to grow as an individual, enjoy my degree, take part in research, and have the experience of a lifetime. I can't express how hard I worked to have this opportunity. The classes also equate to a year's worth of credit hours, and I plan to graduate on time. How much will a medical school value this experience of mine? Are there schools that would favor it over others? How can I make this experience more meaningful? Note: I spent the first two years of my undergraduate career partaking in leadership positions on my campus, as well as a great deal of philanthropy work; I also have clinical experience. Thanks!

Pathdocmd

The answer to any question about this or that experience is better, more impressive, etc. is what did you get out of it, how did it help you develop as a person, and what will it bring to your med school class and future patients? If you can communicate this in writing (i.e., application and essays) and verbally (i.e. the interviews) then you will show the admissions committee the experience's "value."


Do I have a chance to get into a Texas Med School?

I am a current senior at UT Austin with a cum. GPA of 3.21 and science GPA of 3.0. I am a first generation college student from a low-income family in South TX. I went into college not even knowing basic chemistry/math because we didn't really have great teachers and our high school is not in a good area... This all ties into my rough first couple of semesters. However, there is an upward trend in my GPA. I am an EMT with lots of experience in dealing with patients, officer for three organizations, I have done global outreach, research in biochemistry for two years, been involved in a fraternity for 2 years, and a student mentor. I have not taken the MCAT yet however but plan on doing so this summer. What can I do to prepare for my last semester before applying to medical school to ensure I have a chance of acceptance in Texas?

tantacles

Your chances with a 3.21 cum. and 3.0 science gpa are not great, but depend on your MCAT score. I would suggest you take more classes to raise your GPA to a more reasonable level and ensure you do very well on the MCAT; at this point, those are the things that will keep you out of medical school.


Switching MD/PhD to MD

I have received the advice that if I have not received an interview at a school I applied MD/PhD, I likely will not get one, and that if I am interested in their MD program, I am better served by calling the school and switching to MD, as I am more likely to get an interview.

While I am passionate about MD/PhD, there are schools where I would gladly attend as an MD student. Should I call and switch MD only? Will this be viewed as "wishy washy" or uncommitted to my goals?

Pathdocmd

It depends on the school. Some are MD/PhD or nothing and some are OK with you taking "only" the MD acceptance. I would research each school's policy and/or talk to someone in the admissions office. Or you did just fine and you will get accepted to the MD/PhD program.


I need to decide between RowanSOM and PCOM? I live in jersey. Do you have any suggestions that would help me pick the right school

I am a new jersey resident. Accepted to both schools.

tantacles

it very much depends on your preferences. Where do you wish to live? What did you like about both schools? Does one cost more than the other? No expert can make this decision for you as most medical schools are, essentially helping you learn the same material but with a different style. Choose the one that works for you.


How important is a sponsorship letter from your prehealth committee?

How would medical schools view a non sponsorship letter vs a sponsorship letter from your school's pre health committee? Would a non sponsorship letter be viewed badly and put a student at a disadvantage in the application process?

Pathdocmd

I can't speak for other med school admissions deans, but I REALLY prefer committee letters over individual ones. Saying that.. it will not hurt you if you get individual letters.


Ways to get research experience for a non-traditional applicant

I am a non-traditional med school applicant, having completed my MBA and worked as a finance consultant for 3 yrs. I completed 95% of my pre-reqs at the time of appln (3.93 GPA), 513 MCAT, have close to 200 hrs of volunteering, and 40 hrs of shadowing. I am guessing leading teams and projects at work would count as leadership exp. I did not have enough time to devote to research while in school and so do not have any research experience.. I applied this cycle to 9 CA schools (my husband works here), and though I received secondaries from all schools, was rejected post-secondary from 4...still waiting to hear from the remaining 5 schools on their post-secondary decision. It is starting to look like I might have to re-apply. I would like to strengthen my application and get some research experience - Is there a way I can do that without being a student at a university? What other weak points do you see besides the lack of research? Thank you very much!

tantacles

It sounds like there are several issues that are keeping you from getting into medical schools; the main one is that it seems you didn't apply broadly enough; California schools are notoriously competitive even for applicants with great statistics, like you. While it absolutely makes sense to prioritize your family and your husband's career, applying to private schools outside of California would most likely give you better yield; many students apply to medical school outside of California and return for residency.

In terms of research, there are research jobs available at many universities that you can apply for; it would likely be difficult to get significant research experience without doing this, but there are many schools, which can be researched through the MSAR (which you can purchase online) that do not prioritize research; if you apply to 15 schools with mean GPA and MCAT at or below yours in addition to your California state schools, you have a great chance of gaining acceptance.


Interest Letters

If I am on a pre-interview hold at a school and would like to send them a letter expressing my deep interest, but the school does not accept updates, what should I do?

Should letters of interest be short and to the point, or more fully explain why I believe I am so interested in the program?

tantacles

If the school does not accept updates, you should not send them an update; the school does not want an update.

Letters of interest should only be sent to schools that welcome them openly. They should explain why you are interested in the program and be less than one page long.


Advanced inorganic chem before the MCAT?

Hi everyone! I really need some advice. I've seen this question a couple of times, but it was posed before the new MCAT came out and there are conflicting responses. This semester, I'm planning on taking inorganic (not gen chem). Has anyone taken this class and found it helpful for the MCAT? I'm taking the MCAT in may, so I need to dedicate this time to studying for it. I don't want to load up on classes (I'm at 17 hours will physiology, cell phys, biochem, inorganic, and research). So should I take it this semester or wait it out? I know it's not *needed*, but is it helpful? Thank you.

tantacles

If you are able to do well in this class and you would enjoy the material, it would be a good class to take. However, it sounds like you are overburdened based on your current class schedule and that the class is not strictly necessary for the MCAT; your time would likely be better spent studying for the MCAT.


January, Still No Interviews

Hi,

It is January and I have not heard back from the majority of the schools I have applied to. I imagine for most schools I am in the "maybe" pile. For the school I feel most strongly about (and have not heard back from) is it appropriate to send a letter to the admissions committee reaffirming my strong desire to go there?

tantacles

It is absolutely appropriate to write a letter expressing your interest. It is very likely that all interview spots are taken, but writing a polite letter of interest can not hurt.


What is the best way I can approach my goal as a high school student.

This may not be the appropriate place to ask, however, I'm currently a sophomore in highschool. Ever since I was young I aspired to be a (orthopedic) surgeon. I took highschool classes in middle school and didn't care much of it, being immature I did not realize how much it affected me in the long run (lowering my gpa). Currently I have made substantial changes to my life education wise including raising my gpa back above a 3.0, my question is what steps should I take to help give my self a better chance at making it into better schools etc. I'm going to volunteer this summer at a local hospital but I am also looking into shadowing a orthopedic surgeon. There are some classes I saw that I can take but are expensive and not in my state. What is your advice on things I should do or take part in?

casedentalmed

Being an orthopedic surgeon is one of the most competitive medical specialties (in US medical education, so I'm presuming that's your focus). The major focus is to show consistent academic excellence in your math and science classes, maybe even think about doing research throughout your undergraduate period.

But keep in mind, if you are being trained in the United States, your first goal is to get into medical school, which means you have to complete college and get into medical school where you are trained to be a general physician. Then you have to go through the residency match for the competitive process to become an orthopedic resident or surgeon. You have a long way to go, but your foundation starts with being serious about your studies and focused in your science coursework.


Naturopathic to Allopathic Transition

After I graduated from my undergraduate program, I attended naturopathic medical school for one year before having to make a difficult decision to change my educational path to my career in medicine. There were many factors that went into this decision, but a lot came down to the lack of autonomy in practice, limited exposure to the patient population I want to specialize in, as well as lack of training in emergency medicine which I think is really valuable no matter what field of medicine. It was not until I really got into experiencing the program that I realized that I was going to be far more limited in my education and clinical experience than if I had chosen the MD/DO route.

I am wondering if anyone can possibly shed light on how this one year of naturopathic school will look to admissions committees. Will it be viewed as more negative because I did not finish the program or positively because I gained experience and knowledge in this particular aspect of integrative medicine. Also, if there are schools that anyone knows of that might value this experience, that would be great.

tantacles

Your year of naturopathic medicine will be unlikely to affect your admission to medical school unless you had particularly poor performance in your program; I would find something worthwhile to spend your time on in the intervening time between your application to medical school and now so that you can show your commitment to allopathic medicine.


Medical Records Question

Short synopsis: I accidentally overdosed on acetaminophen by taking too much over the course of a day (I had a migraine, was drowsy, wasn't thinking clearly, and took the maximum dosage). After spending a night in the ICU, I now have an "accidental drug overdose" on my medical record. I have never had any other health problems nor any issues with the law.

Is this going to impact my medical career in any way? I guess I'm worried about questions regarding substance abuse....I didn't think this counted, but a friend has me concerned.

I have already accepted an offer for medical school next fall. Is there a reason I should update them about this?

tantacles

Medical schools do not have access to your medical records, and there is no need to inform them of your overdose unless there is a legal or disciplinary sanction that arose because of it.

Pathdocmd

I would say no to both. It was not an illegal drug or abuse of a prescription medication (e.g., opioids). I'm glad you are OK. Be careful!


Organic chemistry 2 question

Hello,

I will be taking organic chemistry 2 next semester and I was wondering how I can prepare for it?Thankfully I did well in organic chemistry 1, and I wanted to know what topics/material I can go over to review which can help me in organic chemistry 2.

Also, is there a lot of sn1/sn2/e1/e2 material in organic 2? I had trouble with that in ochem 1.

Thank you.

tantacles

Generally, the best way to prepare for a class is to make sure you are ready to focus on the material and do well on the pre-requisite material. It sounds like you did well on organic chemistry I, so take a breath, make sure all of your materials are ready for the next semester on day 1, and be ready to focus in class. Doing well in organic chemistry I is a good sign that you're well prepared for the next half of the series.


Gap Year Options

I'll be graduating from Texas A&M in May with a degree in Molecular and Cellular Biology, a 3.67 GPA, a 513 MCAT (127 Chem/Phys, 128 CARS, 128 Bio, 130 Psych/Soc). I've done two summer research internships, worked in an ER as a scribe for a few months (400 hours) and done volunteering and leadership stuff at school. I will be taking a gap year this summer but am wondering what I should do?

My current ideas are: teach MCAT, substitute teach, keep scribing, get a research position, work as a clinical assistant or get my EMT certification

Any advice is appreciated!

tantacles

All of your ideas are fantastic, and all would make for wonderful gap years. It sounds like you already have a well-rounded application, and I would suggest you take the position that you are most interested in.


BCPM GPA vs. cGPA in looking for schools

I had a really poor freshman year (2.5 GPA) and have subsequently done much better throughout school. By the time this coming spring semester is over and I do as well as I've been doing I should have a 3.59 cGPA and a 3.88 sGPA to which I will be applying at this point. What score should I shoot for on the MCAT to still have a fair shot at some mid-tier MD schools if I still have a chance at them? Thank you for your time.

tantacles

You definitely have a good shot at many medical schools. Aim for the highest score possible on the MCAT as it will give you choices and potentially chances for scholarship. Scores of 508 and above are generally considered to be competitive for medical school and should complement your GPA and significant upward trend well. I would strongly suggest purchasing the MSAR to figure out what scores you will need if there are any specific schools you are interested in.


Gap year options

I am a senior with a 3.99 GPA and 500 and 497 MCAT. I have worked with pregnancy/infant heart research for 2 years and am finishing up a thesis. I'm in my school's Honors College and have various shadowing experiences (physical therapy, occupational therapy, optometrist, pediatrician, internist). I volunteered in a hospital for 2 years.

I'm not sure if taking a year off and working (probably in a non-medical field due to where I live; but could take a CNA-I class this summer and look for a job in that or look for getting certified to work for an EMS company) or doing a 1 year Masters program (applied to MBS at Duke and Georgetown's MS in Physiology) will be best? Cost is definitely an issue too.

Also, should I retake the MCAT this summer? I did not study well the first time around, took it 3 weeks later to try to squeeze in the last one of the year which ended up being a big mistake since I went down 3 points due to not studying because I was busy with school. However, I doubt I can get much higher than a 500 due to slight ADD.

I am going to apply DO next year (and to a few MD schools) because I realize that the MCAT ruined my chances for MD. However, I know this is what I want to do so I am not going to give up no matter how long it takes. I would really appreciate any and all advice on how to improve my application for next year!

tantacles

Ultimately, with your MCAT scores, it will be difficult to get into MD schools. DO schools are definitely a possibility, but you would be well served by taking the MCAT again (and doing well). If you truly have ADD, I would recommend that you seek appropriate accommodations, and if you have not been diagnosed and are simply using that as a reason you did not do well, then technically, you do not have ADD and would likely not qualify unless you see a professional.

I don't think you need to give up, but I think you need to give the MCAT a real shot; it sounds like you did not study well either time you took the exam and you may be able to improve. Keep in mind that tests do not stop once entering medical school, and you will have to take standardized exams throughout your career; I would suggest you try to do well now and invest your time in this exam.

In the next year, I would get more research experience and do your best to do better on the MCAT; that will give you your best chance of getting in.


How good are my allopathic chances after retaking a class?

I got an F in pre-calculus but retook in and got an A, I also got an A in calculus the next semester. I know that AMCAS will average the retake together with the first take and it will be a C, which hurts me. Am I better off going osteopathic where they count the most recent grade of the retaken course only?

tantacles

With only one grade, it is impossible to gauge your chances. One F will not make it impossible to get into allopathic or osteopathic medical school, and medical schools will look at your entire application, including every course you have taken. I would suggest that you continue to do well in all of your courses and ultimately do well on the MCAT and then and only then reevaluate where you should apply to medical school.

casedentalmed

Your description of how AMCAS calculates your GPA vs. AACOMAS is accurate. You're going to have to decide how you want to to proceed with your application strategy.


What can I do to increase chances of getting into a med school?

I'm a college student who will be graduating with an overall GPA of 3.35, and 221 credits. The last three semesters have been approx 3.9 GPA and 23 credits per semester. I've yet to take the MCAT (taking the year off, burnout is real), but hope to attend a school in WV or nearby. I've shadowed several doctors in the past three years, volunteered at a nursing home for a year, was the president of a sports club on campus for a year, and have been an active biology club member for four years. I also have a ND degree, dunno if that helps. What do I need to to increase chances of getting accepted into a school?

tantacles

The best way to increase your chances of getting into medical school would be to continue to take some more classes to show you're doing well in science courses and increase your undergraduate GPA (graduate courses won't help!), do well on the MCAT, and get some more volunteering and clinical experience. It seems like the pieces are all coming together for you. I would also suggest you purchase the MSAR once you've taken the MCAT to determine where your application is likely to be competitive.


How bad will a C+ affect my admission to med school?

I am currently a freshman at the University of Florida. Something awful happened to me during finals week. I missed one of my final exams because I wrote the wrong date on my calendar. This resulted in me getting a C+ in General Psychology (I would have probably gotten an A if I had taken the final). Other than that, I got straight A's in all my other classes (Gen chem, trig, english, an elective, and chem lab). This results in me having a first semester GPA of 3.63. How bad will this C look on my transcript going forward. This was a genuine human mistake that I have learned from. It has now motivated me to work even harder. This hit me really hard because I put in so much work this semester but it is behind me. I am just curious about the long-term effect of this C.

tantacles

It is rare that a C, particularly one received in the first semester of college, has a profound effect on medical school admissions. Medical schools tend to look at patterns of grades rather than any single grade, so provided there aren't many more C's in your record, you should see minimal effect from this one grade. If getting C's becomes a pattern, you may find, though, that they have a significant effect on your admissions record. Do your best to recover from this and do well in all of your classes and you will likely see little impact from it.

casedentalmed

One can't answer for all medical schools, but let it be your only C grade if you can. Depending on the schools you ultimately shoot for, some will be more tolerant of that grade (in a non-science course) than others. Don't let missing an exam happen again.


Should I do an SMP, regular masters, formal post bacc, or unstructured post bacc?

I have a GPA of 3.29 and MCAT score of 500. I have been working as a scribe for 2.5 years and volunteered in a free clinic. I know that I need to increase my GPA and retake the MCAT, but I am wondering the best way to do this. I am considering financial options and the most efficient way to make my application more competitive, as it does not seem likely that I will be accepted to medical school this cycle. I would love to continue scribing, but I believe I need to enroll full-time in a program, but I am not sure which program to research. Thank you for your help!

casedentalmed

If you're going to go all-in with coursework, you will need to drop your scribing position. With that low a GPA, and assuming you have a science background, a formal postbac or SMP may be the best way to approach things. You'll need to talk with your previous prehealth advisor if there are other options, but you will need to show maturity and an ability to handle a schedule of challenging science coursework if you are going to give yourself as strong a shot at medical school. And don't work while doing an SMP/postbac.


Anesthesiologist or CRNA?

I am sure that I want to have a career in anesthesia, I am not, however, sure which route to take as I could be a doctor or a nurse and perform anesthesia. Ultimately I want to be the best anesthesia provider I can, and I know that being a doctor would provide me with the best education, but I could also see myself as a nurse anesthetist; I've shadowed both, and they both seem satisfied with their career choice. All of the nurse anesthetists I have met have been happy with their career choice, and they were satisfied with their workload as well, but when I asked them if I should be a CRNA or anesthesiologist most of them said CRNA but 1 told me to be an anesthesiologist. The anesthesiologists I have met said they had longer hours which they were mostly okay with and they all told me to go to med school and be an anesthesiologist. So, with their recommendations split down the middle with one CRNA recommending that I go to medical school, I am still torn. I feel a little apprehensive about attending medical school because I've never considered myself as a genius, but I know I could do well because I am a hard worker. I feel like there is a lot of pressure for me to choose what will make me happiest in the future and I don't want to look back as a CRNA and have to call in the anesthesiologist when I don't know something, and wonder what they know that I don't. I think since I love anesthesia so much I should go all in and become an anesthesiologist. Do you think that the doctor path is the right one for me if I know that I want to do anesthesia and I want to be the best anesthesia provider that I possibly can?

tantacles

I would absolutely say that while the two jobs have very similar tasks, they are different in their scope.

As an anesthesiologist, you would be managing other anesthesiologists and managing a team of CRNAs, perhaps, as well as residents and PAs as well as performing your own cases.

As a nurse anesthetist, your scope would likely be more limited (provided you don't pursue more education), and you would likely find yourself mainly running cases.

Either field is acceptable, so take your time deciding; get some more clinical experience, and also think about whether medical school or nursing school would be more tolerable to you; the education in medical school is grueling, and while certainly thorough, may not be your cup of tea. On the other hand, you might find a more grueling curriculum more stimulating depending on your preferences. It really is all up to personal preference and ability to get in.


Is it okay to quit my job before medical school?

I was recently accepted to my dream medical school. I could not be more happy about that. However, I am currently miserable working at my full-time research job (bad work environment, little pay, etc). I started the position in July, and it is a one year appointment. I would feel bad for my boss if I left early (he is great), but I also want to take some time to travel and relax before I begin school again. Is it okay to leave early even though I put on my med school application that I would be there all year? Also, will I need letters of rec in medical school to get research spots? Thanks so much for your input!

casedentalmed

Congratulations on your acceptance. You'll have to be the one making the final decision. You will need some time for yourself to relax, but you need a conversation with your supervisor about it.

Now if you said in your application that you have committed a full year to the research opportunity, you probably should do your best to stick with your commitment. Granted, no one is going to police you or force you to stay, but if you are going to be true to your word, try to stay.


Applying before finishing one of my pre-requisites

Hello, I still have to take calculus before all of my pre-requisites will be done but likely won't be able to take this until this coming fall semester although I intend to apply in the summer. I will have all of my pre-requisites done by then except for this one. I also may take the pre-req. to calculus if it is a practical option but I would like to know how this will impact my application. Thank you for your time!

tantacles

If you are enrolled for this pre-requisite but have completed all others, not taking this one should not affect your application; just be certain to do well and e-mail schools when you receive your grade.


What can I do to stand out?

A quick look at my background: I am graduating with a BSN in April of 2017, but have decided to embark on the journey into medicine as it has always been my dream and what I feel to be my true calling.

I currently have a 3.87 cGPA with the potential to raise up to 3.89 over this next Spring semester. sGPA is most likely a 3.95+. I have worked in the medical field as an emergency department technician for a little over 3 years now and have worked my way into a leadership position within the ED. I currently lead a team of 12 ED techs. I am married with two children and have worked full-time my entire undergrad. I feel that this shows my ability to balance my work-personal life effectively.

I am currently talking with a local community outreach organization about volunteering with their medical outreach efforts. My plan is to take the necessary prerequisites over the next two years and then take the MCAT in Spring '19 in order to apply to schools in June '19.

What can I do over the next two years to make myself stand out? Aside from my clinical experience, I feel that my holistic picture is lacking.

Thank you.

tantacles

The piece that might be missing from your education is research; Many medical schools like to see applicants make a significant intellectual contribution, and if you can spend some of your time working in a laboratory and getting experience creating scientific questions and methods to answer them, it would help your application to medical school.

Pathdocmd

I do not feel that your picture is lacking in the least. Real clinical experience, good grades, worked full time in school, juggling family responsibilities... You are doing just fine. Hopefully you will do fine on the MCAT.


ASAP: Is My Chance to Get Into Med School Blown?

I know this question is asked a million times, but are my chances of med school blown?

Before this semester, I had a mediocre GPA. I was able to make all A's besides one C. I was going for business before I decided what I really wanted to do and economics class was not my thing. Anyways, this semester I am taking Bio, Chem, and Calc. Not because the work is hard, I just didn't study nor go to class much, I failed Chem and Calc. I missed all the quizzes and never did the homework. I'm not going to make excuses about it because the blame is all on myself, with the exception of scantron. The prof. didn't let us show what we knew so it made it much harder to pass the exams with high scores. I managed to pass Bio with an A+, though.

I am able to retake both those classes, and the new grade will replace the old in my GPA calculation. I already know that MD schools will take an average of both, I know I will have to work my ass off to be able to do it, but I should be able to pass all my classes with A's from now on. If I do that, my GPA should be 3.6 by the time I graduate. I think that is competitive if I earn a high score on the MCAT.

I was just wondering if my chances are blown by failing those two class even if I retake them and continue with good grades from here on out? I am very head strong, and when I put my mind to something I won't stop until I've reached my goal.

I was also wondering, what type of EC should I do? We have a biology/chemistry club on campus that I was thinking about joining, but I was also wondering what type of things I should do off campus? I know volunteering, but are there any specific places I should look for? I also already work at a nearby hospital as a multi tech doing EKG's and phlebotomy.

Thanks for the replies ahead of time.

tantacles

If, in fact, you can get your GPA up to a 3.6 (while counting your failures as 0's in your GPA calculation), you still have a good chance of getting into medical school. In fact, even if your GPA does not improve quite that much, getting all high grades (read: A's and A-'s) can redeem you because medical schools like to see improvement. They tend to be forgiving of academic lapses at the beginning of college and less lenient towards failures near the end. You will still have to do well on the MCAT, and you can purchase the MSAR (google MSAR!) to see what kind of scores you will need to be accepted.

Outside of campus, I would recommend you do several things:

1. Get clinical experience (your current job counts).

2. Get Volunteering experience (Does not necessarily need to be medical) over a long period of time; volunteering 3-4 hours each week with the same organization will serve you well.

3. Do some research. This need not be scientific research, but it can be. Try to do research where you will be involved in the intellectual aspects of asking research questions rather than just doing grunt work in a lab (i.e. washing dishes).

4. Participate in leadership. This could be in any field, but you should ideally be leading your peers and not just doing an activity like teaching, where you teach those who are "beneath" you in terms of knowledge and experience, though teaching is certainly an admirable form of leadership. If you become the leader of a club, that is acceptable, but keep in mind that clubs aren't necessarily the most impressive thing to have on your resume.

Good luck!


What are my chances in texas?

I'm an asian Texas resident. cGPA for tmdsas is 3.83, sGPA: 3.88. First mcat was 503 (130,122,125,126) 2nd MCAT 507 (127,125,128,127). Applying to all texas schools

casedentalmed

Numbers won't be the only thing that determines your consideration, but you probably have a good chance provided you have solid recommendations and have proper maturity and perspective on the profession.


worried about my stats

Hi Guys I am 29 yrs Old African immigrant in the US, I had poor GPA 3.23 at a community college. I was working fulltime and taking care of my siblings and mother during that period it was a challenging task to keep my GPA at above 3.6, two years ago I transferred to a my state University and worked hard I currently my GPA stands at 3.69 with a mcat score of 513 do you guys think I can make it to an MD school in the states? considering my poor gpa in community college. sGPA 3.51

casedentalmed

It's best to talk with some admissions staff at the schools you are most interested in applying to. Knowing more context about your education and your journey to the US may be helpful when it comes to viewing your application.


Should I mention my mental illness as a non-traditional applicant?

I graduated May 2016 from a BSN Nursing program and applied to medical school last cycle (2015-2016 school year) with a GPA of 3.06 and MCAT of 514 (128 Chem/Phys; 128 Crit. Analysis; 129 Bio/Biochem; 129 Psych/Soc). I pretty much waited until the last possible second to get my secondaries in. I was not invited for any interviews, much less accepted by any medical school.

My GPA is low because I had been dealing with depression during undergrad but refused to accept it at the time. It built up at the end of junior year and I ended up failing a (nursing) class, not because I didn't know the material, but because I hadn't shown up to class enough times. I sought professional help that summer and finished out senior year which included 3 hours of class plus 30 hours of clinicals weekly, also retaking the class I had failed and passed with a C+.

Since graduation I've been working on an ICU floor and have distinguished myself among my fellow new hires. Ever since I've actually addressed my depression, I have been doing well. I believe that my high MCAT score as well as work as an ICU Nurse are evidence that I have the pre-requisite knowledge and the fortitude it takes to go to medical school. Will adcoms see it this way? Should I mention my mental illness in an "additional information" section secondaries?

tantacles

If you are using your mental illness as an excuse for poor grades, it won't work. However, mental illness, when properly integrated into a personal statement as a positive part of your ability to identify with patients, can be helpful. Just keep in mind that it's best not to give anyone the idea that you won't succeed in medical school; that will make them much less likely to accept you.

That being said, your GPA is very low for MD schools; I would suggest that you take a year to reflect and take some more undergraduate level courses to boost your GPA. Your nursing background is definitely helpful, but you ultimately have to show that you can handle the academic rigor before applying to medical school.


Medical School admission

Hello, I am a premed student and I have a lot of questions and even though I have found some answers here I would like a more personalized answer.

I did my BA in psychology and then figured I wanted to do psychiatry. I took some of the requirements while doing my BA and I am now enrolled in a second Bachelors that I don't plan to complete, I just want to get the prereqs done. I want to apply for med school this coming June but by then I won't have taken orgo nor biochem, is there a chance to pass the MCAT without those two? Also my GPA was 3.8 when I finished my BA but I think by the end of this semester it may have gone down to 3.5 or 3.6. I did a year of research for a child anxiety and phobia program at my University, I volunteered at a Hospital (but just a few months), and I work at a mental health institution doing psychosocial rehabilitation which has allowed me to get exposure and work directly with SMI patients. I am also a member of Psi Chi the International Honor Society in Psychology(not sure if this helps). I want to know what are my chances first, to pass the MCAT without orgo and biochem? Should I shadow a psychiatrist or is working with patients enough? I live in FL and most medical school here have an average acceptance GPA of 3.7, will I be able to get in if my GPA goes down to 3.5? Any recommendations to make myself a string applicant? Any advise will be appreciated.

Thank you.

tantacles

I would strongly suggest that you take the time to finish your pre-requisites before you take the MCAT. Applying to medical school quickly is not usually the best tactic. Take the time you need to guarantee you need to do as well as possible in all of your classes, and take the time you need to study for and do well on the MCAT. There is no rush; medical schools will still be there if you take a year off, and you will still be able to get into medical school. Please apply when your application is at its best and not before; you will very likely have a better result if you wait.


Can I go to med school in the future if I'm majoring engineering with gpa 3.2?

Hi. Right now I'm a senior college student majoring computer science with gpa 3.2. Is it too late to consider med school? Short story is, I had my dreams to go to med school before I came to college but my mom, who is a med doctor, opposed very hard. Also, I was on f-1 visa at the time so I decided not to pursue. Now I am on green card and as my graduation approaches, I just could not imagine myself working at companies coding 8 hours a day, and I have been having bit depression at the thought that I've chosen the wrong path. If I do one more year + summer, I might be able to get the BA. in biology and complete the necessary course lists, but my gpa for cs major is running at 3.2 now. I'm not sure how much my gpa would get better, but this is the situation. Do you think it's too late, to think about the other path right now? I have my offers and I can graduate next semester with cs degree, but I feel like I won't be too satisfied with my career.

casedentalmed

It's certainly possible, but you may want to look very seriously at postbac/masters options. That would be your best route given your current situation.


What are my chances and what can I do to improve them?

I am looking to apply to med school in about two and a half years and am currently completing an MBA, having already completed a Bachelor's in Politics. My grades in undergrad were mid to high B's and in grad school are now in the A- to A range. Currently serving in the Royal Canadian Air Force.


Nailing the MCAT is a strong part of my plan to compensate for the low undergrad scores and my lack of any science based education since high school but what else can I do? What volunteer work would best complement my file?

What else can I do and what are my chances?

tantacles

I would suggest you do four things to improve your chance of getting into medical school:

1. Do well on the MCAT

2. Do some research, particularly research that may lead to a publication

3. Do some volunteering (any is fine)

4. Get some experience in the medical field (with contact with patients).

Those along with a strong academic application should put you in a strong place to gain acceptance to medical school.


Test Anxiety Advice and Advice on Future

I am primarily nervous for attending medical school for one main reason. I am willing to do all the studying, but I struggle with taking tests. I have a history of being anxious since a young age. It has gotten better, but I have never been a guy who got all A's and never studied. I need to put in the time and even if I study actively, I still often cut it close in terms of my grade.

I have gotten all A's in my Science classes throughout my undergraduate year and have about a 3.95-3.97 due to going to a community college for 3 years and then finishing up 3 semesters at a 4-year college. I've gotten an A in Biochemistry and tougher classes, but it has been tight because I often do not start out the best. I am not trying to brag, just say that I have been successful in my academics but still feel something is missing to take it one more step further to do better.

I have learned a lot but still am concerned about test taking in medical school. I get 1.5x extra time on exams and in classes where you can only get a few wrong on tests I struggle to get an A. Even in Microbiology which is supposed to be easy. I actively study according to Cal Newport but often get stuck on like 8-10 questions on the exam due to wording mainly. A few questions I do not expect but I have learned to do spaced repition. If anyone has any advice for individuals that struggle with test taking, I would appreciate it. I know I can be successful, I just need to change my approach, which I have found is key to studying.

I have one more Microbiology exam left and want to learn some skills for general test taking to do better in the future. I still need to take the MCAT and am nervous about taking longer exams in my undergraduate years, and if they will allow this in medical school.

casedentalmed

While I'm sure there are many people that become doctors that have test anxiety issues, it is a harsh reality to know you will have to get used to being examined often to maintain your license to practice or to progress in your career. I hope you have taken the time to seek learning specialists who can perhaps give you tools and advice, because medical school is warp speed compared to undergraduate (which is faster than community college). Seek the help you have available for you because the stress of medical school, residency, and beyond will not help you adapt if you cannot cope with new ways to deal with your anxiety now.


Applying to medical school in June

Hello,

I am writing today to ask about my competitiveness to medical school and if I should be doing something differently than what I am now. I have a 510 MCAT (129, 126, 128, 127) and a 3.57 cGPA and a 3.5 sGPA. Don't think I'm turning to many heads with that? Currently - In Thailand teaching English, Math, and Science to 7th graders, is this good gap year experience? Would this be seen as favorable? I would plan on staying here until March or August In College: 370 hours volunteering in hospital doing mundane patient care tasks (changing linen, delivering water and the like) - 1.5 yr 300 hours in the same hospital doing leadership (managed a hospital floor of volunteers) - 1.5 yr About 400 hours volunteering in a sports medicine clinic at my university, learning about sports medicine and athletic training (although I didn't really get to do much) - 1 year Researched in a psychology lab, have been published in 2 papers (not the first author, there are about 7 authors on the paper, and I am the fifth) - 1 year Research in a biology lab working with zebra fish, I did not enjoy this - 2 quarters

Have one institutional action taken against me --> Had to study for a midterm, my apartment was very loud, went to the library which requires we present our college I.D.'s. Didn't have one at the time, used my friends, got caught ----> although this has now been removed from my academic report.

What do you guys think, do I have a shot somewhere? Should I be doing something differently at this point in life? Thank you for the help

tantacles

You seem to have a competitive application based on statistics. If you apply, I would recommend applying to a broad selection of schools at which your stats are around the median or above it using the MSAR to narrow your search. Be sure to submit your application as early as possible to ensure your best chances of success.


Unbalanced MCAT, F on transcript, Late application, Should I retake

I got a 509 MCAT but it is really unbalanced C/P 128, CARS 123, BIO 128, P/S 130. cGPA is 3.79, sGPA is 3.66. I applied late in the cycle (secondaries complete in October) to a lot of MD schools (25). My in-state schools are pretty competitive. I have some basic research and extracurriculars but nothing that stands out. 50-100 hours research, volunteering and shadowing. Strong letter of Evaluation. I also have an F on my transcript (online sociology which I retook for an A). Should I start preparing for an MCAT retake? Its been pretty much silence from the schools. Is there any way to make the CARS 123 look a little less scary? I have an English professor who just agreed t write me a letter and I'll be updating the schools with it, but I wonder if they will even look at it...

tantacles

It is hard to say the exact reason that your application is being overlooked, but it looks as if you have several strikes against you - applying late, an unbalanced MCAT, and an F on your transcript. Retaking your MCAT and reapplying next year is probably a good next step. If you haven't already, purchase the MSAR and make sure you're applying to schools where your scores are at or above the median.


Should I do an SMP, formal post bacc, or unstructured post bacc?

I have a 3.4 UGPA, 3.0 sGPA, 519 MCAT. Good research experience, including first author of a publication. Little clinical/volunteering experience- I spent a week in a medical mission trip in south america. I missed this cycle of applications, and want to spend my gap year as efficiently (strengthening my weakest points in as economic a manner as I can) as possible. Would an unstructured post bacc, while getting clinical experience/volunteering, over the year be the best option, or would a formal post bacc or SMP be better?

tantacles

An unstructured post-bacc, if you can do well, would likely be the best way to improve your application as it will improve your GPA along along with giving you a chance to improve your extracurriculars. Your medical mission trip was only a week long and thus will probably have little impact on your application.


I am a junior never taken a ap or honors course is it possibleto be a radiologist if I go all out ap senioryear also I have had 1 year of spanish

I have all A's for all high-school years I have not been in lots of clubs, but I have been in some in sophomore and in my current year (junior). I have never been in a chemistry class but been in biology, general sci1&2 and algebra 1&2 2 is what I'm taking now

tantacles

The first step to being a radiologist is gaining admission to medical school, which will depend on your undergraduate GPA. Focus on getting the best grades you can in college and take all pre-requisites, and you'll be well on your way; radiology is a field within medicine, so you'll have to get through four years of undergraduate and four years of medical school to figure out if you are competitive.


Need Efficient Gap Year(s)

I am graduating from the University of Notre Dame this year with a low science GPA (3.0) and low overall GPA (3.4). I have not yet taken the MCAT or finished physics or biochemistry because of unforeseen circumstances. I am trying to figure out what would be the best way to get into medical school quickly. Would it be better to do a SMP? a normal MP? or should I just take a year of classes such as upper level sciences, finish my pre-reqs, focus on MCAT? I am very torn by the many options and I don't know what would be the best course of action. Please help. I also feel limited by what SMP or MP I can apply to because of my lack of pre-reqs and MCAT score.

tantacles

While I understand that it is easy to feel rushed, it is best to take your time and approach things slowly if you want to have the best possible result with regard to getting into medical school. I would suggest that you improve your GPA with a formal post-baccalaureate program, which may cost less than a master's degree (which will have no effect on your GPA) and take the MCAT when you are prepared and have done several practice tests and know your likely score.


A friend cheated off my labs without my consent and got caught, how will this impact my chances at med school?

A friend asked to look at my labs to verify his format was correct. I had him promise me he wouldn't look at content as that is the part that the labs are graded off of. I was a bit concerned, but he'd been trustworthy and never seemed the type to cheat. Anyhow, I gave him temporary access to my labs to verify his format. It turned out that he'd straight up copied my labs. I took the class last semester and he took it this semester. I was called into the academic Integrity office. I answered their questions honestly and they gave me Academic Integrity Probation and a paper to write. The woman in the office interviewing me assured me it would not be an issue for me besides that I would have to acknowledge what happened in the interview and that it would not impact my odds of getting into med school. Reading through the papers after the fact it appears this is listed as academic dishonesty, even though I never copied anything and this was done without my consent. I also read up on Integrity violations and it seems the med school apps require disclosure on the first page. I read many pages on how badly applications are impacted by records of plagiarism and cheating, but I am not sure how it is affected by being cheated off of. Trusting someone like this was very stupid of me and I am concerned it has ruined my future. How badly will this look for applying to med school?

casedentalmed

If you are being honest and you have support from your Academic Integrity office, you shouldn't have a problem.


Medical Education and Disability

I am a non-traditional medical school applicant applying to med school to practice psychiatry. I also happen to have a disability. I am trying to get a better grasp of disability accommodations in medicine. My disability is not physical, I can do everything a non-disabled individual can- except that, I suffer from extreme fatigue, a side effect of medication I have to take for my condition (psychiatric condition). This is not something I can change by trying a different treatment (I've already tried everything and it is a chronic long term issue) and I am committed to medicine, so I am not willing to go in a different field.

In order to get through medical school and residency, I would likely be requesting an accommodation consisting of a reduced course load (maybe one class less per term), while in medical school. I would also be requesting reduced hours during residency (likely around 35 per week). In psychiatry, the average residency work week is under 50 hours/week, so I would not be working that much less than full time. (I will also likely receive some accommodations to take tests in a semi-private setting due to some ADD type issues, but I don't foresee that being an issue).

I am trying to find out whether there are any other people who in medical school ended up taking a reduced course load and who finished in 4.5 or 5 years instead of 4 and whether there are other people in residencies who are working slightly reduced hours (for medical reasons or otherwise).

My main purpose is to get an idea of how such an accommodation request would be viewed during my medical education and to find out how rare (or not rare) such accommodations are (EDIT: I recently found out that about 10-15% of peds residencies are advertised as part time which is a good sign).

I am particularly worried about getting through medical school with a reduced course load but having problems finding a residency, due to having taken longer than 4 years to complete my education (while I know there are laws to protect against such discrimination, there are also stories out there of ppl who were not able to practice medicine as a result of not finding a residency, directly related to their taking longer than 4 years to graduate).

Let's assume my grades, USMLE scores and everything else would be good- the only issue would be that I would need to have the reduced course-load. (my past educational history is good- JD/MBA, high GPA, etc).

My sense is that the accommodation would be granted by medical schools (as per ADA) but I may have to explain to residencies why it took me slightly longer to finish school, if they asked (although I would not have to disclose the disability or the accommodation request until after I am accepted into the program). I am less clear of what would happen after you are accepted as a resident. Technically, as your employer, they would have to grant your "reasonable" request for less hours unless there is a very good reason not to (and decrease your salary at the same time). So it may take a little longer to finish the residency (which is OK). But given that this type of situation is not common, I am a little apprehensive given the investment of time/money and would welcome any feedback.

tantacles

Many medical schools are willing to make accommodations for disabilities, and it sounds like your disability, if well documented, will qualify you for some sort of accommodation. The main difficulty would not be in getting a medical school to grant you time off but rather getting through difficult rotations, like surgery, OB/Gyn, and internal medicine, where you often work more than 80 hours per week as a student. However, if you are able to get through that part of your medical education, you will likely be able to get through residency as well. I would be forthright after your acceptance about the kind of extra time you will need so that your school can accommodate you appropriately.


I'm stuck between two options for my undergraduate degree.

I am currently a sophomore at a large state university with access to all premed essentials like volunteering, shadowing, pre-med clubs, pre-med advising etc., but I have the opportunity to go to a very, very small undergraduate college in California to play tennis. I'm torn between these two schools because I'm not sure which one will put me in the best position to get into med school. The large state school provides me with almost everything I need but the small school would allow me to play tennis (and I could volunteer, shadow etc. in the summer). I know that in some cases athletics are viewed as a bonus but my main question is what will look better when applying to med school; a well-rounded student from a large university, or a student-athlete from a very small school?

tantacles

My suggestion is that you go to whichever school you feel you will do better at in terms of grades and having time to study for the MCAT. School name does matter, but your academic performance and performance on the MCAT are by far the most important factors when applying to medical school.


MCAT prepartion tips while in school and working part-time.

Hello,

I am currently in my first semester of my junior year and intend to take the MCAT in June 2017. I go to school full time and work part-time. I'm in need of advice on how to study efficiently during the spring semester with my somewhat busy schedule. I plan on taking off the month of May to study for the MCAT, but I feel studying for a month isn't a sufficient amount of time. Any tips or advice would be greatly appreciated.

Thank you.

tantacles

It may be that you need more time to study for the MCAT, but the only way to determine this is to actually take the MCAT. To get an idea of how much time you might need, I would suggest that you take a practice exam and see what your baseline is. Then, don't take the exam until you're scoring at your goal or at least at an acceptable place to get into medical school. If this means that you need to take an extra year off to get in, please take that time; taking a year off is extremely common, and you can use that time to relax and enjoy a year before going to medical school, which is a grueling experience, and you can get a job that will allow you to potentially build up your resume for when you decide to apply to residency.


competing against ivy- league students during admissions

I go to an average university as a bio major. is it possible to match up to ivy league or top tier undergrads when it comes to med- school admissions? how ? what can i do to improve my chances?

Pathdocmd

It is what you do with your time and how you take advantage of the resources of your university that makes a difference. Think "big fish, small pond". If there is something you want to do that isn't at your school (e.g., biomedical research, internship programs) then use your summers to go elsewhere to get those experiences. Each class at my school has alumni from 55-70 different colleges and universities... many I never hear of before starting to work in admissions. If there is a college I am unfamiliar with, then I go online and learn about it. Diversity comes in many forms and having a class made up of all alums from the Ivy League or the "Top 10" liberal arts schools would be going against that principle.


Is it getting too late?

I have high statistics (LizzyM=76-78) and good EC's and great letters but no interesting life story that many top applicants seem to have. I have one IA but it was a minor offense five years in my past. I have been accepted to one state school and WL'd at a top 40. I have not heard from any other schools besides a few rejections and am wondering if you guys think I will get any more II considering it has been 3 months since I was complete? I applied to 30 or so schools and my list was pretty top heavy.

Pathdocmd

It is not too late. Most schools are still sorting though the avalanche of applications that they get in July, August, and September. My school (Wake Forest) schedules interviews one month before the interview date. There are lots of people in our "to interview" list that haven't gotten the call yet and probably will not until January or later.


Asking for Admission Timeline

I have been accepted at a school that I will be thrilled to attend if I don't get into my first choice. My first choice school mentioned notifications not going out until December or later, and a large deposit is due at the accepted school around December. Would it be inappropriate to call the first choice and ask if it would be possible to hear back from them before the deposit at the other school is due? If I were to get into my first choice I would definitely be attending there and withdrawing my application from other programs.

tantacles

I would say that this is not an appropriate request. The school likely has this system because of a large volume of applicants, and they need not be in any rush to accept anyone; don't draw negative attention to yourself by asking for an exception. I would suggest you pay the deposit at your first school and either forfeit it or have it returned when you withdraw.


Is it too late to submit applications (10/26/16)?

It's October. I have submitted 5 DO secondaries and 10 MDs. I have low stats (27 MCAT, 3.3 cGPA, 2.8 sGPA) I have good LORs, ECs, work experience, non trad. Non-URM but minority nonetheless (female, hispanic, immigrant, LGBT)

I am sitting on two rejections and no interview invitations. I know in this game no news is good news but I am wondering if I should apply to a few more DO schools. Or that is money wasted at this point.

tantacles

Your mcat score and gpa and science gpa are probably keeping you from getting interviews. Rather than spending more money on this cycle, I would suggest spending the rest of the year improving your application by taking more science classes and studying for a retake of the mcat if you want to have your best chance of getting into medical school.


Better opportunity for reapplicant- Scribe or MPH?

For a semi-nontraditional reapplicant, what experience would look better, getting an MPH or dedicating that same amount of time to becoming a ER scribe? I haven't received any interviews this cycle yet and am starting to think about plan B. I'm 2.5 years out of college, working full time in healthcare IT. I have ~180 hrs of meaningful hospital volunteering and a 3.6/505.

Any help is greatly appreciated!

Pathdocmd

It is still early in the cycle. If you got your application in late (i.e., before August) then you still may still hear from schools. That aside, do what interests you and were you think that your application is weak. If you think that you need more clinical experience, then being a scribe is a good way to go. You could possibly do that while keeping your IT job. If you are really interested in public health then get an MPH. If not, don't do it.


Should I apply for Osteopathic medical schools this cycle?

Hello I wanted some input. I'm debating on whether I should apply to DO schools this cycle considering it is late. I graduated in August with a BS in psychology and minor in Chem. I have an overall gpa of 3.75 (haven't calculated my sgpa). My mcat score is low at a 499 (125,122,124,128). I have 80 hours of shadowing from 2 physicians (1 MD and 1 DO) and was apart of 2 clubs in college for long periods (3 years). Just wanted to get your guys input to help me figure out if I should apply or wait until after my mcat retake. Thanks so much!

tantacles

It is always best to apply only once to medical schools and only when you are truly ready. A score of 499 is not amazing, and you should not apply with that score if you intent to get acceptances to MD schools. I would suggest taking the year to strengthen your application and applying next year.


Interview question

For interviews, I was curious how most med schools view it if an applicant is somewhat over talkative. Is that a really negative thing or is it okay if you are giving good quality answers but going a little long?

Thank you so much for any insight!

Pathdocmd

It can be a real negative. If you dominate the conversation or the interviewer can't get a word in, that does not look good. They are going to assume that you will talk to patients the same way.


Is it ok to take all premed classes at community college?

I'm planning doing an associate of science degree at CC then transferring to a university to finish my bachelors. If I do my premed courses at the CC will this hurt my admission chances to med school?

casedentalmed

The Medical School Admissions Requirements should identify the handling of community college credit at each medical school you intend to apply to. Some will not accept any, some will not matter. I think given the competitiveness of the pool, taking all of your premed courses at a CC is not going to place your application in a desirable position.


MBS Tufts masters program

Is it hard getting into the program for MBS at tufts or do they accept most people who want to go pre med field and can pay ? Can you start in the summer or it's only fall?

tantacles

The Tufts MBS program can only be started in the fall. In addition, the program has an application process and does not accept all students who apply. I would strongly suggest that you contact the Tufts MBS program to get more answers. Some basic information about the program is shared by the program at the following link:

http://publichealth.tufts.edu/Academics/MBS-Microsite/MBS-Frequently-Asked-Questions


is it a bad idea to take OchemII, Biochem I , physics at once?

I'm a non-traditional applicant. I was a chemistry major then I went to to nursing school. I just graduated this May.

I took some of the pre-reqs years ago. I have BioII, Ochem II, Biochem, Physics I&II left. My GPA before nursing is 3.98 and the nursing GPA is 3.78. (both at a state university)

Is it a bad idea to take Bio II, OchemII, Biochem I , Physics I all at once next semester and take the MCAT in May 2017 ?

I was going to give myself 2 years and apply for the 2018 cycle. But now I want to save one year and apply for the 2017 cycle. Any advice?

Thank you!

Pathdocmd

That's a rough schedule and I have seen applicants fall flat on their face with a schedule like that, but it depends. Are you working at the same time? Family? Volunteering? Also studying for the MCAT? You sound like a strong student, but it is really your call if you can handle it.


How would medical schools view me if I transfer from a UC to community college?

I am miserable here at my UC, I am giving my best effort and I am not doing as well as I'd hope.

I'd like to transfer to community college and then transfer to a private college or CSU after.

This UC is also very expensive, and I don't want to have to get loans. What should I do? I am very stresssed.


Would this decrease my chances of getting into medical school?

I want to be close to my family. I am lonely here at UC Davis, I want to be with my friends and family. I want to be home.

tantacles

Transferring from a four year college to one that is not as rigorous, like a community college, would likely decrease your chances of getting into medical school. However, if you are able to transfer back into a four year college, there should be no problem provided you do most of your medical school pre-requisite courses at the four year college AND do well in them.


Could you help me by possibly endorsing specific schools/institutions who could help me to resolve the issue?

Hello! I'm in the Army and going to apply to F.E. Hebert School of Medicine, Uniformed Services University. I'm almost do with my science prerequisites (career changer). Although, I was granted my Bachelor's and Master's overseas. With that I was told that in addition to having my transcripts evaluated (I already have that done and never got a problem with that before), the school would need to have a U.S. Degree Granting Institution actually award me a Bachelor’s degree based on my transcripts; the institution would have to be accredited to do so. I, also, was told that there are some schools out there that do this quite a bit for the military members.Could you help me by possibly endorsing specific schools/institutions who could help me to resolve the issue? Thank you very much!

Foreign Transcripts - U.S. Degree - Military Medical School

casedentalmed

I don't know which institutions you got your degrees from, but unless you took those classes under a program administered with a US institution, I'm not sure how you would be able to do it. Otherwise, you may have to talk to some transfer counselors at local universities to see if any of your coursework could be applied towards a bachelor's degree in some major area they offer. You might have to take some additional classes in order to fully qualify due to general education requirements for graduation.

USUHS did strike an agreement for a postbac program with George Mason University in Fairfax/Manassas, Virginia. I'd check with the program directors of the EMDP2 (https://www.usuhs.edu/emdp2) about this.


AUA vs SGU (pre-med)

Which medical school is better to attend in the Caribbeans: American University of Antigua or St.Georges?

tantacles

I would strongly recommend against attending any Carribbean school. Your chance of getting a residency slot is very low unless you do exceptionally well on Step 1. I would recommend that you wait another year and apply to MD and/or DO schools. If you are not currently competitive for those schools, I would recommend you take time to improve your application.


More than 1 IA's, Med School DOOMED?

Hello, so currently I am a sophomore undergrad and got a possession of alcohol ticket by the police the very first weekend of my freshman year. I finished the differed prosecution agreement and my case was dismissed, although the alcohol edu classes that I was required to take for the university still remains on my disciplinary record. About exactly a year later, I have been charged with possession of marijuana and marijuana paraphernalia, though I haven't had my case heard yet. Most likely I will have to go through a probation period of 6-12 months, and assuming I don't do anything else wrong during that period, I will still have this on my Institutional Action record as well with my alcohol charge. I am working to expunge both my alcohol and marijuana charges from my criminal record with a lawyer before I apply for medical school, but I know that I still have to disclose on my application that I have had 2 IAs during my first 2 years of undergrad. I know it seems bad, but I have completely changed my life perspective around and am trying to go sober immediately. I have good grades (3.5 GPA, (3.5 Sci GPA), but I know I can do better as my GPA is mainly hurting due to 2 classes where I got low grades (C in Chem 101, B- in advanced physics 118). I know that 1 IA is not the end of the world, but what about 2 different charges (considering I have no criminal background record due to expungement of charges)? And also, what is my best course of action to begin to prove that I am changing my habits? I also know that it is best to write about how it changed me and how I learned from the experiences, but I was wondering if there was a way to prove that I have changed (i.e. pass mandatory drug tests, letter from dean saying I have worked hard to change myself, etc etc). All input is greatly appreciated! Thanks!

Pathdocmd

Two does not help, but you are not doomed. Now, I can ONLY speak from my experience, so I don't know how other schools will view this. Your marijuana possession, combined with the two alcohol violations, would be a MAJOR problem. If you got any other violation, I wouldn't even apply. All of those things to prove that you are changed are fine, but time and a clean record will be the best proof to any committee.


If anyone went abroad for their gap year, how did one deal with interviews?

Hey everyone! So I am taking a gap year for sure before heading to medical school, assuming I get accepted. For my gap year, I really want to join the workforce, get a sense of what the "real" world is like. And I think the best way to do it is for me to work, hopefully abroad. Because of premed requirements and other issues, I could not go abroad. I study Japanese though, so I really want to go there. The problem is that I talked to my advisor about it and he mentioned that doing interviews this way would be bad. He also seemed to support the abroad if I was teaching in developing countries (I did not mention my interest in Japan so he said something like Nepal).

Also, has anyone volunteered abroad in a hospital? I don't have a lot of volunteering hours (have around 150). I am going back to volunteer this year though. I think it would be cool to get to see the healthcare system from a different perspective.

Really appreciate everyone's responses.

casedentalmed

While I don't think it matters where you do your international experience when it comes to teaching, the real question is that you have to figure out for yourself the importance of sacrificing what could be a year of your work abroad for interviewing. There are some but not a large consensus to accommodate applicants who decide to do something overseas during a gap year. You could have an option of a Skype-like interview, but don't bank on that. Otherwise, you'll just have to plan ahead for the few weeks of time you want to anticipate having interviews and schedule your travel within that time block. But in such a process, you can ask how much a school can accommodate you, but don't expect too much of the process to be flexible to you.


Unbalanced MCAT and no committee letter...

Hello, I would really appreciate it if you could give me some advice. I took the July MCAT and received a score of 512 with the following breakdown: 127/131/127/127. I have a 3.8 cGPA and sGPA.

My extracurriculars are 1) I wrote that I will have about 400 hours of clinical volunteering by matriculation to med school (if I matriculate in fall 2017), 2) 1 year of research (no publications), 3) 45 hours of shadowing 1 doc (I forgot to put this info on my AMCAS, so I emailed schools individually about this or uploaded a pdf file about my shadowing experience to the "additional material" file in my secondaries, 3) tutored both English and Chemistry in 2 different centers (for 2 and 1.5 years, respectively), 4) taught a college literature class, 5) do dancing as a hobby (performed at 1 concert), and 5) graduated with honors in my major (English), which I also forgot to put on my AMCAS and just ended up emailing schools about or uploading pdf files detailing my experience

I didn't get a committee letter from my college because it has a new rule that prevents people who haven't had an active file with the pre health office for at least a year from getting a letter. I can't get it next cycle either, because I already graduated.

My question: Do you think that I am competitive for MD schools this cycle? Most of my applications were complete in August. If you do not think so, could you please give me advice on how to strengthen my application for next cycle? Should I retake my MCAT? (It is weirdly unbalanced. I have a really high CARS score and average science scores…) and will my forgetting to include my shadowing and honors experiences in my primary affect me negatively? Will the fact that I didn't get a committee letter affect me badly? Sorry for the millions of questions. I appreciate your help very much.


Thank you so much to anyone who actually had the patience to read all of this!

tantacles

You are competitive for MD schools this cycle provided you have the right letters. Whether or not you will get interviews I can not say. I would recommend that if you have no interviews now, you start working on a backup plan.


Bad interviewer- report? or too bad?

I recently had an interview at a MD school, which uses the MMI format plus one, traditional, 20-minute interview. My traditional interviewer (and only adcom member that I interviewed with that day) was very odd. I expected him to try to get to know me- asking the traditional "why medicine," "why here" questions, but he asked very few questions about me, and spent at least half of the interview failing to meet my eyes and talking about his kids. It was very strange and very frustrating, since this is my top choice school, and I wasn't given an opportunity to talk about that. I'm worried that I didn't have a chance to make much of an impression and that he won't have anything to say about me during the adcom meeting, because he didn't do, what I understand to be his job: getting to know the applicant and seeing if they are a good fit.

My question is, what's the best way to deal with this? Tough luck? And hope that I can write a convincing LOI when I end up on the waitlist? Is there a way to reach out to the school without coming off like a whiny, excuse-making premed? Could this have been a very odd version of a stress interview? Any advice would be greatly appreciated, because, other than this odd encounter, I love the school!

Pathdocmd

That is a tough one. It might have gone better than you think, but you never know. I would wait to see how you did (accept, wait list, reject). If you get in, no problem. In fact, I would send a letter to the Dean of Admissions about this person after you matriculate. If you are on the wait list, I think you are stuck. If you get rejected, I would inquire. Not knowing anything about the school, I would imagine that the MMI and metrics (GPA, MCAT) part has much more weight than the personal interview. I have a feeling that the Admissions admins know about this interviewer's personality and style.


Clinical Hours - Not a lie but will adcoms see it as such

I will have 2000 hours of clinical experience when I enter medical school, which is what I put on my application. I work at a psychiatric hospital and work on avg 26 hours a week and will be there from 1 year and around 9 months by the time I am accepted. Are adcoms going to look at this and believe I am lying because 2000 hours is such a high number?

tantacles

If you actually worked 2000 hours, there is no reason to be worried.


Interviews

Hello, I was complete in mid July to early August at most schools. I have received three interviews, although none at my top choices, and two rejections from bad fits in late August and nothing since. I have a LizzyM of 78 and solid EC's although nothing spectacular. I am just wondering, is this it for me or should I receive at least a couple more (I applied to ~25 with 10 or so in the top 30 according to US News)? Normally I would be incredibly grateful to have 3 but I am applying at the same time as my significant other and nothing has overlapped with her potential cities so far.

tantacles

It is impossible to say whether you WILL receive more interviews, but I can say with great confidence that schools are still offering interviews and that it is likely, assuming that your application is strong, that you will receive more interviews.

Pathdocmd

It takes a very long time to read all those applications. I'm expecting at least 9000 this year and we are still sorting through the ones we got in early September. Some that have been singled out for an interview may not hear until January or later.


AMCAS Courses

Good Evening!

I am currently a sophomore in undergraduate school. I took a year off of uni for military reserve training. The Preprofessional Biology Studies major has changed in the past three years. For the first year, the requirement for degree was Zoology and Botany. I took those two classes my freshman year and made a C+ and D+ respectively. 2 years later and Botany is not a requirement for my degree. It is now listed as a 'course not applying to this program.'

Will AMCAS still record by D+ grade for a class not applied to my major?

Thank you.

tantacles

AMCAS records all grades that were taken as college courses; your d+ will count towards your gpa.


Interviews for Med School

I submitted my secondary applications in the end of July and was complete at most of my schools between 7/26 and 8/3. To date I've gotten 3 rejections (2 of which were from far reach schools) and no interviews. I've been put on hold at one school in my target range. I have not heard back from any others, and I applied to 28 schools in total. I was told by my pre-med advisor that my application pretty much matched the average matriculant at low/mid-tier schools (LizzyM~69-70) and my school list seemed pretty well-balanced.

Is this normal? Should I start mentally preparing to execute my back-up plan?

tantacles

It is early, and many applicants have not received interviews yet. Your application statistics are appropriate, but I can not advise you specifically without knowing the schools you applied to.


MCAT Score Retake effect chances?

I initially took the MCAT and received a 492, my retake wound up being a 500. I know a 500 is only an average score and isn't very competitive. I was wondering though if the fact that I had to retake the MCAT will negatively impact my chances of acceptance, or if schools will look at it from the positive viewpoint that I doubled my percentile score with me retake?

tantacles

Your retake will be viewed negatively compared to if you had gotten a 500 the first time. That being said, your mcat score is not likely competitive for MD schools, and I would recommend another retake if you wish to matriculate to an MD school.


What should I become?

OK, so I'm currently doing my A levels. I'm a fairly smart student, planning to go study in my country (the US) next year. I don't know what I want to become. Many say engineering but I don't like plenty of maths and physics. Parents want medicine but it takes a lot of years and it's not easy by any means + medical school is expensive on top of college fees and most importantly many say MCAT is impossibly difficult? Well, my favourite subject is Biology, but I don't want to do something seemingly a bit minor like microbiology due to low salaries. All-in-all, your two cents on this?

tantacles

There are many careers for those with biology degrees. First and foremost, I would talk to a career adviser about your options.

If you wish to work in research, many with a degree in biology choose to pursue a job as a lab technician. You could do this either at a biotech company or in an academic laboratory.

You could also work in clinical research as a coordinator.

In addition, there are many fields for graduate study available, including dentistry, podiatry, medicine, physician assistant, and pharmacy. It would be useful to think about what you are looking for in a career and what level of education you hope to have, and that will likely lead you to your best career option.


Should I Do an SMP, MS, or just apply to DO Schools given my background?

I will try to keep this as brief as possible but it gets complicated:

-Arrested in 2004 for possession of marijuana at age of 18: Basically, it was a friends bag and he didn't admit to it, so everyone was arrested and charged. I am looking at getting this expunged under Youthful Offenders. -Moved with Father and started college in 2006 and to be quite frank was immature and had no clue what I wanted to do. We had some family issues going on as well. Moved again in 2008 and was focused heavily on staying financially afloat and again put academics on the back burner. Have quite a few WF's from withdrawls. -Moved again in 2013 to buckle down, focus on academics, and get my bachelor's degree. I graduated in Dec of 2015 with 2 bachelors degrees (biological psychology and biology) with 3.771 (considered Summa Cum Laude). Founded a student club and was President/VP for it and President/Vice President for another student club as well. Also won numerous academic awards. This was at a University attached to a Chiropractic College which I originally came here for but found Chiropractic wasn't exactly for me, which is what I originally came here to pursue after my bachelors. I have always wanted to be a Physician and interested first and foremost in medicine, so I explored different aspects of medicine including Dentistry. I was a Dental Assistant for a Dental office for 3 months in my last quarter of my bachelors and a Dental Assistant Volunteer for a low income health clinic for a year. At that same clinic I was able to shadow a DO and volunteer a few times on the medical side and LOVED it. I really love the DO philosophy, that is where my heart is.

So, my graduating GPA from Life University is a 3.771 but it is quarter hours as opposed to semester, so not sure how this plays with grade replacement in AACOM. I do know when combined with my other transcripts from other schools that 3.771 sinks to ~3.0. Given my background, should I do an SMP, a MS, or assuming I do well on my MCAT (I take it in March), should I just apply to DO schools? Do you think I have a chance?

tantacles

If your GPA is below 3.0, you have very little chance of acceptance at any US medical school, though your upward trend is reassuring. My suggestion is that you take science courses in a post-baccalaureate program to raise your GPA until it is above 3.0, making sure you get A's in as many classes as humanly possible (read: all of them). At that point, take the MCAT. That will give you your best chance of getting into medical school.


can i take science classes again after graduation to boost up GPA before applying for medical schools

i am senior year student and my GPA is really low to get qualify for medical schools but i will be graduating this fall. can i take couple science classes after my graduation it will count for medical score require GPA ? i really dont know what to do

tantacles

You are absolutely able to take extra science classes to boost your GPA. However, two words of warning: if you are trying to improve your gpa, make sure that you take undergraduate courses that are not part of a graduate level degree. Graduate level coursework does not count towards your AMCAS GPA. Second, remember that upper level courses will improve your gpa the same amount as retaking your other basic science courses, so consider broadening your education by taking something new.

Pathdocmd

Many applicants take extra classes to boost their sGPA. However, if you GPA is really that low, a post bacc program of graduate or med school level courses may be necessary.


How are new medical in the U.S.A looked upon by residency programs?

Hi, I need some assistance if a new medical that opened recently in the U.S. such as 2015 or 2016 is considered as good quality for future residency programs? This includes if the medical school is accredited. I do not want to offend any new school or anyone, I understand all medical schools are treated equally but some opinion is appreciated if newer medical schools are less appraised. Thank you!

tantacles

It is unclear how new medical schools fit into residency matching, particularly at more selective residencies. That being said, students from new MD schools don't seem to have trouble matching, but how their schools are viewed is only really known to residency program directors. If you wish to go into a competitive specialty, I would advise that you go to a school that has that specialty if possible so that you can get exposure to research in the field.


Confusion About Med school Prerequisites

I am currently a student at a community college in Washington state in nursing. I was looking into prerequisites for medical schools and alot of them require "one year of each of the following: biology, Organic chemistry, inorganic chemistry, and physics."

For biology I am fine I have 2 ANP classes, Cell bio and microbiology under my belt however the next 3 get tricky. In my school classes are offered in one quarter increments very few have class series besides English and math. My school only offers one kind of physics class, which is only one quarter, 2 kinds of organic chemistry which is only one semester and 1 kind of inorganic chemistry. unless I take multiples of the same class I will not be able to achieve the one year requiremen's at my school and due to already being in my current program I simply don't have the time to go to a 4 yr college to get the missing prerequisites without waiting an extra year to apply and I don't want to do that.

Will my being 1 quarter short on organice chemistry and 2 quarters short on my physics and inorganic be a big problem?

casedentalmed

I would check with the schools that you have strong interest in. Many of them may not accept community college coursework, others a limited number of course credits. Repeating coursework would not help you, especially if you are trying to plan on taking the MCAT later on.

I'm not sure what I would do in your situation. You say you don't have the time to transfer into a four-year university, but you have to understand that the vast majority of applicants are university graduates with bachelor's degrees or graduate degrees who have not just taken the prerequisites, but have also gone way beyond the requirements. Would being short on your prerequisites be a big problem? I honestly think if you wanted a real chance for admission to medical schools, the answer would likely be an honest yes.

But to encourage you, there have been many nurses who have gotten into medical school. They found a way to get those courses and other recommended courses in, and showed themselves to be desirable candidates for medical education. Their experiences are really needed when it comes to building a class that is truly empathetic towards their patients. But the requirements must be fulfilled, and there are plenty of candidates that an admissions committee sees that do it and more.


How do med schools view consulting?

I recently received an offer to work at a consulting firm after I graduate in 2017. I will not know what my project is until I start, so there is a fair chance it will not be related to healthcare and medicine. My premed advisor advised against consulting saying that med schools would view this as a lack of commitment to medicine. The physician I shadow however has encouraged me to accept the position because she believes true work experience of any kind will be viewed positively. How do med schools look upon consulting during a gap year?

tantacles

The physician you shadow is correct. While you should certainly seek out medical experience, having varies experience is important, and I would suggest you take this job if it's something you're interested in.


Do schools require all your transcripts, or just the common application?

Do grad schools require you to submit all transcripts, certificates, etc., or do they just look at the AACOMAS application?

I know they would require an updated transcript from current school, but was wondering about all of the other ones. Basically, I made a mistake, a small one, one that my friends don't think is worth submitting corrected info for and delaying everything. Nothing to do with GPA. If I leave it as is, and the application gets bounced back, does that automatically mean I'll look bad, or does this sort of thing happen all the time? I know schools can be notified if there's a discrepancy, just wasn't sure how often that is or how serious it is. Is it a case by case basis or pretty much a coup de grace on an application? I'm already applying late. Just don't want to make it even later.

tantacles

You must submit a transcript from every single school you attended to amcas.


Are high transcripts required for medical school admission

So im a immigrant to USA. I had already completed my high school in Fiji before I migrated. Right now im in a community college trying to my biology degree. I got in the college through doing compass tests. Currently in plan of transferring to a university. I have my high school transcripts with me, but they would like the transcripts to be send directly to the school from the high school. But since all the original transcripts were already given to me i dont how that is possible. I talked to the university and they have agreed to consider my admission based of my college scores. But what i want to know is that is there any chance of me getting into a medical school without any result or transcripts of my highschool or is that im just wasting my time right now doing pre med

Pathdocmd

Medical schools do not look at high school transcripts.


How does AMCAS weight a Health Sciences class compared to the traditional Biology, Chem, Physics, and Math (BCPM)?

I have a high GPA with my Medical Lab Sciences degree (3.97). But once I factor in my previous degree, my GPA is only 3.4. Do health sciences classes factor into your science GPA or not?

tantacles

BCPM refers to biology, chemistry, physics, and mathematics. If the primary content of one of these courses was in that category, it counts as bcpm. If it was not, then it does not count. I can not answer specific questions about your courses as I am not familiar with them, but it is possible that they will not count depending on their content.


Do medical schools take into account grades from abroad and community colleges equally?

When I began undergrad, I was told that any courses that I took from community colleges would only transfer in as credits and would not effect my GPA. That is true, they did not effect my GPA on my transcript, however after applying to medical school I realized that they would be incorporated into my AMCAS GPA. I'm so upset because I graduated with a 3.90 at my university but with the community college courses and abroad courses, I now have a 3.79. Will medical schools take into account that I received B's from the community colleges and abroad (in non science courses) and not at my home university? From my home university transcript, I have all A's, 3 A-'s, and one B. But my abroad/community college grades added 5 B's and 1 B-. My science GPA is a 3.89.

tantacles

All grades from courses taken at domestic universities and universities abroad are counted in the exact same way. As the other responder mentioned, your GPA has not dropped to a point where you are in any danger of rejection solely based on your GPA, so I would not fret over this small decrease in GPA.

Pathdocmd

Medical schools, in my experience, look at the AMCAS calculation for GPA. A drop from a 3.90 to a 3.79 is nothing to worry about.


How should I explain my retroactive withdrawal?

The questions are at the end. Sorry in advance for erring on the side of too much information.

I had an upwards trend in my GPA (3.0, 3.4, 3.7) until I experienced a major depression. (At the time, I didn't know why I was depressed; I now attribute my depression to a growing realization of my homosexuality.)

In the fall semester of my fourth year, I became severely depressed. I could not bring myself to get out of bed, and I failed a class. In the spring semester I saw a psychiatrist, who advised me to withdraw from the spring semester and take a retroactive medical withdrawal for the fall semester. I am currently thinking about how to explain these semsters. (NB: I took Spanish 1 and 2 during these semesters, so I think it will be obvious that the first withdrawal was a retroactive withdrawal.)

My GPA at this point was 3.4. I felt immense shame withdrawing from 2 semesters of college, and it was difficult for me to recover. I returned to college after taking three semesters off, but I probably was not ready to return. I came out to my parents after I returned to college and they told me that they didn't want to speak to me again and that they would remove me from their will. I was living with my parents at the time and was forced to move out and withdraw from all classes. When I finally finished college, my GPA was 3.35. (I received two C's in courses required for my major; Only one professor in the department taught these courses, and he is infamous for only giving a single A every 3-4 years. And yes, I know that does not count as a valid excuse for making a C.)

Obviously, I would simply get an MS in medical science if I had the financial resources and that would solve my issue. I know that an MS program would be my ideal next move, but I don't think I could convince any of my relatives to cosign the student loan bill. I feel that I am over my depression because I have come to terms with my sexuality, but I have no proof of this from an academic standpoint.

Instead, I have worked a year after college as a scribe. I have taken the MCAT and scored 514 (92nd percentile). I also have over 700 hours of volunteer experience (caregiving for dementia patients; providing HIV tests on a medical mobile unit).

I have now completely finished my primary application but I am at a crossroads and cannot hit "submit" yet.

My question is two-fold: 1 - Is it unreasonable for me to apply to medical school at this time? (Interviews will much of my money and I'm financially independent.) 2 - *Most important* How do I explain the withdrawals to put me in the best possible light? I have read over and over on SDN that I should avoid disclosing depression at all costs when applying to medical school. I have also read over and over that I should not reference my sexuality. I'm trying to think of another explanation for all of my withdrawals, but I'm having a difficult time. Do you have any specific and/or special advice?

tantacles

It is possible that given your gpa and mcat combination that you can be accepted to medical school. It is my suggestion that you apply to both MD and DO schools as that likely represents your best bet of gaining an acceptance on your first try.

With regard to your second question, you can absolutely mention major depressive disorder as your reason for your withdrawal, and the most important thing is that you mention that your mental illness is now well controlled. If you wish to mention it in your personal statement, I would remark on how it has given you insight into mental health.

With regard to your sexual orientation, it is wonderful to mention your sexual orientation if it connects in some way to medicine. Many medical schools (Yale, upenn, northwestern to name a few) actively seek out sexual and gender minorities as they add a significant amount of diversity to a medical school class. I'm not certain where you read that you should not mention your sexual orientation, as there are many threads on SDN that suggest exactly the opposite (the lgbt applicant thread, for example).

On a personal note, as a former applicant and now medical resident who is also a gay male, I mentioned my sexual orientation in my interviews, and I was accepted to several schools when I applied. Being lgbt is no longer a mark of shame and is something to be acknowledged and proud of, including in medical school applications.


Do I have a chance this cycle?

cGPA 3.36, sGPA 3.1, MCAT 496. Do I have a chance with MD or DO this cycle?

tantacles

You have a chance for DO, but it is a very slim one. MD is almost certainly not happening unless you have a very lenient state school.

I would suggest you retake the MCAT and apply again for the next cycle. If possible, I would also take more undergraduate science courses to boost your GPA's.


G.E.D. exam

Im 18 and still a sophomore in highschool due to stopping a year and moving from another country, my counselor advised me to go to adult school, but they also said that taking G.E.D.s is another way, so were planning to take the test this year, im planning to take bs nursing, then pre med, then proper, will the G.E.D test affect my chances of going in to medical schools? And this means I dont finish american highschool.

Pathdocmd

No. In my experience, med schools never look at where you went to high school. It is all about how you did in college.


Pre-Med work/volunteer advice

As an undergrad I was a history major(class of 2000) , so I have no exposure to bio or Chem. I will begin a pre-Med course of studies at UCLA Extension in 2017 while I continue working FT in a field unrelated to a medical profession.

Is it therefore advisable to search for opportunities to shadow a physician, or assist in a lab now or can this be undertaken a bit later on as I complete a few pre-Med courses?

tantacles

The best way to pursue this so that it looks good for medical schools is to look for experience early and participate for a few hours each week as you go through your pre-requisites. That way, by the time you apply, you have shown a long-term commitment to medicine. You don't need to volunteer a tremendous amount each week, but showing longitudinal activities that demonstrate altruism and medicine is important.


Adversity Secondary Essay

Hi,

I am facing my secondary applications, and I am currently most concerned about the question about a failure in my life that I've overcome. My initial thought is to put down my frustration that while taking my core classes for my major, my grades slipped consecutively from B's to a C, the first one in my life. However, I have also heard that things such as these are not really considered adversities, and that admission committees might even scorn these kinds of issues, believing that if bad grades were the worst adversity of my life, I have serious issues.

So my second option was to go for the fact that I commuted by public transportation for all four years of my undergraduate career. Round trip, it often took me average of 4 to 5 hours every single day. But that also seems trivial. Moreover, the factors that led up to this decision to commute all four years seem like I brought this upon myself. For example, most people advocate leaving the house and learning to live independently, but I chose to stay with my parents not because I don't want to take up responsibilities, but because I feel obligated to stay involved with their life. I am also probably financially capable of paying for dorming or renting an apartment near campus, or even paying for parking for four years, but personal consideration led to deciding that those fees were too high that as a family, we weren't ready to pay for them.

So I guess my ultimate question is, would my first response about low grades be looked upon in a poor light, because if asked about something I seriously care about where I honestly think I failed, I think that's what I see most important to me. But I also know that I have to choose topics wisely and I'm more than willing to pick something else if that would be seen in a more positive light.

As a last resort question, would leaving the question blank (it was optional, but gave me 3000 characters worth of space) be even more detrimental to me?

Many thanks. I wish I could respond and thank you again after you spend your time answering my question, but I couldn't find a way to. So thank you so very much for your help and time in advance.

tantacles

Your answer about grades isn't extremely helpful; being frustrated about grades, while a personal struggle, isn't particularly unique and doesn't give any extra information about you; admissions officers can see your grades, and bringing attention to your poorer grades will not help your application in the slightest.

Commuting is certainly a struggle, and talking about how commutes affect you might make for a compelling piece, but to be honest, it doesn't strike me as something particularly moving or special. There are people who actually can not attend college because they can not afford it, so it just isn't a tremendous sacrifice to commute when some people are sacrificing far more.

If you truly don't have something to offer, leave the question blank. If you can think of something better, add it. But it's better to leave the optional question blank than to answer with something that would make you seem spoiled.


Is my GPA good enough for med school

Hi, I failed out of college my first year with a .121 GPA. I took a year off, and completely turned my life around. I met my premed advisor, ad he told me to forget about medical school, but that just encouraged me all the more. Since then I have got an A in every class I've taken and my GPA is currently a 3.9 including retakes, and a 3.67 without retakes. My premed advisor is still telling me that I will have a tough time getting into med school because I messed up my first year. Is that true?

tantacles

Medical schools are very interested in candidates who have upward trends, and your course seems like just that. Your GPA is great for medical school, and the only thing that will determine your chances is how well you do on the MCAT.

There may be some schools that are not thrilled with your performance, but I predict there will be just as many that will be intrigued by such an amazing turnaround.


Ipad and Laptop, which one is effective and useful for medical students?

tantacles

Both can be very effective for medical students.

Ipads have the advantage of being lightweight and portable, and a keyboard can be attached to make them easier to use.

Laptop computers have more versatility and memory, but are harder to carry around depending on the model.

Personally, I had both, and I found both useful.


How best to list my Military Medical service on AMCAS

I was an Army Medic for 4 years. I medically retired as a Corporal. During my service I worked in several capacities many of which included closely operating with Army PA's (Both with MS's and MD's) in several environments (Base Clinic, Battalion level Aid Stations, Line Company Medic, EMT Medical Coverage/Ambulence, Flight Medic and in Combat) and with several specialties. I am confused as to how exactly I should list this experience. I can see it as Clinical Hours, Physician Shadowing, Military Service, and even just Leadership... Should I split it up into all those catagories? How best can I do that? What levels of my medical practice would actually count as clinical hours? Should I estimate all the way down to my time on Coverage or in Combat?

tantacles

The military, if you were paid for your services, is paid employment. That is the only way you need to list it in terms of raw categories.

That being said, if you had multiple positions, each could be listed separately, but they would still be listed under paid employment as you were paid for your work. If there were leadership positions involved, you can list those in the description and discuss how you showed leadership.


Bs/md with fullride and med school scholarship or ivy league for undergrad?

If choosing between a bs/md program at an university one likes but is a lower tier with a scholarship for all of undergrad and part of medical school, or an ivy league (about 80k loans will be needed)...which is best? Med school admission will not be binding but if applying out will lack of ivy education hurt in the md cycle?

tantacles

I would strongly suggest you take the combined BS/MD program if you know you wish to be a physician. If you are a fantastic student, all doors will be open, and you will have far less debt and fewer expenses than your peers. In addition, you can limit your medical school search to schools you like more than your medical school you are currently accepted to. These benefits are tangible.

Oh, and don't forget: A bird in the hand is worth two in the bush. There are people with amazing applications who are never accepted to medical school. You are guaranteed a spot, and it would be wise to take it.


MCAT after secondaries?

I have taken the MCAT twice for consecutive scores of 510 and 507. If I potentially decide to retake, does it work to indicate this on AMCAS and take the MCAT after already submitting secondaries? Or is it better to not retake at this point at which I have nearly received secondaries from all schools I applied to?

tantacles

I believe it would be a poor idea to retake the MCAT with good scores having already retaken it once and done worse. You have two acceptable scores, and these scores should make it possible, assuming your GPA is acceptable, to get into medical schools provided you apply broadly.


Can I get into med school after low gpa in exchange program?

I'm an URM and go to a small private college where I was able to do an exchange program my junior year at USC. I had a rough junior year and my sGPA dropped significantly. I did some calculations and the highest my sGPA can reach after senior year classes is a 3.4. Have I ruined my chances of getting into med school? Does it look bad that my grades dropped once going to a more competitive university and then my grades getting better once returning to my home college for senior year? Also my first semester at USC was a 2.13 cGPA and second semester was 3.15cGPA. Do you think admissions committees would notice the improvement or is it just all bad?? Any advise or thoughts would be very helpful:)

tantacles

It is still possible to get into medical school, but it is impossible to tell your chances without knowing your MCAT score. With an MCAT score of 515 or above, it should be possible to offset your GPA. Recognize, though, that a score of 515 is very difficult to attain, and it would be a good idea to take multiple practice tests to find out where you stand before investing resources in the actual exam.


Confused about gpa for med school application

I did 4 semesters of summer school at a university near my home. However, I go to a different college otherwise. I know I still have to submit my summer transcript, but my regular university accepted my grades differently. Would the aamc only look at the grade my university put or the original grade where I took the summer courses? Thanks!

tantacles

Amcas will only look at the grade from the other institution.


How to be qualified to work in physiology with M.SC. aboard

I have completed B.VSc. In Iraq and recently done .MSC. medical physiology in India then I got back to Iraq and have no future of doing any related job the only thing which I did, is giving lectures at medical schools without any payment around three years in past and working as medical representative at other side to live. So I would like to have a precise answer about how to be qualified to work upon my master background outside of my country and have permanent residency in such a peaceful country. Please give me all details about sponsorship or any contract. Thank you in advance.

casedentalmed

I think if you define work as teaching, you probably can do some teaching at high school or community college. Lecturing at medical schools is more difficult but working in a lab is not. Non-academic positions are possible (pharmaceuticals, medical writing, etc.).


Chance of Interview and When?

Residence: California Ethnicity: Asian/Pacific Islander 1st MCAT: 26, 55th percentile (8 PS / 8 VR / 10 BS) 2nd MCAT: 516, 95th percentile (129 PS / 129 CARS / 131 BS / 127 Psych) cGPA: 3.62 (upward GPA trend) sGPA: 3.54

2 years volunteering at major hospital 2 research lab courses 2 weeks shadowing Graduate from top 3 public university

Verified 7/18/16 Sent in most secondary applications (15) late July - early August

What are the chances I receive an interview invite? When should I expect interview invites (if any) based on when I sent in the secondary applications?

Thank you!

Pathdocmd

That all depends on what schools you applied to, details of your experiences, quality of your essays & letters, etc. From what I have seen so far, the interview invitations are just going out. My school, Wake Forest, has not sent out any as of today. I'm sure that they will be coming in soon.


Need Help ASAP Choosing between grad schools

So I'm a little bit lost as to where to go for grad school. I'm the first person in my family to try to go to medical school. As such, they don't really have much advice for me or seem to understand how hard it actually is to get into a medical school in Ontario (I live in Canada). Anyways, I have a pretty underwhelming overall undergraduate GPA of 3.4. My first year of University was absolutely brutal. After changing my program, my grades increased dramatically and I brought my GPA up to a 3.9 in my last two years. However, I should mention I completed my undergraduate degree in Nutraceuticals. I have always been involved in unique extra curricular activities that involved a significant leadership role. I also played on an intramural soccer team and worked throughout the last years of my degree. I have also done a ton of research (worked on two undergrad projects & won an award for the highest grade achieved in these classes) & have a publication as a third author. As such, I thought it would be best to complete an MSc or potentially a PhD to bring my overall GPA up a bit.. Also I'm writing my MCATs for the first time in August so I won't know my score for that until September. So I have been accepted to McMaster University, Calgary University & Western for grad school for a Masters thesis program in Medical Science. All of the programs I was accepted to are in relation to cancer research, I'm just not sure which to pick.

McMaster: - great lab in hospital - advisor has multiple papers in Nature & I will have an opportunity to have multiple publications - 3/5 grads have gone to med school at Harvard - theres an opportunity to participate in rounds at the hospital for every department - animal research working with knockout mice, similar to what I did in my undergrad so it won't be as steep of a learning curve - I can apply to Macs med school with a reference from faculty at the school (not sure if this really that important though) - Downsides: I have to move to Hamilton, the advisor isn't a doctor so its not as good of a reference & I have to TA which takes away time from focusing on school and bringing up my GPA.

Calgary - really nice lab mates (when I went to visit they took me out for sushi & shopping) - also a great lab in a hospital - advisor is an oncologist = great reference - animal research but focuses on animal surgery which is very interesting and as close as I can get to doing actual surgery for now... - If I go here I have the option to apply to two extra medical schools as the GPA lowers to a 3.3 for Alberta residents. - Calgary's med school only looks at best three years so I would actually have a competitive GPA for applying. - I don't have to TA so I can focus on bringing up my GPA. - 5/5 grads gone to UOC med school. - Downsides: Calgary is incredibly far from my home town (London) & rent is going to kill me financially

Western: - great advisor who is a clinician = great reference - I can live at home = save $24 000 in rent money compared to Calgary - in a lab in a hospital with opportunity to participate in oncology rounds - animal research focusing on knockouts, similar to research I have already done. - Don't have to TA so I can focus on bringing up my GPA - Lots of opportunities to apply for external scholarships! - Downsides: unfortunately with my GPA I can't apply to Western's medical school so I'm not sure if its a bad idea to go to grad school somewhere I can't even apply to their medical school at?

ANY HELP would be greatly appreciated. I feel terrible keeping all these schools waiting for me to make a decision. I think I would prefer to just stay at home but Im very worried I'm missing out on applying to two extra medical schools in Alberta if I don't go there. Any thoughts on my chances of getting into med school somewhere in Ontario? Sorry this post was so incredibly long!

casedentalmed

It is difficult for anyone on the internet who doesn't know you to steer your decision. It sounds like you have written down your considerations for each move, and that the question is one that seems to be more with the support you need outside just your classes. In the end, you just need to be in a supportive environment where you can properly focus on your courses to the favor of your faculty who you want to support you in your next application. If you haven't already made your decision, I wish you the best of luck sorting everything out.


IS THERE ANY NO MCAT PATHWAY TO US MEDICAL SCHOOLS

Hi, I am a junior Biochem major. I have 3.79 cGPA, 3.72 sGPA and reasonable shadowing, volunteer and research experience with presentation of research work at three different symposiums. I am really scared about MCAT since I can't score good on the practice exams no matter what I do :( I was just wondering would there be any way to get into US med school using a non traditional route to medical school?

Thanks in advance!Any help would be highly appreciated!

casedentalmed

Pretty much any AMCAS or AACOMAS program is going to require the MCAT if you are applying as a regular applicant. There may be some early acceptance programs that might waive the MCAT requirement, but you probably should have already gotten in by your sophomore year.


Named a school in AMCAS primary app.

I accidentally said, in my disadvantaged AMCAS essay, that "I hope to work hard and succeed at XXX school." Will this destroy my chances at other AMCAS schools that review my app?

casedentalmed

Most applicants are expected to have some idea to proofread and double-check all of their application responses, and lack of attention to detail is significant to varying degrees to different people who might look at your application. It's a mistake that you should own if you get called for it. Hopefully, it's a mistake that you can laugh at later if you get more than one interview and hopefully a subsequent offer.


Work experience for career changer pre-med applicant.

What kind of work should I be doing while getting my prerequisites, that will enhance my med school application? Will entry level jobs hurt my application or enhance it? I am 30. I have 10 years experiences working in surgery on cell saver devices that recycle the patient's blood, owning and managing the company that provides these services, as well as involvement in the regulating agencies' committees and volunteer assessor position. I also own a separate company involved in the clinical training for cell saver. I now want to pursue a medical degree. I will be spending 2016-2017 full-time getting prerequisites from a community college and taking the MCAT. I am financially stable enough to not work for that time, but was told that this would hurt my application. It was suggested that I get a part-time job during this time to show that I can handle the workload of part-time job and full-time school. The part-time job would need to be something entry-level that is the grunt work in the industry. I was also told that I needed more patient experiences, so something like medical assistant, phlebotomist, or medical scribe would work. Should I get the entry level part-time job? Will it hurt me if I have the big difference in job responsibilities? or should I spend that time volunteering and shadowing?

tantacles

Entry level jobs, in and of themselves, will not hurt or help your application. Any medical or research position, however, will most certainly help your application. If you wish to simply show that you can handle the workload of medical school, I would suggest you have one goal: Do as well as you can in your classes. However, as you do this, I would suggest you volunteer in a clinical setting (no need to do this for more than 2-3 hours per week), get experience shadowing physicians, and perform some research. If you need more medical experience, you need not get a job; volunteering is completely acceptable.


mentioning kids in personal statement

I have three kids, all under 5. I withdrew from two classes because I was put on bed rest when I was pregnant with my third child because I had severe complications. my question; is it wise to mention my kids in my personal statement? ( I am married and in my early 30's) if not, how do I explain the W's ? in any case lying is not an option, so I'm officially confused! plus my GPA went sideways when I had my last baby, went from 3.8 to 3.5

tantacles

W's generally require no explanation, and your GPA is acceptable for medical school, so I would not mention your children in the context of your performance. However, I would mention your children if they played a significant role in your choice to pursue medicine as I would find a story about complications of pregnancy told from the first person to be quite compelling, particularly if accompanied by above average insight.


Is there a benefit to taking the MCAT in certain months ?

I am taking a gap year after undergrad to work on strengthening my med school applications, but I will be ready to take the MCAT at the end of August (applying for 2018 cycle). Is there any advantage to waiting a little longer to take the MCAT when it is not prime test taking months (April-August),?

tantacles

It is not known by anyone other than those who administer the MCAT whether there is any distinct advantage to taking the MCAT at any particular point. I would recommend, above all else, doing multiple practice tests while you prepare and simply taking the test when you are ready, even if that means you have to delay your exam and wait a year to apply.


Rural Medical School

What is it like going to a rural medical school such as Dartmouth? Is it considered better to go to a medical school in a more diverse environment? Also, what kind of student typically thrives at smaller medical schools? Thank you for your time!

tantacles

Rural medical schools typically focus less on urban populations, as you would expect, and more on the difficulties that rural populations encounter. It is hard to generally classify these schools as each has a different mission and serves a different population, but they largely seek students who wish to stay and practice in their areas. In addition, it is hard to speculate as to who would perform well at these schools; a good student tends to do well at any school.


Changed my career path at my sophomore year. Is it to late for a career in medicine?

Hello, I am about to begin my junior year in college and during the spring of my sophomore year I decided what I really want to do of my life. Before I was sure I wanted to get into a med school and graduate with a biology degree. Now I know I want to graduate in psychology and become a physician assistant. It is important to point out that I am also a student athlete. I play soccer in order to make school more afordable to me.

The problem is that I am kind scare of not being able to achieve all the prerequesites by the time I graduate. PA schools ask about 2 years of healthcare experience (which I have none) and a lot of volunteer work (which I also have none). But the good thing that I have is a good GPA. Right now I am considering chasing my healthcare experience during the summer but it will be far from the 2 years requiered.

I would like to know what are some of the possible routes that I could take in order to get into PA school after right graduate. Also, am I in a really bad shape for a aspiring PA?

Thank you!!

tantacles

It will be very feasible for you to enter PA school, but without the pre-requisite experience, it will be difficult to gain admission; PA schools are becoming increasingly competitive. I would recommend that you consider taking a year off; this would allow you to improve your application in a relaxed way and get the full clinical experience you need to gain an acceptance; many students in health professions schools are now 2-3 years out of college.


Does working in a nursery count as clinical experience?

I just started working in a church nursery with infants and toddlers and I'm wondering what category that would fall under on the AMCAS application. Can it be considered clinical if I'm hoping to become a pediatrician or neonatologist? In what ways will it benefit my application?

casedentalmed

Usually a "clinical experience" will be located in a clinic or hospital where patient files and information are needed to serve patients. It is better classified as volunteering in an extracurricular activity sense. Will it benefit your application? I'm sure it might, depending on what experiences you have in that nursery.


how will college accredited military course work affect my gpa for medical school?

Hello,

I have a substantial amount of college accredited course work through the military, specifically the community college of the air force. Some of it was transferred to my 4 year university to assist in getting my degree. The rest of this credit went towards my associate degree in a different subject awarded through the community college of the air force.

I know these classes will be seen/required in the admissions process (40ish credits) However the only existing grade for them is an S (For satisfactory?) (Possibly on a S/F grading scheme however i'm unsure)

What would this S translate to GPA wise (all classes on my transcript are an S) if at all?

Thanks for any help

casedentalmed

I think the AMCAS instructions will confirm this. I think the coursework from the CC of the Air Force will not count at all towards your GPA. I'm not sure how the transferred credits will be calculated. You may want to contact AMCAS.


Pre med program

I have graduated in Biology then started pre med fall 2015 in Ohio . I had to move to CA, this summer. Deadlines for pre med in my new city are passes and I have to wait till next year. Should I continue my courses online or in community college here? What should i do?

casedentalmed

It is too difficult to know what classes you are taking. I think if you have moved to California, you have no choice but to try either online coursework or take CC coursework. You may want to check in with some of the admissions officers you have networked with.


Should I apply to med school during my one year Master of Health Science program?

I graduated from a great university, but with a 3.13 cGPA. I have research experience, but have not yet authored any papers and I think I can write a reasonable personal statement. I have attempted the MCAT once with basically no prep and scored in the 61st percentile, which I know, is less than passable. However, I know that if I attempt it again, I can definitely score in the mid to high 90s.

I got into a one year master's program at Johns Hopkins - which I think is pretty great considering my GPA. Now, I can transfer into a two year master's program that is research-heavy, or stay in this one-year program that is based mostly on coursework. I don't want to waste another year, but considering that MD application deadlines are before the year ends, I will only be able to show my grades for 2 or 3 terms instead of the full 4. I would like to go to a top-tier medical school in the US. Assuming that I attain a 3.8 - 3.9 GPA in the terms that I can show to med schools and ~95 %-ile on my MCAT, should I really switch to a 2 year master's program and spend another ~70K and a whole extra year? Will the reputation of JH help my chances?

Thank you for your time!

casedentalmed

You could apply, depending on the type of support you get from faculty in your program. Unless you're going to shoot for a highly competitive medical school, I wouldn't worry about the MCAT so much. I think the JH name can help you but I would not rely on it. You're going to need to be very strong with your prosaic components of your application, including your letters.


I accidentally put the wrong semester (somehow) for 19 of my courses in AMCAS and they were all changed and now have an X...is this going to be a red flag?

I somehow missed that for a lot of my second semester classes, I accidentally put that I had taken them first semester. AMCAS just verified my application and now I have 19 X's where they changed the semester for me. They did not change any course classifications or anything major...my GPA is exactly what I had calculated it to be for BCPM and total. Will this be a red flag to AdComs? Should I call the schools I'm applying to and tell them what happened if there isn't a space to explain in the secondary app? I feel like a moron.

Pathdocmd

I wouldn't worry too much. I would send a separate letter to each of the schools that you get a secondary from explaining the situation. I would not use up valuable secondary space for this. Use the secondary app essays to sell yourself and not draw attention to mistakes.

tantacles

I would not mention this mistake to medical schools anywhere in your application. It was an honest mistake, and not one that should have a huge impact on your application. Your courses still stand as they were, and this mistake seems not to have affected your application, so I don't believe that most medical schools would truly care.


Online Subtitling under Pseudonym

Hi,

I am applying for med school this year, and have been struggling with figuring out whether I should put something down as a work/activity.

I have been subtitling Chinese language dramas for a long time now on a website that streams Asian dramas. It has helped me grow in terms of understanding nuances of language and cultural differences, and I've learned a lot of leadership skills organizing the volunteer work. However, I have worked exclusively under a pseudonym, so it would be hard to find a contact person who can vouch for my volunteer work. I've even recently decided to start my own subtitling and international media distribution company because of my experiences working as a volunteer on this site.

So I guess my two-fold question is: 1. Will it be okay to include this on the application? Will it have a negative impact on my application? Could it potentially make a good impression to adcom?

2. If I do put this down as a work/activity, I'd like to put it down as Artistic Endeavor because it seems like the subtitles I create are being read by a large online audience. However, since I am working on all this stuff under a pseudonym, I think it would be really hard to find a contact person who can vouch for me with my real name. Is there any advice/suggestion for this? Thanks so much!

tantacles

I think that this is a completely appropriate activity to put on amcas, particularly since you've been so involved. Really, anyone you worked with can be a reference; it need not be a boss. However, I would work hard to find someone whose phone number you can give and to whom you can out yourself (meaning your real identity) without issue.


Naval Officer with undergrad in Ocean Engineering gpa>2.67

I'm currently a Pilot in the Navy with an undergrad in Ocean Engineering from the United States Naval Academy, where I was also a varsity athlete. My dream is to become a psychiatrist. My gpa isn't great and I'm willing to take a post ba premedical program. I have 5 years left on my commitment to the military and would like advice on how best to set myself up for medical school. I'm currently deployed but my next tour will be stateside where I will be able to attend school at least part time, and seeing that I graduated 5 years ago, I want to focus on a program that will get me back in the swing of academics while making me more marketable to medical schools.

tantacles

Given your gpa, a strong performance in a post-bacc program would be necessary in order get into medical school. If you are able to get a high gpa (meaning almost all A's), you would have a good chance provided you also get a good mcat score. Keep in mind, however, that these programs can be costly, so it would be wise to have a backup plan. Your experience in the military is a fantastic activity to write about in your application, but remember that it is no substitute for good grades, volunteering, shadowing, experience in medicine, and a good mcat score. To be successful, you want to have all of these things in place before you apply.


GPA Advice

I am an incoming Junior with an overall GPA of 3.47. My GPA drastically went down because of my first C+ in organic chemistry 1. I am currently taking the course again, in hopes of getting a better grade, however, it will most likely be the same. I plan to take the MCAT this January and apply that following cycle. What are my chances in getting an interview into medical school? I also have research (along with my own research project), research scholarship, volunteering, leadership (officer for a pre-med organization), and work (scribe) experience on my resume. Also, what advice could you give me to make my application more competitive?

Pathdocmd

If you do well on your MCAT and your other experiences (research, volunteer work, etc) are solid, then you have a pretty good chance in getting an interview. It would be foolish for a medical school to overlook someone because of one C+. Would be it be better if you had an 'A', of course. Do well in your other science classes and you should be fine.


School List

I have a 3.60 science GPA, 3.65 total GPA, 507 MCAT, was a Spanish major at a well-regarded institution, and a CT resident. Where should I be applying?

My current list: UCONN Quinnipiac UVA Tulane George Washington Georgetown Miami FAU BU Temple Drexel Thomas Jefferson MUSC Wake Forest Emory U. Southern California

NY Schools since I am a CT resident: Einstein Mount Sinai NYU NYMC

tantacles

Your school list looks, overall, good. Emory, Mount Sinai, NYU, UVA, BU, and USC are likely reach schools. I would recommend adding Rosalind Franklin, EVMS, SUNY Downstate, and VCU and possibly removing some of your reach schools.


Can someone give me some advice on how to improve my studying skills when it comes to the sciences? I am goung to be enrolling in Bio 1 this fall for the second time after dropping it last semester.

Study tips are what I need.

tantacles

Everyone has a different study style, so it is difficult to advise you, but here are some tips that helped me.

1. Take a break at least every 60 minutes. Almost no one has the attention span to do anything for more than 60 minutes, and a break can help you recharge.

2. If something isn't working, change your study style. This is something many people have trouble with as it requires introspection and self-reflection.

3. Ask for help. Your professor has office hours, and your TA's do too. Go to them if you you are having trouble.

4. Make sure that you do exercises that force you to test your ability to generate solutions and don't simply study out of a book without practicing. This is why professors often have practice problems: to force you to think.


Additional Information Essay

Multiple school secondaries have an essay asking for "additional information not found elsewhere in the AMCAS application." Is this an appropriate place to explain a GPA trend or below-median GPA/ sciGPA due to early lack of focus? I have an essay prepared detailing how recognizing my passion led to a tremendous jump from my performance as an undergrad (3.6 with 3.1 sciGPA) to my performance in my post-bac (3.9), but I'm not sure whether a) this is a good idea - will it look like I'm making excuses? or b) whether this is the best use of this essay prompt opportunity.

Any thoughts would be greatly appreciated!

Pathdocmd

The answer is.. it depends. At my med school, Wake Forest, the interviewers don't know your grades and MCATs, so they don't know that there is a problem unless you draw attention to it either in the essays or the interview. In my experience it does not help much to write about it. There is a fine line between making excesses and overcoming an academic misstep. However, if you know the interviewers will know your grades then it might be a opportunity to get ahead of them and try to set the tone of the conversation. I always tell applicants to highlight the good in the essays.


High School Conferences?

During high school, I attended both the National Student Leadership Conference on Medicine and the National Youth Leadership Foundation on Medicine. I am currently filling out my work/experiences portion of my AMCAS. I don't think each deserves it's own entry, but when I tried to lump them together, I was only given one date option. Any suggestions for how to enter these?

Pathdocmd

Medical school AdComs don't really want to see a lot, if any, experiences in high school. The exception would be something that you continued on into your college years or they are truly significant. If you still want to put them down then use two entries.


Any disadvantages to reapplying?

While the rest of my application is strong, I have an MCAT score that I am not totally confident in. I am debating whether it is worth applying this year, or waiting until next year in order to retake the MCAT. Are there any disadvantages to applying now, and seeing if I get in? If I don't get in, then retake the MCAT, do better, and reapply, will schools that I applied to in the previous cycle view me less favorably as a direct result of my being a reapplicant? Or will they understand that I reapplied as a result of having to retake the MCAT, and will view that improvement as worth considering? To be clear: if I have to reapply, I would be reapplying to the same set of schools.

tantacles

Most medical schools see reapplicants as a risk and have the view that not getting in the first time is a sign that something was wrong with the application. I would strongly suggest that you wait until next year to apply so that your application is at its absolute strongest. If you don't get in and retake the MCAT and do better, many schools might ask, "Why didn't you just wait until you had a great MCAT score?"


Will not applying to a DO school on TMDSAS hurt my chances into an MD program?

I am applying through TMDSAS to the 2017 application cycle but I applied to every single school in Texas but the UNTHSC School of Osteopathic Medicine. Will that hurt my chances into an MD program with a 3.79 overall GPA, 3.68 BCPM GPA, 510 MCAT, and well rounded extracurriculars in both the clinical and non clinical areas including research?

tantacles

Applying to one school or not does not hurt your chance of getting into another; Schools generally do not have access to applicants' other school selections and thus can not use that as a criterion for admission.


What to do with time off from school before application

I am not taking time off by choice, I was originally pre physical therapy starting college, which meant my school advised me to go down a route that did not require organic chemistry. I am also in the army and they want me to commission as soon as possible so rather than allowing me to take the time I need to finish organic chemistry before I graduate they are pushing me through and telling me that I can finish my necessary coursework on my own time.

With that being said however I did choose to go national guard, that way I have the freedom to go to school, etc. My main question is... what should I be looking to do with this down time in school? I should have a lot of extra free time as I will be taking only one or two classes. I would like to do as much as I possibly could to improve my application to medical school and just prepare myself in general. Is it advised that I volunteer, or try to work at a hospital, etc?

Thank you

Pathdocmd

Exposure to patients as a volunteer in a clinical setting can never hurt. If you have the time try to get a clinical job (e.g., patient transporter). You could also shadow physicians during this time. Can you get clinical exposure or some type of extra medical training through the National Guard? That would be worthwhile.


Chances of admission despite minor misconduct

My overall GPA is a 3.5 and my science GPA is a 3.7. I have yet to take the MCAT but I will be taking it in the next few weeks (for answering purposes assume I score in the 50th percentile). I am commissioning into the army national guard as a medical services officer and I currently run my own personal training company. I have a limited number of hours shadowing and volunteering but I will be taking time off from school once I graduate to gain more experience in those aspects of my application which I feel are weak.

The biggest issue I am worried about is that I have some minor misconduct issues from when I was younger in college. I was caught smoking tobacco in the dorms as a freshman (stupid I know) and I was charged with academic dishonesty one time in an attendance based class as a sophomore. The charge was not serious either, I was not present one day in class and someone accidentally or purposely signed on the space for my name. That day she held everyone after class and called the names that were signed in order to check the attendance, of course I was not there so she accused me of having someone forge my signature which I did not. Do you think these misconduct charges will have a large affect on my admission or do you think it would be something I could explain as a learning experience, or have removed from my record at my university? Also, with all of these factors in mind do you have a best guess as to which Florida medical schools I have a chance of being admitted to if any?


tantacles

While your tobacco issue will likely not affect you so long as you acknowledge it, the academic dishonesty issue will likely not be viewed favorably. Avoid explanations that deny guilt as medical schools will likely not care why the incident happened and will only care that you learned from the experience so that it does not happen again.

It is hard to say what schools will overlook these issues as every school is different, so I unfortunately can not advise you on that piece of your question.


More non-traditional student hopeful of acceptance at a DO school. Help please!

Hi Everyone,

I'm hoping you can help me. I've been preparing to apply to medical school for what seems like my entire life. I completed a human biology bachelor's program and my cGPA is a 3.6 while my science gpa is about 3.4. I have been studying to take the MCAT so that I can hopefully get above the average score considering my gpa's are about average. I plan on applying to only DO schools.

As far as experience... I was captain of my lacrosse team, I was president of the pre med club which did various volunteer activities in the community as well as being a member of a sorority which also added to volunteer experience. I was an EMT for 2 years and I currently work at a hospital as a technician. I'm trying to figure out my chances of getting accepted on my first round of applications going out in 2017. There are no D's or C's on my record, because I did have to retake organic chem one and two. I will be getting shadow experience in the meantime from surgeons and private practice DO's. Please, any advice is welcome.

Thanks for reading!


tantacles

It sounds like as long as you receive an appropriate MCAT score that you have a good chance of being accepted to a DO school provided you have good interviews. However, a poor MCAT will greatly decrease your chances, so do your best.


Post-graduation help with GPA

I just finished with my degree in sociology/psychology with a minor in Biology. I finished with a cumulative GPA of 3.54 and a science GPA of 3.15. Granted these were a little lower when I first applied, but I sent in update for my senior year. I am currently on the waitlist at a great school, but I am afraid I will not make the cut. The school I was rejected from without an interview said that I should work on my science GPA by taking some upper-level science classes. I am currently working full time as a clinical research assistant, and can continue this through my gap year, but I'm not sure how to go about increasing my GPA.

How are these usually done? should I take them from a community college where I will surely get an A, or does that not look as good as taking classes at a 4-year college? And are letters of intent actually helpful?


tantacles

You can take your science courses anywhere, but it's often better to take them at a University, even if you take them a la carte. Any university will do, so don't feel as if you need A's and tremendous rigor; just get the A's.

As far as letters of intent go, it varies from school to school. If you truly wish to be at the school you are on the waitlist for, I would suggest you write one, but just recognize that it may have little impact.


Discrimination based on position working on Executive Board of Gay-Straight Alliance

I worked on the Executive Board for my school's Gay-Straight Alliance for a year and I listed it as one of my activities. I'm not a member of the LGBTQ community, but I am an ally and I was wondering if AdComs may discriminate because of this? Can I focus on this experience for my secondaries? I felt like this was a very impactful experience and I don't want to leave it out if possible.

Pathdocmd

In my experience, listing LGBTQ organizations in the Experiences section or discussing it any essay (AMCAS or secondaries) has never been an issue. If you think working with the organization contributes to understanding why you would make a great doctor, then by all means use it. You can highlight your qualities of leadership, organizational skills, time management, compassion, diversity, grit, etc.


Physics Section on New MCAT

2017 MCAT taker here. I have not had taken much physics beyond Newtonian forces (no electromagnetism, thermodynamics,optics). I've heard that the new MCAT does not have much (if any) physics. A) Is this true? B) I'm hoping that my prep course (Princeton's) will teach me all the physics I need, is this ill-advised? Thank you so much for your time!

tantacles

It is my strong advice that, despite the fact that it is possible to study for and do well on the physics section of the MCAT, it would be wise to take both physics courses before taking the exam. Every school's physics curriculum differs, so it is impossible to tell if your school might cover something that might show up on the MCAT. In addition, while the new MCAT has fewer physics questions, it is still possible that it might cover advanced topics, but it is definite that your knowledge will be tested over fewer questions. Thus, it is important to have a good background in the material.


GPA <3.0 Please Help.

Hi, I have a GPA less than a 3.0 after graduating college. I have done a lot of reading in different forums and a lot of people have stated that they just retook the classes that they have done poorly in. Somehow, I do not feel that will work for me, being that my GPA is so low. I graduated with a degree in Biology and really want to go the premed route, and know that I have to boost my GPA way up, does anyone have any advise for me? Thank You in advanced. regards.

tantacles

It will be very difficult for you to attend an MD school as MD schools do not offer grade replacement, and a GPA less than 3.0 is almost certainly a death sentence. DO schools, however, will replace your grades if you retake courses. There are a few ways you could proceed:

1. If you want to go MD, take whatever courses you want to increase your GPA, including any pre-requisites, and do very well on the MCAT.

2. If you want to go DO, retake your courses you did poorly in (C's or D's) and do well on your MCAT.

3. Take your MCAT, do very well, finish all pre-requisites, and pursue a special master's program (SMP) as these offer direct linkage to many MD schools.

Answered 3 years ago by Pathdocmd (3060 points) I think that the best solution would to be a post baccalaureate program. There are many programs around the country, but they are different in set up and goals. Since you already have a biology degree and presumably the traditional premed requirements, I would suggest a masters degree or certificate composing of either medical school or graduate school courses. The AAMC has a webpage with a searchable database.

https://apps.aamc.org/postbac/#/index

Use "Academic record-enhancers" as one of your search criteria.


Valet Car Accident

I work as a valet attendant and I backed up into a column today with someone's car. I had to wait around an hour as I kept urging manager to tell him or her but he or she kept putting me on hold as it was busy. When I eventually told him or her, the police were called, as per procedure. The officer threatened to give me leaving the scene of an accident (5 points), failure to report an accident (5points), and some 2 point ticket for not being careful. These were all mainly because I "waited" an hour. I was luckily not issued any tickets but the officer did write a report mentioning that I did wait an hour after the accident, which I'm guessing is not too good. This was my first time ever dealing with something like this and really didn't know how to handle it. Will medical schools see this report on background checks or will I have to report this on future medical school applications? Is this considered a crime in any way? Could this hinder my admissions process in any way?

tantacles

Medical schools only require that you report any legal infractions, not warnings. You need not report these issues as they did not result in actual disciplinary action beyond a warning, which is not a conviction.


What can I consider a "non-science recommendation"?

I see many schools are interested in a "non-science academic/faculty" letter of recommendation.

I have two options I was interested in using but both are on the border of the definition. I have one undergrad professor who taught me in "exercise nutrition and weight control" as well as an independent study looking at research into ADHD and physical activity. This definitely does not fit into the hard sciences but I wasn't sure if ti would be accepted.

I also just earned my masters in public health in epidemiology and have worked with two epi faculty extensively but was never taught by either of them. Would either of their recommendations fit this description?

casedentalmed

Usually non-science means not in the category of biology, chemistry, math, or physics. Your courses sound like they work under the Health Sciences. Health Sciences also includes public health/epidemiology.

The listing is known as the "AMCAS Application Course Classification Guide", a PDF document that can be searched for in AAMC.org .

Your specific question on whether health science coursework could count towards a school's/admissions committee's definition for "non-science" should be directed to your schools, but it is likely those references could be considered "non-science."


Low end of 3.0 scale and didn't do too good in Organic Chemistry.

I would like advice from medical students on what I need to do to finish my undergrad successfully.

tantacles

It is difficult to say what you need to do to successfully finish your undergraduate as everyone's experience is different. If your goal is to go to medical school, a 3.0 is not an ideal GPA, and you can increase your GPA by doing your best to get A's in your remaining courses. If you have not been able to get A's in your courses, it may be time to seek help from an academic adviser and/or reconsider medical school. Many students find that it is their study habits that are to blame for their grades, so you may need to seek tutoring or some other assistance to do better in school from this point on.


Withdrawl from a class in freshman year has me panicking big time. Please help!

dropped bio 1 and lab during freshman year because of personal problems (the 17 credits I was taking was too much at the time). My sophomore year, I took Bio 101 (1 semester) because I was majoring in the social sciences and didn't need the longer 2 semester class. I have recently been thinking about medicine and taking prereqs (I am a junior), but am worried that the W my freshman year will make me look like I can't handle med school. Will this look bad to adcoms and lead to rejections? Or will doing well in prereqs amd on the MCAT make the whole thing a non-issue that they don't look twice at? Thanks so much for any help.

tantacles

W's are completely appropriate, and one W will not hurt your application to medical school in the slightest. Continue to do well in your coursework and you will be in a good position.


What is the difference between a bachelor's in Medical Science and MBBS?

I would like to know what the differences are between doing MBBS followed by an MD and doing a bachelor's in Medical Science followed by doctor of Medicine(MD). What are the difference in opportunities after completion of education? Would i be able to practice as a doctor if I study a bachelor's in Medical Science(3 years) followed by MD(4 years)?

tantacles

In the United States, there is no explicit difference between an MBBS and an MD beyond the fact that one is a baccalaureate, or undergraduate, degree, and one is a graduate degree, or one that is completed after college. That being said, MBBS is not offered in the United States as all medical schools are graduate schools. However, upon completing a graduate medical education program, or a residency, anyone with a medical degree may practice.


Post-Bacc and Nursing

Finished my Bachelors in Arts with a Psychology Degree. GPA 2.83 Grades for Pre-requsites

1. Biology 2A: C, Biology 2B: C-, Biology 2C: C 2. Chemistry 2A: C+, 2B: C-, 2C: D

Post-Bacc (not formal)

3. Organic Chemistry (Upper Division) 1:B, 2:A, 3:A ( at a 4 year) 4. Physics 1: A, 2: A ( at a 4 year) 5. Biochemistry A (at a 4 year) 6.Physiology A (community college)

My question is schedule set up that will continue to allow me to get A's. This fall I want to start studying for my MCAT it will be straight for four months do you recommend that I take anatomy and microbiology during this time? I am taking this for PA school (backup). As well as I will be taking Genetics, Cell-Molec ,Immunology, and retaking Chemistry during the Spring and Winter quarters at a four year university. And I want to complete an accelerated BSN program during the same time that I should be able to finish my end of fall or even late summer. I plan on working as a RN for 8 months prior to medical school and duirng my 2-4 year of medical school working during the weekends. Does this plan seem unrealistic or harmful to me doing well in courses for medical school as well as the MCAT.

tantacles

Based on your performance in your science courses, it is definitely worthwhile for you to pursue medical school. However, be aware that unless your cumulative GPA, including your post-bacc, ends up above 3.0, admission to medical school will be nearly impossible, and even then, I would suggest that it will be difficult without an extremely strong performance on the mcat to gain admission to medical school.

With regard to working while you take the mcat and courses, it is possible to do well, but only you can determine your limits. However, I would strongly recommend against working while in medical school; medical school is a full time job.


Thoughts on becoming an MD after RD?

I just graduated with my BS in dietetics. I will be entering a program where I will earn my RD (registered dietitian) credential as well as my MS in Human Nutrition. I have been thinking about pursuing medical school in addition to being an RD. Is it feasible to become both an MD and RD? I want to focus on an area that has close ties between nutrition and health, such as endocrinology. I graduated with a 3.79 GPA (science GPA is lower, around a 3.3 unfortunately), I have a variety of clinical, volunteering, and leadership experiences, and I graduated from undergrad a year early. What are my odds of getting into medical school? I'm just concerned because the program I am in is 2 years and medical school will be another 4+, so I am just wondering if it would be "worth it" to pursue both, both time-wise and opportunity-wise. Also, since I will have a Masters, would it help offset my lower sGPA?

casedentalmed

It's too hard to predict what individual medical schools will do. One thing that is certain is that you need to show how your background gives you a more unique perspective that can be valued by the schools you are applying to. Hopefully you will do sufficiently well to garner the attention of admissions committees and fit with programs that might value someone who is more oriented towards preventative approaches through nutrition education. I wouldn't know if it is financially feasible to do both an RD and an MD, but I would want you to think that getting an MD is actually really hard when it comes to the class loads you have to balance.


What to do when did bad on MCAT and can't apply now?

Hello. I am new to this forum and I am in desperate need of some advice. I just received my score back (501) and it is not good enough to get into the schools I want. I am currently at an internship so I do not have any time to try and study and take it again before applications are due. I do not want to waste the time and money to fail it again, so I have decided I will just wait a year and apply for the 2018 application.

Really, I just don't know what to do at this point. This changes my entire plan and I'm stressed out. What are some options that I have for my gap year? I am going into my senior year so any suggestions would be appreciated.

tantacles

Many who are taking a gap year do research (clinical or basic), a scribing job, or a job as an emt or cna. However, any job that you wish to do is on the table. Medical schools look for unique applicants, so if you're able to find a job in a unique field, it may be to your advantage.


Is it the right time to reapply?

Hello, this is the first time I've ever used SDN for application help. The rejections I had received this cycle really hit me hard and definitely shook my confidence. In the last few months I have tried to gather my wits and get the confidence I need to tackle another admissions cycle. I am trying to reapply to medical school during the 2016-2017 application cycle, and I thought I would try to reach out and ask for some expert advice.

Last year (2015-16 application cycle), I applied to 20 different MD programs. I graduated in May of 2015 with a BS in biochemistry (biology minor), and had a 37 MCAT (2014) score and a GPA from 3.823. In addition to that I spent a couple of years as a volunteer in a hospital and spent a summer with some clinical research facility. During the last application cycle I also got a job as a medical scribe at a GI facility, and have been working there ever since. I also got a job as a part-time online/in person science tutor.

Last cycle I had received 6 interviews and unfortunately had no acceptances. I was put onto three waitlists, but at this point it's become clear to me that I am unlikely to be accepted this round. I have a feeling that what really dinged me were the interviews, because I remember feeling exceedingly nervous and acting apologetic for lacking certain extracurricular/leadership activities.

However, apart from a year's worth of my jobs (and 5 new LoRs from the doctors I scribed for over the last year) I have not added any new experiences or more impressive scores to my application. I had even added the information about my jobs in the last admissions cycle, and had sent the new LoRs to the schools that put me on the wait lists this year. I am not sure whether reapplying to some schools will be right, as it will inevitably invite the "how is this application cycle different from your last one?" question on the secondaries and I am not sure how to address this.

Pathdocmd

I am sorry that you haven't been accepted. I want to ask few questions for you to think about. Where the 20 schools you applied to all "top tier" (E.g., Hopkins, Harvard, Stanford) or did you have a wide range of schools (including all your in state schools)? How much patient contact did you have in your volunteer work? Was it substantial? Where your other experiences (leadership roles, extracurricular, paid employment) substantial or did you spend most of your time in the library studying? Did you have someone objectively look over your essays/ personal statements?

It probably came down to your interviews. Everyone is nervous, but if you are so nervous that you can't function then you are not going to be accepted. Coming across apologetic is BAD. You emphasized the weaknesses of your application. Did you come across as immature, extremely introverted, or arrogant? Could you articulate your reason to become a physician? Could you describe your research well? Could you talk about a meaningful patient experience? Etc, etc, etc... I can't really say since I wasn't there.

I would reapply. DO NOT TAKE THE MCAT OVER! You will most likely do worse and I think it is a red flag for someone with a 37 to try for something better. In your re-application secondary questions emphasize the scribe experience. Emphasize the experience with patients. That is more than enough to write about. Good luck.


I only have Junior Year, Senior Year , Post Bacc and Grad GPA on AMCAS , am I at a disadvantage?

The reason is I have a bachelors degree from a foreign university ( though I am an American ) , I learned and understood that this is not sufficient in applying to a US Medical School, so I transferred some credits and earned a 2nd bachelors degree at a US University.

I understand that based on the rules of AMCAS, those transferred credits , as well as any credit or coursework coming from a foreign institution will not show up on my primary applications. If that is so, then I will only have a junior year and senior year GPA for undergrad ( 4.0 Biology, Physics, Chemistry, Math and 4.0 All other for both years ) listed. Post-bacc GPA of 4.0 BCPM and AO , and a graduate GPA of 3.9 with a Masters of Medical Sciences.

I have 3 publications , 50 hours shadowing , 200 hours + volunteering , Student Council member ( albeit in the foreign school ) , track and field championships, swimming championships , 2 years work experiences as a medical laboratory scientist , military experience , and a decent life story ( Ben Carson-esque )

Will the lack of a highschool, freshman year, and junior year GPA hurt my chances at a top med school in the US?

Pathdocmd

In my experience this should not be a problem. Every school is different in terms of pre-requisites and number of hours, but it looks like you have excellent grades and experiences so it should not be a problem (you didn't mention MCAT, but I will assume that it is a good one).


Do I have to list one class taken at a community college that isn't needed for my major and appears nowhere on my 4-year schools transcript?

I switched to an engineering major at the end of my freshman year, and that summer I took a pre-calculus class at a junior college to get back up to speed in math so I could take calculus 1 at my 4 year school. I ended up getting a B, not realizing I may have to report the grade on my applications. However, the pre-cal class technically is not needed for my major (only Cal 1 - Dif eq is needed), and appears nowhere on my transcript. Do I have to report it when filling out my med school applications? If need be I could look into grade exclusion or something at the junior college just to get it off of everything. I know it may not make much of a difference, but for my BCPM GPA I think it will take it down from around a 3.9+ to a 3.8+, which would be frustrating.

tantacles

You must report all grades on your amcas application. In addition, if this course was taken at another school, you must submit a separate transcript from that institution.


I have some MCAT specific questions ... where do I turn to?

I am self studying for the MCAT using 2015 Kaplan MCAT books. I usually can answer my questions and if need be I can understand why a specific question is right or wrong. But digging more into my studying I am finding I have specific questions about a problem that I cant find the answer to just by looking in the answers. Anyone know of a place I can go to for some help? I cant afford a tutor so that is out of the question. Would some place like Chegg Study or something be good or does anyone have an place they used?

casedentalmed

Hello, it's difficult to be specific because I think you are using proprietary questions from Kaplan or the AAMC (if it's from any AAMC exams) with those study guides. I'm not sure if the questions from Kaplan that you have challenges with can be answered through an online Q&A box. I don't think you would need a tutor for just one question, but if you are taking a class, the instructor should be able to explain the answer (or if not, another instructor). If you're not taking a Kaplan course, you may still be able to see if a Kaplan instructor could answer your basic question, or maybe the Kaplan website may help connect you with someone.


Explaining low GPA

My GPA from undergrad is 3.26. I don't have any stellar reason to explain it other than I went to a school with minimal grade inflation (an adcom member told me this so I'm assuming it's legitimately true) and while I had vague dreams of going to medical school, I wasn't committed nor had any idea of how much I was jeopardizing my future by not taking my classes as seriously as I should have. Additionally, multiple choice tests are not my strength. While I've since worked to address this problem, at the time I did not. I always knew and understood the material, but did not take the additional steps to get over my test taking difficulties.

After spending senior year abroad I came back to the US truly committed, and took a year and a half of science classes to finish my science prereqs and boost my GPA. Overall I had an average of 3.7 from those classes.

I recently finished a Masters of Medical Science program with a 3.36. My grades were significantly better second semester. During the Masters program I worked incredibly hard so it was not a lack of motivation, but rather a) had not taken classes for 9 months b) general test taking issues and c) I wanted it so badly that I psyched myself out.

Given all of that background, how should I address the low GPA? Would you recommend submitting an addendum to my primary application? I didn't know that was an option until today. I've already submitted my primary but only selected 2 medical schools since I'm waiting on my MCAT (I'll take it June 18). Should I address it in secondaries?

Thank you for any advice!!!

tantacles

I would avoid making any excuses for your GPA as honestly, "I wasn't committed" and "My school has minimal grade inflation" are just that: excuses. My suggestion is that for the time-being, you try to get the highest MCAT possible. Ultimately, the numbers do matter, and your low GPA will hurt you regardless of any explanation you try to make.

I would strongly consider applying DO and possibly using grade replacement (only available through DO schools, not MD schools) to raise your GPA if you are not accepted this cycle. Applying to DO schools, assuming your MCAT is acceptable, will allow you to become a physician in the United States, and GPAs of matriculants tend to be lower.

I would not submit an addendum to your primary application; just do your best on the MCAT and improve your grades later if possible.


My Chances

My MCAT score is 503 and my GPA is 3.68. I'm currently working on my Master's thesis and my med school application. I was wondering if I should apply this cycle or retake the MCAT (I really do not want to do this). Personally, I'm not trying to get into a top Medical School but I want to know if it's even worth it to apply. I think that my ECs and LORs will really help with my application but sadly the numbers are really important.

tantacles

With a 503 on the MCAT, it will be very difficult to gain admission to an MD school but very possible to gain admission to a DO school. If you are open to DO, feel free to apply. If not, I would suggest retaking the MCAT and applying next year.


Auto-rejection of high stats app from lower tier school?

I heard recently that if you have high stats as an applicant, you shouldn't apply to lower-tier safety type schools because they will likely just reject your application outright on the assumption that you will likely get accepted and a more prestigious school and go there instead. Is there any truth to that?

For example, I have a 3.84 gpa and a 524 on the MCAT with 2.5 years of research and then what I think are average ECs after that. I was told that I should drop some of the lower tier schools I was planning on applying to for this reason.

Pathdocmd

Not necessarily. There are really no "safety" med schools. I don't care how great your stats are and if you don't apply to a whole range of schools, then you might find yourself on a lot of prestigious wait lists. If you really think that schools will not interview you because of your good stats, you need to tell them WHY you are interested in that school. Looking at their mission statement is a good place to start. You still might not get an interview because maybe their mission is rural primary care and your 2.5 years of basic science research doesn't make you a good fit.


How should I go about becoming an anesthesiologist?

I am 20 years old and currently going to CC in order to obtain an associates degree (only obtaining 38 credit hours thus far with a 3.76 GPA), while talking to many different doctors and counselors they have instructed me that I should become CNA certified in order to get a patient-doctor type relationship. I have done tons of research and tried to talk to multiple doctors but I am still confused as to how I should plan for my future. I love working as a CNA, I was wondering if I should working my way up in nursing and then apply to medical school after I receive my RN certification or if that would be deviating from my goal of becoming an anesthesiologist. I plan to go to Chapel Hill in North Carolina for my medical schooling (if that helps any). I planned on working on nursing because it adds to my medical experience as well as paying better than a factory job. To be more specific what would any of you personally recommend. I have nothing to go off of except to work as a CNA and get an associates in applied science. What should I do for work? Should I obtain a BSN?

casedentalmed

The route to being an anesthesiologist means going to medical school and getting into a residency in anesthesiology, which is no small task because it is a very competitive specialty track. So I would encourage you to do whatever you can to show you can succeed in rigorous courses. That means going for a BSN and maybe an additional BA/BS in science first. You may have to sacrifice working while going to school, or you may have to save up a lot of money to carry you through undergraduate courses.

There is also the anesthesiology assistant route, which is an allied health profession similar to physician assistant, takes about 2 years, and has a practically perfect employment rate (most get jobs by the time they graduate from an Anesthesiology Assistant program at this time). You should definitely look up that path and ask people employed in that role.


I am a reapplicant for the 2017 cycle is there anything I should be doing specifically?

I had applied this previous cycle to many schools but could not complete the secondaries in time due to unforeseen circumstances with my graduation requirements. Now i am reapplying and was wondering if there is anything I should be doing to help me reapply and if I should be wary of anything that might happen. I am doing research during this time and trying to get my name published. Is there anything else I should be doing besides reapplying to through amcas and aacomas?

Pathdocmd

I would continue any volunteer work and/or any experiences that involve patient contact (or start new ones). Committees want to see that you are still actively pursuing medicine and not being idle. You are working in research, which is great, but keep up the "people part" of your activities.


HELP! What should I do this fall?

Sorry, this will be long..

So, I'm really not sure what I should do next semester. To give a little background info, my undergraduate GPA is a 2.77 from a pretty large and competitive school. But then I did a post-bac program at a smaller and less competitive school and got a 4.0 gpa for the two semesters that I was there. I was then accepted into Case Western's Medical Physiology Masters program for this fall, and I'm not sure if I should do it.

I have shadowing experience and I've volunteered and interned in Africa, but I'm really lacking in research experience and I feel like my application could benefit from a year of work experience doing research that I'm really interested in. What do you think I should do? Should I just do the masters program and show that I can handle upper level science courses? (By the way, we also take a physiology shelf exam to prove our competence.) Or should I just save a lot of money and get some work experience?

Thoughts?

tantacles

With a GPA of 2.77, it will be almost impossible to gain acceptance to medical school without an extremely strong performance in an SMP program. I assume that your post-bacc grades increased your GPA; if so, and it increased it to above 3.0, you may potentially be in a good position as long as you have a stellar performance on the MCAT.

That being said, if you do this program and do not do well, you will be shooting yourself in the foot in terms of acceptances to both MD and DO schools. Consider grade replacement and applying to DO schools as that would allow you to raise your GPA more than you will be able to for MD schools.

I can not make a definite recommendation for this program itself as I am not familiar with it. Either way, doing well will help, and doing poorly will detract from your chances. This master's program will not contribute to your AMCAS GPA, however, so it may have limited utility.


Received a C in college Algebra...next step?

This is my first C ever! I am very concerned that this will be a red flag to the medical sxhool committees... So... My questions are:

Should I retake it, move on to pre-call and get serious help or...other suggestions?

I pretty much gave up around my midterm due to my grandmother being diagnosed with cancer and other issues...

casedentalmed

While I think it's really optional to retake college algebra, you should do whatever you can to place your grandmother's diagnosis in the right place when it comes to handling the stress of the situation. Rely on the support systems that are available to you, with your family and with your school. The red flag will come if you are unable to balance that stress with the requirements expected of you as a student, as it will not be any easier once you become a physician.


Should I re-apply to medical school with a low MCAT score with intentions of re-taking the MCAT in August?

I applied to medical school this past cycle (2015-2016) and was only able to afford to apply to 6 medical schools. I received interviews from 4 of the 6 and was placed on the waitlists for all 4. I was not successful in getting off any of the waitlists so far. I would like to re-apply this coming cycle (2016-17) but I am not sure whether or not I should wait until the following year (2017-2018).

I believe one downfall of my application was that I did not submit it early. It was submitted around July 15. Another downfall was my low MCAT score (501).

The only thing that would be different with my initial application is my added Americorps experience (I did this during my year off) and my new MCAT score (which schools wouldn't receive until Septemeber).

My three questions are: 1. In your opinion, would it be wise for me to re-apply this coming cycle (2016-17) or should I wait another year for the 2017-18 cycle? 2. Does the fact that I received interviews and was placed on waitlists mean that there was nothing detrimental with my primary application and MCAT score? 3. If I did re-apply, would it be wise to revise my personal statement or should I just keep the same personal statement?

Thank you in advance for your help and for taking the time to answer my questions.

I hope you have a great day.

casedentalmed

It's too hard to say whether being a reapplicant really hurts your standing. If you think your downfall is only your MCAT, then I would reapply and expect no movement to happen until after your new scores come in (though you may still have interviews). You were good enough to be on waitlists, so you may have a good shot to get in off the waitlist now or in a reapplication.

As for revising your personal statement, I would generally revise it to include some mention of the activities you added (specifically Americorps). I would really consider a serious rework just to give people a new impression of what is important now to you.


Do I need a LOR from an old PI whose lab I was in for a year if I don't think it'll be strong?

During my junior year, I was in a lab doing electrophysiology. Due to the mostly independent nature of the lab work I was doing, I rarely was in contact with my PI and I didn't really enjoy the lab too much so I decided to change labs after I came back from a study abroad semester in Fall of my senior year. I doubt that this PI would be able to write me a very strong/not generic LOR, but should I still include one just so that I don't have a suspicious lack of LOR about that experience?

Thanks!

casedentalmed

I think you answered your own question. Don't feel obligated to ask someone to write a letter that you think would not help your application. A lot of people do short research stints while in college.


There isn't a lot of talk about a high GPA and a lower MCAT -- what are the chances of getting in?

For example, a 3.81 GPA with a 505 MCAT. It seems like the GPA would be more representative of your overall scholastic ability...

casedentalmed

I am not sure that a 505 is a "low" MCAT for many medical schools, but I guess it depends on where you are looking to apply. GPA may be a bit more representative of your ability, but for many who struggled and had to adjust to college coursework, an overall GPA may mask his/her ability. Furthermore, there are many schools that have classes with serious grade inflation. Basically a GPA is just a tool that measures an outcome of coursework that presumes a strong ability to study, prepare, and take exams (or projects). It doesn't do much to predict how well one does in medical school beyond just the first year. Neither does the MCAT for that matter.

tantacles

I would take a look at this set of tables, which detail your chances of getting in with a 3.81 GPA and a 505 MCAT (converted to the old MCAT):

<a href="https://www.aamc.org/data/facts/applicantmatriculant/157998/factstablea24.html" rel="nofollow">https://www.aamc.org/data/facts/applicantmatriculant/157998/factstablea24.html</a>

While I can not speak to why medical schools do what they do, I can say that a caucasian person with a 3.81 and a 505 has somewhere between a 59% and a 76% chance of gaining admission to medical school.

If you find your ethnicity and go to the table, you can find your chance of gaining admission as well.


Submitting AMCAS before MCAT is in?

I am not taking my MCAT until July 9th. However, the rest of my AMCAS application is complete and I want to get it out to schools as early as (safely) possible. Should I still submit my application on June 7th when it opens or should I hold out for my scores to come in?

Pathdocmd

I would only wait if you intend to completely withdraw from this year once you get your MCAT score. It could save you a lot of money and time in secondaries (many schools send secondary application to everyone), if you do not think your MCAT is competitive. If this isn't an option, then send it in. Your application will all ready be verified by AMCAS and waiting at the schools you applied to by the time your MCAT scores arrive. It will give you some valuable time that you would otherwise lose if you only apply after you get your scores back.


AMCAS: Work and Activities

Trying to figure out what to put in my work and activities. I want to make sure I properly represent my achievements while still being relevant.

I have the usual, undergraduate research, clinical and volunteering, but there are a few hazing things I'm unsure if I should include:

1. membership in a fraternity. Alpha Chi Sigma professional chemists fraternity, a service fraternity in which I held two positions.

2. Membership in phi beta kappa

3. Artistic endeavors? I was a fashion design major my first semester of college, and did a huge science themed mural in high school. As well as several fundraising events for various organizations using my artistic ability.

4. Hobbies!? I knit and made a quilt that were donated to women in a halfway house. I visited that halfway house and got to meet those women and present my gifts to them.

5. Medical employment: it was as a receptionist in my parent's medical practice. Not really sure if I should include it, because i don't know how it will represent me: positively in that medicine has played a large role in my life for a long time or negatively in that although i went to other places to get clinical experience at later times, i was payed by my own parents (probably gonna have to list my mom as the contact because she's the manager)

tantacles

1. Sounds good. I don't think this will be particularly helpful, but the leadership positions may hold some minimal weight.

2. PBK is great, and including membership can signal to ADCOMs that your GPA is in the top 10% of your class; this is important as at grade deflating schools, where a lower GPA may still be in the top tier of grades.

3. Medical schools love to see activities like this.

4. This would work well as volunteering.

5. This activity has debatable merits; because the practice was owned by your parents, it may not be the greatest to include. However, if you have kept working there throughout college and up until medical school, go ahead and include it, particularly if you can speak well about how it impacts you.

All of these activities could be worthwhile to include; if you must choose between them, be sure to select activities that you are able to speak eloquently about.


Need advice about my next step.

I have been out of college for a year and a half. I went to medical technologist school for a year and currently work at a local hospital. My cGPA is 3.3 and sGPA is 3.0 and 3.75 at MT school. Do you think I should go do a postbac to streghten my gpa or could I balance out my gpa with a strong mcat score and lots of clinical and volunteer hours. Please and thank you.

tantacles

1If your MCAT score is absolutely exceptional, it may balance out your performance in undergraduate. However, if your science GPA and cumulative GPA will demonstrably increase in a post-bacc and you can get all A's, it would definitely be worthwhile to pursue.


Handling a disciplinary action as a second time re-applicant? =

I am a pre-medical student applying for my second time, the last time I applied was three years ago. On my last application, I disclosed a disciplinary action and what it resulted from, a domestic dispute with my girlfriend that resulted in arrest (one charge of criminal trespassing and another for interfering with an emergency phone call). Since this happened my girlfriend and I have made amends and come to terms with the incident, the university and law enforcement have both expunged the record. The only existing record that this ever happened is on TMDSAS because they keep a record of applications. I have done a lot since this to better myself and gain more healthcare experience (1 year of scribing, an MPH and just retook the 2015 MCAT).What can I do this time around to make sure that this no longer reflects poorly on me?

tantacles

This particular event may be death knell to your application. You can not leave this event off of your application per AMCAS' and TMDSAS' rules if there was any conviction or University disciplinary action (an arrest without a conviction does not count), but explaining it may help to mitigate its effect on your application provided everything else is stellar. When you explain this disciplinary action, explain that you were involved in a domestic dispute with your significant other and that you realize that your behavior was inappropriate. Be sure not to make excuses for your behavior. Your reconciliation with your girlfriend is immaterial to this event; what is most important is your understanding of the event.

No matter what, an arrest will reflect poorly on your application, but if you explain what happened, don't make excuses for your behavior, and talk about what you have done to change during the intervening period. Have you gone to anger management? Seen a psychiatrist or other mental health professional? Done some activity to gain a better understanding of domestic violence and how it impacts people, like working at a women's shelter? These pieces of the puzzle are important and may help to show that you have changed.


Applying 2016-2017: Asian, MCAT 513, cGPA 3.62, sGPA 3.55. Chances? Advice? =

MCAT Overall - 513 (90th percentile) CARS - 126 (70th percentile) Bio/Biochem - 128 (87th percentile) Chem/Phys - 129 (93rd percentile) Psych/Soc - 130 (97th percentile)

cGPA - 3.622 sGPA - 3.55

3+ years of combined research experience at 2 labs 3 months volunteering at major hospital 2 weeks shadowing experience

tantacles

Your MCAT and GPA will get you a foot in the door at many schools, and I would say you have a good chance of getting interviews. Just be sure to apply broadly geographically and try to find at least 12 schools at which you are above the median for MCAT and GPA, and you should do well as long as your interviews go well.

If you can, continue to volunteer; your volunteering experience is a bit short, and medical schools like to see volunteering that lasts for a longer time.

Best of luck, and I hope you get in!


I am looking for feedback on activities I have planned for "growth year" as to how they would be viewed by ADCOM. =

I am a re applicant to MD programs (mostly in FL where I live). I applied to ~10 schools, but my application was WAY late ( I dont even like saying when I was "completed" at even my first choice schools). I had two interviews but no acceptances. I've re worked and seriously improved my personal statement. Since last app, I have taken 31 science credits with ~3.9 GPA for all of senior year, completed another clinical internship (~40 hours shadowing surgeons + ARNP), completed two independent research projects, and excelled (highest score) in a class I was chosen for, gross anatomy/dissection techniques. My overall and science GPA went up to 3.56, and my MCAT score is 514 (92%). For my year off, I am working full time as an anesthesia technician, and holding a non paid position as a research intern in the anesthesiology dept. of a nationally recognized research hospital. I'll be spending a LOT of time in the OR and even doing projects on cadavers, with a chance to present at the next conference. Are these positions going to reflect well on my "growth?" I am looking to improve EVERY aspect of my application to ensure an acceptance this time around and avoid a THIRD cycle.

tantacles

Your achievements will definitely show on your application. No one can tell you how this application season will go, but I would strongly suggest a year off. If the previous cycle did not go well, it is unlikely that your application will have improved significantly while you were interviewing.

That being said, the fact that you submitted late May have contributed to your failure to gain an acceptance. Your stats are good for allopathic admission. If you try to apply this year, be sure to apply to multiple schools that were not on your list last year.

In addition, if you can, call the schools that interviewed you and politely see if they have any feedback on your application as you have not yet been accepted. This may yield fruit and you may be able to remedy whatever weakness caused you not to be accepted.


Question regarding medical schools =

how do medical schools screen applicants with two MCAT scores? if the recent MCAT score is significantly higher than the first MCAT score, will medical schools still screen the old MCAT score and reject you? Please let me know.

tantacles

This varies widely by school. Some schools will look at your best score, some will look at your best composite score, and some look at your latest score. Some schools even average your scores together. Most schools do not make this information public. Notably, though, if your application is complete and you already have an mcat on file but you plan to take the mcat again for your current cycle, you may be screened out by a school unless you inform them that you are retaking the mcat on a particular date.


mcat 307, with reading comp 124, and GPA 3.9 -should I apply? =

Hi I recently received my MCAT score and am unsure whether to retake the test or apply.

overall MCAT: 507 (76th percentile) CARS - 124 (49th percentile) Bio - 127 (78th percentile) Chem/Phys - 127 (79th percentile) Pysch - 129 (92

I have a 3.9 GPA and a 3.85 science GPA. I have 1 year of research in college and two years in high school/summers. I have ample clinical experiences and service trips.

My reading comp is low, but I also put my all into studying this semester, so I am not sure what I would do differently to improve my score :/

What do you think?

casedentalmed

Your numbers are tell us part of your fit with medicine, so the rest of your application needs to shine through, including your possible fit with the schools of your choice. That said, I'm not sure about your CARS score being so different from the rest of your performance. I think it's worth applying since there are so many schools to choose from, but I hope you have other evidence that can support any argument that your CARS score is a slight aberration.


Question regarding MCAT scores =

How will medical schools view an applicant who took the MCAT twice, first try got a 484, while the second try got a 520? Will the first MCAT score still be a red flag? My GPA is a 3.68 overall so far with a science GPA of 3.55. Is there a good chance to get accepted to an instate MD school? Please let me know. Honest answer please.

tantacles

This varies widely by school. Some schools will look at your best score, some will look at your best composite score, and some look at your latest score. Some schools even average your scores together. Most schools do not make this information public. Notably, though, if your application is complete and you already have an mcat on file but you plan to take the mcat again for your current cycle, you may be screened out by a school unless you inform them that you are retaking the mcat on a particular date.


When taking MCAT twice? =

When taking mcat twice, is there an option on the medical school application to list your most recent score or do they just look at your scores and you don't have to manually put in the scores onto the application?

Pathdocmd

Medical schools will see all MCATs taken. It depends on the school if they take the highest score, most recent, an average, etc.


Waitlist Letter of Intent Length Question =

I have had three interviews to medical and am on the waitlist at all three. I just found out the other day that I have been put on waitlist for my number one choice. I want to write a letter of intent to number one choice and letters of interest to the other two. I have a letter drafted but it is 1.5 pages and I am not sure if that is too long? It is very straight forward, I give an update of my recent grades and graduation and activities then give three reasons why they are my number one choice. Is that too long? Also should I send it in the mail, as an attachment to an email, or as the body of an email? Lastly, I mention in the letter the other two school I am on the waitlist for and say despite being on their waitlists I would accept if accepted to this medical school, is that ok to name drop other schools I am on waitlist for? Thank you for any help or advice.

Pathdocmd

I had a philosophy professor in college that said "Brevity the essence of a focused mind" and I agree with him 100%. I cannot speak for other med schools, but I would keep it short (no more than 1 page). I would e-mail it. It is easier to keep track of since paper folders are becoming a thing of the past. I would not mention the other schools.


Is the (somewhat arbitrary) information provided regarding hours for each activity/work experience taken into serious consideration? =

The total hours for some experiences that I'm listing on the AMCAS application feel arbitrary based on a personal estimation of time spent working on a project. I'm not sure how to approach this section for experiences that don't conform to the provided categories. For example, I'm listing a poster that was presented for research that I contributed to extensively. I wasn't able to attend the conference, but I am still listed as an author in the conference abstract. Should I be listing the hours spent working on this research (an arbitrary estimation) or the hours of the conference (that I did not attend)? Maybe I'm over thinking this...

Unfortunately I won't have the space to communicate my reasoning for the estimation of hours in the experience description.

Pathdocmd

Try to estimate the number of hours you spent on the project: background research, IRB time, doing the experiments, time writing up the abstract, etc. I wouldn't log the hours at the conference. If you did present at the conference I would only estimate the number of hours you spent standing in front of the poster, not the total time of the conference.


Should I use my gap year to bring up my g.p.a. or work for a physician? =

I just received my undergraduate degree. Unfortunately, I didn't apply to any SMPs or post-bacs, as I'm using this summer to re-take the MCAT. My current GPA is a 3.29c 3.13s, with 144 total hours (some of my credits from research didn't go towards my degree, as well as re-taking two classes).

I'm planning on applying to SMPS and post-bacs for Fall 2017. Should I use this coming year to bring up my GPA or work in a medical setting (such as a scribe or medical transport)? Any advice is greatly appreciated, I'm very torn on what I should be doing to increase my chances of being accepted to medical school in the future.

tantacles

Depending on your financial situation, you may wish to improve your GPA. If you plan to attend an allopathic school, increasing your GPA will almost definitely be necessary without an almost impossibly strong MCAT score. If you're willing to take a long route, you may wish to work while taking 2-3 classes per year, simultaneously improving your GPA while improving your experience.

If you are unable to improve your GPA, an SMP may be the only route that will work for you, but keep in mind that if you do not perform extremely well in an SMP, you will have ruined your chances of admission.


Will I need to retake a lab? =

I am a psychology graduating this semester and I have a 3.73 GPA I have taken all the pre requisites for medical school except biochemistry which I am taking as a post bachelor class. I am also taking the mcat in july. I have passed and done relatively well in all my classes, except the lab for Biology 2 I received a C- but for Biology 2 I had received a B. It is a 1 credit lab and now I don't know if I need to be retaking that because I am hearing mixed answers about it.

tantacles

It depends on the school you apply to. Some schools will not accept a grade in any course lower than a C. I would look on schools' websites for individual requirements as many may accept a C- in a lab, but others will not.


worried about GPA =

Hi! I am a junior Biochem major. I had a 3.89 GPA but due to some family issues, I couldn't focus these last two semesters and messed up my GPA. It's 3.79 now. I am not sure if I am going to able to get into a good med school this cycle. I really want to get in this cycle though. But due to this trend in my GPA, I am not sure what to do? I haven't taken MCAT yet. I am thinking of taking them in June. (P.S. I do have research, volunteer and other reasonable extracurricular). Any help would be highly appreciated!

tantacles

3.79 is a great GPA for medical schools. To ease your mind, you may wish to invest in the MSAR; this resource will tell you the median GPA and MCAT of all US medical schools and give you an idea of which schools may be good fits for you to apply to.


advice for 3.62 c/3.43s, low mcat =

Asian, Graduated in 2014 with 3.62 c gpa and 3.43s gpa. I retook MCAT in may 2015 with score of 503 (126b, 127c, 125 cars, and 125 psych). My previous score was 26: 8V/8P/10B.

I have a lot of volunteering experiences : 300hrs in pediatrician clinic and some social work volunteering in hospital. I also shadowed both DO and MD before.

I think my biggest weakness is my mcat score. Would you suggest that I retake it again? Do you think I can still apply to schools this cycle?

tantacles

I would strongly suggest that you retake your MCAT. A score of 503 is equivalent, approximately, to a 27 on the old MCAT.

From this table:

https://www.aamc.org/download/321516/data/factstablea24-3.pdf

34.9% of Asian applicants with your stats were accepted to a US allopathic medical school. If you were to apply this cycle, I would suggest mainly focusing on DO schools.


Is medical school realistic for me? ANY help is welcome! =

Hello all! I am new to this site, thought I would use another resource as I plan ahead in my career path.

I am just finishing up sophomore year of college. Female, age 20, (if any of that matters). 

So. I started college not really sure what I wanted to do, in the back of my mind I always wanted to be a physician (my uncle is a MD and huge influence) but never felt smart enough or thought I would be able to go to school that long without going nuts. Basically, at 17-18 I thought I wanted to rush through college, get married, blah blah- all that terrible stuff (kidding, kinda).

Ya. Well here I am, microbiology major now after taking 2 years to decide, and medical school is the current plan (also considering AA and possibly dental school, so any input there is great too). My mother is a dental hygienist so that's where the dentistry influence comes in- I love shadowing the dentist she works for.

However. Here is my issues/worries. I have now received credits from 4 colleges, 2 are public state universities, the other 2 are community colleges. Some of my credits I received in high school as a dual enrollment student. I have 4 "W's" on my transcripts, however, I am still slightly ahead on credit hours and have a 3.8 GPA.

I work at Vanderbilt University Hospital, worked in a surgery unit & now in an ICU, have a bit over 1,000 direct patient care work experience hours, and my hours will increase simply due to the fact that I have to work and cannot just go without a job. Also have about 300 hours of shadowing between my uncle (MD in pain management) and the dentist my mom works for. Planning to start volunteering maybe at Vanderbilt as well? Should I?

Any input is so greatly appreciated. Be as brutal as needed, I can handle it, just please be honest. Thanks so much.

tantacles

Your GPA is fantastic for medical school. I would strongly suggest that you get some experience volunteering as opposed to just shadowing as this will show a commitment to altruism. The multiple schools should not be a huge issue as you have done well throughout, and the W's should not affect you either, again because your GPA is so high. Volunteering at Vanderbilt sounds like a good idea, and I would suggest some non-clinical volunteering as well. Research would be helpful for a medical school application, but may not be necessary at certain schools (see their missions to find out which ones).

Truly, though, I can not advise you until you have an MCAT score - the combination of GPA and MCAT will determine whether you are able to get interviews for medical school, so do well on the MCAT and you have a great shot.


Should I reapply this cycle? =

I am currently waitlisted at two medical schools and am wondering if it would be wise to reapply this cycle. I believe that my selection of schools from last cycle was what lead to an unsuccessful cycle and wanted to know if reapplying with an adjusted selection of schools would lead to more success. I am also considering taking an extra year to take post-bac classes and continue volunteering in hospitals.

My schools I applied to last year: Baylor Boston University Case Western Columbia UCLA Drexel Duke Emory Dartmouth George Washington Georgetown Harvard Johns Hopkins USC Mayo Michigan Michigan State NYU Northwestern Upenn Stanford Tufts Tulane-waitlisted UCSF Uchicago UIC-waitlisted Maryland Cornell Yale

Here are the schools I plan to apply to this year (Illinois resident): UIC Tulane Rosalind Franklin Rush Southern Illinois Wisconsin Medical Western Michigan Jefferson U Miami VCU Albert Einstein Wake Forest Toledo Loyola Rochester Creighton NYMC Qpac Hofstra EVMS Cincinatti Louisville Saint Louis Virginia Tech Vermont

My cGPA is 3.6, sGPA is 3.33, MCAT is 31, and undergraduate school was Stanford These are my activities from this year: 1000 hours working to restructure South Africa's emergency medical system in Cape Town 100 hours ambulance rides in South Africa 300 hours working with children with Austism in clinics 100 hours volunteering at Alzheimers Clinic 100 hours shadowing doctor in ER

Activities on my application last cycle: -started own 501c3 to fund heart surgeries for Vietnamese children -1000 hours working for NGO that initiates first aid training sessions in Ho Chi Minh City -500 hours of research for improving health care system in South Africa -EMT training and certification -300 hours research on spinal rehabilitation therapy -4 years club soccer -1000 hours running charity program at school -4 years musician for student band

Pathdocmd

Your second group of schools is much more realistic with a 3.33 sGPA and a 31 MCAT. You seem to have the activities down, but I would continue with something domestically with direct patient/ people interaction this year. I would work on the sGPA and maybe retake the MCAT. If need be, work on your interview skills.


AMCAS Application =

Hi, I am taking MCAT on June 2, 2016. I really want to apply for this cycle. So, my question is, lets say I submit my AMCAS Application for verification before my MCAT score comes and God forbid if I do bad on my MCAT and i change my mind of applying this cycle. So, I can just cancel my application ? If no, how it works?

Thanks in advance! Your help is appreciated!

tantacles

You can not cancel your application, but you can choose not to complete any secondaries.

In addition, what I would suggest is that you apply to one school on your first go, submitting all transcripts as well, and depending on your MCAT score, either add schools so that you then have a complete and verified application (submitted early, no less) and can receive secondaries immediately should you choose to add more schools. This will ensure your best chances of being accepted.


Path from RN to MD =

Hello,

I graduated last year with Bachelor of Science in Nursing (BScN). I have been working in the Intensive Care Unit for the past couple of months. I am interested in pursing a career in medicine. My GPA is 3.95 during nursing school. I am thinking about apply to University of Alberta & University of Calgary (neither require science pre-reqs).

My first question is if you would recommend I go back to school to take a full course load of science for one academic year (fall & winter) of biochem, physics, math, etc. with labs? Before I completed my BScN, I obtained a 3 year General Science degree as well - but I was not focused in school during that time and did very poorly in biochem, math, physics, etc. (grades in 70s). So I am wondering if I should re-take the whole 2 semesters to help me prepare for the rigorous course load during medical school. As well as helping me refresh my knowledge in preparation for the MCAT exam.

My second question is that I am interested in working/volunteering in a lab, so how/where can I find supervisors that provide this opportunity?

Lastly, it appears the vast majority of pre-med applicants volunteer abroad for a ~month, assisting local physicians and healthcare team. Would you recommend undergoing this experience? Are there any specific groups/organizations/locations you would recommend?

Thank you very much. Any other suggestions/opinions for nurses interested in pursing medicine would be much appreciated.

tantacles

With your GPA, there is no need to go back to school, but you will need to make sure you have completed all pre-medical requirements for your applications and you will need to take the MCAT. No need to retake the entire pre-medical sequence if you've already passed and have a 3.95 GPA total (assuming that includes ALL college courses you have ever taken). In fact, if your GPA is above a 3.8, there is no reason to retake anything. You can study for the MCAT on your own (or take a course) and do well without retaking courses.

If you wish to work or volunteer in a lab, you can look online for job postings or cold e-mail science professors at your University or another to see if there are any positions or opportunities.

Finally, no need to volunteer abroad. Short clinical experiences, like volunteering abroad for a month, tend not to be helpful for admission to medical school.

Lastly, if you plan to apply to medical school, I would suggest applying to more than two; medical school admissions are extremely competitive, and you want to give yourself the best chance of getting in the very first


Disadvantaged status or no? =

[GPA: 3.88, Science 3.9 ; Work: 600 hr internship that led to a job (2,000) at a home health care organization. Worked 400 hours in Dr.___ private pediatric office; Volunteer: 200 hr in homeless shelter, 200 in cancer hospital, 300 shadowing Dr.G, 100 shadowing Dr.L., 100 hr building homes with my club Habitat, mission trip to Peru, etc. ; MCAT: July (late!!), hoping for 510 or ~32 (not sure of old scale); No research

Am I qualified for the disadvantaged status?

My parents and I left behind our whole family when immigrating from Ukraine to the U.S. We arrived in with enough money to afford half of a one bedroom apartment, sharing it with two other adults, in a impoverished city. Everett has the highest opioid death rate in MA, and I live next door to a barber shop that circulates drugs and a bar that brings violence every weekend. By eight, I was left home alone all day where I learned to take care of myself, while both of my parents worked during the day and spent their nights in college classes. Though my public school had the lowest MCAS scores in the state, underpaid teachers, and was called a ‘communist’ daily never affect my love for school. Though the environment and stress of living in Everett has impacted our health. I eventually developed asthma from the enviroment, and my mother has been relapsing with depression for the last 7 years. In 2014 she was diagnosed with severe fibromuscular dyplexia, leaving her unfit to work for large periods of time. My mother’s condition has left my father and I as our only financial support for ourselves, and our family in Ukraine. But taking care of my mother has never affecting my grades, my time for volunteering, shadowing, tutoring, and leading my club to success.

tantacles

Below is the information from AMCAS with regard to disadvantaged status:

"Disadvantaged status is self-determined and each medical school has their own policies for how they use this information.

This information is voluntary and will not affect the processing of your application.

To help determine if you are disadvantaged, click the How do I know if I should be considered disadvantaged? link, which displays the following information:

Underserved: Do you believe, based on your own experiences or the experiences of family and friends that the area in which you grew up was inadequately served by the available health care professionals? Were there enough physicians, nurses, hospitals, clinics, and other health care service providers?

Immediate Family: The Federal Government broadly defines "immediate family" as "spouse, parent, child, sibling, mother or father-in-law, son or daughter-in-law, or sister or brother-in-law, including step and adoptive relationships."

State and Federal Assistance Programs: These programs are specifically defined as "Means-Tested Programs" under which the individual, family, or household income and assets must be below specified thresholds. The sponsoring agencies then provide cash and non-cash assistance to eligible individuals, families, or households. Such programs include welfare benefit programs (federal, state, and local); Aid to Families with Dependent Children (AFDC or ADC); unemployment compensation; General Assistance (GA); food stamps; Supplemental Security Income (SSI); Medicaid; housing assistance; or other federal, state, or local financial assistance programs."

In other words, you must determine for yourself whether you would be classified as disadvantaged. Unfortunately, this is not a determination we can make for you. Essentially, it boils down to this:

Was your area served by an appropriate number of physicians?

Were you enrolled in a means-tested program for which the requirements were related to assets and income?

If the answer is yes, you qualify as disadvantaged.

If the answer is no, then you would likely not qualify as disadvantaged.

However, I would recommend you speak to an adviser about this.

casedentalmed

In addition to the above answer: Have you been able to apply for any MCAT, AMCAS, or AACOMAS fee reductions? Have you ever participated in any enrichment programs in the summer targeting disadvantaged students?


Late applicants disadvantaged? =

Okay I know this question has been asked many times, but maybe I could even find more information.

I am taking my MCAT on July 22, and I think this is way to late. - I have a 3.88 GPA, and 3.9 Science GPA - I have over 2,000 hours of related work experience, 300hr in volunteering, and another 400 in shadowing doctors. I have lived in the middle east and Peru to work with the underserved community. My passion for the underserved community sparked while living in the projects as an immigrant. I do mention some of this in my personal statement but I focus on who I want to be, instead of who I was. - I do not have research experience, which I regret. - I know my LOR are great because I asked professional people who truly know and care for me.

Do you guys think I have a change if I send in my application in mid Aug? I know clearly "I have a chance", but honestly with your experience, do you think a late applicant can really disadvantage me ?Am I even considered a strong my applicant compared to everyone else because I did not do research?

tantacles

Your MCAT is not too late. What I would suggest is that you submit your AMCAS application on the day it opens with your personal statement, activities, and all of your transcripts. This will allow your application to be verified.

Subsequently, when your MCAT score returns, add as many schools as you wish, using information from the MSAR to guide your decisions. This will allow you to get secondaries immediately.

Your lack of research may hold you back from top research universities, but with your GPA and clinical experiences, you stand a good chance of being admitted as long as you get your application verified early and you do well on the MCAT.


Low GPA because of surgeries. SMP or retake for MD? =

Hello, I just finished my junior year as a biology major at a top 10 public university (known for low GPA). My undergraduate situation is unique so I posted this question again, despite the overwhelming number of posts about SMP vs retakes on SDN.

History: I did receive some low grades during my freshman year, but I had been improving it during my sophomore year. Unfortunately, during both of the summer breaks I had so far (summer break after freshman and sophomore years), I had to undergo surgeries due to pneumothorax (one on each lung each summer). I was able to somewhat handle the course load during the sophomore year, but after the second surgery, my grades this year (junior) dropped severely, with several Cs and two Ds. These courses (Cs and Ds) include physics (electricity and mag), anatomy, biochem 1, evolution, and immunology.

My Stats and EC: Numerically, I had cGPA around 3.45 at the end of sophomore year with strong upward trend, but as of now, I have a 2.89. I took the MCAT last month (April 2016) and got 511 (128/128/129/126). EC wise, the surgeries essentially made me take "breaks" from the EC activities I was doing. I volunteered for a semester at a children's hospital during sophomore year, stopped and currently restarted volunteering at a large midtown hospital. Research wise, I was in a cancer research lab for two semesters during freshman-sophomore year, stopped, and joined a different lab (studies muscle aging with mice) during second semester of my junior year (spring 2016). These "stop" points coincide with the fall semesters when I returned from surgeries so I got tired easily. As a semi-replacement to shadowing, I have been taking seminar courses where we have guest physicians come in every week and talk about their experiences. I am also one of the founding members of a badminton club and teach beginners how to play.

Summary: My history aside, I understand very clearly that something needs to be done about this GPA. I am interested in the allopathic MD schools, not DO, or dental. Currently, I am preparing a med school application that will be ready for submission this summer (I was expecting much better grades this semester). I have a solid belief that I can retake courses where I received Cs and Ds and get As, proving that I do know the material.

With this in mind, I would like advice on how I should proceed for best shot at med school acceptance. Should I take another undergrad year and retake the low grade courses or is my time much better spent at just going to a strong SMP? Another question is whether I should even apply to medical schools this summer.

Please give me your opinions. Thank you.

tantacles

With a 2.89, it is almost certain that you will need to either raise your GPA above a 3.0 and do extremely well on your MCAT (>95th percentile) or go through an SMP. Retaking courses will not help you as MD schools do not offer grade replacement, so if you wish to improve your GPA, you may be better off taking higher level courses.

Note, however, that many schools will not accept pre-requisites that have been completed with a grade below a C, so regardless of whether you do an SMP, you will have to retake any of your pre-requisites with grades below C.


Should I retake General Chemistry 2? =

My name is AD and I am currently a rising sophomore in the college of Arts and Sciences at Vanderbilt. It has always been my dream to go to medical school and almost everyone in my family has attended. That said, my second semester turned out to be pretty rough. I got a C in my intro to neuroscience course (which I intend/intended to major in) and a C+ in Gen Chem 2 (after a B- last semester), due mostly to poor performances on finals.

   From talking to some of the representatives at the medical school fair, I understand that the admissions process is holistic: they look at MCAT scores as well as extracurriculars and service. I plan on shadowing this summer and doing research. I also took a research assistant job in a neuroscience lab last semester, which I will continue next semester.
     That being said, I still have doubts about my grades and planned curriculum. What course of action do you suggest I take with regard to my curriculum to improve my chances to attend an allopathic medical school? Should I be retaking General Chemistry 2 next semester instead of continuing with organic chemistry? Should I retake neuroscience as well, or just switch my major altogether to something less demanding, such as MHS? Any advice you may offer would be great. Sorry for such a rambling email but I was really not expecting such poor grades.

Thanks again, AD Vanderbilt Class of 2019

tantacles

The main thing you need to work on is making sure that your cumulative and science GPAs are over 3.5. Allopathic medical schools do not allow for grade replacement, so it would not be in your best interest to retake these courses, but rather take courses in the future in the sciences that you know you can do well in.

As far as your major is concerned, the major itself is not particularly important, but choose a major you can do well in and that interests you. A low GPA, no matter the major, will make getting into medical school much more difficult.


I just took my MCAT and I'm pretty confident, but my GPA isn't stellar. =

The person I was living with in junior year of college got really sick and I tried to help as much as I could. Unfortunately my grades dropped because of that and my overall GPA is 3.35. I just took the May 6 MCAT and I feel pretty confident I made around a 508 (at least based on my practice tests). I don't have volunteer experience (because I was supporting myself in college working 35+ hours a week) but I've been working in a transplant immunology research lab at a prestigious private university with residents for almost 2 years. I've been to clinic with the PI once and seen a few transplant surgeries. What do you think my chances are for getting into a med school if I apply early June?

tantacles

Take a look at the following:

https://www.aamc.org/data/facts/applicantmatriculant/157998/factstablea24.html

These tables give you an idea of your chances. For example, for a caucasian applicant with your stats, 828/2431, or 34% of applicants were accepted.

Is a 1/3 chance of getting into medical school acceptable to you? It might be! If it is, then apply. If not, you may wish to take some more undergraduate classes and/or retake your MCAT to give yourself a better chance of being accepted. Try to make your first application cycle your last one by applying when you are most qualified.


Should I double major in Biology & Philosophy? =

I want to major in both biology and philosophy. Is that a good idea? Approximately how many years will that take? What are some pros and cons for double majoring?

casedentalmed

Depending on where you go to undergraduate, you should be able or unable to double-major in anything. You would need a lot of planning with faculty or staff advisors to balance your electives and requirements. If you like both topics really well, or like the connections between the topics, you can double or triple major if the schedule works itself out. This also means you would have to be very disciplined and committed to the scheduled core of classes you would take, as you would be less likely to take any "fun" elective classes outside your majors. So ask your advisors at your home school to see if it's something you want to do.


After being accepted to a university I do now do not want to attend I am debating on applying next year but have heard terrible things about a 'blacklist'. Is all the 'blacklist' talk true? =

I have recently been accepted to a MD program I interviewed for quite late in the interviewing season and got an acceptance within days of interviewing. This acceptance took me by surprise as I did not have enough time to think about my decision and withdraw my application between interviewing and getting in, despite deciding that I would not attend and was no longer interested. I have been told about a National Acceptance Report that would release my name whether I attend this medical school or not and was advised by my pre health advisors to accept - I received the acceptance April 28th so I didn't have much time to think about this either. After more research, I've been told that my chances of getting in after reapplying are not good since I will have 'turned down' and acceptance. Do you know how much truth there is to this? I did not think I would get in and have become extremely personally invested in my research, so much so that I would now be interested in applying MD/PhD. I also do not want to attend this medical school because of personal reasons - my family is no where near there and I really wanted to stay in the northeast to be near them. My MCAT is a 505 and my GPA is a 3.61 with two W's and 1 F. I have an engineering degree and a large amount of shadowing/volunteer hours.

I am asking for genuine advice - thank you so much for your help.

Pathdocmd

Unless you have a very good reason in turning down an acceptance, this is not a good idea in my experience. I cannot speak for other schools. Have you thought about asking for a deferment to spend a year on your research? Also, are you on the wait lists at other schools? Let them know that you are still interested in their school even if you have been accepted to Med School X.

If you REALLY want to be a physician, then you are going to have to make sacrifices, and going to this school away from your family is going to one of many. Only you can make this call.

tantacles

I would strongly suggest that if you wish to be a physician that you attend this school. Other medical schools will be able to see that you turned down an acceptance, and this school will certainly not accept you a second time if you withdraw on this occasion.

In addition, with your stats, your chances of being accepted this year were low, and getting into medical school results a huge triumph. Any medical school will put you on the path to get a residency near your family, particularly an MD.

In addition, if you wish to do an MD/PhD, you can apply from within and ultimately get a spot that way, which will have the same result and not give you the risk of not being able to attend medical school at all due to declining an acceptance.


How does the influence of my effort compare to the influence of the prestige of certain Universities when applying to Medical Schools =

I'm in the process of applying to 4-year colleges for pre-med.

Due to my lack of performance in Freshman and Sophomore year, my GPA isn't great for my high school transcript.

Realizing that I cannot get into the great medical universities like University of Rochester, I am left with a limited selection of colleges that are not as prestigious.

My question is: Is it much less likely for me to get into a great medical school since I'm attending an OK 4-year school? Or will the prestigious med schools see my stellar performance in my OK college and my good MCAT scores and judge me based on my individual effort, rather than based on which university I attended?

Pathdocmd

For a class of 120, my school takes applicants from 55-65 different colleges and universities. They come from "big name" schools (Ivy League, JHU, Stanford, Williams, Duke, etc) and places most people have never heard of. Don't worry so much about the brand. Prestige counts for very little in my experience. Also, a local college may have a great placement rate with med schools in the area. You need to go to a place where you will thrive and make the most of your 4 years. If you are successful then doesn't matter that much where you went to college.


Failing Concurrent class. =

I have been worrying about my chances of getting accepted. I failed my concurrent highschool class my senior year. I failed because of absences from going to a funeral. Do I have a chance of getting accepted?

tantacles

One failure in a course will not completely ruin your chances of being accepted to medical school. The important thing is that you work hard to improve and have an upward trend in your grades.


MCAT exam =

I took the MCAT exam but didn't receive my scores yet, If I take the MCAT in June and apply this cycle will medical schools see my first MCAT score as my June results will come later? Is there a way I can not send my first MCAT score? Let me know.

tantacles

When you apply to medical school, all administrations of the MCAT will be reported unless they have expired. Each MCAT takes 3 years to expire.

Pathdocmd

I do not believe that there is a way to not send MCAT scores. If that was an option, about many, many applicants would do it.


Taking June 2016 MCAT and if I am too late to apply in the cycle for this year. =

I already took the MCAT during the April 23rd 2016. I don't think I did well because of the timing. Should I take the MCAT in June if I want to still apply this cycle. And if so, what should my study methods be each day till the June 18 exam. The scores come out July 23, will I be at a disadvantage in the application cycle? My overall GPA is 3.64 so far. Please help me. I am really confused.

tantacles

If you feel your score on the MCAT will not be ideal for this application cycle, you might consider signing up for a June date. Keep in mind that if your score comes back and is appropriate, you can always cancel your test date and not take the exam.

If your scores are released on July 23, you are in a perfect place to apply this cycle and will not be at a disadvantage. My suggestion is that you submit your AMCAS application with transcripts to just one school on the first possible day, and only add more schools when you have your new score. This will ensure that your application is verified in a timely fashion and will allow you to complete secondary applications as soon as possible.

As far as studying for the MCAT, it is hard to say. Every person studies differently using a combination of review books, courses, and practice tests. My greatest suggestion is that you take multiple practice tests to assess your progress, and if your score is not changing, change your study method.

Finally, if neither MCAT goes well (or even if they do!), consider waiting a year to apply. There is no rush to enter medical school, and a year off can often provide growth that can not be gained in a professional program. This would also allow you more time to study for the MCAT and improve your resume.


is doing MAVNI during medical school a good idea? =

I am a fourth year pre-med student currently applying to DO medical school this year. My GPA is not superb (3.65) but I did a lot of EC activities (more than 500 hours of clinical experience, > 100 hrs of shadowing, drug discovery lab research presentation at conference, disability study publication, etc).

Last semester, I came to know the expedited US citizenship program called Military Accessions Vital to the National Interest (MAVNI) program, and I'm currently in process one step away from signing enlistment contract to Army Reserve ( a physical exam and signing the contract next couple weeks or so). According to my recruiter and research, I will be naturalized in an exchange of 5 weeks of Basic Training, 8 weeks of Specialty 68W (10 weeks for 68D) Training, 24 drills a year (approximately 1 whole weekend a month), mandatory 6 days of annual training plus possibility of deployment(s) ranging from 6 months to 1 year. My recruiter told me deployment is very unlikely, which got me hooked in the program at the first place, but my further research has revealed that it IS likely to be deployed at least once or twice during 6 years of service. This put me in a very uncomfortable situation because neither sources, recruiter and internet, are reliable...to make such important decision that will have a significant impact on my path to become a physician, and I'm looking for someone who went through the medical school with F-1 visa status and matched with a residency program in US.

Will gaining US citizenship overcome the downsides of spending one whole weekend every month throughout the medical school and possible repeat(s) of school a year or two, and be worthwhile when it comes to getting into a residency program? I'm not particularly looking for any competitive specialty (e.g. radiology or general surgery), but very interested in Emergency Medicine. Currently leaning towards withdrawing my MAVNI process because this bet looks too risky for me...

I know this is very wordy question, but I thought I would post this question because I believe this would apply to many other pre-med students out there around the country, and benefit SDN discussion diversity. If this is a redundant, please forgive me and kindly direct me to a correct forum. I would really appreciate any advice given!

tantacles

It is very difficult to say if this program will help your chances of getting into medical school. Getting US citizen or green card status is worthwhile, so if you are an international student, it can very much help you to have a green card or citizenship. Your GPA is appropriate, and if you get an excellent MCAT score, you would have a good chance of admission. Consider, though, that you may want to push off applying for medical school right now as "I will be a US citizen" doesn't mean too much to admissions committees, while "I AM a US citizen" is not at all a gamble for a medical school.


I was recently accepted for a research coop at Johns Hopkins Applied Physics Lab (JHUAPL) so I was wondering my chances of being accepted there for medical school? =

My GPA is about 3.8 and the average at Hopkins is 3.9. I was wondering if there are any other members who have been in a similar situation. I was told if the coop went well, I could be offered a job. Would a gap year of working at JHUAPL further help my application? I am a bioengineering major and as most people know, engineering does hurt a person's GPA but I don't think top medical schools care about that.

tantacles

It is extremely hard to say whether this co-op will help your application. Each medical school views these experiences differently. Some may look at an in-house experience favorably, and others may not particularly care where your research is as long as you publish. That being said, research is highly sought after in a medical school application, so any research you do should contribute positively to your application. In addition, your GPA is excellent, so you stand a good chance, as long as your MCAT goes well, of getting into any medical school.


Should I choose a career in medicine or research? Or how is the best way to combine the two? =

I am a microbiology and neuroscience double major (GPA: 3.63, haven't taken the MCAT yet). I have been very involved in research as an undergrad. I have worked in an environmental science lab for 3 years and have 1 publication from that. I have been a summer research intern at two different hospitals (including one internship at a prestigious, world-renown hospital). I have another publication from my time at the prestigious institution. I love the research lifestyle and could never completely forgo research. But I really want the medicine applicability of a MD. Plus, I have heard many bad things about the job outlook and pay grade for PhDs. I have also heard that many MDs can do research and can get more funding for their labs than some PhDs, but I've also heard that MD research is usually more clinical related. I have considered the MD-PhD route but I know that is very competitive. I also have heard the MD-PhD route is useless in some cases because MDs can do a research fellowship, and you rarely graduate with a strong MD and strong PhD, usually one part of the degree is lacking. What advice do you have?

Neuronix

Greetings,

Check out: https://students-residents.aamc.org/choosing-medical-career/careers-medical-research/md-phd-dual-degree-training/

Also, the SDN Physician-scientist forum here: http://forums.studentdoctor.net/forums/physician-scientists.32/

First you need to decide if you want to be a physician and treat patients. This is a serious time commitment. An MD without treating patients is not something I generally recommend because of the time investment, cost investment, and intellectual investment into patients. The degree is essentially useless without residency, but your shortest residency and fellowship options for MD/PhDs are at least 5 years.

So I assume you want an MD and want to be a physician-scientist. You can do MD alone or MD/PhD. There are pros and cons to both pathways.

For MD alone the pros are: 1. You can decide whether you want to do research later, no upfront commitment. 2. You can tailor your research in residency and fellowship directly to your clinical interests, which will evolve during your medical training.

Cons are: 1. You still need extensive research training (PhD later or equivalent several years of research). 2. You typically will have to pay for medical school.


For MD/PhD the pros are: 1. Free med school. 2. Thinking like a physician-scientist from day 1.

Cons are: 1. Upfront time commitment (8 year MD/PhD program, PhD before you have any significant clinical experience). 2. PhD research is often not directly applicable to your medical career later.


Job outlook is bad right now for anyone trying to do lab based research. An MD gives you a fallback career in clinical medicine. Even academic medicine nowadays involves mostly patient care and not much research, including for most new MD/PhD grads. It is a common myth that MDs and MD/PhDs get funding more easily than PhDs (see: http://forums.studentdoctor.net/threads/top-choice-phd-vs-very-good-mstp.1191667/#post-17595493).


As for "you rarely graduate with a strong MD and strong PhD, usually one part of the degree is lacking." Haters gonna hate. Both sides will give you issues for the rest of your life--MDs for thinking you're not committed to medicine, PhDs for thinking you're not committed to research. Get used to it. You have to decide what training you want for what career you think you want and go for it. Your GPA isn't bad, try not to let it go down at all. Score high on the MCAT (515+).


How does having been in the Army Reserve during college look on a medical school application? =

I am currently a freshman in college, and I was kind of interested in joining the Army Reserve because I want to serve my country and also its only one weekend a month and two weeks during the summer. If I do join, I would be part of the Army Reserve for my sophmore, junior, and senior years in college. I was wondering it would look on a medical school application. Does it help me or hurt me or does it not really matter?

casedentalmed

It's too hard to say that it would ever help or harm your application on face value, but realize you may want to consider the Health Professions Scholarship Program when you are planning your studies for medical school. Do whatever you can to be an outstanding student and reservist.


Im stuck on a undergrad =

I will be starting college in the fall, I'm looking to make college as affordable as possible. I was looking at William Patterson university, yes I know its not the best school but I feel like I would do good here. If I get a great GPA and MCAT score and have a lot of ECs, would a medical school discourage me based on where went to for undergrad?. I plan on majoring in psychology and minoring in biology, would this be a wise major to? thank you for your answers

casedentalmed

The college you attend is your decision and what you do with it would matter more. Make sure you have solid advising for your career direction.

tantacles

In terms of planning ahead for medical school, the best decision you can make is to choose a college or university where you will be able to do well. That means something different for everyone, but the most important thing you can do to ensure admission to medical school is to do as well as possible in your courses and do well on your MCAT as these make up the base of your application. Some medical schools may look at school prestige, but GPA and MCAT score are largely the most important parts of your application no matter which medical school you attend.


Dominican University vs UIC =

Hi, so right now I have to decide between Dominican (small private school) and UIC (big state school). I am not sure if I want to do pre-med or pre-pharmacy. If I go to Dominican I am guaranteed a spot in Midwestern's pharmacy school so not stress later because I wont have to apply or take the pcat. If I change my mind to pre-med, I think UIC is better because it is well-known and has more research opportunities, so that will look good on a college app but I am not guaranteed n=anything for pharmacy and will have to go through the application process all over again. I just cant decide, any thoughts?

Pathdocmd

I would not base where you go to college based on not taking the PCAT or having to go through the process of applying to grad school. The two schools are so different that you must have some type of preference: small vs large, Catholic vs state school, urban vs suburban, etc. Pick the school that you are most comfortable with and where you will be happy and thrive. If you do well (and you will do better academically and personally if you are at a school where you are happy) then you have a good shot at med school or pharmacy school.


Reapplying for the 2017 cycle as a non-traditional student =

I applied last year to 7 medical schools, and one DO school. I received secondaries from each of them but no interviews and I completed my secondary applications in mostly late August/early September. My sGPA is 3.83, cGPA is 3.72 undergrad, and 4.0 Graduate. 1st MCAT 25, 7PS, 8 VR, 10 BS on 1/8/15. Second MCAT 506 with 125 CP, 131 CARS, 125 BIO, and 125 Psych on 5/22/15. I currently work full time as an athletic trainer, I also also teach between 4-7 credit hours a semester, and take between 3-8 credit hours for pre-med courses. I've been doing this for the past 3 years. I have, I think, a lot of hours working with our team physicians. Primarily with taking notes/shadowing while they see our athletes, but also with formal shadowing during surgical days and clinic days. I do not have much vonunteer/community service but I do have about 30 hours. So I guess my biggest question is besides more volunteering is there anything else that I need to do before the next cycle?

casedentalmed

First suggestion as a reapplicant: Apply to more schools if you can. Just one DO school??? Why are you dead-set going to MD programs and not giving yourself a chance with DO programs as well? I'm not sure how much more volunteering you can do since you work full-time, but if you can squeeze in a consistent opportunity, I suppose that would be good.

tantacles

There are several important things you should do to improve your application for your next cycle. First, if at all possible, you may want to retake the MCAT. Your score is acceptable, but may be preventing you from getting a look from many schools who would otherwise accept you.

Second, 8 schools is too few. Invest $15 in the MSAR, and apply broadly to at least 15 schools (at least 10 of them ones you haven't applied to before) where your GPA and MCAT are above, at, or at least near (though ideally above) the median for accepted students at those schools.

Finally, I agree, that your volunteering needs some buffing. Try to find an opportunity that you can do over the long term.

It may be a good idea, rather than jumping into this coming cycle, to take a year off before reapplying to schools can see that you've had time for your application to improve.


I was thinking about majoring in Respiratory Therapy and then going to med school ? I am not quite sure if this is a good idea. =

I know you're not supposed to have a certain major in order to get into medical school as long as you take the correct classes. But I find Respiratory Therapy interesting although I want to go to med school to become something along the lines of a Neonatologist.

casedentalmed

Why are you thinking about this major? If you are willing to take a much longer journey to medical school by gaining a few years of experience as an RT to better understand what physicians do on a health care team, perhaps. But why do you want to go to medical school if you can make a difference as an RT more quickly and less expensively? You should talk to professionals in the fields of your interest to help you with this decision. RT is not a stepping stone to medicine.


Reapplicant. Red flags 5+ years old. Please advise! =

I apologize in advance for the length, but the text is important to supplement my questions below. For those reading, thank you for your time.

I am planning on reapplying this coming cycle as an MD/PhD (I applied for 2015 entry as MD). Reason for my rejections I found out through my interview process, were my institutional actions.

Below I will give you my stats profile and red flag profile. Here I would like to say that my time since rejection was a blessing in disguise, because it gave me time to explore other alternatives, and really make sure this was what I wanted to commit to (In the interim, I worked for a biotech company, took more classes, started a nonprofit, did more shadowing, and learned 3D printing). In this time I learned that I am interested in research informing my (hopefully) practice of medicine, and want to pursue this path.

Without further adieu, my current stats profile: GPA: 3.54 (upward trend from 3.0-4.0), B.A. Neuroscience, top 10 Liberal Arts college MCAT: 33 (11,11,11); expires after this cycle Notable EC stuff: 4 publications, 5 years (4000+ hrs) research experience, 100+ hrs shadowing (free clinic, hospital, international), 100+ hrs volunteering (nonprofit work, sports with kids with mental disabilities, and free clinic), captain of DIII soccer program (with numerous awards), and recipient of $100k social humanitarian prize. LORs: could not be more fortunate. I have had great mentors and role models, including 1 Nobel Prize winner.

my prior (2014-2015 cycle) stats profile: GPA: 3.46 (upward trend from 3.0-3.8), B.A. Neuroscience, top 10 Liberal Arts college MCAT: 33 (11,11,11); taken April 2014 Notable EC stuff: 1 publication, 3 years research (2000+ hrs) experience, 60+ hrs shadowing (free clinic, hospital, international), 100+ hrs volunteering (sports with kids with mental disabilities, and free clinic), captain of DIII soccer program (with numerous awards). LORs: Could not be more fortunate. I have had great mentors and role models who wrote me spectacular recs.

Now for the huge red flag: I have 6 "institutional actions". I know, sounds awful, and there is no way to wash myself of them. To defend myself in "the do you just not learn?" reflex to this, I was naively unaware my warnings were considered institutional action. Had I known, I would not have had even one. Still my mistake for not knowing, and now I have to live with it.

The best I have is a letter from my college administration saying that I am in good academic and disciplinary standing with the school, and that they endorse my application.

That all said, it is now 5 years after my latest warning. It took me a bit to adjust to the freedoms and responsibilities of college life and subsequently, acknowledging wrongdoing, even if only by witness. For that I am deeply regretful.

Context on the warnings (and why I thought nothing of them at the time): My college gives students a warning if they are present at the site of an action that is not in line with the school policies, or if the infraction occurs in their living space. I received the majority of my violations due to presence, and that is why they are warnings, with no consequence beyond notification of receiving a warnings.

The warnings (w/dates): 10/2010, 2/2011, 11/2011, 12/2011, 2/2012, 12/2012 12/2012

2 were fire safety: soccer bag in the hallway and non-regulation extension cord. 1 was presence at an unregistered party. 1 was for alcohol possession under the age of 21. I am now about to turn 26, and would now never jeopardize my future by doing anything illegal. 1 was for the co-possession of substance. There was marijuana found in the dorm room I shared with my roommate. Like above, I would now never jeopardize my future by doing anything illegal. The reason for this not being a probation or anything more severe is because it was not being smoked. It was my roommate's, but I have been urged not to bring that up as it shows a lack of responsibility. 1 was for harassment. I received this warning because I volunteered myself as a witness to a harassment event that took place on campus. Due to my presence, the disciplinary panel found me guilty of harassment as well. It was unfortunate, as I could not have stopped the incident in question, and I merely stepped forward to protect another student from suspension because I knew he was innocent. I urged the individuals that I knew were present to be responsible for their actions, but unfortunately none stepped forward. My role as an uninvolved party was corroborated by all other accounts.

How I have sought to address my acknowledgement of responsibility and personal growth: Since the summer of 2015, I have been working with a 17-year-old from the Bronx named Bibi, who does not understand the repercussions of his actions, but wants to be the first in his family to go to college. I was the second in my family, and it was a proud day when I was accepted. His position was more severe than mine, as he trafficked various drugs, was involved with a Bronx gang, and was a marijuana and cigarette addict. Since starting to work with him, he has obtained a job, finished his SATS, and is no longer smoking. Leaving the gang is difficult – after trying, they beat him so badly he has ended up in the ER, missing school and work. Unforseen to him, because these guys were his friends, it is hard to walk away.

He recently was accepted to college, and I hope that he can use that as a way to leave his neighborhood and gain some level of safety. Regardless, my advice about my past seems to have resonated with him on some level, as he is set to pursue a college degree and become a legitimate businessman.

Questions: 1) I have followed the advice of a director of admissions I was fortunate to speak with and separated myself from the warnings as much as I could before my MCAT expires, improved my GPA with upper level science classes, and fulfilled my grant through completion. Her advice was for MD programs. Are my chances worse/better for MD/PhD?

2) I only have 1325 characters to address and take responsibility for my IAs. Is my tone appropriate above? Should I discuss this at all in my PS?

3) I am planning on applying the first day the application opens and be diligent with secondaries. Do you have any other advice for my situation? Should I be reaching out to admissions now to inform them of my IAs/application?

Thanks again for reading, your advise is greatly appreciated.

casedentalmed

What you have written as your strategy seems reasonable. Followup with the admissions folks and MD/PhD staff. Think about other options though as well; IA's are not as big a deal for Ph.D.-only programs, and you need to think about how you want to be a clinical/translational scientist in an academic setting. You should think about MD programs that may welcome your experiences (including the IA's) as well.


Can't decide which school to attend! =

Hi everyone. I've been accepted to NYIT-COM and New York Medical College. I'm having trouble deciding which one to attend. I really enjoyed both schools but am not sure which school will give me a better education and greater opportunities. Does anyone have any insight on this?

Thanks!

tantacles

MD schools, by and large, will provide more opportunities after graduation as there is still a lingering stigma among many MDs about allowing DOs in their training programs.

That being said, it is difficult to say which school will give you the "better" education. At any medical school, there are exceptional, dismal, good, bad, and just ok instructors. This assessment is probably best made by students who attend these schools, though it can sometimes be difficult to find students who will be truly honest about their negative experiences. My suggestion is to reach out to as many students as you can and find out what they think! SDN as a whole is ill-equipped to give opinions on schools as very few people on SDN will have attended these particular schools (I certainly didn't!)

Another avenue you could pursue is to visit the school for a second look. Even if the formal second look weekend is done, you can call the schools and ask to come back and talk to current students about the experience, or a student may be willing to host you for a night so that you can get a less formal exposure to its environment.


Hi, I've attend two schools and have two undergraduate GPAs. One low, and one high. Would I list these separately when I apply or average them? Also, can I have some direction on what to do based on my GPAs? Whether to apply to DO, MD, Postbac , or SMP programs? =

My 1st school GPA is 2.9, 64 attempted, 54 earned credit hours.

My 2nd school GPA is 3.6, 78.5 attempted, 78.5 earned credits.

Pathdocmd

AMCAS will average the two undergraduate GPAs. I assume that the 3.6 GPA is later in your college career and upward trends are viewed more favorably than downward trends in grades. Grad school grades, if any, will be not averaged in with your undergrad GPA.


As a reapplicant, how much should I change my personal statement and activities sections? =

I am technically a third-time applicant, although due to unforeseen financial circumstances in my first application cycle I was unable to complete secondary applications for all but one school. I am currently waitlisted at two schools and have begun the process of reapplication and am stuck for how to change my activities and personal statement. As for the personal statement, all of the reasons why I want to become a physician are the same and I am not sure in what way and how much to change it, since I have heard that you should if you reapply. In addition to that, there are several activities that I am going to be putting on my application again and I am wondering if I need to change the section where I explain why things were some of my most meaningful experiences. If so, what advice would you have for going about that?

Pathdocmd

I would leave the activities section the same and add anything that you did between your last submission and now. You can change your personal statement and secondary essays to reflect the changes that have occurred. Did you you get only 2 interviews? It might be MCAT or GPA that is holding you back. I hope you get in to one of those two med schools, but if you don't I could contact those schools and ask for feedback. It might not be your essays, MCAT, or GPA. It might be a problem with your interviews. Good luck!


Personal statement =

Hi! I'm in the process of writing my personal statement, and I have hit a wall. I'm normally an exceptional writer, but I'm evidently not good at writing about myself.

I have a few ideas to choose from, but I keep going back to the story of when a loved one died by suicide. Other options that lead me to medicine are my grandfather's HSP, sister's pancreatic division, my mom (a nurse) always being the one that sick family member come to for advice, and my job as a nursing assistant at a hospital.

The main question here is what are admissions committees tired of seeing? I thought about beginning with the story of the day my loved one died by suicide, but I think the "I felt helpless" line is over used.

Recently, I have attempted to begin the paper by briefly reminiscing on when I was learning to drive (stay with me here). My grandfather taught me to drive with the notion that "the whole road is yours until something is coming," so I thought about using that to say that losing my loved on that way was like seeing headlights heading straight at me, so I had to pick a lane.

I just want to make this interesting without sounding cliche, cocky, boring, generic, etc. I have to turn in my rough draft to my advisor by Friday, so I would appreciate any feedback/paper review. Thanks!

tantacles

It is difficult to say what medical schools are "tired of seeing," as you never truly know who is reading your personal statement. It could be someone who went through a similar experience, and it could be the director of admissions that is checked out completely and about to retire, and it could be someone who just started on the admissions committee who hasn't seen anything.

With regard to your particular query, it is absolutely fine to include your loved ones mental illness as a piece of your application, but it would likely help if you approached it from the standpoint of how that experience helped to shape your perspective and how it informed your pursuit of medicine. It is understandable that you might be helpless in the moment, but remember that the goal of medicine is ultimately to prevent, treat, and manage many types of illnesses and that that thought should come through in your personal statement along with how your experiences will help you navigate the field.


How should a career-changer go about the 15 activities section as a reapplicant? =

The one thing I can't find information is how to go about the 15 "Meaningful Things" part of the application as a reapplicant.

I have some new things to add from shadowing, supplemental work, volunteering, etc as an update. As an "Old Pre-med"; however, most my 15 things are based on my [former] career and other activities that have spanned 15 years or so and have not changed much. For example, I didn't write about being on the "Student Government" in college because I thought writing about being a school principal was more relevant and meaningful. I have done more shadowing and volunteering this year (which I'll add), but my surgery internship at UC Davis with 500+ hours is still the "most meaningful"

I know it's important to write a new personal statement and new essays for secondaries. What have people done/heard about the 15 things essays? Is it acceptable to keep some of the same in this case? Do I keep the activity and write about a different aspect or retrospective thoughts?

tantacles

Your most recent activities tend to be the most important activities. Of course, your prior work experience is helpful, particularly if it spanned many years, but I would absolutely recommend pulling out some of your older activities and replacing them with new ones. There's no one right way to go about this process, and I am sorry that I can not give more specific advice.


School list for Reapplicant =

Hello! I will be a 3rd time applicant, and am trying to make this cycle my last application cycle, so I'm seeking all the help I can get. I applied TMDSAS and AMCAS in 2013 (as non TX resident), AMCAS in 2014, did not apply during summer of 2015, and will be applying in the upcoming cycle.

cGPA/sGPA: 3.79/3.69 B.S. Biochemistry MCAT 2012: 33Q (V10, P11, B12) MCAT 2016: 521 (130, 130, 130, 131) 100 hours volunteering in ER that started October 2015, will be ongoing 40 hours shadowing in Orthopedics and Neurosurgery in March 2016 50 hours shadowing oncology in summer 2012 2.5 years of undergrad research in biochemistry non-clinical lab 3 major service projects during junior and senior year (ASB participant, leader)- put in ~500 hours IM Sports captain for 3 years in college Religious group participant/leader- 2.5 years College ACS chapter office- Junior/Senior year TA for 2 undergrad courses (sophomore, junior year) Currently in 2nd year teaching in inner city school Coaching soccer for 2nd year at varsity level Member of adult soccer team

May possibly be not teaching next year and instead be a researcher in a pediatric hospital department Will be TX resident (so needless to say, I will be applying to ALL TX MD schools)

What OOS schools should I be applying to with my numbers? (I know there may be a bias present against TX residents, but for the purpose of discussion, let's say I'm not a TX resident; especially since I am a reapplicant, I want to apply to a broad range of schools to help maximize spread and possibility of acceptance)

Thank you!

tantacles

Some schools that I would recommend applying to given your unsuccessful application cycles are the following:

Rosalind Franklin Georgetown GWU Drexel Penn State Tulane The Commonwealth Medical College Loyola Dartmouth University of Rochester

These are schools that have long track records of accepting students from out of state.


Should I pursue a special master's program? =

I am currently going for my EMT-Basic Certification. But I am having doubts on the best path to get to medical school. Should i pursue a full time job as an EMT and retake the mcat? Or should I apply to enroll in a Special master's program. Would i be able to get into one with these stats??


I am a Computer Science Major. I received a 504 on my MCAT. I have a 3.7 GPA and a 3.5 sGPA. I have 200+ hours of volunteering in my hospitals ER. I have 100+ hours of shadowing (90 with a M.D and 20 with a D.O) I have done research in my undergraduate institution. 100+ hours research in a my local hospital. I have received letter of recs from both the M.D and D.O i shadowed and my pre-med committee. I am an eagle scout, athlete, and worked at a few different jobs since the age of 17 while going to school.

casedentalmed

What suggestions have your premed committee given you? I'm not sure there is ever a "best path" and they reviewed your credentials for an application.


Psychology as major =

I want to go into surgery. Can I major in psychology in college? Or should I stick with biology??

casedentalmed

You need to succeed in medical school first before you think about surgery as a specialty, unless you are interested in more direct route to do surgical procedures (dentistry or podiatry). As long as you have a strong foundation for medical school regardless of your major, you should be fine. Then find mentors in medical school that can give you more insight into surgery.


Stuck between Vet School and PA school =

Hi Everyone! I'm sort of indecisive and cannot make a decision. When I think I've made one then I go back and think about the other one. I'm not sure if I want to do vet school or pa school. For vet school I have some experience (science research, vet assistant since last year for a mobile clinic, and some shadowing hours) but I don't have any shadowing for pa school. I'm sure y'all already know but vet school is 4 years plus more if you want to specialize, and I wanted to do the Vet corps. I know I will love being a vet plus being in the army since I've done rotc previously but I've heard vets do not make that much. Then pa'a only have 2-3 years of school and could make more than a vet right out of school, and I'm sure I would be happy doing that too. I struggling because I want a job in medicine that I would love but not be struggling financially in. Could y'all help me?

Also, I will have a master's in cancer biology in one year, if that helps.

dyachei

Either way, you should get experience hours in both. That's the only way you will be able to make an informed decision.

If lifestyle is important to you, it is very difficult to make a lot of money in vet med. The debt to income ratio is high and can be crushing in some cases. Even if you do decide to do vet corps, there is no guarantee you will get this while in school, and therefore school costs may be high. The general thought is that if you'd be happy doing something else, you should probably do that instead. However, there are major differences between vet med and PA. 1) what kind of responsibility do you want? As a vet, you would be the responsible party. As a PA, your physician would be. This is both good and bad. As a vet, you'd have a bit more autonomy. However, if there's a legal issue, you're at fault. 2) Would you be happy with a limit to the scope of your practice? PAs are often limited in what they can do without physician involvement. Vets are not as limited (except by client finances or similar).

There's no reason that you cannot gain hours in both (and even apply to both programs) if desired.


Help I have 4 W's =

I have 3'Ws on my transcript and I might be on the verge of getting mother one I'm taking a class I don't need and I don't want a B in the class will the W's affect me when I'm applying for medical school. I am really scared this will mess me up my GPA is not that impressive at a 3.69 being. Junior. How will I fair . With these W's

casedentalmed

Have you talked with your academic advisor? For admissions committees (depending on the school you apply to), the preponderance of W's will outweigh any benefit you think you will get with a high GPA; one may get the impression you cannot handle difficulty or criticism, especially if you are afraid of getting a B in a class.


Rejection Feedback =

Ok so I got rejected last cycle and one of the feedbacks I got was this:

"I would mention that many of your essays talk about similar things. I would recommend diversifying these answers so we can get to know you better. Include thoughts about recent activities, etc, that you are involved in so we can get a better feel for who you are."

I gave my all last cycle to diversify my answers, but seems that it did not work. I took help from a phD person to check my grammar and content.

Now if I want to further bring out points of diversity in myself through my essays, what should I do? How should I go about it? I am having difficulty with this, any help would be greatly appreciated!

casedentalmed

It doesn't appear you understand what is some pretty straightforward advice. You should use your writing center and/or career services office to give you some insight on answering your question. If you have a health professions advisor, that would be even better. If you are having difficulty talking about yourself beyond what sounds like a limited scope of activities or experiences, then you will come across as being very wooden as an applicant. Remember the criticism by Gov Chris Christie against Sen Marco Rubio for a debate in New Hampshire of always saying the exact same "catch phrases" over and over again? (Here is a link for a reminder: https://www.youtube.com/watch?v=a0WUtNJAo9k .)


Should I disclose a summary offense (less than a misdemeanor) if it is never asked for in primary or secondary applications? =

I recieved a summary offense (less than a misdemeanor - like a non-traffic version of a speeding ticket) for disorderly conduct for being under the influence of marijuana. I plan on applying to medical school in 2 years. The primary AMCAS application only asks about misdemeanors and felonies, which this is not. If a secondary does not word a question in such a way that I would have to disclose this, should I still disclose it voluntarily? I don't want to be dishonest but I also don't want to put myself at a disadvantage unless necessary.

casedentalmed

While you don't have to disclose on the AMCAS application, it is probably wise to be forthcoming on secondary applications. It's very likely that such summary judgments won't really amount to much against you, but it is better to disclose when it comes to those gray areas. Now, you may have a question on a secondary application that asks you "Is there anything else you wish to disclose to us?", and I would read that prompt as an open invitation to disclose any of the gray areas. No, you are not required to but it may be professional courtesy to do so.


What college would be best for me? SOS =

Hi everyone, my name is Andrea and I am currently a high school senior. Initially, I began applying to schools with the general intention of becoming some type of doctor. I selected biology/biological sciences as a major for all the schools I applied to. I eventually settled on the idea of becoming an optometrist. However, now, I want to become something more. I think I might want to become an ophthalmologist or idk some type of surgeon or physician. I haven't really figure it out yet but I definitely want to go to med school. The life of an optometrist seemed way too boring for me. Anyways, since I didn't initially plan on going to med school, I applied to some non-rigorous and non-prestigious undergrad schools and got accepted into all of them. These schools consist of UCSB, UCI, UCSC, UCR, SDSU, CSUF, CSUSM, and NAU. I have been doing a lot of research and most people say that the undergrad school you attend doesn't really matter as long as you earn good grades, do well on the MCAT, etc. However, I am still very indecisive and just want to do what would be best for my future, especially if I want to get into a good med school. I really regret not applying to UCLA or UCB, haha. My stats were pretty decent... 4.2 weighted UC gpa, several extracirriculars and community service, above average sat/act scores. Oh well, what can I do? Please help!

casedentalmed

I guess I'm confused with your situation: I presume you are trying to choose a college from the choices you gave. Really the only thing that can be advised to you is to pick a school where you feel the most support and the most opportunity. Find a program that you feel has the best fit for advising prehealth students like you. You can only do what best prepares you for your future, so being in a supportive, safe environment where you can really learn and apply what you learn is going to prepare you.

tantacles

Ultimately, any of the schools you were accepted to will make it possible for you to enter medical school. I would suggest that you choose the school at which you feel you will be happiest and at which you most feel you will be able to succeed and do well in classes, whatever that means for you. Your major, similarly, isn't particularly important, and you should choose a major that will allow you to remain interested and do well.


Med school application advice (Post bacc vs clinical experience) =

I applied to around 17 DO schools this cycle (applied mid July). I had one interview and have been placed on the waitlist. I am also taking a prep course and plan to retake the MCAT this spring. I am currently debating if I should apply to a post bacc program to boost my GPA, or if I should try to do something to gain more clinical experience? I have considered continuing to work as a scribe for another year, or possibly applying to a one year surgical assistant program to gain more clinical/work experience. What would make me a more competitive applicant when I apply the second time around? Any advice would be appreciated!

cGPA: 3.34 sGPA: 3.02 MCAT: 497

Other experience: -medical scribe (one year) -CNA (7 months) -Molecular biology research (3 semesters) -2 overseas health/public health service trips -27 hours shadowing a DO -Leadership in extracurriculars (2 years)

casedentalmed

Is your calculated GPA's AACOMAS calculated? If so, then perhaps a postbac program could help, but you are on the waitlist, and you never know if you might be taken off it (though you cannot bet on that). Clearly your clinical experience seems to be solid, but if it is a question of grades and MCAT, you know where you have to work to be a more competitive applicant.

tantacles

Currently, the academic parts of your application seem to be weak -as you know, your GPA and MCAT are low for medical school admission. I would recommend retaking some courses, as DO schools allow for grade replacement, and retaking your MCAT.

While you do this, you can continue to do longitudinal volunteering. You can continue to work as a scribe, but there must be more new pieces to your application than that if you wish to be successful as a reapplicant.


Post-bacc advice/options for applicant with JD/MSPP & BA (no science courses) =

-Undergrad cGPA: 3.36; BA in Political Science w/ minors in Philosophy and Sociology - *no science* -Graduate May '17 with a JD (health law concentration) and MSPP (in healthcare policy)

-Relevant work experience: 1) U.S. Dept. of Health and Human Services, Office for Civil Rights - Regulatory compliance; 2) Leonard Davis Institute of Health Economics at UPenn - Research assistant; 3) Rothman Institute - Orthopedics - Legal and regulatory compliance

-Volunteer experience (to date): 1) Ronald McDonald House Charities - Executive Board: Volunteer Coordinator 2) Medical-Legal Partnership at Hahnemann Hospital/Radiation-Oncology Unit - Pro Bono legal work for underprivileged patients below federal poverty line

-Clinical experience - shadowed various specialists throughout the past several years: -Cardiology (father is interventional cardiologist) - observed several diagnostic caths, angioplasties (both femoral and trans-radial), TAVRs, and mitral clips -also: Pulmonology; Infectious disease ; Neurology; Emergency/Critical Care medicine/Palliative Care medicine

I am seriously deciding to do a post-bacc program- after extensive deliberation, taking into consideration all of my experiences, true passion about a career in medicine, and the understanding that such a career change will require at least another decade of school

I'd most prefer a program in the Philly area (Temple/Jefferson/Drexel/Bryn Mawr/LaSalle/etc.)

My question(s): -Primary Q: Considering the above information, would I be a competitive applicant to any of these programs?

-Are grades from post-bacc coursework calculated into undergrad cGPA? -I do not fully understand the logistics of linkage programs; are post-bacc's with linkages more desirable/preferable for med school acceptance?

Any other advice that you may have would be greatly appreciated. Also, I apologize for the long post; there is certainly no need to address all of my questions.

Thank you so much.

casedentalmed

The first thing to do is look at all your postbac options and talk with their program officers. In Philadelphia, there are programs at Drexel and Penn, as well as Thomas Jefferson and Bryn Mawr (among others). You will need a curriculum offering that builds your biomedical science foundation. As for what makes you competitive, only those directors will know, but for a career-changer track, you probably have as strong a chance as anyone else.


Should I transfer? =

I am a sophomore at a 4 year liberal arts university, Western Washington University (quarter system). There is not that much pre-medicine competition and a lot of opportunities for research.

My only issue is that a lot of departments are understaffed and not very developed. For example, the biology 204 class which I just completed, I busted my butt and got a B because they have not edited their lab manuals that require 6+ hours of weekend work plus the lab is inconsistent with class work.

Not only is it infuriating and time consuming but it has costed me the grades of classes that I actually care about, and personally I feel like it looks bad that I got these grades in a introductory class when in fact I didn't blow this class off and I worked really hard. This cost me the grade of my organic chemistry class which I got a C in because I spent too much time on the bio labs on the weekend and my introductory communications class. English 101 similarly was a huge time commitment with high expectations from the staff but not much reciprocation. (In english 101 when I tried to argue my grade he said they already shredded my rubric and that only a few select like 3 students get an A.. now I have to go through a petition with the dean)

I went into college feeling like this University was one that I could really handle. I went to a competitive high school and the level of challenge of this university seemed appropriate.

But I find myself spending hours trying to figure out exactly what is expected of me and why my proffs/teachers are nitpicky and grading me on things that were never introduced to me. When I bring it up to them they always reimburse me with the points and apologize for the under staffing but this is taking a huge toll on me. They clearly know I am right and the little points I am getting back is not worth the added stress I am getting and the lack of commitment to my other classes.

I am striving for perfection in an environment that is not allowing me to. I love this school. I love the community. But honestly I am crying as I type this. Do you think I will have a chance anywhere else? Are all schools like this? In your experience do you think this incompetency will end with higher level classes (I have heard at my school that they "get better")

I currently have a 3.6 GPA which I would like to be way higher. If I redo a class here my grades are averaged. I want to optimize my chances to get into medical school and if that means transferring I will do it. It is also very rainy here and even if it costs more to go outside of state, I wouldn't mind somewhere sunnier (help with seasonal depression). Maybe a semester system would help as well? What are great schools you have went to that addressed some of my issues?

I feel alone in all this and any advice at all is very appreciated. Tell me your experiences. Thank you so much in advance.

casedentalmed

Compared to other problems, transferring to another school is an inconvenient hassle. In my opinion, I would focus on the quality of letters of evaluation you would be requesting. Are your relationships with faculty or at least a faculty mentor strong enough that such a letter can be included in your application packet? If not, then transferring out should be a consideration. I assure you not all schools are in the situation you describe.


Hi, I am admitted to CSUF (Cal State Fullerton) and University of Missouri Kansas City (Out of stat) with majoring Biology. Which school has a better for Medical School Admission? Any advice is welcomed. =

Hi, I am admitted to CSUF (Cal State Fullerton) and University of Missouri Kansas City (Out of stat) with majoring Biology. Which school has a better for Medical School Admission? Any advice is welcomed.

casedentalmed

Did you talk with the prehealth advising staff at each institution, and have you communicated with prehealth students at those schools? There are also a LOT of medical schools, so it's too challenging to surmise that one's university choice alone can determine success in the medical school admissions process.

tantacles

It is hard to say which of these schools is more likely to get you into medical school because ultimately, your undergraduate school is far less important than your GPA and MCAT score. I would suggest that you choose the school where you feel you will be able to get a high GPA and where you feel you will be most adjusted. Feel free to reach out to other students who are pursuing medicine to get more insight.


Whats wrong with my application? =

I am a graduate from a UC school (Biology) with a 3.87. I took my MCAT twice first time I got a 27 (11/6/10) and second time I got a 29 (12/7/10). English is my third language and I came to America from India in eight grade so most of my actual English education occured after 8th grade. I have about 13 EC's which include community service, a part time job, shadowing, and hospital volunteering.

I applied to medical schools twice for past two years. The first year only UCR invited me for interview and then I was rejected. This year I was offered no acceptances. Do you have any advice for me as what I should do? Your help would be greatly appreciated.

I am considering to apply to D.O. this year. Should I think about admissions consultants? I know that SDN is mostly against this but in my situation do you think its necessary to consult one?

casedentalmed

Usually if you had an interview at UCR but was rejected, there may be some feedback your received. The reapplication for UCR could involve how much you took any advice they could have given you to improve your application (among other things). So apparently something could have happened where you did receive specific advice and did not follow through.

You probably should have applied DO with your reapplication last cycle, so definitely apply this cycle.


Should I do a Special Masters Program? =

Hello,

I am in the process of putting in an application this application cycle and Im really half and half on whether I should do an SMP or not.

My school just opened one up and its only 6 classes next to the med students (my top choice) and they promise an interview if you finish with a 3.7 gpa.

My stats are

MD cGPA: 3.08 sGPA: 3.3

DO (With grade replacement) cGPA: 3.7 sGPA: 3.9

MCAT: 514 (Balanced)

I have 2 years experience in my current employment as a nurse in a cardio unit and have extensive shadowing and volunteer like 2 years of science tutoring and a year of playing guitar for my local nursing home. BTW the low md gpa is for a few semesters freshman year with no direction and leaving the university and not dropping (got like 10 F's in my first few semesters). But my last 2 years after transferring back is a straight 4.0 (includes pre-reqs/upper level sciences)

I know for DO i am competitive but do you think an SMP is worth it for a shot at an MD or should i just bite the bullet and apply broadly everywhere. Do you think my application shows that I am ready for medical school or will an SMP be required to show them without a doubt?

Would appreciate any guidance and thank you so much.

casedentalmed

I guess the first question is to figure out what your endgame is. Did you actually apply to or are you considering DO programs? If you are excluding DO programs, why? If you really think the metrics game works much better for DO applications and you think you can get into medical school sooner, then great! I hope you have shadowed a DO and know what their holistic approach is all about. If you haven't done so, then you need to check with admissions officers for the MD programs you are considering to see if your application could be considered due to an upward trend in your grades.

The endgame is important: do you want to spend more money on a postbac/master's program when you could use that money towards medical school tuition? What do you really want in medicine? Is it that important to get an MD over a DO?


Should I disclose my narcolepsy diagnosis when asked about =

I'm applying to medical school this cycle. I have narcolepsy and this has negatively affected my academic performance. I was a top student in high school and was always studious so it surprised me immensely when I started to do poorly in my classes. My worst year was sophomore year when I received a failing grade and 3 barely passing grades. Those were my pre reqs but I repeated all of them and did significantly better. Also, over the past year I've taken harder upper level classes (molecular genetics, anatomy, microbiology, upper level bio class, etc.) and have done better. Whoever else, despite doing better my overall and science gpa is still low (im a bio major) because of those poor grades 2 years ago.

I guess my question is, how do I tell admissions committee about my narcolepsy? Narcolepsy really has effected my academic performance negatively and I really feel that if I could be in medication (i can't because I suffered an autoimmune disease from one of them and thus all the meds that treat it I am allergic to) I could do better. I'm not making excuses this really has affected me. However, I don't want to come off as making excuses. Also, I don't want to disclose my condition if they are only going to look down on me because they may see my condition and thus deem me as unfit for medicine,

So I guess my question is, should I disclose it? Or will it only make it worse? Thank you!

casedentalmed

I don't think you are making excuses if you have a documented health issue. Only you know how it affects you in your studies and you know how to discuss this with your health team. Have you gotten encouragement from your physicians or have they also given you the impression the severity of your condition will prevent you from doing well in professional school? I would ask a dean of students at some of the medical schools you may be interested in and certainly a few where you are not as interested in attending. There are opportunities in your initial AMCAS/AACOMAS application to describe your situation as a personal difficulty you had experienced while going through undergraduate, and I would expect similar opportunities in school-specific applications.


A reapplicant for medical school trying to figure out what is wrong with my application. =

Hello, I applied for the medical admissions cycle this past year, and got only one II. At this point, I am planning to reapply this coming cycle and was wondering what are potential issues with my application. Information regarding my 2015-2016 application: MCAT: 33 (12, 12, 10) on my 2nd try, received a 31 (11, 11, 9) on my 1st try. sGPA/cGPA: 3.68/3.78 (Cumulative GPA was 3.74) note: I took both semesters of intro physics at a cc after graduation. I am currently taking human biology and biochemistry at a cc as well. Psychology major-graduated in 2013 Turning 25 this year, California resident, Asian, male Submitted my primary end of june, verified end of july, submitted all secondaries by end of august (is this considered late?) Note: a potential problem is that I have too many reach schools. I will aim lower this time around.

Research: -research assistant at USC for 3 years, researched stem cells. Have manuscript and abstract published -research assistant at MGH/Harvard over a single summer, conducted transplantation research. No publications -Lead research at Seoul National University conducting stem cell research for one year (Fulbright research grant). Only presented at Fulbright conference. -PI/lead researcher at Westmont college conducting psychology research, supervised 4 undergraduates for one year. Presented at a conference in my alma mater. -research assistant at a Korean biotech company (1 year)

Clinical/shadowing: -shadowed in the pre-op, operating room, and post-op for a hospital (>200 hours)

Volunteering: -volunteered in a post-rehabilitation center for acquired brain injury (~50 hours) -volunteered at a private medical clinic that administered clinical pharmaceutical drugs (~50 hours) -volunteered at a hospital. Essentially I played with kids and distracted them from being lonely (patients were usually international) and from the constant surgical treatments. (~200 hours)

Other ECs: -Asian Student association (2 years), was the president (1 semester), vice president (1 semester), and officer (1 year). Was also on the Intercultural Organization board for one year. -social outreach in Mexico over spring break for 4 years -worked as a tutor for over seven years

Awards: -Fulbright Student Fellow (research grant to South Korea conducting stem cell project) -Presidential Scholar and Global Family Scholarship Endowment (academic awards at my alma mater) -Deans List (for 1 or 2 years, I believe)

Activities since submission: -taking human biology and biochem at a cc. -shadowing at a hospital and conducting outreach to the local Korean American society and South Korean organizations. -working as an instructor for an after school program (teaching SAT, ACT, and AP courses)

Medical Schools I applied to: Albany Einstein BU Emory Harvard USC Temple Uni. Morehouse Oregon UPenn Rutgers New Jersey Rutgers Robert Wood Stanford Stony Brook Uni of Miami Tufts USF Uni. Of Arizona UCSD UC Davis Uni of Colorado Uni of Connecticut Uni. Of Illinois (interviewed) Uni of Min (Minneapolis) Uni of Nevada Uni. Of Virginia Uni. Of Washington Vanderbuilt Virginia Commonwealth University

casedentalmed

What do you mean you got only one??? One interview, or one acceptance, but you're not happy? Assuming you mean one interview out of the entire list, it's too hard to know since you do have a relatively robust resume profile. I don't know if you think you have too many reach schools, but if you do, then I think you don't have an idea what schools you want to go to in the first place. I have no idea about your letters and how strongly they help highlight your personal characteristics and ability to truly overcome challenges or work with others. I don't know if your personal statement is unfocused, or if your message of your interest in medicine is too diffuse. I don't know if you just copied and pasted your application answers and didn't really show any specific interest in the schools you really fit at (and maybe you don't know what comprises a desirable fit for you).

Hopefully you had a prehealth advisor at your previous undergraduate institution because he/she/they may be able to give you better insight. If not, there is a network of volunteer prehealth advisors at the NAAHP under Student Resources > Find an Advisor (http://www.naahp.org/StudentResources/FindanAdvisor.aspx) that might be able to connect you with one.

tantacles

It is my strong opinion that you need to add some more mid-tier and low-tier schools to your list. I would recommend the following:

Remove:

Harvard UPenn Emory Vanderbilt Stanford University of Washington University of Nevada University of Colorado Oregon University of Minnesota Morehouse University of Connecticut

Add:

Rosalind Franklin Loma Linda (assuming you are willing to put up with their non-medical requirements) UCLA (It's your state school!) UCSF (Another state school!) Georgetown GWU Drexel Penn State Tulane The Commonwealth Medical College Loyola Dartmouth University of Rochester

While your list was a little top heavy before, your list is very top heavy for a reapplicant. Use the MSAR to help you whittle your list down more if necessary.


Mission trip that will delay applications to Med school by one month =

Hello,

I am returning to Mexico for an extended mission trip this summer (5/28-7/1) to work with orphanages in the same area I went last year for spring break. I felt a strong desire to return knowing more work had to be done. I will be working with other interns who will facilitate other groups who come in to volunteer and assist in serving the needs of impoverished kids. I committed to this trip late last summer but I am concerned about not being able to submit my primary until July after receiving a 503 on the new MCAT. Should I expect the delay to hurt my chances? Is there any way that admissions committees will look favorably on this? Thank you.

tantacles

Applying in July is not a death sentence in terms of medical school applications, and submitting your primary in July (ideally, on July FIRST on the nose) would be ideal.

That being said, your MCAT score is a bit low for admission into medical school, and that will likely hurt your application far more than your choice to delay your application. My earnest recommendation would be to put off medical school applications until you are able to improve your MCAT score.

In terms of your mission trip, it is hard to say how admissions committees will view it. A month-long mission trip is a relatively short experience, and many committees like to see evidence of a longitudinal commitment rather than a short term service trip, the likes of which have become reasonably common among many college students. However, it certainly will not be viewed negatively.


What to do in Gap Year =

I am currently completing a joint bs/mph degree at a state flagship school. I will be taking the June MCAT and applying for matriculation in med school in Fall 2017. What is the best way to utilize my gap year? Should I be a scribe? Do something related to public health? Do medical research or try to get more clinical experience. What do the adcoms want to see?

casedentalmed

Admissions committees want to see someone who performs at his/her best in doing something he/she is passionate about. Assuming there is no real issue with grades, then the question is what would the gap year activities do to deepen that passion and understanding. Scribing is a great opportunity, as is doing biomedical research or additional focused work in clinical settings. Don't make it look like you're doing something just to get into medical school, and so the effort you invest in the activity is minimal or short-lived.


What Should I do Next? =

Hello. Im a white male from new york that applied to 15 Allopathic Schools and 1 Osteopathic School (NYCOM) because it was the only in state school I liked and the out of state ones seemed too competitive for out of state students. I am a Computer Science Major. I received a 504 on my MCAT. I have a 3.7 GPA and a 3.5 sGPA. I have 100+ hours of volunteering in my hospitals ER. I have 100+ hours of shadowing (90 with a M.D and 20 with a D.O) I have done research in my undergraduate institution and in a hospital setting. I have received letter of recs from both the M.D and D.O i shadowed and my pre-med committee. I am an eagle scout, athlete, and worked at a few different jobs since the age of 17 while going to school. I have been rejected from about every allopathic school but I am still waiting to hear from NYCOM (I completed my application in August and still have heard nothing)


My dream is to become a doctor no matter what happens this cycle. I am currently enrolled in an EMT-Basic training program so that i can work in my local hospital or the FDNY and I would like to retake the MCAT in 2017 to obtain a higher score and apply again. Is there anything else I should be doing to help boost my application to apply again?

Thank you for your time.

SDN Pre-Medical experts

One of the parts of your application that stands out to me as sub-par is your MCAT score: A 504 converts to a 28 on the old MCAT, which likely is the reason you have not received any interviews. That is the main part of your application that is lacking.

A score of 507 or above should give you a much better chance of getting into medical schools. In addition, while it may be expensive, it would likely be a good idea to apply to more medical schools the next time around. I would recommend investing in the MSAR to determine which schools are most likely to accept you with your combination of MCAT and GPA (look at the median and apply mainly to schools where you are at or above the median MCAT and GPA).


Where should I apply? =

Hi- not sure if I should post here but I am desperate! I am graduating from a northeast public university in the spring with BS in pubhlth. Science GPA 3.98 Nonscience GPA 4.0, MCAT 512. Currently enrolled in a 1 year accelerated MPH at the same university, but not impressed with the classes I am taking for it currently so I may just take time off and work as a CNA in a hospital (is that a bad idea?). Experience - 500 hours as a CNA in a nursing home, 1.5 years research/program development on a youth development program in northwest Alaska (travelled there on grant), 300 volunteer hours (non medical, more like big brother/big sister, unpaid non CNA nursing home volunteer, sexual assault community education, homeless shelter).. unfortunately no shadowing as I thought CNA would be better use of my time. I really want to be a PCP in underserved communities.. do I have chances at UMass med, tufts, dartmouth? I have heard these places are best for that. I am having trouble on MSAR figuring out what private universities beyond northeast accept people despite geographic distance. Any recommendations?

casedentalmed

I think the MSAR does give you a lot of information in the school's mission statements if you are looking for being in underserved communities. The next step is following up with your shortlist of schools and seeing on their websites what activities they have that support underserved communities. I also would not disregard osteopathic programs which also tend to have a strong focus on serving underserved communities.

The bulk of your work/volunteer experience would appeal to many medical schools. I would suggest some time observing in a community health center clinic to get a better idea of the challenges it brings compared to what the nursing home challenges are, but that's not a deal-breaker if it were me.

Most other private medical schools in the US have some component where public health or community service is a major part of their curricula, and I would just suggest the MSAR again for a first pass then contacting schools themselves. You may also ask for information on the National Health Service Corps and see if they have lists of their scholarship recipients and where they went to school.


27 and bad academic history, should I even begin? =

I am a practicing architect, which I studied in college but did not complete. While I was in school, I was suffering from severe depression. Ended up in the hospital a few times, and got horrible grades. I have been in therapy for awhile now and am getting my life back on track. However, I am miserable as an architect and I know I want to become a doctor. I have several semesters full of bad grades but I want to go back to school. Will my grades prevent me from getting into medical school when the time comes? I am 27 years old and I don't want to screw around, is it too late for me to start pre-med?

Thank you for your help

casedentalmed

It's not too late at this point to start taking prehealth courses if this is really something you want to do. Check out some postbac programs as options.


Steps to take to get into medical school (plastic surgeon hopeful) =

I'm very new to this website, but my question is what my options would be to get into medical school. I have a bit of a unique situation, but maybe someone can relate or has experience with this. My dream is to become a plastic surgeon.

I graduated two years ago, unfortunately with a 2.5 GPA. I had a lot of family issues which resulted in a parent becoming incarcerated, lost our home, etc. I was also working and was a division 1 athlete. I loved biology, and things didn't really click for me until it was too late and was about to graduate.

I have industry experience and am currently doing Research at a Harvard Medical School affiliated hospital and I love it! I'm doing really well here. I'm hoping to be on a few publications by the time I'm done here. I'm currently looking to take a few graduate classes (which they are paying for, thankfully). I've also worked a night job for two years so I have a bit on my plate.

I'm hoping to not have to retake classes in order to boost my uGPA, but would rather move on and overcompensate in other areas to demonstrate that I AM capable of succeeding and that my past academic experience shouldn't reflect on my potential.

What are the possible routes I can take? Any help is greatly appreciated!

casedentalmed

I am not sure you can do much to boost your undergraduate GPA anymore since you already graduated. I would look into postbac programs and coursework in your area. You should be able to do well in a master's program geared to premed students, given your experience in research, though it's not a 100% given.


Associate's degree in High School? =

I have recently been accepted to a boarding school that has a program with on of our in-state colleges. This program allows me to receive an associate's degree in General Studies when I graduate high school. I am curious as to how this will matriculate. Will I be able to graduate in two years, or will it take three or four? If I do graduate early, should I take a gap year?

casedentalmed

It's hard to say since it depends on the major you intend to pursue. I would check with the advisors at the undergraduate program. You may be able to graduate relatively early but it's more important to have an idea of your degree plan and the courses you have to take in your major.


Size of school setting...a factor for training? =

I was recently accepted to Rochester and UVA and am torn making up my mind. I really like UVA/Charlottesville, but one of the things I keep asking myself is - since Charlottesville has a population of 40,000 (and a metro pop of 200,000) and Rochester has a population of 200,000 (metro 1 million) - will UVA afford me enough opportunities for patient contact and clinical experience?

I'd love feedback from current students/doctors on this. Does anybody feel/have any insight on how the size of the school's setting has affected their med school training? Does a small population give you "enough" opportunities to hone your clinical skills?

Thanks so much!

tantacles

While it may seem that a small city does not have enough opportunities for patient care, hospitals that serve these communities often act as safety nets for larger regions. While I can not speak to UVA specifically, I can say that you will almost definitely have a varied and rich clinical experience at both institutions. What generally varies between institutions is the characteristics and socioeconomic and racial makeup of the population, and that would be a great question to ask of administrators or students you have contact with from your interview day.


Haven't taken any upper-level major courses, or finished pre-reqs upon applying? =

I transferred schools in my undergraduate career, and my new school does not recognize all of the premedical courses I have taken. Because of this, I had to retake quite a few prerequisites, and haven't had a chance to take any upper level courses. Will this hurt my chances? I've already taken the MCAT exam and received a score of 509, 80th percentile. Thanks.

tantacles

So long as you have done well in all of your courses and you have completed all pre-medical requirements successfully per the requirements of each individual school you are applying to, not taking upper level courses should not hurt your medical school application.


How can I improve as a re-applicant to med school? =

I am graduating one year early from my undergraduate university, so I applied to 27 med schools after my second year of undergrad (2015-2016 cycle). I did not get a single interview, which was pretty disappointing, so I was wondering if someone could point out what I'm missing? For my activities, I'm putting down everything I have now since I'm reapplying soon.

Also for the gap year I am now going to have to take, I plan on completing at 1-year Masters of Public Health. Will that help me during reapplication? What should I work on improving?

Major: Neuroscience related (it's an interdisciplinary major) Total GPA: 3.83 BCPM GPA: 3.79 AO GPA: 3.94 MCAT: 515 (93rd percentile)

      • Research:

-4 years research in physiological neuroscience lab (some of that time before college) -2 years at a behavioral neuroscience lab at my university -5 months working as a student researcher for a public health study for a health impact assessment of a Native American tribe

      • Clinical volunteering:

-2 years volunteering weekly at local hospital (OB department and oncology) -1 week medical brigade in Central America with my university -1 month shadowing a pediatric neurologist

      • Non-clinical volunteering:

-1 year working for global health related NGO -3 years working for global health & mobile health related NGO

      • Paid jobs:

-2 years working as a graphic designer for my university dining services marketing team -4 months working as a design and data consultant for a health technology firm

      • Leadership:

-1 year as layout and design editor of my university's multidisciplinary student research journal -2 years as officer for my university's undergraduate research network -2 years as public relations committee chair for my university's chapter of Global Medical Brigades -Co-founding a new student research journal focusing on global health projects for my university

      • Other:

-Entered my university's Entrepreneurship Cup with an idea for an HIV/AIDS support related mobile health application -Graphic and user experience designer for a student entrepreneurship group developing mobile applications -I am a scholar at my university

casedentalmed

Your activities are very impressive, so the question to me is how you selected your schools and did you really tailor your application towards those programs? At your school, do you use your prehealth advising infrastructure (including a committee letter if available)? If anything you have a lot of accomplishments but perhaps you need to focus a bit more on your ability to relate to patients or to caregivers? It's too challenging to say if it might be a perceived maturity issue.

Have you made any contact with admissions officers before you applied to get some impression of what they would like to see in applicants? Are you trying for research-intensive schools? MD/PhD programs? Any evidence that shows you can persevere through challenges and have a mature world view of health, disease, and society?

I just ask these questions because it seems that you need to talk with your prehealth advisors and some admissions officers for the schools you have the greatest interest in. I don't know if the interview cycle is over for MD programs, but it never hurts to check.


Should I apply this cycle, and where? =

I am trying to decide whether I need to complete a second post-bacc program or if I should just try to apply to medical school this cycle.

I am a non-traditional student, I majored in Psychology at a UC. I come from a rough upbringing and struggled in college initially, failing some classes. I am African American. I finished very strong however, with an upward trend. My undergraduate GPA is 3.0, and cumulative is 3.1 including post bacc.

I completed a post-bacc with a GPA of 3.5, but didn't do as well the last semester getting a C+ in biochem and B- in physics.

I am considering applying for medical school this coming cycle, however I am concerned about my competitiveness due to my GPA and weak last semester in post-bacc.

I will be taking the MCAT soon and have been scoring around 500 on the Kaplan diagnostics, and I have a couple of years of volunteer and research experience working in underserved communities.

I asked another person on SDN and they encouraged me to apply but I would just like one more opinion as the application process is so expensive.

So should I apply this cycle or take more classes to improve my GPA first? I would like to apply to about 30 schools.

casedentalmed

It's hard to tell you where you should apply, but I think depending on what type of training you are looking for, you probably would have a good shot at any MD/DO school that fits your own goals for training. You may want to talk with some admissions staff at schools you have particular interest in, but I don't know if applying to 30 is "too many" for you (as you are a California resident); it might be just a bit more than what you may want (maybe 20).

tantacles

I'd like to direct you to the following table, which provides some statistics for applicants with your characteristics:

https://www.aamc.org/download/321514/data/factstablea24-2.pdf

Somewhere around 40-60% of African Americans with stats similar to yours (assuming a 500 on the MCAT) have been accepted to allopathic medical schools in previous cycles. This suggests that you may be able to be accepted to medical school with your current stats, but that it would be beneficial to take more classes (even if not science ones) to improve your GPA.

I would suggest you purchase the MSAR, and if you decide to apply this cycle, apply to as many lower-tier (low MCAT and GPA acceptance medians) as you can. In addition, if you have time to take more classes, it might be wise to take a year off to take more classes. You need not complete a full post-bacc; you could simply take some "filler" courses to raise your GPA. Your upward trend will be helpful.


What is the big deal if I go to a med school overseas? Either way, my father told me that if I do not get accepted into a medical school here, he will talk to a medical school in my home country Iraq. Any flaws with that? Any better ideas? =

So I really want to be a doctor and I am really into medicine. However right now, I am in the 8th grade and I don't think my grades are high enough. If I do not get accepted here and I go overseas, what are the pros and cons with that?

casedentalmed

Medical education outside the United States/Canada is drastically different from the rest of the world. You probably won't get accepted to medical school in the US/Canada unless you are about to complete a bachelor's degree. In the rest of the world it's different because you probably have to test well as a "high school" (or some equivalent) student to be admitted to medical school, which would include topics that are normally covered in the undergraduate US curriculum.

The other challenge is that if you go through the six-or-so years of medical school overseas, you would need to take an exam in order to be able to practice in the United States, including being eligible for any residency positions in US/Canadian hospitals. Consequently it is a much tougher road if you ultimately want to practice in the US/Canada.

So I would say first things are first: do well in strong math and science classes in high school, go to a strong college/university in the US, and then be prepared for the application process to the US medical school system.


Scholarships for pre-med =

Does anyone know of full scholarships for premed studies in the U.S?

casedentalmed

Most of the full scholarships for medical school are federally funded, such as the National Health Service Corps, the Health Professions Scholarship Program, and the MD/PhD MSTP. If you are talking about for premedical studies, you will have to rely on the undergraduate school's financial aid scholarship funds; if you are talking about postbac programs, you will need to talk with graduate financial aid offices.

tantacles

Most schools that offer pre-medical courses will offer some scholarships to their best applicants. It is difficult to answer this question without knowing your level of training, but not knowing that, I would suggest that if you apply to colleges, you apply to colleges that are not highly ranked - ivy league schools may not give out scholarships based on merit due to their agreement with other ivy league schools, and other prestigious schools will only offer scholarships to the best of the best. If your credentials are fantastic, you may be able to get scholarships from more prestigious schools.

If you are done with undergraduate and pursuing a post-baccalaureate degree, I would suggest you look into the websites of the individual schools - their policies likely vary widely.


Is nursing a good premed =

I would like to study medicine and become a doctor. I need a scholarship because I am an international student but I am also an American citizen. There are more scholarships available for nursing that an undergrad scholarship for premed. Is nursing a good idea for a premed?

casedentalmed

While any major can prepare people for a career in medicine, I would not prefer having one begin with nursing coursework. You would get a lot of experience working in the clinic, but you would approach things from the professional perspective of nursing. You might not get as many science-rigorous courses if you took on nursing as a major and then tried to apply to medical school soon thereafter.

That said, you should know what nurses do and how they differ from physicians. You might be surprised how much you can do as a nurse or physician assistant under the supervision of physicians. Before you make a decision on your major, make sure that the short-term career of being a nurse is something you want to do as opposed to other allied health professions.


Traffic Misdemeanor =

I recently got accepted into medical and filled out the background check. I had no idea that I had a traffic ticket that was considered a misdemeanor in that state, so I reported that I had no misdemeanors on my application. Now it looks like I lied will the school rescind my offer of acceptance? What should I do?

tantacles

It is in your best interest to self-report to schools that this misdemeanor exists. It will show up on your background check. Given that you did not lie but simply did not realize this traffic violation was a misdemeanor, a school will likely overlook this small mistake. However, it is on you to apologize for overlooking this particular event before and not after your school finds out on its own.


Road to Psychiatry: Postbacc Psych program or 2nd bachelors? =

Hey guys, I'm currently finishing my last quarter of my BA degree in Public Health at the University of Washington. During my undergrad I switched majors a lot because I was unsure of what I wanted to do (grades reflected this). It wasn't until I took a three quarter break and worked as a Mental Health Counselor for at risk foster kids with behavior issues, when I discovered my deep passion for the mental health field. Working with the kids totally changed my perspective which led me into wanting to become a psychiatrist. I truly wish I could've realized this earlier rather than the end of my senior year, as I would be more goal driven. Because I didn't really have any sense of direction, I'm graduating with a 2.5GPA. Trust me I understand my mistake. I feel really stuck as no postbacc premed programs or masters programs will accept such low GPA. I simply don't know where to go from here. I did some research online and discovered post bacc psych programs (http://www.apa.org/ed/precollege/psn/2014/09/post-baccalaureate.aspx) which helps students wanting to eventually get their masters. Because my GPA is so low I don't know if it's better to do the postbacc program and get my masters in clinical psych or counseling then do a postbacc for premed certificate that satisfies premed prereqs OR just take a 2nd bacc in Chemistry to get my prereqs done and get A's? Financially speaking the second option is best but, I won't be able to do that at the UW bc space for non nonmatriculated students is scarce. Is it necessary to get another B.A in psych, then masters, then prereqs for psychiatry? I am curious in hearing other plausible routes or advice/words of encouragement. Working with the youth has fueled my drive to succeed just need a realistic plan to follow? Thanks in advance!

tantacles

It is my strong opinion that with your undergraduate GPA, it will be very difficult to enter allopathic medical school without an absolutely stellar performance in a post-baccalaureate program. Osteopathic (DO) medical school is possible and would provide a very viable route into psychiatry. DO schools also offer grade replacement - in other words, you could retake courses you had previously not done well in and they would count your new and improved grades as a replacement for your old ones.

That being said, doing another bachelor's degree would be costly and may not lead to the result you want. I would suggest doing a pre-medical post-baccalaureate degree. If you can not get accepted to one with a linkage program, you may be able to do a post-bacc "a la carte" at a state school. You then must perform exceptionally well on the MCAT.

Another option once you have finished your pre-medical studies (and done well in them) is to do a special master's program, which may provide direct entry into a medical school. Keep in mind, however, that these programs are costly and may not ultimately lead to an acceptance.

Finally, it seems likely given your track record that you may not be able to "make the cut". Medical schools typically will not even take a close look at applicants with GPAs below 3.0, so if you find yourself unable to reach this cutoff, I would consider other paths - clinical psychology would be very viable and would offer you a great chance to work in the field of psychology.

For more information on psychology, I would suggest reading posts from that forum (linked below).

http://forums.studentdoctor.net/forums/psychology-psy-d-ph-d.57/

Best of luck!


Dropping below full time =

Hello, I am currently a Junior with a Biochemistry Major. Due to my professor's unwillingness to let me take a make-up exam before his exam, I had to drop his course since I will be going to a school sponsored conference on the day of the exam. I was hoping to add another class but I dropped after the deadline so now I ended up with only taking 9 hours instead of 12. Will medical school say anything about me being a part-time student for one semester? I will still be able to graduate on time with extra classes to spare. Another thing I could do is take another 3 hours class during session 6 of the semester and that will count towards my total hours for this semester but I don't really need any of those classes. I'm also planning to take my MCAT in May. I have a 3.9 cumulative gpa and I'm currently in a sorority. I'm the VP of academics for my sorority and I work 20+ hours a week. I'm also researching about 5-10 hours per week on prostate cancer. If I stick to the 9 hours, I will also be volunteering a lot while studying for the MCAT. Will that be enough or will medical school question the easy workload. I have been taking 14, 16, 13, 15, and 13 hours per semester and some classes over the summer. I will be taking 8 hours this summer and 12, 12 senior year in order to fulfill my degree

tantacles

Medical schools typically don't take note of how many courses students take in a semester. The most important thing is that you continue to do well in all of your courses and graduate with the highest GPA possible as well as do very well on the MCAT. Your extracurriculars look great, but make sure you have some clinical exposure and non-clnical volunteering.


Will a college write up affect getting into medical school? =

So I am a sophomore in college and I have been doing great school wise. However, my room got in trouble for smoking marijuana. I rarely ever smoke however I did this one time and of course we were caught by campus police. These were not real police so it will not be on my actual record or anything. Although, I will have a write up from the school. I obviously am not going to make the same mistake again and I will continue to study even harder now but I am worried this will ruin/hurt my chances of getting in. I was just wondering if med-schools can see this information of write ups from schools or if it is something that I need to tell them. I have heard information for both sides so I'm not sure whats true. If anyone knows anything about this and can let me know I would really appreciate it because this means a lot to me. Ans also because if it is going to ruin my chances I would rather know sooner than later. Thank you.

casedentalmed

You need to talk with your university student conduct board about its policy on disclosure of student conduct issues. It also would depend on the school on how they would treat such behavior. The best advice is to own your mistakes at all opportunities to disclose this information.


Haven't finished Pre-reqs prior to Application but will finsh before matriculation-How much of a disadvantage? =

I have taken Biology (1 Year), General Chemistry (1 Year), Calculus (1 Year), Statistics (1 Year), Biochemistry (1 Semester), But have not taken English, Organic Chemistry, and Physics. Although the websites for individual schools say that you can complete prior to matriculation, realistically, how much of a disadvantage am I at? (CGPA>3.9, SGPA> 3.9)

casedentalmed

Since you haven't taken organic chemistry or physics, the first question I would have is how you are preparing/prepared for the MCAT?

Secondly, while technically it shouldn't matter if you don't have all of your prerequisites finished before applying (as long as they are all done by matriculation), you have to understand you are competing against thousands of applicants who will have those grades in hand. I do think your GPA trend may work in your favor, but realize that you may overestimate your competitiveness against the other applicants who have those prereqs completed.


Not sure if competetive enough.. =

My freshman and sophomore year were terrible for me. I goofed off and payed no attention to school. Currently, my GPA is a 3.2. I have step up my game this semester. I am looking for internships and research oppurtunities, and will start studying for the MCAT soon. Just wanted some advice on anything else I should focus on/do differently. Also, what is considered a "good" GPA for med school?

tantacles

The most important things you can do are to make sure you do as well in your courses and MCAT as possible. The higher your GPA, the better. Typically, medical school matriculants have GPA's at or above 3.5 before applying.

In addition, make sure that you have varied extracurriculars, including clinical experience, shadowing of physicians, volunteering, and research. All of these activities would ideally be longitudinal and last at least one year, but you do not have to participate for a huge number of hours each week.


Is Health Science (Microbiology) the best major for me to go with as a Pre-Med? The med school (Quillen @ ETSU) that I'm going for doesn't have any specific pre-reqs. =

tantacles

Major isn't particularly important when applying to medical school; in terms of statistics, medical schools look primarily at cumulative GPA, science GPA, and MCAT score when considering applicants for admission.

Therefore, the best recommendation I can make about major is that you choose a major that you know you can do well in. Getting A's in English classes is fantastic and looks great on a med school transcript. Similarly, getting A's in microbiology courses looks great on a medical school transcript. The only thing that can hurt you is if you don't do well in your coursework.

So to make a long post much shorter: Choose whatever major you are passionate about that you will be able to do well in.


What to do about loans during gap year? Do I have to pay them when I attend medical school? =

Im thinking that to be able to have enough time to study for the MCAT and have a strong application im going to need to take a gap year. Which means my grace period will end 6 months after. When I attend medical school in the following fall will I have to keep paying my loans? Or is there an option where I can put them off while I return to school? Im just worried about taking a gap year and having to pay my loans when I return to school. What are my options?

tantacles

You must pay your loans while you're out of school, but while you're in medical school, you can typically defer student loans until your education is completed. I would speak to a financial adviser at your institution when you matriculate to medical school to get this process started, and in the meantime, I would speak with a financial adviser at your home institution to see what your options are for repayment.


Should I apply for DO schools this year? =

I am currently a junior. I had a rough freshman year with a whole bunch of C's and D's in my gen bio/chem classes. I have a strong upward trend from there on, getting B/B+ in my upper lever science classes. (Orgo, Genetics, Biochem, Molecular Bio). My extra-curricular activities are strong as well. My cGPA right now is a 2.9 and I wanted some insight on whether it is worth applying this year or should I take a gap year.

tantacles

With a GPA of 2.9, it seems unlikely that many people will give your application a look. I would recommend that you continue to get the best grades you can and, yes, I recommend one (or more! take your time!) gap years to strengthen your overall application and possibly improve your GPA with more undergraduate coursework if necessary. Remember that this process is not a race, and you only want to apply once - when you're best prepared to get in!


Engineer Applying to MD/PhD in Engineering =

Hello! I have a question that might best pertain to a physician-scientist.

I have been working in engineering R&D for a large and well-known medical device company for a couple of years and am interested in applying to MD/PhD programs. I have some academic research experience in basic science molecular biology, and experience as a researcher for an academic-industry collaboration project (biology/engineering). I have a shadowing/clinical/volunteering experience as well.

Stats wise, I was (unfortunately) not a responsible student in undergrad, where I studied bioengineering. Since, I have shaped up and re-taken the pre-med science courses independently at a well-regarded public university (post-bacc GPA: 3.84). My overall AMCAS GPAs are (cGPA: 3.06) and (sGPA: 3.46). I am doing a master's degree in engineering (grad GPA: 3.90) at a highly ranked engineering school. My MCAT score is (38).

I know that stats wise, there is a big weakness; however, my clinical interests, along with my industry experience and research background are a strong case for an MD/PhD engineering, since the engineering research is usually adopted into industry. I generally get a positive response when I speak with faculty about my background, but I am afraid that I might get filtered out before a human reads my application due to my low GPA.

So questions:

1. I do plan to elaborate on my case for pursuing this path in my personal statements. What are the chances it will get read? I have heard that MD/PhD programs generally take a better look at their applicants, since it is such a specialized path. Is that true, and how can I use that to my advantage?

2. Although I was not a good student in undergrad, I am a better student now. Will programs see or care about the upward trend? Are there places in the application for me to emphasize this and my recent academic record that you can recommend?

3. How is industry research experience viewed for MD/PhD applicants in engineering? My school's pre-med advisor says schools don't care, but faculty I have spoken with say that it is a unique advantage. How should I interpret this?

I haven't been able to find much information, or other people in this kind of path, so far, so I would greatly appreciate any and all advice. Thank you for reading this, and I appreciate your time.

casedentalmed

I would ask the directors of MD/PhD programs for their advice. Target programs where there is a strong engineering program at the same university as the medical school. Ask your questions to those directors for their general advice; I suspect that they would be interested in your industry experience and value your graduate coursework. But ultimately you need to know why you need the MD/PhD degree to do what you want. You need to have some actual clinical experience knowing what the routine is for a general physician. Depending on where you want to target for your MD/PhD training, you need to figure out how much that clinical experience is valued in the evaluation of your application.


Chance of interview at NEOMED =

I have a 3.42 GPA 3.22BCPM and a 500 MCAT what are my chances at NEOMED?

tantacles

Without more information, I can't answer this question definitively, but based on solely your GPA and MCAT, your chances are not high. See the following document:

http://www.neomed.edu/admissions/medicine/direct/COMM1ClassProfile201515thDay.pdf

Per NEOMED, the mean matriculant GPA was 3.7, and the mean MCAT was 29 (corresponding to approximately a 505 on the new MCAT).

It is my strong recommendation that you continue to improve your GPA and retake the MCAT to have an optimal chance of garnering an acceptance at NEOMED.


Am I competitive for high-tier PostBac programs? =

Hello,

I plan to apply to PostBac programs for entrance in the fall of 2017. I graduated from a service academy in 2008 with a 3.07 GPA and a B.S in Applied Physics. I had an ACT composite score of 27 and SAT score 1260.

I'm currently Active Duty (I'll discharge about 2 months prior to entering postbac) and I've flown combat aircraft for the past 7 years. I'm also a URM.

While I feel these factors make me pretty unique, I'm concerned my lower GPA will take me out of the running for the higher tier postbac programs I want to attend (UVA, JHU, Mich, Goucher, Bryn Mawr..)

Do I have a shot at these schools? Is there anything I can do to make my application stand out in a good way?

Edited to add- I haven't taken the MCAT's. It's been 8 years since I graduated college and my understanding is that it takes you out of the running for most Post-Bac programs.

casedentalmed

The one piece of information that is missing is your MCAT or GRE scores (more likely MCAT). Your ACT/SAT scores don't really say much to your current situation.


Undergraduate research application help =

A few of the undergraduate research programs (CNUP, Amgen Columbia...) I'm applying to need me to provide a list of potential mentors I would like to work with. Would it be helpful for my application to get in contact with those mentors? Do they have a say in the application process?

tantacles

Unfortunately, there is no easy way to determine what these programs are looking for from an outside perspective. It is my strong suggestion that you call the program and ask what they are looking for with regard to their application. Unless you have an "in" in the program, there is almost no way to know exactly what the expectation is.


Considering a second degree and/or graduate degree =

There is so much wonderful advice flying around here I was hoping that someone might be able to help me out. :) So I first started out as a transfer student to a big name school and the class sizes were huge and my major was really tough. My GPA went down from a 3.9 to a 3.0 cumulative and I freaked out (being young and without a support network to coach me through it). I ran out of money and went to work before finishing my degree. I took classes on the side constantly because I like to learn but not to get anywhere in particular thinking that all hope of ever getting into medical school or doing research was completely lost. So now I have all these credits. Some of which I failed because of silly reasons--working too much, stress, family commitments, bad reactions to prescribed medications, etc. In hindsight, it was really silly to go to school without an actual plan but I have an insatiable curiosity. Eventually, I decided to go back to school to actually finish my degree and I realized there might be a chance of going to medical school if I get a second degree and re-take classes to bring my cumulative GPA up from a 2.6 to a 3.4. It will be two years starting next fall and by the end of it, I would have degrees in Microbiology and Chemistry with 7+ years research experience, 3+ years patient care (CNA and taking care of my sister), and hopefully by then I will have published some of my research. My concern is that because I failed classes and got a couple D's, if I re-take them my school replaces the grade with the second time you take the class which factors into the GPA but medical schools can still see them as flunked/D the first time. It looks horrible. Furthermore, the medical schools in my state are ultra competitive--a 3.7 is the average GPA of matriculates. Should I pursue a master's before I apply in addition to re-taking my classes/getting my second degree in Chemistry? I really wouldn't mind doing research first. :) I love research about as much as I love patient care and would like to be able to do both. Thank you so much in advance for any advice!!! :)

tantacles

The most important parts of a medical school application are the GPA and MCAT score. MD do look a trends in scores, but only to an extent; below a certain GPA often gets screened at many schools. DO schools, on the other hand, tend to be a bit more lenient and also incorporate grade replacement, which would allow you to boost your GPA by retaking classes you did not do well in.

Ultimately, the most important thing you can do to ensure that you are able to go to medical school is to continue to do well in your undergraduate classes and do very well on the MCAT; a master's degree will not factor into your GPA on AMCAS, the medical school application service used by medical schools in the United States, so will be of less utility, but may still be a useful investment if you are doing a master's degree in something you truly enjoy.

I would recommend that when you apply, you apply broadly to MD and DO school to ensure the best chance of matriculating.


I need help with life choice =

I am currently very discouraged undergrad student majoring in biology. I would like to know what I could do to become a doctor. Currently I am going in my first semester as a Junior and I am academic probation and have 2.5 GPA. I would like to go to UAG, American university of Guadalajara. I wanted to know what are options? Should I give up? Is the dream I'm chasing unreachable? What can I do?

casedentalmed

It is not easy to advise anyone with just numbers alone. I am hoping you already sought the help from a health professions advisor on campus, or perhaps someone from the NAAHP who volunteers his/her time to help counsel students like you. To connect with an advisor, go to http://www.naahp.org/StudentResources/FindanAdvisor.aspx .

The questions that pop into my head are very school-specific. How large are the class sizes? What tutorial help have you gotten to help you with classes? Have you actually talked with your professors? Are you appropriately balanced in your focus on your studies versus other activities? Are you far from home? There are a lot of things you have to have done and not done to fall into probation. The first thing you have to do is shape up and finish your studies.

I am also curious why you want to go to UAG. What information do you have that makes UAG appealing? Do you want to practice in the United States, and if so, in what capacity? Have you talked to the doctors you have shadowed about UAG or other foreign programs?

In short, it seems to me that you are completely lacking an advisor and champion/coach who can help you focus on the right priorities. You need to have that skill (focusing on priorities) to succeed in medicine. If you are unable to do that, your aspirations to become a physician will either never happen or you may be grossly underprepared for the challenges in the career.


Accepted at school after writing Letter of Intent, but then got an additional interview... =

Accepted at school after writing Letter of Intent, but then got an additional interview. Letter of intent said I would withdraw other applications and acceptances. Day after accepted, got the interview before I had a chance. If the school were to somehow find out that I went on an additional interview post acceptance, am I risking the acceptance? And is that even possible?

casedentalmed

While I don't think the other medical schools can find out, my suggestion is to do the right thing and honor your intentions and commitment. Your career as a health care professional is established on a foundation of integrity, honor, and respect for others. If you were sincere in writing your letter of intent, then you have articulated for yourself your commitment to attend that school for richer or poorer, etc. Why would you rescind on your commitment to the school that has accepted you now? Be fair to others in this process who are not as lucky as you and turn down the other interview invitation. You have made your bed.


Pre-MD-PhD Sophomore Check-in =

Hello, I'd be very grateful if I could get a little "check-up" (no pun intended) after my third semester in college. In addition to some *QUICK questions* under this very condensed repertoire, this is what I accomplished so far as a pre-MD-PhD college sophomore:

- Completed 2 years of chemistry (including orgo), 1 year of biology, 1 year of English, 1 semester of psychology, 1 semester of medical sociology. Avg GPA 3.8 - Had a 10-week translational research experience at a cancer center last summer. Presented research twice since then. - Started research during the school year in a biochem lab this year. - Started working as a peer tutor in chemistry and biology this year. - Shadowed two surgeons during clinic and surgery cases in winter break for two weeks

Plan to: - Take statistics junior year - Take 1 year of physics junior year (in case I do not want a gap year) - Start hospital volunteering this spring semester - Work with a local nonprofit for cancer outreach to the hospital I will work in - Sign up for the one-week spring break service project this year, ran by my school's community service club - Work in a cancer research lab again for 10 weeks in the upcoming summer

QUESTIONS: - Is physician-shadowing clinical experience? - How much clinical experience do I need as a pre-MD-PhD? E.g. should I sacrifice a 10-week summer research internship opportunity for a semi-long-term clinical experience... or is hospital volunteering during the school year enough? - Would the spring break trip be appreciated by adcoms? We would most likely build houses and public facilities for a neighborhood. Thus, it is not medicine-related at all. -Any other advice?

Thank you!!

tantacles

- Is physician-shadowing clinical experience?

Physician shadowing, while clinical experience, doesn't really cut it. Ideally, you want to be in a position to DO something. Serve in some way or get a job that requires you to interface in a more primary way with medicine.

- How much clinical experience do I need as a pre-MD-PhD? E.g. should I sacrifice a 10-week summer research internship opportunity for a semi-long-term clinical experience... or is hospital volunteering during the school year enough?

Hospital volunteering during the year counts as clinical experience, and there is no need to sacrifice your research internship if you continue that experience. That being said, if you can do both, that is ideal.

- Would the spring break trip be appreciated by adcoms? We would most likely build houses and public facilities for a neighborhood. Thus, it is not medicine-related at all.

It might be. Charitable work is seen positively. However, a longer commitment would be viewed more positively. I would suggest that you do not take a charitable trip if the cost of said trip would be a burden and instead look for domestic experiences, which may actually be more robust.

-Any other advice?

Do well in your courses and do well on the MCAT; these things are your priority. Everything else, including extracurriculars, can be made up by taking an extra year if necessary.


I do not know what to do. I need advise! =

I am currently a junior in college but I graduate in the fall. My problem is that I am having troubles with organic chemistry. I have taken the course last spring semester and did not pass because I missed it by one percent. While taking this course, I was dealing with a lot of landlord problems that made it difficult for me to focus on school. It got so bad that I had to get a lawyer involved. Anyways...I moved on to organic chemistry 2 over summer thinking that I could get a good grade because I almost passed organic chemistry 1 but received a D. All I needed was 2 percent this time. I retook organic chemistry 1 online with a professor who has never taught it online before. It was awful. He did not teach organic chemistry at all. He just created tests that he never taught us. Also, I was hoping this course would be more flexible because it was an online course. I needed more flexibility because I am currently working 32 hours while going to school full time. Anyways, I ended up with a D in the course without showing how I received the grade.. Now I am stuck on what I should do now. I am not sure if my school allows us to retake a course more than once. Should I move on to organic chemistry 2 and retake organic chemistry over the summer or next fall? or should I retake organic chemistry 1 and take organic chemistry 2 over the summer? My problem about taking organic chemistry 2 over the summer was that it was super fast paced and was taught in 5 weeks. I need to take the organic chemistry sequence before I can take biochemistry in the fall. Biochemistry is only offered int he fall. What should I do from here? HELP!

casedentalmed

You should ask your academic advisor at your school what the policies are on retaking classes. You also should go to your health professions advisor and be prepared for recognizing that you need a longer-term plan to build your academic transcript than you originally intended. Remember that application services evaluate all grades on the transcript regardless of any "retake" policy at each individual school, so both of your D's will count against you. It is vitally important that you seek advice from your school's resources or you will waste time and money on fruitless applications.


Research Experience =

I am currently a college junior, and I recently received offers from two professors asking me to become a research assistant in their labs. I was wondering if this would look bad on my CV if I do not produce anything from these two experiences by the time I submit my application. Generally, at my school, most professors have undergrads help grad students out before starting their own projects. Also, how much research are top med schools looking for? I have been an RA in a clinical research study for 1.5 years and have participated in a summer research program. Currently, I have 3 posters and I expect to publish 2-3 1st author pubs.

casedentalmed

The answer to your question is going to depend a lot on who you believe are your "top" choices for medical school. I will say there is no upper limit since you will often have other applicants with way more experience in research labs than you. Many of them did research after graduating as postbacs or were actually employed full-time. Others will have other work employment experiences in other fields before turning to medicine as a lifelong career.

If your goal is to ultimately get accepted to a medical school that requires research to graduate with your medical degree, then I think having experiences that result in poster presentations and publications is probably "satisfactory" but it may not make you the cream of the applicant crop. Quantity is not as important as quality and impact. If your goal is more focused on actually caring for patients, being an advocate for their well-being, and understanding that a patient is not a problem to be solved, you may have a wider array of schools to apply to that may not be so focused on research in your background.


what are my chances? =

I have an associates in biology, bachelors in biology and a minor in chemistry. My GPA is 2.8 and I am have not yet taken the MCAT. Is there a chance for me to get into a caribbean medical school? Or should I do my masters first and will that help my chances? What is that MCAT score that I should try for? Is there any hope for me? Please help........ Thank You.

casedentalmed

The first question is how serious you are in getting to medical school. It is too challenging to just look at your GPA and say whether you have a chance or not, though I will suspect that with your degrees, you're going to have a look at a serious postbac masters program and show you can really handle the work. As for your MCAT, what makes you think you can score within the range of matriculating applicants (around 30 in the old scoring system, and I would suspect at least in the 50th-60th percentile in the new scoring system? You need to get some help from a bonafide prehealth advisor from your old university and be prepared for a reality check.


Would you recommend working if you are in post-Bacc Pre-Med programs to pay for basic expenses? =

I'm going to apply for post Bacc Pre Med programs in the East Coast. I'm still working on my application but i'm thinking if I can work while studying to pay for expenses and not spend all my savings in 1-2 years of study. Is there a clinical job that I can take though my background is IT (not in Health Science) while having these classes and yet get an scholastic grade? Anyone here experience this? I appreciate your thoughts.

tantacles

If you can work during your post-baccalaureate program and simultaneously receive excellent grades, then working is absolutely a viable option. Many find that they are capable of working during a post-bacc program and performing academically excellently.

It may be wise to apply for jobs during your first semester in a post-bacc program rather than starting your job at the beginning. That way, you can see how you do in the program and then subsequently accept a job if you've received excellent grades and felt that you had time to spare.


Bad Scholastic History: Report or not report? =

On AMCAS and TMDSAS applications I have been informed the I will have to supply all educational history from every institution ever attended. Below I explain the situation and I have no problem submitting my transcripts from the previous institution that I attended from 2001-2003. However, I do not want to submit my transcripts from the community college. What should I do?

I am now a premed student with a GPA of 3.7. Previously I attended a private university in a different state from 2001-2003 with a GPA of 2.5 and did not obtain a degree. Following this I attended a community college for one semester with a transcript paraphrased below:

FALL 2003 -Course 1: D -Course 2: W -Course 3: F -Course 4: F

from 2001 - 2005 I was a teenager / early 20's. At the time I did not know what I wanted to do with my life. I have grown very much since then and will be graduating with honors from a well known state school. I look forward to your answer.

tantacles

You must submit every single transcript from every single graduate institution you attended, regardless of when you attended. Thus, these grades must be submitted. However, one semester of awful grades more than ten years ago is unlikely to ruin your application.

Not submitting these grades, however, will likely have a dreadful result; medical schools can and will find all of your transcripts, and if it is revealed that you lied in your application, you may be expelled.

You may have to explain these grades, so just be sure to highlight the growth you've made since you attended this institution if you're asked.


I selected the wrong ending year for one of my activities on the AMCAS application, what should I do? =

In the Work/Activities section of the AMCAS application, applicants must select the starting and ending month and year of each activity. For one of my activities, I selected the correct start date (August 2014), and the correct ending month (December), but selected 2015 as the year instead of 2014. I have already submitted my application, and I am unsure as to whether I should notify schools of the error via email/secondary applications.

tantacles

I would not worry about this. Mention it as a mistake if you are specifically asked details about the end date, but otherwise, this kind of error is very small and will ultimately have little impact on your medical school application. There is no need to notify schools about it pre-emptively.


How many volunteer and shadowing hours do top tier medical schools prefer? =

Hello experts,

I recently graduated with a undergraduate degree in business but I have decided to pursue medical school after undergoing brain surgery last summer.

I've jumped through all of the hoops so far to make this transition possible, all I have left to do is finish up my pre-reqs and take the MCAT over the next year.

I currently have a 3.95 GPA, and I know I haven't finished the science classes or taken the MCAT yet, but my dream is to attend a top notch medical school like Johns Hopkins.

So far this summer, while taking summer courses, I have been shadowing a local neurosurgeon and volunteering at two local hospitals.

My question to you experts, is how many volunteer and shadowing hours do top notch medical schools like Johns Hopkins prefer? I know that shadowing and volunteering hours is just a bonus on top of GPA and MCAT scores, but I would like to give myself a competitive advantage before I apply to medical school.

Thanks!

tantacles

The "top tier" medical schools don't put strict cutoffs on the number of hours of volunteering you need. That being said, do your best to do as much as you can in terms of volunteering and clinical experience and it will serve you well not only in terms of getting into medical school, but also as you continue your medical education. Top medical schools look for well rounded students with great grades and MCAT scores in addition to myriad extracurricular activities that would ideally include research, volunteering, and clinical experience. Having something else you do on top of that, whether it be arts, sports, or some other professional endeavor, is also a bonus.


What is the timeline for someone taking a gap year? =

So I've graduated with a B.S. in Biology. woo! I've decided to take gap year to build my portfolio up and earn some experience. My school pre-med adviser was really just a biology teacher, so I've been trying to learn most of this stuff (admissions process) on my own the last year and it's just an awful lot to navigate through. I've been studying for the MCAT and plan to take that in January. Also I plan to do EMT work starting in the fall. My question is about the timeline of applications and how to work around certain things.

So if I'm taking the MCAT in January and admissions opens next May for the next round, what can I do with letters of recommendation in the mean time? How can I hold on to them until I can register a 2017 AAMC account? Do I need an active AAMC account for my writers to submit their letters? Any information on this process would be much appreciated.

A side question! While I'm here, I'm having trouble understanding the AAMC admissions checkpoints because they are 1.) written for students as if they were attending medical school the next semester after they graduate and 2.) Only in terms of years and the actual months aren't listed. So admissions open in May 2016. When can I expect to hear back? and When exactly do classes begin?

I'm sorry If this is too broad or very basic. I'm doing this all with little support from family and advisers and it's been a little frustrating gathering information. If you have a better suggestion for me to go find this information, by all means, let me know. Any advisers for sale? XD

tantacles

It sounds as if you have a good start. I would recommend this thread as you begin your research. It answers your questions about letters of recommendation and gives instruction pages about AMCAS:

http://forums.studentdoctor.net/threads/2015-2016-applicant-sticky-ps-amcas-secondaries-interviews-etc.1126830/

Admissions open in May 2016, and the earliest you may hear back is October 2016, though admissions are rolling and you may hear back from schools any time before classes begin. You will then begin medical school, if you are accepted, in the summer of 2017.


Where can i find opportunities for ECs during summer? =

I am currently a freshman and an upcoming sophomore in Seattle, WA and i am trying to find ECs or a job or a research opportunity to do during the summer, I'm currently enrolling in two classes during summer but i feel that i have enough time to do something else. I would like to get some hospital volunteering done but i don't mind doing something else

tantacles

Volunteering is an excellent idea. In addition, you can always e-mail advisers at your school to ask for research opportunities. You will be best served by finding several PI's whose research needs fit your interests and then e-mailing all of them, hoping one e-mails back.


when to expect secondary applications after being verified by AMCAS =

I turned my primary AMCAS application on June 5th. It has been verified, when should I expect to get secondary applications from medical schools

tantacles

Each school runs on its own schedule, but most schools begin sending out secondaries in July and will send them out almost immediately after your AMCAS application is verified.


Is a degree in MLS/CLS a good degree for medical school? =

tantacles

Any major, provided you complete the pre-requisites for medical school, is perfectly fine. What is most important is that you choose a major you find enjoyable and one in which you can get a high GPA.


Deciding a school =

Hello, I am having trouble deciding between 2 schools mainly because of the cost difference. I am a chemistry major who will be transferring in as a junior from a community college. The two schools that I am deciding between for a chemistry major are Slippery Rock University and Pittsburgh University. I am currently enrolled in both schools but I have to make a decision eventually. I will suffer 20k debt at SRU because I can commute there, while at Pitt I am liable to suffer 50k - 60k debt (15-20k is living expenses). SRU's chem department is pretty small but they said they do research. Pitt probably has more networking opportunities. For Pitt my parents can take out Plus Loans from the government, however, if I go to SRU I only have to take out a Direct Stafford loan. Thanks for the tips From what I gather it seems the general consensus is that your undergrad does not affect your application to med school very much. I just want to make the right decision.

tantacles

Your undergraduate education matters, but only insofar as you go to a program you can do well at. If you feel that you can get mostly A's at Pitt and the debt is worth it to you, then that is where you should go. However, if the financial aspect worries you, it is just as acceptable to go to SRU; as long as you do well in your classes and on your MCAT, you will not be held back by your undergraduate.


What are my chances at top medical schools? =

I am an African American student applying with a 3.87 GPA and a 30 on the MCAT. I have done extensive volunteering and shadowing. My research is however, lacking; only one year of research with nothing significant coming from it. I have plenty of extracurricular activities; Organic Chemistry TA, president of my class in the service fraternity "Epsilon Sigma Alpha," volunteer work at local elementary school in a program called "After School Gators," Capoeira (a Brazilian fighting style), dance groups, parkour, etc. What are my chances of getting into a top medical school? Johns Hopkins, Stanford, Harvard, etc.?

Thank you for all your help!

casedentalmed

Metrics-wise you should be a strong candidate, but there needs to be a reason you are looking at "top" medical schools. Research is not required for entry to many top medical schools, but frequently you will find that many (but not all) require research for graduating with an MD program (assuming that is what you are looking at and not DO programs). Your letters of evaluation will make a big difference in showing that you have benefited from environments where strong mentors have helped you get to the point where you are confident in applying to professional school.


August 2015 MCAT, too late? =

Dear all,

I am planning to apply this 2016 application cycle for MD programs. I have a very strong application as I was selected as the valedictorian of my college with the GPA of 3.993, have an extensive research experience with a publication, many clinical and non-clinical volunteering hours, as well as great leadership roles through on campus organizations. My concern is that I have yet to take my first MCAT, and I do not think I will be ready until July/August. I am currently signed up for the July 17th MCAT, but I would really like to postpone it to August 22nd. Would that be too late for my MD application?

I will be submitting my AMCAS application by 6/15 to have it verified. I will also be writing the secondaries before my MCAT score comes out, so I can submit them asap.

Please help!

tantacles

It would be ideal to take the MCAT in July so that your entire application will be available to medical schools in August. Many schools start offering interviews in August, and this information would help them to make an earlier decision. Also, medical school admissions are rolling, so while an August date is not "too late," it may put you at a disadvantage.


Please help deciding my MCAT date =

Hello,

I am planning to apply for MD programs this 2016 application cycle. All parts of my application are strong except that I do not have an MCAT score yet. My GPA is 3.993; I was the valedictorian of my college; Have extensive research experience with a publication; many clinical and non-clinical volunteering experiences; active leadership roles through on campus clubs; etc. I have yet to take my first MCAT, but I am barely starting to study for it recently. Would you consider taking the August 22nd MCAT to be too late when applying for MD programs? I know the score won't reach the medical schools until September 22nd, but I will be writing the secondary applications before the score comes out so I can send the secondaries out asap.

Please help!

casedentalmed

Usually you want to give yourself about 3 months minimum to study for the MCAT before taking/writing the exam. Whatever you do, make sure you have enough time to practice and see any trends in your practice results before going in to take the exam for the first time. Every medical school will state its deadline or latest exam date for consideration for the entering class.


Should I become a Biology major or Nurse major? =

So I will be attending a for year university as an undergraduate in the fall. My ultimate goal is to become an OB/GYN. I recently took an Anatomy & Physiology class at my high school and I fell in love with it. I have a passion for learning about the human body and I want to continue it. I chose to become a Biology major but I feel like they do not get a more clinical experience as nursing students do which is why I am uncertain to which major I should go for.

Although I may be wrong, I feel as if Biology doesn't really cover the human body. I know it is more advanced but I am not interested in learning about ALL living things, but would rather focus on human beings. I know that in order to get accepted into med school I have to fulfill the pre-reqs and Biology along with other science majors fulfill that while nursing doesn't. Should I stick with the Biology major or should I change to a nursing major and take the med school pre-req classes as electives despite the extra work? Which would be the right decision. If I could receive more information on what it is like being a biology students and nurse student that would be great!

tantacles

In terms of majors for medical school, any major will suffice; the most important thing is that you choose a major that you enjoy and that you can excel in. Otherwise, as long as you complete the prerequisites for medical school, any major will do.


Realistic chances of med school with 2.8 science gpa. =

Title pretty much says it. I have a 2.8 science GPA and a 33 on the MCAT. Do I have any chance of going to medical school in the U.S.? Should I look elsewhere, such as the UK, Ireland, or Eastern Europe? Or should I try to improve my GPA.

Neuronix

You have very little chance in the USA today, but that depends on your cumulative GPA. Your best bet may be to retake your worst science courses or take more science courses to get the sGPA above a 3.0 and then apply. I don't recommend leaving the country at this point given the increasing shortage in residency positions and difficulty getting back into the USA after you leave. It is also difficult to score well on the USMLE, an exam for which foreign schools have no incentive to teach to or give you time to study for.


Chances of admission =

I scored in the 56-66th percentile on my MCAT and have a 3.64 GPA and a really strong application otherwise. Is this too low of a score?

Neuronix

https://www.aamc.org/students/download/430684/data/finalpercentileranksfortheoldmcatexam.pdf

This would correspond to an old MCAT score in the mid-high 20s. You may be best served by a re-take, though there are otherwise stellar students who get in with scores like that.


Should I apply NOW!?? =

Hello all,

So i am registered to take my MCAT in two weeks and I am still studying. I planned on waiting to send my application in afeter my june 20th 2015 exam, but for some reason, i have really been panicking and i have this urge to send it in now. I have completed most of the application but if i send it in now, i would have to look over my personal state ment, edit it more, do my activities section. This would take some of my study time. Do you think it would be too late for me if i put in the application late june. The reason why i wanted it in really early because my GPA is on the lower end (3.5). PLease let me know what you think.

tantacles

Late June is not too late for an application. However, note that your MCAT will likely not return until after you have submitted your applications. The best idea would be to submit your application to one school and then, when you receive your MCAT score back, apply to schools within your range.


A recommendation from a doctor whom I was a patient of =

I had a lifelong health concern that was mended, for the most part, by a certain urologist. He is very much an important role model when I think about the type of doctor I want to be. Is it appropriate to ask from him a recommendation? I was a patient of his, however, he doesn't know much else about me. However, he is very important for my choice into the profession. I know I would like to include him in my personal statement. How should I include him into my application. He might be happy sending a recommendation for me.

casedentalmed

In my opinion, no, it is not appropriate if the relationship is solely that of doctor-patient. You need to show you have a long-standing commitment to serving others and a strong mentoring relationship where the doctor shows you both the highlights and the lowlights of the profession. If you are just applying to medical school, picking a specialty in advance solely because of one physician or one group of physicians reflects some immaturity that will be a yellow flag to many.


Community college after graduation to take extra science classes? =

Hi I'm going to be a senior in college and my current gpa is 3.0 and my bpcm is 2.67. Before graduation I can possibly bring my overall gpa to 3.4 and my bpcm to 3.0 if I get As in all my remaining classes. I want to know how I can bring my gpas up? I'll be taking a gap year and I'm hoping to take some science classes at my local community college. Would that look bad for medical schools?

tantacles

The only way to bring your GPA up for MD schools is to continue to take college level courses and get high grades. DO schools, on the other hand, offer grade replacement; you may retake courses you did not do well in, and the schools will replace those grades with your new grade in the second version of the course you took.


which is a good job for pre med student other than EMT. =

Does pharmacy tech works good for medical school? I am really not interested EMT so pharmacy tech works for medical school ?

tantacles

Having a job is not necessary for admission to medical school, but a pharmacy technician position would give you exposure to patients and would qualify as clinical experience on your application, which could be helpful.


help please. I have 1.9 science GPA =

I just ended undergrad. its not on purpose. I worked hard but i qasnt prepared for college from highschooI and mind you MeD school Is harder than college. I took MCAT classes to help score the highest percentile on the MCAT. I volunteer at different health places. any ideas on what else I can do to get into med school? and which school's weighs more on the MCAT? I have no money for Caribbean school cause I need financial aid. Ugh I'm sad help!!!!

tantacles

While it will be difficult for you to enter an MD school, DO schools often offer grade replacement, which will allow you to replace courses in which you did poorly with the new grade; this is likely your best chance of raising your GPA to a point where medical school is accessible.